module 3 financial management and management accounting 1

464
CMA Accelerated Program MODULE 3 Financial Management and Management Accounting 1

Upload: others

Post on 11-May-2022

4 views

Category:

Documents


0 download

TRANSCRIPT

Page 1: MODULE 3 Financial Management and Management Accounting 1

CMA Accelerated Program

MODULE 3

Financial Management and Management Accounting 1

Page 2: MODULE 3 Financial Management and Management Accounting 1

Module 3 - Financial Management & Management Accounting 1

Page 2 © CMA Ontario, 2011

Table of Contents

Financial Management

1. Scope and Environment of Financial Management 3

2. Valuation 12

3. Financial Planning and Dividend Policy Decisions 26

4. Special Topics in Corporate Finance 46

5. Problems with Solutions 53

Management Accounting 1

1. Role of the Management Accountant 99

2. Cost Classifications 122

3. Job Order Costing 154

4. Process Costing and Spoilage 199

5. Cost Behaviour 249

6. Activity Based Costing 287

7. Service Department Cost Allocations 321

8. Joint and By-Product Costing 344

9. Cost-Volume-Profit Analysis 367

10. Pricing 397

11. Budgeting 426

Page 3: MODULE 3 Financial Management and Management Accounting 1

Module 3 - Financial Management & Management Accounting 1

Page 3 © CMA Ontario, 2011

FINANCIAL MANAGEMENT

1. Scope and Environment of Financial Management Introduction The financial manager is an intermediary between investors (the market) and the firm's need for financing. Financial managers typically make two major types of decisions: financing decisions relate to generating funds and managing the liabilities/equity side of the balance sheet while investing decisions are concerned with the allocation of funds and the asset side of the balance sheet. In large part, investing decisions are dealt with through the capital budgeting model that will be addressed in Lesson 18 of this course. The major emphasis in this lesson, therefore, will be on financing decisions. The financial manager must understand how wealth is created and measured. This requires an understanding of how financial assets are valued and how the value of productive assets are measured. Also required is a thorough understanding of the time value of money, uncertainty, taxation, and capital markets. It is generally assumed that the overall goal of all financial management decisions is to maximize shareholder wealth. While at first this may seem to be relatively straightforward, in reality the maximization of shareholder wealth is highly dependent on assumptions which are made regarding the timing, risk and expected value of future earnings. Finally, it must be noted that finance is a living discipline that is constantly undergoing refinement. There is much discussion on the part of financial experts regarding many of the theoretical issues discussed in this lesson. Although definitive answers to many questions may not be possible, the theories discussed will help illustrate the issues that financial managers must deal with and identify approaches for problem solving. This lesson attempts to cover a fairly wide spectrum of material that would ordinarily cover 600 or so pages in a textbook. Consequently, the discussion is abridged and aimed at addressing the main concepts . Students wishing to develop a more in-depth understanding of the topic are encouraged to refer to an introductory Corporate Finance textbook.

a. Objectives of Financial Management As stated above, the objective of financial management is the maximization of shareholder wealth. In order to measure shareholder wealth, we must be able to measure the size of future cash flows, when these cash flows are expected to be received and the

Page 4: MODULE 3 Financial Management and Management Accounting 1

Module 3 - Financial Management & Management Accounting 1

Page 4 © CMA Ontario, 2011

amount of risk or uncertainty that is involved. Note that the emphasis is on cash flows not income flows; financial management is concerned with the long-term flow of resources and, therefore, need not restrict itself to accounting measures of income based on generally accepted accounting principles. Essentially managers maximize shareholder wealth (and by definition, maximize firm value) by making the following three decisions: 1. The Investment Decision - investing in projects that yield a return greater than the

minimum acceptable return, 2. The Financing Decision - choosing a financing mix that will maximize the value

of the firm and that will match the assets being financed, and 3. The Dividend Decision - if there are not enough investments that meet the

Investment Decision criteria, returning the cash to the shareholders. b. The Canadian Financial System The Canadian financial market is comprised of participants and conventions that govern the exchange of financial assets. The market functions to:

• expedite the allocation of funds from surplus units (those with excess funds) to deficit units (those that need more funds than they presently have),

• provide a system where exchanges of resources can be performed efficiently,

• increase the liquidity of non-monetary instruments,

• facilitate the implementation of monetary policy, and

• provide a means of placing a value on financial assets. The term financial market is generally used to denote the total market for financial instruments. This market can then be subdivided into the primary and secondary markets. The primary market is the usual place where instruments are sold to provide funds for investments in plant and equipment. It is also where a firm selling a bond or share issue would offer its securities for sale. The primary market is comprised of both public offerings and private placements of securities. The secondary market deals with previously issued securities. The purchase or sale of existing shares on the Toronto Stock Exchange (TSE) is an example of trading in the secondary market. The importance of the secondary market cannot be stressed enough as it brings together buyers and sellers in an efficient setting. Without the secondary market, the liquidity of most financial instruments would decline substantially; who, for example, would want to invest in 20-year bonds or buy shares in a corporation if they knew that they could not sell these securities when they needed cash resources?

Page 5: MODULE 3 Financial Management and Management Accounting 1

Module 3 - Financial Management & Management Accounting 1

Page 5 © CMA Ontario, 2011

A second method of looking at the financial market is to classify investments based on the average length of time from initial investment to maturity. Generally, we think of the money market as encompassing short-term securities, such as treasury bills, commercial paper and certificates of deposit. Treasury bills (or T-bills), for example, are sold by the government through the Bank of Canada and are highly liquid (easily convertible to cash without significant loss). The terms capital market and bond market refer to instruments which are long-term in nature such as stock and bonds. In order to facilitate the functions of the markets, various financial intermediaries exist to bring together borrowers and suppliers of funds. The major financial intermediaries or institutions in Canada include: banks and trust companies, insurance companies and pension funds, investment brokers, and financial cooperatives. c. Agency Theory There are basically two agency relationships that matter to corporate finance: the relationship between managers and shareholders and the relationship between managers and creditors. We will limit our discussion in this section to the agency relationship between managers and shareholders. The agency relationship between managers and creditors will be discussed in Section 3 of these notes. The basic assumption we make is that shareholders, by virtue of their capacity to hire and fire managers and to design their compensation packages, exercise control over the managers. In return, managers consider the maximization of shareholder wealth as their primary objective in making decisions, even if it conflicts with other objectives managers may have. This assumption is subject to debate. Three major types of potential conflict can occur: 1. Managers may use corporate resources to provide themselves with 'perks' or to

embark upon expansions (empire-building) that are not in the shareholders' best interests.

2. Managers may have shorter time horizons than shareholders. For example, managers may make decisions that increase short-run profitability at the expense of maximizing firm.

3. Managers and shareholders may have a different evaluation of risk. There are several means available to reduce these potential conflicts: - establish incentive compensation plans that are tied directly to shareholder

interests (i.e. maximize firm value), - establish a strong corporate governance model, - monitor managerial effort, and - threat of takeover - firms whose share prices are below potential are much more

likely to invite a takeover attempt. Takeovers usually result in the replacement of management.

Page 6: MODULE 3 Financial Management and Management Accounting 1

Module 3 - Financial Management & Management Accounting 1

Page 6 © CMA Ontario, 2011

d. Market Efficiency and the Efficient Market Hypothesis An efficient capital market is defined as one where prices reflect all available information. Technically, in an efficient capital market, an investor purchasing a security will receive a normal return that is commensurate with the risky nature of that security and will not receive any abnormal returns. What makes a market efficient is the level of competition between investors. If a stock is considered to be underpriced, informed investors will buy this stock putting upward pressure on the price until the price reaches an equilibrium where it is neither under- or over-priced. The Efficient Market Hypothesis (EMH) states that it is impossible to obtain abnormal returns consistently with either fundamental or technical analysis. • Fundamental analysis is the evaluation of a security's future price movement

based upon sales, internal developments, industry trends, the general economy, and expected changes in each factor.

• Technical analysis is the evaluation of a security's future price based on the sales price and number of shares traded in a series of recent transactions. Technical analysis, when applied to the stock market, attempts to predict future share prices based on the movement of past share prices.

Under the EMH, the expected return of each security is equal to the return required by the marginal investor given the risk of the security. Moreover, the price equals its fair value as perceived by investors. In actuality, certain information impacts stock prices more quickly than other information. As such, the EMH has three forms: • Strong form efficiency. All public and private information is instantaneously

reflected in securities' prices. Thus, insider trading is assumed not to result in abnormal returns.

• Semi-strong form efficiency. All publicly available data is reflected in security prices, but private or insider data is not immediately reflected. Accordingly, insider trading can result in abnormal returns. Most experts conjecture that it is reasonable to assume that the market is semi strong efficient.

• Weak form efficiency. Current securities prices reflect all recent past price movement data, so technical analysis will not provide a basis for abnormal returns in securities trading.

Some implications of market efficiency: • changing accounting policies that increase income will have no impact on share

prices • managers cannot time the issue of securities, (i.e. the market will always price the

securities correctly). • large blocks of stock can be purchased and sold without affecting the price

Page 7: MODULE 3 Financial Management and Management Accounting 1

Module 3 - Financial Management & Management Accounting 1

Page 7 © CMA Ontario, 2011

e. Interest Rates An interest rate is a price, and like all prices it is determined by the laws of supply and demand (see Figure 1). Unfortunately, the market that determines interest rates is very complex, resulting in various theories regarding how interest rates are determined. Included in the list of complicating factors are inflationary forces and a host of risk factors, including maturity risk, default risk and foreign exchange risk.

Interest % Supply of Money Nominal Rate Demand for Money Supply = Demand Amount of Money Available

Figure 1: Supply and Demand for Money and Interest Rates Maturity risk refers to the duration of the investment. The longer that funds will be locked into the security, the greater is the risk that interest rates will change significantly over the life of the investment. Default risk measures the likelihood that the borrower may become unable to meet the terms of the investment agreement. Exchange rate risk deals with the chance that the real return will change due to fluctuations in the foreign exchange rate between two or more countries. As a general rule, one can describe the interest rate that will prevail in any situation as follows:

Nominal Rate = real rate + expected rate + risk of Interest of interest of inflation premium

It is important to note two things at this point. First, the expected rate of inflation is built into the interest rate that is quoted on the market. If the real rate of interest is 4% and the Canadian government sells T-bills to yield a return of 9%, it is reasonable to assume that the expected rate of inflation must be in the neighborhood of 5% because the risk premium is likely close to zero (there is little chance that the federal government will not pay the stated interest and principle at maturity). In fact, most financial analysts refer to the return on federal government T-bills as being the risk-free rate. Secondly, the greater the risk that is associated with a particular investment, the greater will be the required rate of return. As an investor, we would desire a higher return from a

Page 8: MODULE 3 Financial Management and Management Accounting 1

Module 3 - Financial Management & Management Accounting 1

Page 8 © CMA Ontario, 2011

speculative gold mining stock than from an investment in federal T-bills. This fact is one of the fundamental rules by which financial management decisions are governed. The risk premium is generally comprised of the following: • the default risk premium – the risk that the issuer of the security will default on

either the interest payments or the repayment of principal. It is generally assumed that debt issued by governments has a default risk premium of zero,

• the maturity risk premium – the longer the maturity, the higher the risk, and • the liquidity risk premium – if the security can be sold promptly, then the liquidity

risk premium is very low or close to zero. Note that for short term securities that are subject only to inflation risk, the relationship between real and nominal rates of return are governed by the Fisher effect: (1 + i) = (1 + r)(1 + π) where i = nominal (i.e. observed) interest rates r = real rate of return π = expected inflation rate The term structure of interest rates An interest rate curve can be obtained by graphing the interest rates (y-axis) vs. the maturities (x-axis) for debt securities of similar risk. The curve obtained is called the term structure of interest rates. More often than not, interest rates increase with maturity (due to the maturity risk premium). There are much less frequent periods when the reverse is true and the term structure is downward sloping. It will be downward sloping when investors expect interest rates to go down. The rationale for an upwards sloping yield curve is that, all else being equal, longer maturities are riskier. Much of this risk depends on inflation expectations, which are an important determinant of interest rates. This is the basis for the idea that the term structure should usually be upward-sloping.

Page 9: MODULE 3 Financial Management and Management Accounting 1

Module 3 - Financial Management & Management Accounting 1

Page 9 © CMA Ontario, 2011

f. Measurement of Risk In its simplest terms, risk is the chance that things won't turn out as expected; in other words, the return may be more or less than expected. The measurement of risk can be objectively or subjectively. In its subjective state, we see statements about organizations or individuals as being risk averse. As you have learned in previous courses, it is generally assumed that all individuals are rational, that is they behave in such a manner as to maximize their satisfaction given income, market opportunities and individual preferences. Note that we do not mean that those who are risk averse are not rational; it means only that those who are risk averse will behave in a manner consistent with that risk aversion. Risk can also be measured objectively by determining the variance of the probability distribution of expected returns from a certain investment. The greater the variance, the greater is the degree of uncertainty in the return or, in other words, the greater the risk. For example, look carefully at the expected returns for stocks A and B in Figure 2. The risk associated with Stock A is obviously greater than that for Stock B.

Probability Probability 0 30 60 0 25 30 35 Expected Return % Expected Return % Stock A Stock B

Figure 2: Expected Returns for Stocks A and B If one were to calculate the standard deviation for Stock A and Stock B, one might arrive at figures of approximately 9% and 0.25% respectively (this assumes that the probabilities are 25%, 50% and 25% respectively). The larger number indicates more diversity in the distribution of Stock A's expected returns and thus indicates higher risk. A "blue chip" stock such as Bell Canada would be an example of a low risk stock; stocks in an exploratory oil-drilling venture would represent a high-risk stock. Combining the notions of risk and return, we can conclude that investors considering investing in Stock A would demand a greater expected return in order to compensate for the greater risks involved.

Page 10: MODULE 3 Financial Management and Management Accounting 1

Module 3 - Financial Management & Management Accounting 1

Page 10 © CMA Ontario, 2011

Quantitative Measures of Risk The preferences of an investor in terms of the mean-variance theory can be expressed as follows: Stock A is preferred to Stock B… If E(Ra) > E(Rb) And σa < σb Where …

E(Ra) and E(Rb) are the expected returns of stock A and B respectively σa and σb are the standard deviations of stock A and B respectively.

g. Capital Asset Pricing Model By combining securities or investments from different firms, the investor can reduce the total risk associated with various investments. This notion of risk reduction is commonly referred to as diversification. Financial models can be employed to determine the expected return for a given group of securities (or portfolio) and the degree of risk associated with such an investment strategy. The expected portfolio return is simply the weighted average of the expected returns for the individual stocks. Similarly, the portfolio risk for a two-security portfolio incorporates the risks of both securities and the degree of correlation between them. Correlation refers to the degree to which the expected returns move together. If two securities increase and decrease in value together, they would be positively correlated; if they moved in opposite directions, they would be negatively correlated. Therefore, if an investor purchased one stock that went up when inflation rose and another which went down when inflation rose, the risk associated with holding the two stocks together would likely be less than that which would exist by holding either one individually. Financial analysts use a simple model known as the Capital Asset Pricing Model (CAPM). This model (see Figure 3) shows clearly the relationship between the degree of portfolio risk, the expected return of the portfolio and the risk-free rate of return. The straight line shown on the graph is known as the Security Market Line (SML) and represents various combinations of security investments that will yield a given return for a given level of risk. If an investor acquires a set of investments that matches the proportions of risk-free and risky investments available in the market, we would refer to this as a market portfolio. The expected return from this portfolio is called the market rate of return and is often approximated by the rate of return on a broad-based stock exchange index such as the Toronto Stock Exchange (TSE) 300. Investors may reduce their risk below that which is prevalent on the market by investing in less risky securities, but their returns will fall. Additional returns beyond the market return may be earned by acquiring a higher percentage of risky investments. The main application of the Capital

Page 11: MODULE 3 Financial Management and Management Accounting 1

Module 3 - Financial Management & Management Accounting 1

Page 11 © CMA Ontario, 2011

Asset Pricing Model and portfolio theory to a financial manager is the portrayal of the relationship between risk and return and how diversification can reduce risk.

Return % Security Market Line Market Return Risk-Free Rate Market Risk Risk

Figure 3: Capital Asset Pricing Model and SML The idea underlying the CAPM can also be extended to individual securities as they exhibit the same relationship between risk and return. Financial analysts are able to determine the risk associated with individual securities by determining the nondiversifiable risk or beta of the security. Note here that some risk is eliminated by the formation of the portfolio (this is known as diversifiable risk ) and the remaining risk (nondiversifiable risk) captures the amount of risk remaining for the security in question. Beta is a measure of the volatility of the returns for a particular investment relative to the market portfolio. The beta of the market-based portfolio is 1.0. Securities which exhibit less volatility than the market have betas of less than 1.0 (for example, Bell Canada and Trans-Canada Pipelines); stocks which are more volatile have betas of greater than 1.0 (for example, Alcan). Furthermore, we can say that if a particular stock has a beta of 1.3 it will rise in value 30% faster than the market when the market rises and fall 30% faster than the market if the market falls. The determination of the beta for a particular security, therefore, can tell us a lot about the relationship between risk and return for that particular investment. The capital asset pricing model can be quantified as follows: E(Re) = E(Rf) + β [E(Rm) - E(Rf)] Where E(Re) = the expected return of a given common stock

E(Rf) = the expected return of a risk free security (usually a long- term government bond)

β = the beta of the common stock E(Rm) = the expected return of the market (the TSE 300 is often

used as a surrogate for the market return)

Page 12: MODULE 3 Financial Management and Management Accounting 1

Module 3 - Financial Management & Management Accounting 1

Page 12 © CMA Ontario, 2011

The beta value of a firm's stock can generally be calculated by regressing the firm's returns against market returns.

2. Valuation a. Time Value of Money Many financial decisions involve the comparison of a cash outlay now with cash inflows into the future. Because one dollar received now is worth more than one dollar received in the future, future cash flows have to be discounted in order to compare them with today's dollars. This section goes over the mechanics of calculating discounted cash flows. The format for solutions using a financial calculator used throughout this module and other Accelerated program modules is as follows:

N I/Y PV PMT FV Enter 5 6 1000 Compute X

In the above example, we are trying the calculate the present value of $1,000 to be received in 5 years from now at an interest rate of 6%. If you are using the Texas Instruments BA II Plus, you need to do the following: - set the calculator to accept one payment per year as follows: 1 2ND N You only need to do this once. - clear the Time Value of Money memory as follows: 2ND FV You should do this every time you do a time value of money calculation. - enter the numbers above in the TVM memory registers - to solve, press CPT and the TVM register you are attempting to solve for, in this

case PV - the answer provided is -747.26. This means that if you were to invest $747.26

today (money out of pocket and therefore the negative sign) and invest it for 5 years at 6% compounded annually, the amount would grow to $1,000.

If you are using the Hewlett Packard 10BII, you need to do the following: - set the calculator to accept one payment per year as follows: 1 then Orange Button then PMT You only need to do this once. - clear the Time Value of Money memory as follows: Orange Button C ALL

Page 13: MODULE 3 Financial Management and Management Accounting 1

Module 3 - Financial Management & Management Accounting 1

Page 13 © CMA Ontario, 2011

You should do this every time you do a time value of money calculation. - enter the numbers above in the TVM memory registers - to solve, press the TVM register you are attempting to solve for, in this case PV - the answer provided is -747.26. This means that if you were to invest $747.26

today (money out of pocket and therefore the negative sign) and invest it for 5 years at 6% compounded annually, the amount would grow to $1,000.

Present Value of a Single Sum to be received in the future Say we expect to receive $100,000 in 10 years from now. What is the present value of this sum using a discount rate of 7%, i.e. what is this sum of money worth today?

N I/Y PV PMT FV Enter 10 7 100000 Compute X

PV = -50,834.93 Present Value of an Annuity Say we expect to receive $1,000 per year for 5 years (to be received at the end of each year). What is the present value of this stream of payments? Assume an interest rate of 8%.

N I/Y PV PMT FV Enter 5 8 1000 Compute X

PV = -3,992.71 Assume now that you expect to receive $10,000 per year for the next 10 years and a final payment of $100,000 at the end of the 10th year. Assuming an interest rate of 6%, the present value of these cash flows is $129,440:

N I/Y PV PMT FV Enter 10 6 10000 100000 Compute X

If you were to draw a timeline for the above, it would look as follows:

0 1 2 3 4 5 6 7 8 9 10 10,000 10,000 10,000 10,000 10,000 10,000 10,000 10,000 10,000 10,000 100,000

Page 14: MODULE 3 Financial Management and Management Accounting 1

Module 3 - Financial Management & Management Accounting 1

Page 14 © CMA Ontario, 2011

So, unless you tell it otherwise, the financial calculator always assumes that the timing of the cash flows is at the end of the year. Future Value of a single sum invested today Assume you have $1,000 to invest today and want to invest it for 5 years. Assuming a discount rate of 6%, how much will you have accumulated at the end of 5 years.

N I/Y PV PMT FV Enter 5 6 1000 Compute X

FV = -1,338.23 Future Value of an Annuity Assume now that you are willing to invest $1,000 per year for 5 years, but starting next year. How much will you have accumulated at the end of 5 years?

N I/Y PV PMT FV Enter 5 6 1000 Compute X

FV = -5,637.09 The difference between an annuity and annuity due Annuities assume that cash flows occur at the end of the year whereas annuities due assume that cash flows are at the beginning of the year. Most financial calculators have the capabilities of dealing with annuities due I would recommend you not use these since the function must be cleared before doing simple annuities and often this is forgotten leading to countless errors.

Page 15: MODULE 3 Financial Management and Management Accounting 1

Module 3 - Financial Management & Management Accounting 1

Page 15 © CMA Ontario, 2011

Uneven Cash Flows Both the TI BA II Plus and the HP 10B allow you to enter cash flows that are uneven. For example, say you are offered an investment with the following cash inflows. You want to calculate the present value of these cash flows at a discount rate of 6%.

Time Cash Flow 1 $20,000 2 30,000 3 40,000 4 40,000 5 40,000 6 50,000 7 50,000

Using the TI BA Plus, you proceed as follows: 1. Press 2nd RESET ENTER to clear all registers 2. Press CF 3. The first register is for CF0 – the cash flow at time=0. Press 0 and ENTER.

4. Using the ↑ and ↓ keys, find the register for C01 (the first cash flow), enter 20000 and press ENTER.

5. Using the ↑ and ↓ keys, find the register for C02, enter 30000 and press ENTER.

6. Using the ↑ and ↓ keys, find the register for C03, enter 40000 and press ENTER. 7. Because the $40,000 repeats three times, we want to make sure that the frequency

for this cash flow reflects this. To do this, using the ↑ and ↓ keys, find the register for F03, enter 3 and press ENTER. (Note that the frequencies assigned to all cash flows is 1 by default so you only need to enter a frequency if it is other than 1.

8. Using the ↑ and ↓ keys, find the register for C04, enter 50000 and press ENTER.

9. Using the ↑ and ↓ keys, find the register for F04, enter 2 and press ENTER. 10. Press the NPV key. The calculator will ask for the interest rate. Press 6 and press

ENTER.

Page 16: MODULE 3 Financial Management and Management Accounting 1

Module 3 - Financial Management & Management Accounting 1

Page 16 © CMA Ontario, 2011

11. Using the ↑ and ↓ keys, find the register for NPV= and press CPT. The calculator should give you a present value of $209,227.55. Using the HP 10BII, you proceed as follows: 1. Clear all: Orange Button C ALL 2. Enter the cash flow at time = 0: 0 and press CFj 3. Enter the cash flow at t=1: 20,000 and press CFj 4. Enter the cash flow at t=2: 30,000 and press CFj 5. Enter the cash flow at t=3: 40,000 and press CFj You now need to inform the calculator that this cash flow repeats three times: Enter 3 Orange Button Nj 6. Enter the cash flow at t=6: 50,000 and press CFj You now need to inform the calculator that this cash flow repeats twice: Enter 2 Orange Button Nj 7. Enter 6 I/YR 8. Press Orange Button NPV The calculator should give you a present value of $209,227.55.

Page 17: MODULE 3 Financial Management and Management Accounting 1

Module 3 - Financial Management & Management Accounting 1

Page 17 © CMA Ontario, 2011

Perpetuities and Growing Perpetuities A perpetuity is an infinite annuity, i.e. a constant cash flow to be received at regular intervals forever. Present value of a perpetuity =

PMT1

r

Examples of perpetuities are preferred shares - they have no maturity and pay a fixed annual dividend. For example, if you were to purchase a preferred share that pays a constant dividend of $7.50 per year and you require a 12% rate of return, then the preferred share would have a value of $7.50 / 0.12 = $62.50. A growing perpetuity is a constant cash flow growing at a constant rate at regular intervals forever. The present value of a growing perpetuity is equal to:

PMT1

r − g

where PMT1 = next year's payment

g = growth rate. For example, a preferred share whose dividend is growing at a rate of 5%, and generates a returns of 10% and will pay a dividend of $10 next year should have a value of: $10 / (.10 -.05) = $200 b. General Valuation Model The value of any asset is the discounted value of its expected future cash flows:

V0 =

CF1

(1+ r)+

CF2

(1+ r)2 + ... +CFn

(1+ r)n

where V0 = Value of the asset at time 0 CF = cash flow at period 1, 2, …n r = discount rate Three things should be noted with regards to this model: 1. only cash flows are relevant 2. past cash flows are irrelevant. It is the expected cash flows that determine the

current value of an asset.

Page 18: MODULE 3 Financial Management and Management Accounting 1

Module 3 - Financial Management & Management Accounting 1

Page 18 © CMA Ontario, 2011

3. the discount rate must reflect the asset's underlying risk - the higher the risk, the higher the expected rate of return.

c. Valuation of bonds A bond is a long-term obligation for borrowed money. It is a promise to pay interest and to repay the principal on terms specified in a contract called the bond indenture. A typical indenture will usually include the following provisions: • the face value of the bond - the amount of money borrowed (usually in

denominations of $1,000) • the coupon - the amount of stated interest to be paid - usually semi-annually • the maturity - the end of the bond life • a call provision - gives the issuer the right to redeem the bonds prior to their

maturity by paying a call price. A bond's market price will only be equal to its face value when market interest rates are equal to the coupon rate. If the bond pays a coupon rate that is less than the required market rate, the bond will sell at a discount (i.e. less than $1,000) since investors need to be compensated for the difference in interest rates. If the bond pays a coupon rate that is more than the required market rate, the bond will sell at a premium since investors are willing to pay more for a bond that pays higher than required interest. The required market rate is called yield to maturity. This can be shown through the bond valuation formula: Bond Price = P0 = Present Value of the Coupon Stream (Annuity or PMT)

+ Present Value of the Face Value (Future Value) Example: suppose a bond has a coupon rate of 8.5%, a remaining maturity of 12 years, and a face value of $1,000. If the yield-to-maturity on this bond is 10%, what is the current price of this bond? The bond pays coupons semi-annually. Coupon = $1,000 x 8.5% / 2 = $42.50

N I/Y PV PMT FV Enter 24 5 42.50 1000 Compute X

Bond Price = $896.51

Page 19: MODULE 3 Financial Management and Management Accounting 1

Module 3 - Financial Management & Management Accounting 1

Page 19 © CMA Ontario, 2011

Computing the Yield to Maturity Example: A bond paying a coupon of 9% with 10 years remaining to maturity sells for $1,223. What is the yield to maturity?

N I/Y PV PMT FV Enter 20 -1223 45 1000 Compute X

Note that: (1) the present value is entered as a negative on the assumption that we would have to outlay $1,223 to purchase this bond and then receive the coupon payments and face values and (2) the coupon payment is calculated as: $1,000 x 9% / 2 = $45. The yield-to-maturity provided by the calculator is 3% is for a six month . This converts to an annual yield of 3% x 2 = 6%. Interest Rate Risk Whenever the required return of a bond changes, the price of that bond also changes. This is referred to as interest rate risk: the inverse relationship between interest yields and bond prices. When yields increase, bond prices drop. When yields decrease, bond prices increase. This sensitivity is dependent on two things: 1. time to maturity - the lower the time to maturity the lower the interest rate risk 2. coupon rate - the lower the coupon rate the greater the interest rate risk For example - take two bonds with a face value of $1,000 with 10 years to maturity. The current yield to maturity is 8% - the only difference is the coupon rate: Bond A, with a coupon rate of 12%, will sell for $1,271.81 (N = 20, i =4%, , PMT = $60, FV = $1,000) Bond B with a coupon rate of of 6% will sell for $864.10 (N = 20, i =4%, , PMT = $30, FV = $1,000) Now, assume that the YTM drops one percentage point to 7%, then the new price of Bond A will be $1,355.31 and Bond B will be $928.94. The percentage increase in the price of Bond A is 6.6% and the percentage increase in the price of Bond B is 7.5%. Therefore, Bond B is riskier.

Page 20: MODULE 3 Financial Management and Management Accounting 1

Module 3 - Financial Management & Management Accounting 1

Page 20 © CMA Ontario, 2011

d. Valuation of stocks Using the general valuation model, the valuation of a stock is the present value of future dividends expected to be received over the time the stock is held plus the present value of the proceeds when the stock is sold:

P0 =D1

(1+ r)+

D2

(1+ r)2 + ...+Dn

(1+ r)n +Pn

(1+ r)n

where P0 = the price of a share today Dn = dividend to be received at time n Pn = the price of a share at time n r = rate of return required by common shareholders Generally, however, we tend to conceptualize the value of a share as being the present value of all future dividends to be received in perpetuity:

P0 =D1

r

If we assume a constant growth rate in dividends, the equation becomes:

P0 =D1

r − g

where g is the dividend growth rate and r > g This model is known as the dividend growth model (or the Gordon model). If we rework the above equation, we can solve for the required rate of return:

r =D1

P0

+ g = Dividend Yield + Capital Gain Yield

Example 1: A stock currently paid a dividend of $4 which is not expected to change. If investors require a 12% rate of return, what is the selling price of the stock? P0 = D1/r = $4 / .12 = $33.33 Example 2: Assume that PTH Co. just paid a cash dividend of $5.00 per share and that dividends are expected to grow at a rate of 8% per year forever. What is the current market value of a share of PTH Co. stock is the required rate of return is 12%?

Page 21: MODULE 3 Financial Management and Management Accounting 1

Module 3 - Financial Management & Management Accounting 1

Page 21 © CMA Ontario, 2011

P0 =

D1

r − g=

D0(1+ g)r − g

=5.00(1.08).12 − .08

=5.40.04

= $135.00

Example 3: The XYZ Company is planning excessive growth over the next three years and expects to pay the following dividends:

End of year Dividend 1 $2.00 2 3.00 3 4.50

After year 3, it is expected that the growth rate will stabilize at 6%. If investors require a 14% rate of return, what is the stock selling for today? What will it be selling for in a year from now? Show that the required return of 14% is the sum of the dividend yield and the capital gain yield, using year one data.

P3 =

D4

r − g=

D3(1+ g)r − g

=4.50(1.06).14 − .06

= $59.625

To calculate Po, use the cash flow features of your calculator and discount at 14% (don’t forget to set the first cash flow as zero). The following are the cash flows:

T Cash Flow 1 $2.00 2 3.00 3 $4.50 + 59.625 =64.125

Your calculator should return a value of $47.35 If we calculate the expected price of the stock in one year from now (P1), we can see how investors receive their 14% return.

T Cash Flow 1 3.00 2 $4.50 + 59.625 =64.125

The value of P1 = $51.97

Dividend yield in year 1 =

D1

P0

=2.0047.35

= 4.2%

Capital Gain yield in year 1 =

P1 − P0

P0

=51.97 − 47.35

47.35= 9.8%

Page 22: MODULE 3 Financial Management and Management Accounting 1

Module 3 - Financial Management & Management Accounting 1

Page 22 © CMA Ontario, 2011

Dividend yield + Capital Gain Yield = 4.2% + 9.8% = 14% = expected rate of return (r)

e. Cost of Capital The cost of capital or Weighted Average Cost of Capital (WACC) represents the cost of funds for the firm and is the rate which firms use as a discount rate in evaluating capital budgeting proposals. It is dependent on the capital structure of the firm and assumes that the risk associated with any particular investment will be close to the average risk of the firm. As will be seen later in this lesson, there exists an optimal capital structure for every firm. This optimal capital structure is defined as the proportion of debt and equity that will maximize the value of the firm (and minimize its weighted average cost of capital). In this section, we will take the capital structure for granted. In order to determine the WACC, we must first determine the cost of each source of financing. Beginning with debt, the cost of debt is equal to the after-tax cost of debt financing. This is because interest expense is a tax deductible expense. The two formulas are as follows: After tax cost of debt = kd = k(1− t)

Where: k = interest rate, t = corporate tax rate

The cost of preferred shares is similar to that of debt except, of course, that preferred dividends are not deductible by the firm. The basic formula (derived from the present value of a perpetuity formula) is as follows:

Cost of preferred shares = kp =

DP

Where: D = annual dividends paid, and P = the market value of one preferred share

The cost of equity capital is typically derived using either the Capital Asset Pricing Model (section 1g) or the Gordon Model (section 2d). The weighted average cost of capital can then be determined by weighting each cost by its relative proportion in the total capital structure. Note that this figure represents the current marginal cost of new funds that can be raised. If no new funds are to be raised or the acquisition of incremental funds will not cause the current capital structure to change, the current weighted average cost of capital and the true marginal cost of capital will be the same.

Page 23: MODULE 3 Financial Management and Management Accounting 1

Module 3 - Financial Management & Management Accounting 1

Page 23 © CMA Ontario, 2011

WACC =

BV

kd +PV

kp +EV

ke

Where: kd = cost of debt, kp = cost of preferred shares, ke = cost of equity,

B = market value of debt outstanding, P = market value of preferred stock outstanding E = market value of common equity outstanding and V = B + P + E.

Example: Suppose a firm had a capital structure of $40,000 debt, $20,000 preferred and $40,000 in equity. Also assume that the interest rate on long-term bonds is 10% and that the tax rate is 40%. $100 preferred shares can be sold to yield 12% with issue-related costs of $4 per share. The current dividend on common shares is $4 per share. The current market price is $44 per share. The long-term growth in dividends is expected to be 10%. Following is the determination of the weighted average cost of capital:

Cost of Debt = kb = k (1 - T) = .10 x (1 - .4) = 6.0%

Cost of Preferred = kp = Dp / Pp = $12 / $96 = 12.5%

Cost of Common Shares = ke = (D1 / Pe) + g = ($4(1.1) / $44) +.10 = 20% = $4.40/ $44 + .10 = 20%

The weighted average cost of capital is as follows:

Weighted Average Cost of Capital = (40 / 100 x 6%) + (20 / 100 x 12.5%) + (40 / 100 x 20%) = 2.4% + 2.5% + 8% = 12.9% or 13%

The WACC of 13% would be the appropriate rate to use in the analysis of prospective capital budgeting projects. If the net present value of the project is positive, therefore, it will cover its financing costs, including a normal return for common shareholders. If the proposed investment represented a higher than normal degree of risk for the firm, it would be appropriate to employ a discount rate greater than 13%. One final note on the capital structure. As mentioned earlier, the weights that should be used are based on the optimal capital structure of the firm. These weights (if known) should be used regardless of the actual capital structure. The reason for this is that presumably, the firm is moving towards this capital structure and that WACC is used to evaluate long-term capital projects. Often, these optimal weights are not available. In this case, one can assume that the existing capital structure is the optimal structure. The issue before us now is whether to

Page 24: MODULE 3 Financial Management and Management Accounting 1

Module 3 - Financial Management & Management Accounting 1

Page 24 © CMA Ontario, 2011

use market value or book value weights. If market values are known, they should be used. Book values should be used only if market values are not known. To summarize, the best weights to use in the WACC Calculation are the following:

1. if the optimal capital structure is provided, then we use these weights; 2. if not, then if market values are provided for all of the elements, then market

value weights should be used. Note that the in this case, the market value of the common equity (E) is simply the number of common shares outstanding times the market value per share; or

3. if not, then we have to resort to book values. When using book values, the common equity (E) is calculated as common stock plus retained earnings.

Example: The James Company has the following partial balance sheet as at December 31, 20x9: Bonds Payable, face value $20,000,000, 20 years, semi-annual, coupon 8%, issued on January 2, 20x3 when the market interest rate was 10%

$17,120,000 Preferred shares, 6%, par value = $100, currently trading at $80, 100,000 shares issued and outstanding

10,000,000

Common stock, 1,200,000 shares issued and outstanding, currently trading at $37

18,000,000

Retained earnings 21,500,000 $66,520,374

Additional Information – • the bonds are currently trading at a yield to maturity of 12% • the corporate tax rate is 35% • the common stock’s most current dividend was $4.00 per share (annual) • financial analysts expect the firm to grow at a constant rate of 4% Note that we are not given the optimal capital structure of the firm. Therefore we must calculate the WACC using market values.

Page 25: MODULE 3 Financial Management and Management Accounting 1

Module 3 - Financial Management & Management Accounting 1

Page 25 © CMA Ontario, 2011

The present value of the bonds is $14,798,733 rounded to $14,800,000

N I/Y PV PMT FV Enter 26 6 800000 20000000 Compute X

P = 100,000 shares x $80 = $8,000,000 Kp = $6 / 80 = 7.5% E = 1,200,000 shares x $37 = $44,400,000 Ke = D1/P0 + g = (($4.00 x 1.04)/37) + .04 = 15.24% WACC = (14,800/67,200)(12%)(0.65) + (8,000/67,200)(7.5%) + (44,400/67,200)(15.24%)

= 12.7% Note that if book value weights had been used, then B = $17,120,000, P = $10,000,000 and E = $39,500,000 ($18,000,000 + 21,500,000).

Page 26: MODULE 3 Financial Management and Management Accounting 1

Module 3 - Financial Management & Management Accounting 1

Page 26 © CMA Ontario, 2011

3. Financial Planning and Dividend Policy Decisions a. Financial Forecasting and Planning This topic is covered in the Management Accounting Module (Budgeting). b. Working Capital Management This section of the lesson deals with the management of working capital. Working capital is made up of current assets and current liabilities and is commonly measured by reporting the absolute amount of working capital (current assets - current liabilities) or the current ratio (current assets/current liabilities). The degree of liquidity in every organization is usually dependent on the predictability of cash flows; firms with highly uncertain cash flows will likely have to hold more working capital than those which can accurately predict their cash inflows and outflows. Finally, because working capital is not free, efforts to reduce the level of working capital will, all else being equal, cause profits to increase. c. Management of Cash and Marketable Securities A certain amount of cash is necessary for the firm's operations to cover the normal deficiencies between payments and receipts of cash. Additional cash may be retained, at the discretion of the financial manager, to cover unexpected events and to provide a fund for speculative purposes (i.e. buying additional materials if it is expected that prices will rise dramatically in the near future). A financial manager must ensure that all needs for cash are met without incurring undue costs. The most important tool in the management of cash is the cash budget (see Lesson 19 of this course). Here, the expectations regarding the inflows and outflows of cash are combined with minimum required balances to minimize borrowings of short-term funds and keep interest costs as low as possible. When excess cash is available, it is the responsibility of the financial manager to invest these resources to earn a return. The duration of these investments is typically short-term, and in most cases such investments are in securities (such as T-bills) which are risk-free and easily convertible to cash (despite this fact, many firms do invest in short-term securities for which the level of risk approaches that of a market portfolio).

Page 27: MODULE 3 Financial Management and Management Accounting 1

Module 3 - Financial Management & Management Accounting 1

Page 27 © CMA Ontario, 2011

There are three reasons to hold cash: (1) for speculative reasons - there needs to be enough cash so that the firm does not

pass up good opportunities, (2) for precautionary purposes - to meet unspecified and unexpected contingencies

that may arise, and (3) for transaction purposes - the firm must have enough cash to be able to pay bills

as they become due. Generally, the firm will keep enough cash in the chequing account to meet its transaction obligations. Any cash held for speculative or precautionary purposes is usually invested in short-term securities such as Treasury Bills (or any relatively safe short-term security that is highly liquid). Alternatively, some firms do not keep any cash for speculative purposes but rather maintain borrowing capacity in order to meet these cash needs. Firms will differ significantly in terms of their needs: • the greater the uncertainty about operating cash-flow needs, the greater the cash

requirements for transaction and precautionary reasons • the higher the seasonal factors affecting product demand, the greater the cash

requirements for transaction and precautionary reasons • firms that can borrow easily and at low cost are much less likely to keep large

balances to cover unexpected events • the speculative motive typically is not a big factor for firms that usually invest in

long-term projects but may be a significant factor for firms faced with the possibility of short-term profit making opportunities that have to be taken advantage of quickly

In cash management, firms should be more concerned with managing float than with managing the cash book balance. Float is the difference between book and bank balance and is caused by the outstanding cheques and deposits. Disbursement float refers to cheques written but not yet cashed. Collection float refers to cheques received but not deposited. Net float is equal to disbursement float less collection float. If positive, then the available cash balance exceeds the book balance. Float can also be broken down as follows: Float = Mail float + Processing Float + Availability Float

The time cheques are in the mail.

The time taken between the cheque is received and deposited

The time taken between when the cheque is deposited and is cleared by the banking system.

Page 28: MODULE 3 Financial Management and Management Accounting 1

Module 3 - Financial Management & Management Accounting 1

Page 28 © CMA Ontario, 2011

A firm with a positive net float can use it to its advantage and maintain a smaller cash balance than it would have in the absence of the float. d. Management of Accounts Receivable The management of accounts receivable is important to the overall success of the organization. The key to the management of accounts receivable is the careful monitoring of the extension of credit terms. Selling on credit involves the risk of non-payment. More lenient credit terms may stimulate sales, but the firm will likely suffer losses if the accounts prove to be uncollectible. This is not to suggest that credit sales should not be made; the objective must be to earn a return which is commensurate with the risks undertaken. Various procedures may be employed to measure and monitor the credit worthiness of current and potential customers. In addition to operating an internal credit department, many firms employ the services of commercial credit rating agencies. The careful monitoring of the liquidity of new clients and clients with high volume accounts is especially important. The main variables in the establishment of a credit policy are the length of the credit period, the quality of credit standards, the size of the cash discount offered for early payment, and collection procedures. There are no simple answers as to what is best, as each depends on the situation facing the firm at any point in time. Lengthening the credit period for customers will usually stimulate sales (for example, "don't pay a cent for six months") and result in increased operating profits, but these receivables must be carried on the balance sheet at some cost (most analysts employ the interest rate on short-term bank loans as a surrogate measure of the opportunity cost in accounts receivables). The evaluation of whether or not to extend credit , and if extended, at what terms can be analyzed using the 6 C's of credit: 1. Capacity to pay: consider the size of the firm, the interest coverage ratio. 2. Capital: to what extent is the firm financed - the debt/equity ratio. 3. Collateral: nature of the assets offered for collateral. 4. General economic Conditions: the degree of linkages for the firm to economic

cycles. Consider the degree of riskiness the firm's business is in. 5. Character of management - willingness to pay. 6. Communication : how well the potential customer communicates their true

financial information to creditors.

Cash discounts for early payment benefit the firm by reducing accounts receivable sooner rather than later, but the cost is the reduction in the profit margin which the discount creates. A firm offering a cash discount of 2/10, n/30 (2% discount if payment is received within 10 days, otherwise the whole amount has to be received within 30 days) is faced with an

Page 29: MODULE 3 Financial Management and Management Accounting 1

Module 3 - Financial Management & Management Accounting 1

Page 29 © CMA Ontario, 2011

interest cost of 44.6%. For example, take an order for $100. The buyer can pay $98 in 10 days or $100 in 30 days. If the buyer chooses to pay in 30 days, the buyer is effectively borrowing $98 for a period of 20 days and repaying $100. The effective annual interest on this borrowing can be calculated as follows: EAR = (1+ Cash Discount / (Price – Cash Discount)) 365 / Extra Days - 1 Extra days is the number of extra days to make payment, in this case 20 days.

1+

2100− 2

⎝ ⎜ ⎜

⎠ ⎟ ⎟

36520

− 1= 1.020408( )18.25−1 = 44.6%

Most firms have established procedures for dealing with overdue accounts receivable. Reminder notices are sent to all overdue accounts at periodic intervals with increasing pressure to make a full or partial payment. If no satisfaction is reached on this informal basis, the firm may consider taking legal action or turning the account over to a collection agency. e. Management of Inventory Inventories often represent a significant portion of the total current assets in many organizations, and determining the right amount of inventory to hold is difficult at the best of times. In a retail setting, carrying inventory in excess of current needs provides a safety measure which prevents customer dissatisfaction from stockouts; in the manufacturing arena, excess inventories allow the firm to smooth out production runs and reduce setup costs. There are, however, costs associated with the holding of inventories, including handling and storage costs, the risk of obsolescence, spoilage and the opportunity cost of funds used to finance their acquisition. Various inventory planning models can be employed to aid in determining the right amount of inventory to hold. Most firms track the level of inventory against sales activity by monitoring the number of days of inventory on hand or the inventory turnover ratio. One of the earliest quantitative models is called the Economic Order Quantity (EOQ) model. In the end, however, it is important to remember that the adoption of a Just-In-Time (JIT) approach to inventory management can significantly reduce the funds tied up in inventories and the total cost of acquiring, holding and using inventory. The EOQ model can be helpful in inventory planning, but should not be relied upon as the sole management tool. When we cannot accurately schedule demand for a product or material, then we have to carry inventories. For example, most retailers cannot predict when specific goods will be sold. The firm must therefore maintain sufficient inventories to cover the normal fluctuations in demand. Determining the optimal level of inventories involves a tradeoff

Page 30: MODULE 3 Financial Management and Management Accounting 1

Module 3 - Financial Management & Management Accounting 1

Page 30 © CMA Ontario, 2011

between the costs of holding inventory and the incremental cost of placing additional orders to acquire inventory. Let's now examine the data requirements and the factors we need to consider when choosing an inventory policy. Assume that our firm plans to produce 50,000 units of product next year. Production will be erratic and on short notice. Each unit requires 2 kg. of a particular material, so we anticipate needing 100,000 kg. of this material. Our objective now is to determine when to buy the materials. Two extreme solutions are available: (1) buy all the materials at the beginning of the year and store them until they are needed, or (2) buy each day's requirements as needed. The first approach will minimize the costs to acquire the materials (costs to type purchase orders, delivery charges, and so forth) but may involve some significant costs to store the inventory (handling costs, spoilage, insurance, property taxes, and so on). At the other extreme, the second approach will minimize storage costs but at the expense of increasing acquisition costs. Our objective is to try to minimize the sum of these two costs. Assume that management has arbitrarily chosen to order 2,500 kg. of material each time it places an order. First, let's look at what our annual inventory ordering and storage costs will be. If it costs $10 to place an order, then the total costs to place orders will be $10 multiplied by the number of orders placed. Since we assumed that we would need 100,000 kg. of inventory during the year, we will have to place 40 orders if we order 2,500 units at a time:

number of orders = annual demand / order size = 100,000 - 2,500 = 40

Thus our total costs to place orders will be $400 (40 orders x $10/order). Assuming further that it costs $2.50 per year to store a kg. in inventory, let's now calculate our annual storage costs. To do so, we need to know the average number of units that will be held in inventory over the year. We will simplify our estimate of the average number of units in inventory by assuming that demand for these materials is uniform. That is, we use exactly the same number of units every day. If demand follows this pattern, then we can time the ordering of new units so that they arrive at precisely the moment we run out of stock. This, in turn, means that the largest number of units that we will ever have on hand is 2,500. That is, at the moment a new order arrives we will have 2,500 units in inventory. The smallest number of units that we will ever have is zero, which will occur just before a new order arrives. With uniform demand, we also know that precisely halfway through the time period between orders 1,250 units will remain in inventory. At a cost of $1.50 per unit per year, our annual storage cost will be $3,125 (1,250 average inventory x $2.50 carrying costs per kg. per year). Thus our total ordering and storage costs will be the sum of the ordering storage costs, or $400 + 3,125 = $3,525. That is, our arbitrary decision to order 2,500 units each time we place an order should result in total ordering and storage costs of $3,525 per year.

Page 31: MODULE 3 Financial Management and Management Accounting 1

Module 3 - Financial Management & Management Accounting 1

Page 31 © CMA Ontario, 2011

Economic Order Quantity Our objective in inventory management and purchasing policy is to try to minimize this total cost. We can do so by finding the optimal number of units we should buy every time we place an order. This optimal order size is known as the economic order quantity. Let's define the following terms: D = demand in units for a specified time period (usually one year) P = costs incurred per order C = carrying cost of one unit of stock for a specified time period (usually one

year) The economic order quantity formula is:

2DP

C

Continuing with our example, the optimal order size would be:

2(100,000)(10)

2.50= 894

Thus, we should order 894 kg. at a time. This would result in a total of 112 order per year. The total ordering and carrying costs would be:

Ordering costs: 112 orders x $10 $1,120 Carrying costs: 894/2 x $2.50 1,118 $2,238

(note that the ordering costs and carrying costs should be the same, the differences above are due to rounding)

Inventory Order Point Once the economic order quantity has been determined, management must decide when to place the order; i.e., the order point must be established. If the lead time and the inventory usage rate are known, determination of the order point is very simple. Lead time is the period between the placement of an order and the receipt of the materials. Inventory usage rate is the quantity of materials used in production over a period of time. The order point should be where the inventory level reaches the number of units that would be consumed during the lead time.

Page 32: MODULE 3 Financial Management and Management Accounting 1

Module 3 - Financial Management & Management Accounting 1

Page 32 © CMA Ontario, 2011

Safety Stock Since it is almost impossible to estimate lead time and average usage rate with certainty, many companies prefer to carry a safety stock (or additional inventory) as a cushion against possible stockouts. In such a case, the order point is computed by adding the safety stock to the estimated usage during the lead time. A safety-stock calculation should arrive at a figure which properly balances the risk of a stockout against the additional carrying costs incurred by the extra inventory. One method is to determine safety stock on the basis of experience or traditional rules of thumb. Although this method is less involved, it does not ensure that the optimum amount will be chosen. Another method of calculating safety stock is to provide for the extreme boundaries of lead time and usage variance. Estimates are made for the longest possible time for delivery and the greatest possible usage rate. Safety stock is the number of additional units needed above the order point if the lead time and usage rate should increase to their estimated maximums. f. Sources of financing The basic sources of external financing for a firm are short-term and long-term. In addition, financing is provided by the basic business activity of the organization. Short-term sources of financing include accounts payable and secured/unsecured bank loans. Typically, the cost of borrowing short-term funds is greater than long-term funds, but they provide more flexibility than long-term securities. The firm may acquire additional short-term funds when needed and repay them with relative ease when excess cash resources are available. As a general rule, most financial managers think about matching their use of short-term sources of financing with their level of short-term assets. For example, if accounts receivable levels were to rise during the month of December and then fall in January, one would expect the firm to cover any shortfall in cash resources by borrowing money from a bank rather than trying to float new common shares. Long-term sources of financing typically include preferred and common stock and various forms of debt instruments such as mortgages and bonds. In most firms, these financial instruments are tied to the existence of long-term assets such as plant, building and equipment. The expectation on the part of the firm and potential investors is that the firm will need additional financing over an extended period of time. Typically, the cost of long-term sources is less than short-term sources, but issuing such instruments can be expensive and time consuming; furthermore, a failed issue (one which does not sell well on the market) can be even more expensive and damaging to a firm's reputation. There also exists several additional sources of financing which are intermediate in term, such as leases and intermediate-term bank loans, which may be employed to further match the firm's year-to-year financial requirements and required resources. The

Page 33: MODULE 3 Financial Management and Management Accounting 1

Module 3 - Financial Management & Management Accounting 1

Page 33 © CMA Ontario, 2011

financial accounting part of this course covers the basic bookkeeping issues with respect to leases. Operating leases are for relatively short periods of time and because the lessor bears the risk of obsolescence, the annual payments are usually higher than for financial leases that cover a substantial portion of the useful life of the asset. What is important to remember from a financial management perspective is that leasing allows the firm to acquire the use of an asset without investing large sums of cash to physically own it. g. Management of Current Liabilities and Bank Loans Trade credit is more flexible than other means of short-term financing in the sense that firms do not have to negotiate a loan agreement, pledge collateral and adhere to a rigid repayment schedule. Also, the consequences of delaying trade credit is much less onerous than those resulting from failure to repay a bank loan on schedule. Trade credit also has no cost to the firm, unless cash discounts are involved (see section d) in which case there is a generally high cost to the firm of not taking these discounts. Trade credit is particularly valuable to small firms who may have difficulty in obtaining credit elsewhere. The main source of short-term financing is the line of credit. Normally, these have a fixed maximum and allow firms to draw to meet unanticipated and seasonal working capital needs. Interest rates on line of credits tend to be tied to the bank's prime rate. Lines of credit provide firms with funds that are accessible as needed and no interest cost is payable on unused funds. h. Leasing This topic will be covered as part of the Capital Budgeting segment of the Management Accounting module. i. Venture Capital Financing Venture capital financing is usually provided to private firms by investors in exchange for a share of the ownership of the firm. Generally, this form of financing is sought out when all alternatives have been exhausted. Consequently, venture capital financing is generally obtained by small and usually risky businesses. The proportion of a firm that ends up with venture capitalists will usually depend on two factors:

Page 34: MODULE 3 Financial Management and Management Accounting 1

Module 3 - Financial Management & Management Accounting 1

Page 34 © CMA Ontario, 2011

1. the amount of venture capital financing: the venture capitalist will demand a proportion at least equal to a fair proportion of the firm value.

2. alternative sources of financing: the bargaining position of the firm vis-à-vis venture capitalists is enforced if alternative sources of financing are available.

Generally, venture capitalists expect (1) high returns commensurate with the risk taken and (2) expect to cash out in a relatively short-period of time (5-10 years), usually when the firm is taken public. j. Financial Management of Small and Medium-Sized Businesses Small to medium-sized business do not have ready access to capital markets and therefore are restricted to the following sources of financing: 1. financing provided by shareholders 2. financing provided by internal growth 3. trade credit financing 4. short-term financing provided by financial institutions 5. long-term financing provided by financial institutions, and 6. venture capital financing. Financing obtained through financial institutions is facilitated by the 6 C's of credit (see section d) and for small and medium sized business, it is vitally important that firms: • get to know their banker - a banker will have an easier time supporting the firm in

cases of doubt when the banker has an established relationship with the firm, and • be transparent and timely in its financial reporting - it is in the best interests of the

firm to produce more information than the bank requests and do so in a timely fashion. If the banker is well appraised of the financial situation (in good and bad times) of the firm, it will be easier to support the firm.

k. Long-term Financing Debt Financing Advantages and disadvantages of debt The advantages of debt are as follows: 1. Tax deductibility: interest payments are deductible, dividend payments are not 2. Discipline of debt: Harvard University's Michael Jensen believes that managers

that have substantial free cash flows (cash flows remaining after all financial obligations have been paid off) and little or no debt have such a large cushion against mistakes that they have little incentive to be efficient. In this sense, borrowing creates commitment to make interest and principal repayments and

Page 35: MODULE 3 Financial Management and Management Accounting 1

Module 3 - Financial Management & Management Accounting 1

Page 35 © CMA Ontario, 2011

instills a certain discipline in managers. This obviously presumes that there is a separation between management and shareholders.

The disadvantages of debt are: 1. Bankruptcy costs: the probability of bankruptcy is a function of the size of

operating cash flows relative to the size of cash flows on debt obligations and the variance of operating cash flows. Costs of bankruptcy can be direct or indirect. Direct costs include legal and administrative costs of liquidating the company and the present value effects of delays in paying out the cash flows to the various stakeholders. Indirect cash flows include the perception on the part of customers that the firm is in financial trouble (who wants to buy a product from a company on the verge of failure), stricter terms demanded by suppliers and the difficulties of raising fresh capital.

2. Agency costs: an agency relationship develops between the bondholders (or any

holders of long-term debt) and shareholder interests. There are basically two types of agency problems: (1) investment decisions: the tendency of stockholders (managers) to take on riskier projects than bondholders expect them to and (2) financing decisions: once money has been borrowed, shifting to a strategy of high leverage and default risk leaving the lenders worst off. Covenants and protective puts are usually given to bond holders to protect them from these agency problems. Covenants are limitations placed on the company with regards to minimum ratios, dividend payouts, debt issue, etc… Protective puts allow bondholders to return the bonds before maturity under a series of conditions.

If bondholders believe there is a significant chance that stockholder action might

make them worse off, they build this expectation into bond prices by demanding higher rates. This translates in higher borrowing costs to the company. Direct agency costs include direct costs of monitoring.

3. Loss of flexibility: this refers to the capacity of firms to meet any unforeseen

contingencies that may arise (recessions, sales downturns) and take advantage of unanticipated opportunities using funds they have on hand and any debt capacity that may have been nurtured. One of the reasons firms do not use their debt capacity is to keep it for a rainy day. Firms that borrow to capacity lose this flexibility.

Characteristics of long-term debt General bond terminology has been covered in Lesson 3. The issue of long-term debt is usually characterized by an indenture. This is the contract between the bondholders and the company that specified detailed provisions of a debt

Page 36: MODULE 3 Financial Management and Management Accounting 1

Module 3 - Financial Management & Management Accounting 1

Page 36 © CMA Ontario, 2011

issue. It is very detailed and often several hundred pages long. The indenture agreement may contain the following provisions: • pledge of assets: pledging certain specific assets as collateral • negative pledge: limitations on the creation of additional debt with more or equal

status the bonds in question • covenants: restrictions on specific financial ratios (usually current ratio,

debt/equity and times interest earned) • provision of sinking fund whereas the company puts aside specified amounts with

a trustee that is eventually used towards the redemption of the bonds • call provision: gives the company the right to repurchase its securities at a given

call price during all, or part, of the period that the bonds are outstanding. The call price is almost invariably fixed above par, with the price declining, as the maturity date approaches, according to a scale that is fixed in advance.

Bond Ratings A bond rating represents an external assessment of a firm's credit worthiness, basically an assessment of probability of default. The factors that affect the assessment are the six C's of credit (discussed in section d). The Dominion Bond Rating service uses the following bond rating categories: AAA Bonds which are rated AAA are of the highest investment quality. The

degree of protection afforded principal and interest is of the highest order. Earnings are relatively stable, the structure of the industry in which the company operates is very strong, and the outlook for future profitability is extremely favourable. There are few qualifying factors present which would detract from the performance of the company, and the strength of liquidity ratios is unquestioned for the industry in which the company operates.

AA Bonds rated AA are of superior investment quality, and protection of interest and principal is considered high. In many cases, they differ from bonds rated AAA to a small degree.

A Bonds rated A are upper medium grade securities. Protection of interest and principal is still substantial, but the degree of strength is less than with AA-rated companies. Companies in this category may be more susceptible to adverse economic conditions.

BBB Bonds rated BBB are medium-grade securities. Protection of interest and principal is considered adequate, but the company may be more susceptible to economic cycles, or there may be other adversities present which reduce the strength of these bonds.

Page 37: MODULE 3 Financial Management and Management Accounting 1

Module 3 - Financial Management & Management Accounting 1

Page 37 © CMA Ontario, 2011

BB Bonds rated BB are lower-medium-grade obligations and are considered mildly speculative and below average. The degree of protection afforded interest and principal is uncertain, particularly during periods of economic recession, and the size of the company may be relatively small.

B Bonds rated B are "middle" speculative. Uncertainty exists as to the ability of the company to pay interest and principal on a continuing basis in thc future, especially in periods of economic recession.

CCC Bonds rated CCC are considered highly speculative and are in danger of default of interest and principal. The degree of adverse elements present is more severe than bonds rated B.

CC Bonds rated CC are in default of either interest or principal, and other severe adverse elements.

C C is the lowest rating provided. Bonds rated C differ from bonds rated CC with respect to the relative liquidation values.

As bond ratings get worse, so does the interest spread. A bond rated AAA will have a .3% spread between the government bonds rate. Bonds rated BBB have a 2.00% spread. Preferred and Common Shares Characteristics and rights of common and preferred shareholders were discussed in Module 1. The following is a brief description of the process of issuing new securities in Canada: 1. Approval by the Board of Directors. If insufficient authorized capital exists, an

extraordinary meeting of the firm's shareholders may also be required to approve the necessary increase in authorized capital.

2. The underwriting agreement: this is the contract between the issuer and the underwriter. The underwriter serves as a 'middleperson' between the public and the issuing corporation. The underwriter gets paid by the spread between what the corporations nets on the issue and what the investors pays for the shares. The underwriter will either purchase all the securities at an agreed upon price and resell these to the investing public, or in cases of a risky security issue, accept the issue only on a best efforts basis. In this case, the underwriter serves as an advisor, but does not insure the issuing firm against the risk that the securities will not be sold.

3. The prospectus. The issuing firm prepares a preliminary prospectus, also called a red herring, which contains all relevant information except the offer price. After registration, a final version of the prospectus, which includes the offer price and any changes required by regulatory bodies, is sent to potential purchasers.

Page 38: MODULE 3 Financial Management and Management Accounting 1

Module 3 - Financial Management & Management Accounting 1

Page 38 © CMA Ontario, 2011

l. The preemptive right provision Instead of selling a new equity issue through a public offering, some firms will offer the securities first to their current shareholders on a privileged-subscription basis. This is known as a rights offering. A rights offering is mandatory in those firms in which shareholders have the preemptive right, or right of first refusal, to purchase new common stock issues in proportion to their current ownership position. Shareholders receive one right for each share of common stock that they own. The terms of the offering specify the number of rights required to buy a new share, the subscription price per share, and the expiration date of the offering. Shareholders have three choices: (1) to exercise the rights, (2) to sell them, or (3) to let them expire unused. The third choice usually doesn't make sense because the rights have value. To purchase stock under a privileged subscription, an investor sends the rights plus the subscription price to the company's agent. Stockholders who choose not to exercise their rights can sell them. At the time of the rights offering, the board of directors sets a date of record. After that date, the stock is traded ex-rights, that is, without the rights attached. Before the ex-rights date, the stock sells rights-on, meaning that the new owner receives the rights. Value of a Right The value of a right equals the difference between the rights-on price and the ex-rights price. The ex-rights price, in turn, depends on the rights-on price, the subscription price, and the number of rights required to buy one new share. For example, suppose a share of stock is currently selling rights-on for $50; the subscription price is $45; and it takes four rights to subscribe to one share. This means that there will be five shares ex-rights for every four rights-on shares, i.e. if you own four shares, you will receive four rights and will need four rights in order to acquire one new share. In order to calculate the value of the rights and the ex-rights price of the stock, imagine that a firm has only 4 shares outstanding (i.e. the number of rights required to acquire one share). The value of the firm before the rights offering is therefore $200 (4 shares x $50). Upon the issue of the additional share in exchange for the rights, the value of the firm will increase by $45 – the subscription price – to $245. Since there will be 5 shares outstanding, the value per share will equal $49.00. The $1 drop in the share price will be equal to the value of one right. The above discussion can be summarized in the following table: Number of Shares Market Price per Share Firm Value Before 4 $50 $200 After 5 49 245 Alternatively, The general formula for the value of a right is

R = (P - S)/(N +1)

Page 39: MODULE 3 Financial Management and Management Accounting 1

Module 3 - Financial Management & Management Accounting 1

Page 39 © CMA Ontario, 2011

where R is the value of one right, P is the rights-on price, S is the subscription price, and N is the number of rights required for one new share. Applying the equation to the previous example we get: R = (50 – 45) / (4 + 1) = $1 What is the effect on shareholders of changing the subscription price? Suppose that in the preceding example the subscription price is lowered to $40, the value of a right should now be $2. When the stock goes ex-rights, its price drops to $48. As before, the sum of the ex-rights price plus the value of the right remains constant. Changing the subscription price just changes the relative distribution of value between the stock selling ex-rights and the right. The value of the right is exactly offset by the decline in the stock's price when it goes ex-rights. This example illustrates a more general principle: The rights offering is irrelevant to stockholders so long as they don't allow their rights to expire unused. As mentioned above, a rational shareholder will do one of two things when the shareholder receives notice of a rights offering: (1) the shareholder will exercise the rights or (2) the shareholder will sell the rights. Example (continuation of the above) - assume that a shareholder owns 1,000 shares of the stock of the corporation. The shareholders’ wealth before the rights transaction is $50,000 (1,000 shares x $50). If the shareholder exercises the rights, then the shareholder will have to disburse an additional $11,250 (1,000 / 4 rights to acquire one share x $45 subscription price) and will receive 250 new shares. The shareholders’ wealth after the shares go ex-rights will be $61,250 (1,250 shares x $49). This is equal to the original wealth of $50,000 plus the additional cash invested of $11,250. Is the shareholder sells the rights, $1,000 will be received (1,000 rights x $1 per right). When the shares go ex-rights, the wealth of the shareholder will be equal to $49,000 (1,000 shares x $49) in shares plus $1,000 cash on sale of the rights. One way or the other, the total wealth of the shareholder is unaffected by the rights issue. m. Financial Leverage and Capital Structure Policy Because interest payments on bonds or mortgages are tax deductible, the cost of such long-term sources of financing tend to be lower than share issues [the true cost of debt is equal to the nominal interest rate x (1 - tax rate)]. Common stock tends to be the most expensive source of long-term financing because the risk assumed by common stock investors is greater than that of preferred shareholders and bondholders. It is, therefore, reasonable to ask why firms bother to issue stock when debt is less expensive. The

Page 40: MODULE 3 Financial Management and Management Accounting 1

Module 3 - Financial Management & Management Accounting 1

Page 40 © CMA Ontario, 2011

answer lies in the existence of business risk, leverage and target capital structure. We shall begin with a simple example and show how the existence of various forms of long-term financing impact on earnings per share. In each case, the firm's total long-term debt and capital equals $100,000, the value of stock is $10 per share and the interest rate on debt is 10%. Level of Earnings Low Medium High No Debt; $100,000 of Stock: Earnings Before Interest $5,000 $25,000 $50,000 Tax (40%) 2,000 10,000 20,000 Earnings After Tax $3,000 $15,000 $30,000 Earnings per Share $0.30 $1.50 $3.00 $30,000 Debt; $70,000 of Stock: Earnings Before Interest $5,000 $25,000 $50,000 Interest Expense 3,000 3,000 3,000 Earnings Before Tax 2,000 22,000 47,000 Tax (40%) 800 8,800 18,800 Earnings After Tax $1,200 $13,200 $28,200 Earnings per Share (7,000 shares) $0.17 $1.89 $4.03 $60,000 Debt; $40,000 of Stock: Earnings Before Interest $5,000 $25,000 $50,000 Interest Expense 6,000 6,000 6,000 Earnings Before Tax (1,000) 19,000 44,000 Tax (40%) (400) 7,600 17,600 Earnings After Tax $(600) $11,400 $26,400 Earnings per Share $(0.15) $2.85 $6.60 A careful review of these figures indicates that in good times the return to shareholders is greater if more debt is employed; earnings per share in the high column increases from $3.00 to $6.60 as the level of debt is increased. On the other hand, the earnings per share falls in periods of low profitability (from $0.30 to a loss of $0.15 per share) as the level of debt increases. This relationship is known as financial leverage and can be quantified as follows:

Degree of = Earnings before Interest and Taxes Financial Leverage (DFL) Earnings before Interest and Taxes - Interest = EBIT EBIT - I

Page 41: MODULE 3 Financial Management and Management Accounting 1

Module 3 - Financial Management & Management Accounting 1

Page 41 © CMA Ontario, 2011

In our example, the degree of financial leverage increases as more debt is introduced into the capital structure as follows (calculations are for the Medium level of earnings): No Debt: $25,000 / 25,000 = 1.00 30% Debt / 70% Equity $25,000 / 22,000 = 1.14 60% Debt / 40% Equity $25,000 / 19,000 = 1.32 Interpretation of the degree of financial leverage: the DFL measures the relationship between a change in EBIT on the resulting change on Earnings after Taxes. Using the 30% debt scenario, if the company’s EBIT were to increase by 10%, then the company would experience a 11.4% (10% x DFL of 1.14) increase in earnings after taxes (and Earnings per Share). Therefore, the greater the degree of financial leverage, the fluctuations in earnings per share, and the higher the risk. Using our example, if EBIT were to increase by 10%, the new Earnings after Taxes would be $14,700 which represents a 11.4% increase over the previous Earnings after Tax of $13,200. Earnings Before Interest: $25,000 x 1.1 $27,500 Interest Expense 3,000 Earnings Before Tax 24,500 Tax (40%) 9,800 Earnings After Tax $14,700 Earnings per Share (7,000 shares) $2.10 A related notion is that of operating leverage which measures the responsiveness of the firm's net income to changes in the level of sales. The degree of operating leverage is dependent on the relative amounts of variable and fixed costs and can be calculated as follows:

Degree of = Contribution Margin (Sales – Variable Costs) Operating Leverage (DOL) Earnings Before Interest and Taxes = CM EBIT Firms with no fixed costs have no operating leverage and will increase their profitability if sales increase by investing in fixed assets. On the other hand, firms with high fixed costs will see their profits decline at a faster rate than firms with low fixed costs if sales decline. Consider two firms, X and Y, with a base level of sales of $50,000 as follows:

Page 42: MODULE 3 Financial Management and Management Accounting 1

Module 3 - Financial Management & Management Accounting 1

Page 42 © CMA Ontario, 2011

Firm X Firm Y Fixed Costs of $25,000 Fixed Costs of $10,000 Variable Costs of 40% Variable Costs of 60%

Sales $30,000 $50,000 $70,000 $30,000 $50,000 $70,000 Variable Costs 12,000 20,000 28,000 18,000 30,000 42,000 Contribution Margin 18,000 30,000 42,000 12,000 20,000 28,000 Fixed Costs 25,000 25,000 25,000 10,000 10,000 10,000 EBIT $(7,000) $5,000 $17,000 $2,000 $10,000 $18,000 Degree of Operating Leverage

6.0

2.0

Obviously, Firm X is more highly levered in the operational sense. Its net income will rise and fall faster than the net income of Firm Y given an identical change in sales levels for both firms. Interpretation of the degree of operating leverage: the DOL measures the relationship between a change in Sales on the resulting change in EBIT. For example using the Firm X example, if sales were to increase by 10%, then EBIT would increase by 60% (10% x DOL of 6.0). As shown below this would generate an EBIT of $8,000 which represents a 60% increase over the previous EBIT of $5,000.

Sales: $50,000 x 1.1 $55,000 Variable Costs: $20,00 x 1.1 22,000 Contribution Margin 33,000 Fixed Costs 25,000 EBIT $ 8,000 Taken together, the degree of operating leverage and the degree of financial leverage represent the total leverage for the firm as follows: Degree of Total Leverage (DTL) =

Degree of Financial Leverage X Degree of Operating Leverage Interpretation of the degree of total leverage: the DTL measures the relationship between a change in Sales on the resulting change in earnings after taxes. For example, if a company’s DTL is 3.0, then a 10% increase in sales would result in an increase of 30% in earnings after taxes. In recent times, the degree of leverage has increased for many Canadian firms. The upgrading and automation of production processes has lead to increased fixed costs while, at the same time, the ratio of debt to equity has increased. This explains why the profitability of many firms is so sensitive to changes in general economic activity as

Page 43: MODULE 3 Financial Management and Management Accounting 1

Module 3 - Financial Management & Management Accounting 1

Page 43 © CMA Ontario, 2011

measured by sales revenue. This trend can be reversed by substituting new equity for debt in the capital structure, but many financial managers believe that now is not an optimal time for new equity issues. The financial manager must, therefore, balance the level of long-term debt and equity in order to maximize the expected return to shareholders. Inherent in this balancing is some assessment regarding the variability in earnings of the firm or business risk. As a general rule, firms that enjoy relatively stable earnings can employ more debt than those whose earnings fluctuate significantly from year to year. Furthermore, the existence of long-term debt and its required interest payments creates financial risk for firms with a cyclical earnings pattern. In the end, this combination of forces results in an optimal capital structure (or target capital structure) for individual firms. One of the most important responsibilities of the financial manager is to determine the mixture of low cost long-term debt and equity which will likely maximize the return to shareholders without increasing the firm's risk to the point where bankruptcy is likely to occur. n. The impact of the Debt-to-Equity ratio on the value of the firm In its most basic form, the value of the firm is equal to the expected future cash flows of the firm discounted at the weighted average cost of capital (WACC). Assuming the firm’s cash flows are constant, if the WACC increases, then the value of the firm will go down and vice versa. A firm with no debt in its capital structure can lower its debt-to-equity ratio by substituting debt for equity. This is due to the fact that the cost of debt financing is lower than the cost of equity financing (at least for a firm with little or no debt). As the debt-to equity ratio increases, several things happen: - the cost of debt increases since debtors are taking an increase in risk, - the cost of equity also increases because as debt increases, the firm is faced with

an increasing cost of interest – a fixed cost which increases the riskiness of the firm.

The firm’s beta used in the Capital Asset Pricing Model is equal to its levered beta βL (i.e. the beta of a firm with a certain amount of debt in its capital structure). The beta of a firm without debt in its capital structure is referred to as its unlevered beta βU. The relationship between a firm’s levered and unlevered beta is as follows: βL = βU (1+ [(D/E)(1-t)]) We can see that the firm’s cost of equity will therefore increase as the debt-to-equity ratio increases. Substituting debt for equity in a firm’s capital structure will decrease its WACC but only to a point. At one point, the debt level becomes high and the combination of the increased

Page 44: MODULE 3 Financial Management and Management Accounting 1

Module 3 - Financial Management & Management Accounting 1

Page 44 © CMA Ontario, 2011

cost of debt and equity financing will make the WACC increase. The firm’s optimal debt-to-equity ratio is the one which will minimize its WACC. o. Dividend Policy and Valuation The market price of a firm's shares is determined by the supply and demand for securities. The valuation of the common shares for a firm is a complex process for which many estimation models are available. In its simplest form, we can say that an investor would be willing to buy stock at an amount equal to the discounted present value of the future dividend receipts and the market value of the stock at some future point in time. Assume, for example, that an investor is considering purchasing a stock which promises to pay dividends of $3.00 per year for the next four years. The investor expects to sell the stock at the end of four years for $20.00. The relevant discount rate for securities of comparable risk is 10%. The maximum that this investor would be willing to pay for the stock would be as follows:

Value of the stock = Present Value of dividend income + Present Value of Share Price = $3.00 x (PV factor for annuity at 10% for 4 years) +

$20.00 x (PV factor at 10% for 4 years) = $3.00 x 3.170 + $20.00 x .683 = $9.51 + $13.66 = $23.17

If the investor was considering holding the share for a very long period of time, we could think about the value of the share as being a perpetuity of dividend incomes (the present value of the share price would be zero if we expected to hold it for a long period of time) as follows:

Value of the stock = Present value of dividend income in perpetuity = Annual dividend / required rate of return = $3.00/.10 = $30.00

If the stream of dividends was expected to grow at a constant rate of 5% per year, the value could be determined as follows

Value of the stock = Annual dividend / (required rate of return - growth rate) = $3.00/(.10 - .05) = $60.00

Page 45: MODULE 3 Financial Management and Management Accounting 1

Module 3 - Financial Management & Management Accounting 1

Page 45 © CMA Ontario, 2011

In all cases, the valuation of the shares is in huge measure influenced by the timing of the receipt of expected dividends and the risk associated with the investment. While it may at first appear that the amount of dividend payout is critical in this matter, financial theory holds that in the long run the value of a firm's shares will remain stable as long as the rate of return on new investment is equal to the investor's required rate of return. In other words, dividend policy is irrelevant; it is the return on the firm's investments that determines the value of the firm. Nonetheless, financial managers devote much effort to determine a desirable dividend policy. The factors to be considered in determining dividends include the amount of cash available for dividends, the amount of cash required internally for investment opportunities, legal reasons or contractual covenants limiting the amount which can be distributed, and investor expectations regarding the receipt of dividends. In the end, most large firms attempt to establish a stable or smoothed dividend policy to instill investor confidence in the firm and facilitate the sale of additional capital in the future.

Page 46: MODULE 3 Financial Management and Management Accounting 1

Module 3 - Financial Management & Management Accounting 1

Page 46 © CMA Ontario, 2011

4. Special Topics in Corporate Finance a. Mergers and Acquisitions

Up to this point in our discussion of financial management, we have implicitly assumed that growth comes only from new internal investments. It is, of course, common for firms to expand through mergers and acquisitions. An acquisition takes place when a large firm absorbs a smaller firm while a merger usually refers to the union of two firms of approximately equal size. Regardless of the term used to describe the external expansion, the basis of evaluation remains the same as for internal expansion proposals; a merger or acquisition should take place if it will increase shareholder wealth by generating a positive net present value. The reasons for business combinations are numerous and often involve in-depth analysis of strategic management considerations. A short list of possible reasons for external expansion would include the following:

• to integrate horizontally (acquire a competitor's operations)

• to integrate vertically (acquire an “upstream” supplier or a “downstream”

customer)

• to integrate concentrically (acquire an operation which employs a similar technology or channels of distribution)

• to acquire experienced managerial skills

• to reduce costs by combining similar operations (gain economies of scale)

Rationales for Mergers and Acquisitions The shareholders of an acquiring firm can benefit from a merger only if the two firms are worth more in combination than separately. The net advantage of merging (NAM) to the acquirer's shareholders' equals the difference between (1) the total market value of the firm post-merger net of the cost of completing the acquisition and (2) the total market value of the firms before the merger. Quantitatively: NAM = VAB – (VB + PB) – E – VA = [(VAB - (VA + VB)] – PB- E Where VAB = value of the combined firm (post-merger) VA = value of firm A (pre-merger) VB = value of firm B (pre-merger) E = various expenses associated with the merger PB = premium offered to shareholders of company B

Page 47: MODULE 3 Financial Management and Management Accounting 1

Module 3 - Financial Management & Management Accounting 1

Page 47 © CMA Ontario, 2011

If the net advantage to merging is positive, the merger would increase the wealth of the acquirer's shareholders. The term in the square brackets in the above equation represents the synergistic effect of the merger (discussed below). The whole is worth more than the sum of its parts when this expression is positive. Motives for Merging Valid motives for merging - there are at least four situations when a merger can be economically beneficial: 1. achieve operating efficiencies and economies of scale - the merged firms can

eliminate duplicate facilities, operations, sales forces, or departments. Achieving operating efficiencies is more likely to result from a horizontal or vertical merger than from a conglomerate merger. A horizontal merger combines two firms in the same line of business. A vertical merger involves integrating forward toward the consumer, or backward toward the source of supply, in a particular line of business. A conglomerate merger joins firms in unrelated businesses.

2. realize tax benefits in the form of tax carryovers 3. free up surplus cash - the acquirer may be motivated to use the acquiree's cash 4. grow more quickly or more cheaply - it is generally quicker to obtain new

products, new facilities, or a national distribution network by acquiring a firm that has already developed them

Dubious motives for merging 1. diversification - diversification will benefit the acquirer's shareholders only if

they could not achieve such diversification more cheaply on their own. This is not the case since investors can diversify their portfolios much more cheaply than it costs two firms in unrelated industries to merge.

2. lower transactions costs when issuing new securities - if the merged firm makes larger issues than the two firms would separately, the merger may result in lower transaction costs. However, lower issuance expenses are not likely to contribute much to the value of a merger.

3. enhanced earnings per share - a firm can increase its earnings per share by exchanging its shares for those of another firm whose shares have a lower price-earnings ratio. Such share-for-share exchanges will increase earnings per share even if no real gain results from the merger.

Synergies Synergies occur when a target firm controls a specialized resource that becomes more valuable when combined with the bidding firm's resources. When synergies exist, the bidding firm is often willing to pay a premium for the target firm. In horizontal mergers, synergies come from economies of scale (reduction of costs) and/or increased market

Page 48: MODULE 3 Financial Management and Management Accounting 1

Module 3 - Financial Management & Management Accounting 1

Page 48 © CMA Ontario, 2011

power (increased profits and sales). In vertical mergers, synergies come from more complete control of the chain of productivity. There are two forms of synergy: (1) Operational synergy: increases in value that accrues to a combined firm either

from economies of scale or increased sales/profit, or both; (2) Financial synergy: increases in value that accrue from a purely financial

perspective: lower taxes, higher debt capacity, or better use of idle cash. Tax free vs. Taxable acquisitions A tax free acquisition occurs when the acquirer issues shares in exchange for the shares of the acquiree. The shareholders of the acquiree do not receive any cash, consequently, there are no tax consequences to this transaction. There will be tax consequences only if the shares are eventually sold. A taxable acquisition occurs when the acquiree company's shareholders receive cash for their shares. A disposition occurs and capital gains/losses have to be calculated and taken into shareholder income. b. Business Valuations There are generally three valuation methods: (1) Asset Based Valuation Approaches (2) Earnings Based Approaches, and (3) Discounted Cash Flow Based Approaches. Generally, discounted cash flow based approaches are technically superior to the other two approaches. There are special circumstances, however, when the two other approaches should be used. Each approach will be discussed. Asset Based Valuation Approaches These are used when (1) the company is no longer a going concern (liquidation value) or (2) the company, while a going concern, does not possess any apparent value for its intangible assets (adjusted net assets). Liquidation Value = Liquidation value of assets - selling and disposal costs - liabilities paid ± operating profits/losses during liquidation period - costs associated with liquidation ± corporate income taxes

Page 49: MODULE 3 Financial Management and Management Accounting 1

Module 3 - Financial Management & Management Accounting 1

Page 49 © CMA Ontario, 2011

Adjusted asset value is equal to the net assets of the firm at book value plus or minus any differences between the fair market value and the book values of assets and liabilities. Asset based approaches are also used as a secondary approach when dealing with a going concern valuation approach (earnings or cash flow based). It provides (1) a comparison with other valuation approaches and (2) provides a measure of going concern risk: the higher the difference between the adjusted net assets and the going concern value (goodwill), the higher the risk. Concepts applying equally to earnings and cash flow based approaches The value of a firm's assets is equal to the value of the firm's debt plus the value of the firm's equity. Going concern approaches calculate the value of the firm's assets. Consequently, if we want to know the value of the equity we must deduct the value of the debt. E = V - D When calculating projected earnings/cash flow we must adjust for the following: • costs and revenues derived from redundant assets. Redundant assets are defined to

be any assets not required for operating purposes and include: excess cash, luxury automobiles used by shareholders, etc… (it is generally assumed that redundant assets will be sold by the acquirer)

• any unusual and non-recurring items, and • any excess remuneration paid to existing shareholders. The resulting adjusted net income after taking into account the above is called normalized earnings.

The discount rate used should be adjusted for the riskiness of the earnings involved. Do not use the acquiring firm's weighted average cost of capital. Earnings based approaches This methodology should only be used when (1) there are no significant non cash elements in income (or if depreciation expense closely equals annual capital acquisitions) and (2) if the acquired company is not growing and does not expect to have any future growth. If one or both of these conditions is not met, the discounted cash flow model must be used. Value of the assets = Normalized net earnings x multiplier (1 / required rate of return) see note = Going Concern value + value of redundant assets

Page 50: MODULE 3 Financial Management and Management Accounting 1

Module 3 - Financial Management & Management Accounting 1

Page 50 © CMA Ontario, 2011

Note - it is assumed that these earnings will generate in perpetuity at a zero rate of growth. Discounted Cash Flow Analysis The following steps are normally taken when determining firm value using discounted cash flows: 1. Estimate future free cash flows in the future. Note that you predict year to year

cash flows using available information. Normally we estimate cash flows for 3-5 years and assume that the final year cash flows will grow to perpetuity at a given growth rate. Free cash flow is the sum of the cash flows available before any interest or dividend payments are taken into account.

Free cash flow = Earnings before interest and taxes x (1 - t) + Depreciation - Capital expenditures ± Changes in working capital

2. Discount these free cash flows using an appropriate discount rate. The discount

rate must be risk adjusted for the riskiness of these cash flows. 3. Estimate a terminal value of the firm. This is the present value of future cash

flows starting with the year after which detailed estimates are calculated. 4. Subtract the market value of any long-term debt. The resultant figure is the value

of the equity. Comprehensive Example Assume that you are interested in determining the value of the Rho Company and have accumulated the following information: Five year income statements - (all data in millions) 20x1 20x2 20x3 20x4 20x5 Sales $48.5 $49.1 $53.7 $54.8 $55.3 Cost of goods sold 32.6 33.1 35.9 37.2 37.9 Selling, general and admin 10.7 11.1 11.5 11.9 12.3 Depreciation 2.0 2.3 2.4 2.3 2.1 Interest 0.4 0.7 0.8 0.8 0.8 Other 0.3 0.1 0.2 0.2 0.2 Net income before taxes 2.53 1.85 2.97 2.42 2.02 Income taxes 0.60 0.84 1.31 0.88 0.67 Net income $1.93 $1.01 $1.66 $1.54 $1.35

Page 51: MODULE 3 Financial Management and Management Accounting 1

Module 3 - Financial Management & Management Accounting 1

Page 51 © CMA Ontario, 2011

Balance Sheet As at December 31, 20x5 (millions of dollars) Cash $1 Accounts payable $2 Accounts receivable 8 Other 2 Inventories 18 4 Other 1 28 Long-term debt 12 Plant and equipment - net 19 Shareholders' equity 31 $47 $47

You estimate that you would be able to reduce the cost of goods sold as a percentage of sales to 65% and that by combining your sales force with that of Rho, you could reduce the selling, general and administrative expenses to 19% of sales. Analysts predict that sales growth will be approximately 6% over the next four years and 5% thereafter. The cash flows can be discounted at the after-tax risk adjusted rate of return of 10%. The tax rate is expected to be 40% from 20x6 on. The first step is to estimate the cash flows. Given that the reduction in the cost of goods sold % represents a 4 percentage point drop, we assume that such a drop would not occur in the first year but rather gradually over the next three three years: 68% in 20x6, 66.5% in 20x7 and 65% in 20x8 and on. Similarly, the drop in selling, general and administrative would be spread out over the next four years as follows:

20x6 22% 20x7 21% 20x8 20% 20x9 19%

We assume that depreciation and other expenditures will remain constant and that capital expenditures will equal depreciation expense. Working capital is estimated to be equal to the same percentages as per the most recent year: accounts receivable at 14.5% of sales, inventory at 47% of cost of goods sold and accounts payable at 5% of cost of goods sold.

Page 52: MODULE 3 Financial Management and Management Accounting 1

Module 3 - Financial Management & Management Accounting 1

Page 52 © CMA Ontario, 2011

Calculation of Free Cash Flows: 20x6 20x7 20x8 20x9 Net Sales $58.6 $62.1 $65.8 $69.7 Cost of goods sold 39.8 41.3 42.8 45.3 Selling, general and admin 12.9 13.0 13.2 13.2 Depreciation 2.1 2.1 2.1 2.1 Other 0.2 0.2 0.2 0.2 Earnings before interest and taxes (EBIT)

3.6

5.5

7.5

8.9

Taxes @ 40% 1.4 2.2 3.0 3.6 Earnings after taxes 2.2 3.3 4.5 5.3 Add back depreciation 2.1 2.1 2.1 2.1 Less increase in working capital (1.2) (1.1) (1.2) (1.6) Less capital expenditures (2.1) (2.1) (2.1) (2.1) Free Cash Flow 1.0 2.2 3.3 3.7 Discounted at 10% 0.9 1.8 2.5 2.5

Working Capital at year-end Accounts receivable 8.5 9.0 9.5 10.1 Inventory 18.7 19.4 20.1 21.3 Accounts payable (2.0) (2.1) (2.1) (2.3) 25.2 26.3 27.5 29.1

Value of Rho Company

Sum of discounted free cash flows (20x6 - 20x9) $7.7 Terminal value: Present value of post 20x9 free cash flows * 53.1 Value of the firm 60.8 Less long-term debt 12.0 Value of equity $48.8

* Calculated as:

Terminal value = Year 20x10 Free Cash Flow / (R - G) = 3.7 (1.05) / (.10 - .05) = 77.7

Present Value at the beginning of 20x6 = 77.7 PVIF 10%,4 = 77.7 (.683) = 53.1

Page 53: MODULE 3 Financial Management and Management Accounting 1

Module 3 - Financial Management & Management Accounting 1

Page 53 © CMA Ontario, 2011

5. Problems with Solutions Multiple Choice Questions 1. For any given security, total variability, or risk of returns, is equal to a) market-related and non-diversifiable risk b) diversifiable risk and firm-specific risk c) firm-specific risk and unique risk d) systematic risk and non-diversifiable risk e) none of the above 2. XL Inc. has just issued new long-term debt which sold at face value and has a

maturity of 3 years. If interest rates drop over the next year, what will be the impact on the market value of the debt at the end of the year?

a) It will be higher than face value. b) It will be lower than face value c) It will be unchanged. d) There is not enough information to determine the outcome. 3. Mindy's, a chain of fast food restaurants, just paid a dividend of $4.00 per share.

Dividends have been growing at a steady rate of 5% per year and are expected to continue to do so indefinitely. If investors require a 12% rate of return on shares of this type, at what price should Mindy's shares trade?

a) $50.00 b) $53.57 c) $57.14 d) $60.00 4. Mr. Dubofsky just won a 'Name that Tune' contest with a grand prize of $100,000.

However, the contest stipulates that the winner receives $10,000 immediately, and $10,000 at the end of each of the next 9 years. Assuming he can earn 10% on his money, how much has he actually won?

a) $100,000 b) $78,225 c) $67,590 d) $61,446 e) $34,988

Page 54: MODULE 3 Financial Management and Management Accounting 1

Module 3 - Financial Management & Management Accounting 1

Page 54 © CMA Ontario, 2011

5. J&J Manufacturing just issued a bond with a $10,000 face value and a coupon rate of 7%. If the bond has a life of 30 years, pays annual coupons, and the yield the maturity is 9%, what will the bonds sell for?

a) $ 7,936 b) $ 7,945 c) $ 8,143 d) $ 8,387 e) $12,482 6. Suppose you have just purchased a share of stock for $58.50. You expect a

dividend next period of $2.50 which will grow at a rate of 15% indefinitely. What must your expected rate of return be on the stock you have just purchased?

a) 4.27% b) 15.38% c) 19.27% d) 19.91% e) None of the above 7. Boomer Products, Inc. manufactures "no-inhale"`cigarettes. As their target

customers age and pass on, sales of the product are expected to decline. Thus, demographics suggest that earnings and dividends will decline at a rate of 8% annually forever. The firm just paid a dividend of $4.00; the required return is 12%. Given these parameters, the price of the stock in 5 years will be

a) $ 17.58 b) $ 15.01 c) $ 14.22 d) $ 12.13 e) $ 5.46 8. Suppose that Topstone Industries has a cost of equity of 12% and a before-tax cost

of debt of 7%. If the target debt/equity ratio is 60%, and the tax rate is 34%, what is Topstone's weighted average cost of capital?

a) 5.64% b) 6.76% c) 7.57% d) 9.23% e) 10.76%

Page 55: MODULE 3 Financial Management and Management Accounting 1

Module 3 - Financial Management & Management Accounting 1

Page 55 © CMA Ontario, 2011

9. Treasury bills currently have a return of 8%. The market risk premium is 9%. If Firm A has a Beta of 1.35, what is its cost of equity?

a) 4.78% b) 9.42% c) 12.78% d) 20.15% e) 20.78% 10. Given the following information, what is WBM Corporation's WACC? Common Stock 1 million shares outstanding $40 per share, $ 1 par value ß = 1.0 Bonds 10,000 bonds outstanding $ 1000 face value for each bond 8% annual coupon 20 years to maturity Market price = $830.12 Market Risk Premium: 8.6% Risk-Free Rate 6% Tax Rate 40% a) 7.89% b) 9.90% c) 12.90% d) 13.12% e) 15.78% 11. According to finance theory, which of the following best states what should be the

primary goal of a large public company’s vice-president of finance?

a) Minimize the risks taken by the company. b) Maximize the current value per share of the company’s existing stock. c) Maximize the asset value of the entire company. d) Maximize the company’s current accounting profit. e) Maximize the company’s current earnings per share.

Page 56: MODULE 3 Financial Management and Management Accounting 1

Module 3 - Financial Management & Management Accounting 1

Page 56 © CMA Ontario, 2011

12. In the financial management literature, diversifiable risk includes risk related to

a) government monetary policies. b) government fiscal policies. c) company product development. d) inflation. e) none of the above. 13. Which of the following will decrease the present value of ten equal annual annuity

payments, all else remaining equal?

a) Decreasing the interest rate. b) Increasing the number of payments. c) Increasing the amount of each payment. d) Making the annuity payments at the start of each period instead of at the end

of the period. e) None of the above. 14. A $100,000 par, 10 percent coupon rate bond, with a remaining life of 8 years, pays

interest semi-annually. Assume that the bond’s yield to maturity is 12 percent. The market price of the bond should be

a) $89,894

b) $90,080 c) $110,720 d) $110,828 e) impossible to determine with the information provided. 15. Which of the following is NOT generally a right of common shareholders of a

corporation?

a) The right to vote for members of the board of directors. b) In bankruptcy, the right to residual assets of the firm. c) The right to receive a regular dividend from the firm. d) The right to limit claims on their personal wealth by the corporation’s

creditors to the amount of money invested in the company’s shares. e) The right to attend shareholder meetings.

Page 57: MODULE 3 Financial Management and Management Accounting 1

Module 3 - Financial Management & Management Accounting 1

Page 57 © CMA Ontario, 2011

16. Acme Limited offers credit terms of a 2% discount if paid within 10 days or the full balance is due within 30 days (2\10, net 30). If 20% of Acme’s customers pay cash on delivery, 60% pay on day 10 and 20% pay on day 30, the average collection period is

a) 10 days. b) 12 days. c) 20 days. d) average receivables divided by average daily sales. e) annual credit sales divided by average receivables. 17. A capital structure that maximizes share prices generally will at the same time a) maximize the weighted-average cost of capital. b) minimize the cost of equity. c) minimize the weighted-average cost of capital. d) have no effect on weighted-average cost of capital. e) do both a) and b) above. 18. Spotech Co.'s budgeted sales and budgeted cost of sales for the coming year are

$212,000,000 and $132,500,000, respectively. Short-term interest rates are expected to average 5%. If Spotech could increase inventory turnover from its current 8.0 times per year to 10.0 times per year, its expected cost savings in the current year would be

a) $165,625 b) $0 c) $3,312,500 d) $828,125 19. By using the dividend growth model, estimate the cost of equity capital for a firm

with a stock price of $30.00, an estimated dividend at the end of the first year of $3.00 per share, and an expected growth rate of 10%.

a) 21.1% b) 11.0% c) 10.0% d) 20.0%

Page 58: MODULE 3 Financial Management and Management Accounting 1

Module 3 - Financial Management & Management Accounting 1

Page 58 © CMA Ontario, 2011

Problem 1 - Risk Assume you are an investor considering investing in Canada Grocer Ltd. (CGL) and Frontier Oil and Gas (FOG). Information on the expected returns for both firms is given below.

CGL FOG Probability Return Probability Return

.20 $4,000 .30 $1,000

.60 $5,000 .40 $5,000

.20 $6,000 .30 $9,000 Comment on the degree of risk associated with each firm. What impact would this assessment have on your approach to investing in either firm? Problem 2 – Present Value Exercises 1. Life & Health Insurance Company offers you the following plan: during the next

30 years you pay $2,000 at the end of each year. Starting at the end year 31, you (or your heirs) will receive a pension for the following 15 years. The discount rate used by the company to calculate your pension is 5%.

a. What is the size of your annual pension? b. If you could take a one-time lump sum payment at the end of year 35

instead of the pension, what would the lump sum payment be? 2. You estimate that you will need $65,000 to send your child to a university in eight

years. You have about $25,000 now. If you can earn 15%, will you make it? At what rate will you just reach your goal?

3. You think that you will be able to deposit $4,000 at the end of each of the next

three years in a bank account paying 8% interest, You currently have $7,000 in the account. How much will you have in three years? In four years?

4. You are offered an investment that will make three $5,000 payments. The first

payment will occur four years from today. The second will occur in five years, the third will follow in six years. If you can earn 11%, what is the most this investment is worth today? What is the future value of the cash flows?

5. After carefully going over your budget, you have demonstrated you can afford to

pay $632 per month towards a new sports car. You call your bank and find out that the going interest rate is 1% per month for 45 months. How much can you borrow? What will be the balance of your loan after you have made the 32nd payment?

Page 59: MODULE 3 Financial Management and Management Accounting 1

Module 3 - Financial Management & Management Accounting 1

Page 59 © CMA Ontario, 2011

6. Your younger brother has come to you for advice. He is about to enter university and has two options open to him. His first option is to study engineering. If he does this, his undergraduate degree would cost him $12,000 a year for four years. Having obtained this, he would need to gain two years of practical experience: in the first year he would earn $20,000, in the second year he would earn $25,000. He then would need to obtain his master's degree, which will cost $15,000 a year for two years. After that he will be fully qualified and can earn $40,000 per year for 25 years.

His other alternative is to study accounting. If he does this, he would pay $13,000 a year for four years, and then he would earn $31,000 per year for 30 years.

The effort involved in the two careers is the same, so he is only interested in the earnings the jobs provide. All earnings and costs are paid at the end of the year. What advice would you give him if the applicable interest rate is 5 percent? A day later he comes back and says he took your advice, but in fact, the applicable interest rate was 6 percent. Has your brother made the right choice?

7. Assume that all cash flows occur at the end of the year.

Part (a) Your younger sister, Barbara, will start college in five years. She has just informed your parents that she wants to go to Eastern University, which will cost $ 15,000 per year for four years (assumed to come at the end of each year). Anticipating Barbara's ambitions, your parents started investing $2,000 per year five years ago and will continue to do so for five more years. How much more will your parents have to invest each year for the next five years to have the necessary funds for Barbara's education? Use 10 percent as the appropriate interest rate throughout this problem.

Part (b) Barbara is now 18 years old (five years have passed), and she wants to get married instead of going to school. Your parents have accumulated the necessary funds for her education (as per part (a)). Instead of her schooling, your parents are paying $7,000 for her upcoming wedding and plan to take a year-end vacation costing $4,000 per year for the next three years. How much will your parents have at the end of three years to help you with graduate school, which you will start then?

Page 60: MODULE 3 Financial Management and Management Accounting 1

Module 3 - Financial Management & Management Accounting 1

Page 60 © CMA Ontario, 2011

Problem 3 - WACC The following information relates to the operations of Data General Limited (DGL). The current capital structure of DGL is as follows:

Long-term debt 8% $20,000,000 Preferred Stock ($100 par - 12 % dividend) $40,000,000 Common Stock $35,000,000 Retained Earnings $5,000,000

The current market price of DGL preferred and common stock is $100 and $50 per share respectively. Common dividends just paid are $5.75 and are expected to grow at the rate of 4% each year thereafter. The corporate tax rate is 40%. DGL expects to maintain its current capital structure for the foreseeable future. Determine the weighted average cost of capital for DGL and comment on how this figure should be used in managing the financial activities of the firm. Problem 4 - WACC The Complex Corporation's capital structure is composed of

Long-Term Debt (with an interest rate of 10%) $40,000,000 Preferred Shares (par $100, 12% dividend) $80,000,000 Common Shares $70,000,000 Retained Earnings $10,000,000

The common stock has a current market price of $50 per share, a current dividend of $6.75 and an expected dividend growth rate of 4% per year. Preferred shares have a current market price of $100. The company expects no major changes in its basic capital structure for the foreseeable future. The corporate tax rate is 40%. Determine the weighted average cost of capital for the Complex Corporation. Problem 5 - WACC Yemko Inc. is a manufacturer of lawn mowers. The following information relates to the capital structure currently in place for Yemko Inc.: Long-term debt (8.5% annual interest rate) $22,000,000 Preferred shares ($150 par value, $16.50 annual dividend) 45,000,000 Common shares (500,000 shares outstanding) 30,000,000 Retained earnings 3,000,000

Page 61: MODULE 3 Financial Management and Management Accounting 1

Module 3 - Financial Management & Management Accounting 1

Page 61 © CMA Ontario, 2011

Currently, Yemko Inc. can raise long-term debt at an annual interest rate of 7.5%. The current market prices of Yemko Inc.'s shares are $170 per preferred share and $70 per common share. Common dividend per share is $6.50 and is expected to grow at a rate of 2.5% per year. Assuming the current capital structure will remain constant and that Yemko Inc. has an income tax rate of 40%, what is the weighted average cost of capital that Yemko Inc. should use to evaluate investment opportunities? Problem 6 - Bond Valuation A bond having a face value of $ 1,000, an annual coupon rate of 12% payable semi-annually and six years remaining to maturity, sells in the market for $ 1,220. Calculate the effective annual yield of this bond. Problem 7 - Gordon Model Common shares of ABC Ltd. currently trade at $27.50. The most current dividend was $1.00. Current dividends are expected to be 10% higher or $1.10. The 10% growth rate in dividends is expected to continue for the foreseeable future. What is the expected return of ABC Ltd.'s common shares? Problem 8 - WACC The Clarke Company has the following partial balance sheet: Bonds payable, face value $10,000,000, coupon rate = 8% payable semi- annually, 12 years left to maturity

$9,750,000

Preferred shares, 8%, par value = $1,000 8,000,000

Common stock, 1,200,000 shares issued and outstanding 25,000,000

Retained earnings 16,000,000 The common shares are currently selling for $13 per share and the preferred shares are selling for $833 per share. The company’s beta is 1.3, the risk free rate is 4% and the current rate of return on the market is 12%. Bonds with a similar maturity and risk level to our bonds are currently yielding 10%. The tax rate is 35%. Required – Calculate the firm’s weighted average cost of capital.

Page 62: MODULE 3 Financial Management and Management Accounting 1

Module 3 - Financial Management & Management Accounting 1

Page 62 © CMA Ontario, 2011

Problem 9 - Common Stock Valuation A certain stock is expected to pay the following annual dividends for the next 3 years. After this, it is expected that the dividends will grow by 5% per year. Shareholders require a return of 18%.

End of year Dividend 1 $1.00 2 2.00 3 3.00

For years 1-4, show that the required return of 18% is the sum of the dividend yield and the capital gain yield.

Problem 10 - Common Stock Valuation ABC Company's common shares currently trade on the Toronto Stock Exchange at $32 per share. A new issue of ABC stock could be sold at $30 per share, less underwriting and issue costs of $3. The company's tax rate is 40% and its beta is estimated at 1.2. The current long-term risk-free rate for the market portfolio is 14% and the return on the market is expected to exceed this rate by 6%. Required - a) Calculate the cost of equity for ABC Company using the Capital Asset Pricing

Model. b) What are the limitations of using the Capital Asset Pricing Model for calculating the

cost of equity? c) Under what circumstances is it appropriate to use the weighted average cost of capital

for investment considerations? Problem 11 - WACC Walker Co. Ltd. 's latest financial statements show the following capital structure: Debt $4,000,000 Preferred shares 1,000,000 Common stock 3,000,000 Retained earnings 2,000,000 $10,000,000

Page 63: MODULE 3 Financial Management and Management Accounting 1

Module 3 - Financial Management & Management Accounting 1

Page 63 © CMA Ontario, 2011

The company is currently considering a major plant expansion, estimated to cost $2,000,000, which it intends to finance solely through an issue of 15-year, 13% long-term debt. Two alternative financing arrangements were considered: (1) issue new preferred shares, and (2) issue additional common shares. Preferred shares could be issued at a net price of $8.50 with an annual dividend of $0.80. Common shares could be issued at $25. Walker Co. has a beta of 1.2 and a tax rate of 40%. The current yield on long-term government bonds is 10% and the expected yield on the market portfolio is 16%. Required - a) Using the Capital Asset Pricing Model, calculate the expected return on common

equity. b) Assume your solution to (a) above is 18%. Calculate Walker's weighted average cost

of capital using book value weights inherent in the latest financial statements summarized above.

c) Comment on the appropriateness of using the weighted average cost of capital as calculated in (b) above in all of Walker's capital budgeting decisions.

Problem 12 - Common Stock Valuation You are considering the purchase of 1,000 shares of Western Oils Ltd. that, in your opinion, have excellent growth potential for dividends over the next four years. Dividends currently are being paid at the rate of $2 per share and are expected to increase by 10% in each of the next four years; thereafter, the dividend growth rate is expected to stabilize at 6% per annum. The required after-tax return is 12%. Required - Given that you require an after-tax rate of return of 12%, what is the maximum price that you would be willing to pay for one share of Western Oils Ltd.? Show all calculations. Problem 13 - Credit Policy Designer Lighting Ltd. manufactures high quality lighting fixtures. However, as a result of a recession, sales dropped from $1.6 million for the previous year to $1.0 million for the current year. Furthermore, gloomy forecasts for consumer spending and continued reluctance of retailers to carry high inventories have led the management of Designer Lighting Ltd. to conclude that improvement in sales will be gradual, averaging about 10% for the coming year.

Page 64: MODULE 3 Financial Management and Management Accounting 1

Module 3 - Financial Management & Management Accounting 1

Page 64 © CMA Ontario, 2011

However, management believes that sales can be further increased if the company changes its credit policy to allow extended terms of payment. The present policy is n/30 with all sales made on credit. All credit applications are carefully reviewed and, as a result, no bad debts have been experienced. The alternative credit policies being considered for the coming year are:

Average Estimated Credit Policy Collection Period Annual Sales

A 60 days $1,400,000 B 90 days 1,500,000 C 120 days 1,700,000

Variable costs are 75% of sales and the opportunity cost of carrying additional receivables is 20% before taxes. Assume that inventory levels will remain constant. Required - a) Which credit policy should Designer Lighting Ltd. adopt? Why? (Show all calculations.) b) What other changes in the credit policy should Designer Lighting Ltd. consider? Problem 14 - Inventory Management The Atkinson Corporation produces product X. The estimated annual materials requirement for finished goods is 2,500 units. The cost of materials per unit is $4.50. The order cost per purchase order is $6.75. The annual inventory carrying cost per unit is equal to 11% of average inventory cost. Required: a Calculate the total ordering and carrying costs for order sizes of 50, 75, 100, 150,

250, 400, and 600 units. b. Calculate the EOQ. What are the total ordering and carrying costs at the EOQ?

Page 65: MODULE 3 Financial Management and Management Accounting 1

Module 3 - Financial Management & Management Accounting 1

Page 65 © CMA Ontario, 2011

Problem 15 - Inventory Management The Goode Company has in the past ordered raw material GRC in quantities of 3,250 units, which is a 26 weeks' supply. Management has decided to change over to an ordering system based on economic order quantities. Assume the following information pertaining to the company's purchasing and production activity: Inventory usage rate: 125 units per week Lead time: 3 weeks Unit price: $1.50 Order cost: $7.50 per order Carrying cost: $0.30 per unit per year Required: a. Calculate the economic order quantity using the EOO formula. b. Calculate the order point. c. Calculate the order point if safety stock is desired to cover a possible lead time of

5 weeks and a usage increase to 200 units a week. Problem 16 - Leverage The following information relates to the earnings and capital structure of Spencer Electronics Ltd. (SEL).

Current Projected Debt (10%) $50,000 $50,000 Equity $50,000 $50,000 Revenue $28,000 $33,600

In addition, it is known that: • variable costs are 50% of sales volume, • fixed costs are $7,000 (excluding interest and taxes), • the corporate tax rate is 40%, and • every $10.00 of equity represents one common share.

Required –

(a) Prepare an income statement for both the current and projected scenarios. (b) Determine the degree of operating leverage, financial leverage and total leverage

for SEL. (c) Show that the percentage increase in sales times the degree of total leverage

equals the percentage increase in net income after taxes.

Page 66: MODULE 3 Financial Management and Management Accounting 1

Module 3 - Financial Management & Management Accounting 1

Page 66 © CMA Ontario, 2011

Problem 17 - Leverage The following information describes the earnings and capital structure of the Maxim Corporation for the current year and projections for the following year:

Current Projected Debt (12%) $50,000 $50,000 Equity $100,000 $100,000 Revenue $60,000 $90,000

In addition, it is known that variable costs are normally 60% of sales, fixed costs (excluding interest and taxes) are $12,000 per year, and the corporate tax rate is 35%. Every $10.00 of equity represents one common share. Required - a. Determine the degree of operating leverage for the Maxim company. b. Determine the degree of financial leverage for the Maxim company. c. Determine the degree of total leverage for the Maxim company. d. Prepare an income statement for both the current and projected scenarios. Problem 18 - Capital Structure A company presently financed by 1,000,000 common shares having a market value of $8 per share is considering two financing alternatives, both raising $3,200,000 in additional capital. Alternative A is to issue 400,000 new common shares at $8 each and alternative B is to issue 15 year 9% debt at face value. Ignoring issue and underwriting costs and assuming a tax rate of 40%, what is the level of earnings before interest and taxes (EBIT) at which the company would be indifferent between the two alternatives?

Page 67: MODULE 3 Financial Management and Management Accounting 1

Module 3 - Financial Management & Management Accounting 1

Page 67 © CMA Ontario, 2011

Problem 19 - WACC Coril Manufacturing Limited (CML) is considering the following proposals for new investments in capital projects for the upcoming year. Initial

Investment Annual

Cash Flow Project

Life Project A $50,000 $10,000 10 Project B $150,000 $30,000 8 Project C $100,000 $30,000 6 CML's current capital structure is composed of $70,000 in long-term bonds, $30,000 in preferred shares, $80,000 in common shares and $20,000 in retained earnings. The long-term debt carries an interest rate of 9%. Each preferred share has a par value of $50 and pays an annual dividend of $7. The market price of the common shares is $60, and the dividend for the upcoming year is expected to be $9 per share. Common dividends are expected to grow at an annual rate of 3%, and the corporate tax rate is 45%. CML can sell new debt and equities at current rates, and its capital structure will likely remain unchanged in the foreseeable future. Required:

(a) Determine the weighted average cost of capital for CML. (b) Determine the net present value of each project. (c) Which projects should CML undertake?

Problem 20 - Leverage The Dunwood Corporation currently has $90,000 in long-term bonds outstanding with an interest rate of 8%. It also has $110,000 in 11,000 common shares on the market. Last year, the company had sales of $216,667 with a contribution margin percentage of 60% and fixed costs of $70,000. The corporate tax rate is 50%. Required:

a) Determine the degree of operating, financial and total leverage for the Dunwood

Corporation if sales are expected to rise to $250,000 assuming that the financial and cost structure of the firm will remain unchanged. (Note: the calculations should be based on a sales level of $216,667 and the DOL/DFL/DTL calculations are used to determine the impact of an increase in sales to $250,000)

b) What would happen to Dunwood's degree of financial, operating and total leverage if it issued $30,000 in new common stock and used the proceeds to reduce its long-term debt by the same amount? Assume that the expected sales level of $216,667 and cost structure of the firm would remain as before.

Page 68: MODULE 3 Financial Management and Management Accounting 1

Module 3 - Financial Management & Management Accounting 1

Page 68 © CMA Ontario, 2011

Problem 21 - Proforma Financial Statements Englehart Limited (EL) manufactures auto parts. Last year, its profits fell for the first time in its twenty-year history. In order to reverse this trend, management has decided to buy $5,000,000 in new special manufacturing equipment to produce 100,000 component for Major Motors Inc. (MMI) at a total sales price of $25,000,000. MMI needs 40,000 components next year and 60,000 components the year after. The special machinery will last for ten years and can be converted to regular production after this order is completed. EL has determined that the net present value of the investment in new machinery is positive, but it does not have the cash to acquire it. As it is not possible to lease the equipment, EL has decided to ask its bank for a loan. The bank manager examined the financial statements of EL (see Exhibit 1). His discussions with the EL controller revealed that the expected level of sales for the next year was $50,000,000 plus the MMI contract. While the variable costs of sales and production were expected to remain unchanged for the next year, fixed manufacturing overhead would likely increase by 5% plus an additional $750,000 (excluding depreciation) related to the new manufacturing equipment. Fixed selling and administrative costs were expected to increase by 5% plus an additional $300,000 for the contract. Total capital cost allowance charges for the upcoming year were expected to be $6,950,000. Sales are made uniformly throughout the year and all customers, including MMI, pay 60 days after delivery. EL's policy is to maintain a minimum cash balance of $200,000 and inventory of $10,000,000. Average accounts payable are expected to be maintained at 8% of total variable costs incurred during the year. All fixed costs are paid in the year they are incurred. The current bank loan requires a yearly payment of $5,000,000 plus interest at the rate of 12%. If the new loan request is approved, a rate of 12% can also be used. Required: The bank manager is interested in lending EL the necessary $5,000,000, but first wants to see a pro forma cash budget for the upcoming year as well as a list of things that management plans to do over the next twelve months in order to improve its working capital position. Assume the role of the EL controller, and prepare the documents requested by the bank.

Page 69: MODULE 3 Financial Management and Management Accounting 1

Module 3 - Financial Management & Management Accounting 1

Page 69 © CMA Ontario, 2011

Exhibit 1 Englehart Limited

Financial Statements

Balance Sheet As at December 31

Cash $200,000 Accounts Payable $2,736,000 Accounts Receivable 10,000,000 Bank Loan Payable 30,000,000 Inventory 10,000,000 Machinery & Equipment 70,000,000 Common Stock 12,500,000 Accum. Depreciation 28,000,000 Retained Earnings 16,964,000 Total Assets $62,200,000 Total Equities $62,200,000

Income Statement For the Year Ending December 31

Sales Revenue $60,000,000 Cost of Goods Sold: Variable manufacturing overhead $30,000,000 Fixed manufacturing overhead (including depreciation)

15,000,000

45,000,000

Gross Margin 15,000,000 Selling & Administrative Expenses: Variable selling & administrative 4,200,000 Interest 4,200,000 Fixed selling & administrative 4,500,000 12,900,000 Net income before taxes 2,100,000 Taxes (see note) 414,400 Net income after taxes 1,685,600 Dividends paid 500,000 Transfer to Retained Earnings $1,185,600 Note re tax expense: Income before tax $2,100,000 Plus depreciation 7,000,000 Less capital cost allowance (8,064,000) Taxable income $1,036,000 Income taxes @ 40% $ 414,400

Page 70: MODULE 3 Financial Management and Management Accounting 1

Module 3 - Financial Management & Management Accounting 1

Page 70 © CMA Ontario, 2011

Problem 22 - Working Capital Management Canadian Toy Retailers Ltd., a medium-sized, privately-owned retail chain with stores throughout Canada, is reviewing a number of its financial policies with a view to improving the company's profitability. The controller has asked you, the senior financial analyst, to take a look at the company's policy regarding payment of accounts payable. She has asked you specifically to include in your review consideration of the alternative policies of stretching payments on accounts payable to 60 days, and taking discounts. The company's current policy is to pay all accounts when due (i.e - on the 30th day). All purchases from established suppliers are made on account and all purchases from new suppliers are C.O.D., the latter accounting for approximately 10% of all purchases. Terms of purchases on account are 2/10 net 30 for 80% of suppliers and n/30 for the remaining 20%. Currently, accounts payable averages $250,000 and is expected to remain at that level for the next two or three years, assuming no change in the accounts payable payment policy. Although the company does not expect significant penalties for late payment of its accounts payable, if it adopts the policy of stretching payments on accounts payable to 60 days, it does anticipate that 10% of the amount of accounts payable that would exist under this policy will attract interest penalties of 24% per annum. The company currently has a sizable revolving line of credit financed at 12% per annum. Required - Compare the proposed and present accounts payable payment policies and recommend the one that will best achieve the company's objective of improving profitability. Show all calculations. Problem 23 – Working Capital Management Denis Electronics has annual sales (all credit) of $8 million and an average collection period of 30 days. The current level of bad debts is $240,000 and the firm's cost of funds is 15%. The firm only produces one product with variable costs equaling 75% of the selling price. The company is considering a change in credit terms from its current policy (net 30) to a new policy (1/30 net 60). If this change is made, it is expected that half the customers will take the discount and pay on the 30th day while the other half will not take the discount and pay on the 60th day. Previously customers paid their accounts in 30 days. However, the new policy will generate an additional $1,000,000 in sales. Also, while bad debts on current sales will remain at $240,000, bad debts on the new sales will be 6% of those sales. An additional $25,000 of inventory will also be required. Should the change in policy be made?

Page 71: MODULE 3 Financial Management and Management Accounting 1

Module 3 - Financial Management & Management Accounting 1

Page 71 © CMA Ontario, 2011

Problem 24 – Rights Offerings The Robinson Company has proposed a rights offering. The stock currently sells for $40 per share. Under the terms of the offer, stockholders are allowed to buy one new share for every five they own at a price of $25 per share. What is the value of a right? With is the price of the stock after the rights offering? Problem 25 – Rights Offerings The Mitchelmore Corporation is in the process of selling common shares through a rights offering. Prior to the rights offering, the firm had 500,000 common shares outstanding. Through the rights offering, it plans on issuing an additional 100,000 common shares at a subscription price of $30. After the shares when ex-rights, the market price was $40. What was the price of Mitchelmore Corporation common shares just prior to the rights offering?

Page 72: MODULE 3 Financial Management and Management Accounting 1

Module 3 - Financial Management & Management Accounting 1

Page 72 © CMA Ontario, 2011

SOLUTIONS

Multiple Choice Questions 1. e Total variability (risk of returns) is equal to systematic (non-diversifiable or

market-related) risk and unsystematic (diversifiable, unique or firm specific) risk.

2. a 3. d P0=D1/(r-g) = 4(1.05) / (.12 - .05) = $60.00

4. c N = 9, I = 9, PMT = 10,000 Solve for PV = $57,590 $57,590 + 10,000 = $67,590 5. b N = 30, I = 9, PMT = 700, FV = 10000 Solve for PV = $7,945 6. c r = D1/P0 + g = 2.50/58.50 + .15 = 19.27% 7. d P0 = D1 / r-g = [4.00(.92) ÷ (.12 + .08)] = 18.40 N = 5, I = -8, PV = 18.40, solve for FV = $12.13 8. d (3/8)(.07)(.66) + (5/8)(.12) = 9.23% 9. d 8% + 1.35(9%) = 20.15% 10. d E = 1,000,000 shares x $40 = $40,000,000 ke = 6% + 1.0 (8.6%) = 14.6% D = 10,000 x 830.12 = $8,301,200 To find kd, we use the financial calculator: N = 40, PV = -830.12, PMT = 40, FV = 1000, Solve for I = 10% Kd = 10% (.6) = 6% WACC = (8,301,200/48,301,200)(6%) + (40,000,000/48,301,200)14.6% =13.12%

Page 73: MODULE 3 Financial Management and Management Accounting 1

Module 3 - Financial Management & Management Accounting 1

Page 73 © CMA Ontario, 2011

11. b The shareholders of large public companies are usually not directly involved with the day-to-day operations of a company and, instead, employ managers to represent the owners. interests and make decisions on their behalf. Therefore, the vice-president of finance is mainly concerned with maximizing the current market value of the company’s existing shares. The market value of the company’s shares is a reflection of the present value of future cash flows. Other Choices: a) Minimizing the risks taken by the company could result in profitable projects being overlooked by management. This could result in sacrificing growth potential, which could have a negative effect on share price. c) Maximizing the size (i.e. asset value) of the company could lead to decisions that could have a negative effect on share price and shareholder wealth. For example, a company could pay an excessive amount for another firm in an effort to increase the acquiring company’s size, but because the amount paid was excessive, the value of its shares could go down instead of up. d) Maximizing accounting profit is not the same as maximizing cash flow. It is cash flow that will ultimately determine the value of the company’s shares. As well, accounting profit for the current year does not consider the future profits of the company. e) Maximizing current earnings per share is similar to choice d) Current earnings per share reflect the current accounting profit of the company and do not consider the future profits of the company or its cash flows.

12. c Diversifiable risk relates to events that affect individual companies, such as

product development (choice c), patents, labour unrest, and the like. These events are specific to an individual company and can, therefore, be reduced (or even eliminated) in a well-diversified portfolio. Alternatively, nondiversifiable risk includes risk related to general economic conditions, inflation (choice d), monetary and fiscal policies (choices a & b) and other such events that are not specific to any one company.

13. e All of choices a) to d) will cause the present value of an annuity to increase,

not decrease; therefore, choice e) is the correct answer.

Page 74: MODULE 3 Financial Management and Management Accounting 1

Module 3 - Financial Management & Management Accounting 1

Page 74 © CMA Ontario, 2011

14. a N = 16, I = 6, PMT = 5,000, FV = 100000, Solve for PV = 80,894 15. c The payment of regular dividends is discretionary on the part of management

and is not generally a right of common shareholders. Choices a), b), d) (more commonly known as limited liability) and e) are generally all rights of common shareholders.

16. b The average collection period is the weighted average period within which the

value of receivables is collected. Acme’s average collection period is 12 days: 0.2 x 0 + 0.6 x 10 +0.2 x 30 = 12 days.

17. c A firm’s capital structure is optimal if it maximizes shareholder wealth.

Generally, a capital structure that maximizes the market value of a firm’s common equity will, at the same time, maximize the total value of the firm and minimize the weighted average cost of capital.

18. a If cost of sales is $132,500,000, and the inventory turnover rate is 8.0 times

per year, the average inventory is $16,562,500 ($132,500,000 8). If the turnover increases to 10.0 times annually, the average inventory will decline to $13,250,000 ($132,500,000 10), a decrease of $3,312,500. At a 5% rate, reducing working capital by $3,312,500 will save the company $165,625 (.05 x $3,312,500). Answer (B) is incorrect because the faster turnover reduces working capital and releases funds for other uses. Answer (C) is incorrect because $3,312,500 is the decrease in average inventory. Answer (D) is incorrect because $828,125 is 5% of $16,562,500.

19. d The dividend growth model determines the cost of equity by adding the

expected growth rate to the quotient of the next dividend and the current market price. Thus, the cost of equity capital is 20% [10% + ($3 $30)]. This model assumes that the payout ratio, the retention rate, and the earnings per share growth rate are all constant. Answer (A) is incorrect because the growth rate (10%) is added to the dividend yield (10%). Answer (B) is incorrect because 11.0% equals the growth rate (10%) plus 10% of the current dividend yield (10%). Answer (C) is incorrect because 10.0% is the growth rate.

Page 75: MODULE 3 Financial Management and Management Accounting 1

Module 3 - Financial Management & Management Accounting 1

Page 75 © CMA Ontario, 2011

Problem 1

The degree of risk associated with FOG is greater than with CGL. This conclusion is based on the degree of variability in the earnings estimates for the two firms. Even though the average expected return is the same for both firms ($5,000), there is a 30% chance that the earnings of FOG will rise or fall by $4,000. The maximum expected change in CGL's earnings is only $1,000. Given the increased risk which would therefore be associated with the FOG stock, rational investors would demand a higher return on FOG stock. This higher return would most likely be manifested in, all else being equal, a lower price on FOG stock.

Problem 2 1a. Amount will accumulate to $132,878:

N I/Y PV PMT FV Enter 30 5 2000 Compute X

Annual pension amount is $12,802:

N I/Y PV PMT FV Enter 15 5 132878 Compute X

1b. $169,590:

N I/Y PV PMT FV Enter 5 5 132878 Compute X

Page 76: MODULE 3 Financial Management and Management Accounting 1

Module 3 - Financial Management & Management Accounting 1

Page 76 © CMA Ontario, 2011

2. In eight years you will have $76,476:

N I/Y PV PMT FV Enter 8 15 25000 Compute X

Rate required to achieve goal is 12.7%:

N I/Y PV PMT FV Enter 8 -25000 65000 Compute X

3. In three years you will have $21,804:

N I/Y PV PMT FV Enter 3 8 7000 4000 Compute X

In four years you will have $23,548:

N I/Y PV PMT FV Enter 1 8 21804 Compute X

4. The present value of these investments at t=3 is $12,219:

N I/Y PV PMT FV Enter 3 11 5000 Compute X

The present value at t=0 is $8,934:

N I/Y PV PMT FV Enter 3 11 12219 Compute X

The future value at t=6 is $16,710:

N I/Y PV PMT FV Enter 6 11 8934 Compute X

Page 77: MODULE 3 Financial Management and Management Accounting 1

Module 3 - Financial Management & Management Accounting 1

Page 77 © CMA Ontario, 2011

5. The loan amount will be $22,812:

N I/Y PV PMT FV Enter 45 1 632 Compute X

The loan balance after the 32th payment: $7,669

N I/Y PV PMT FV Enter 13 1 632 Compute X

6. Assuming a rate of 5%: Option A – Study Engineering: The present value of the undergraduate studies is $42,551:

N I/Y PV PMT FV Enter 4 5 12000 Compute X

The present value of the two years or practical experience is $15,671 + 18,655 = $34,326. N I/Y PV PMT FV Enter 5 5 20000 Compute X =

15671

N I/Y PV PMT FV Enter 6 5 25000 Compute X =

18655

The present value of the masters’ degree is $20,813

N I/Y PV PMT FV Enter 2 5 15000 Compute X =

27,891

N I/Y PV PMT FV Enter 6 5 27891 Compute X

Page 78: MODULE 3 Financial Management and Management Accounting 1

Module 3 - Financial Management & Management Accounting 1

Page 78 © CMA Ontario, 2011

The present value of the earnings is $381,574

N I/Y PV PMT FV Enter 25 5 40000 Compute X =

563,758

N I/Y PV PMT FV Enter 8 5 563,758 Compute X

The net present value of the engineering option is:

PV of undergraduate studies -$42,551 PV of earnings between undergraduate and graduate studies 34,326 PV of graduate studies (masters) -20,813 PV of post graduate earnings 381,574 $352,536

Option B – Study Accounting:

The present value of the undergraduate studies is $46,097:

N I/Y PV PMT FV Enter 4 5 13000 Compute X

The present value of earnings is $392,056: N I/Y PV PMT FV Enter 30 5 31000 Compute X=

476546

N I/Y PV PMT FV Enter 4 5 476546 Compute X

The net present value of the accounting option is: PV of undergraduate studies -$46,097 PV of post graduate earnings 392,056 $345,959

Study Engineering.

Page 79: MODULE 3 Financial Management and Management Accounting 1

Module 3 - Financial Management & Management Accounting 1

Page 79 © CMA Ontario, 2011

Assuming a rate of 6%: Option A – Study Engineering: The present value of the undergraduate studies is $41,581:

N I/Y PV PMT FV Enter 4 6 12000 Compute X

The present value of the two years or practical experience is $14,945 + 17,624 = $32,569.

N I/Y PV PMT FV Enter 5 6 20000 Compute X =

14945

N I/Y PV PMT FV Enter 6 6 25000 Compute X =

17624

The present value of the masters’ degree is $19,387:

N I/Y PV PMT FV Enter 2 6 15000 Compute X =

27,501

N I/Y PV PMT FV Enter 6 6 27501 Compute X

The present value of the earnings is $320,817:

N I/Y PV PMT FV Enter 25 6 40000 Compute X =

511,334

N I/Y PV PMT FV Enter 8 6 511,334 Compute X

Page 80: MODULE 3 Financial Management and Management Accounting 1

Module 3 - Financial Management & Management Accounting 1

Page 80 © CMA Ontario, 2011

The net present value of the engineering option is:

PV of undergraduate studies -$41,581 PV of earnings between undergraduate and graduate studies 32,569 PV of graduate studies (masters) -19,387 PV of post graduate earnings 320,817 $292,418

Option B – Study Accounting:

The present value of the undergraduate studies is $45,046:

N I/Y PV PMT FV Enter 4 6 13000 Compute X

The present value of earnings is $337,994: N I/Y PV PMT FV Enter 30 6 31000 Compute X=

426710

N I/Y PV PMT FV Enter 4 6 426710 Compute X

The net present value of the accounting option is: PV of undergraduate studies -$45,046 PV of post graduate earnings 337,994 $292,948

At a 6% rate, your brother should study accounting.

Page 81: MODULE 3 Financial Management and Management Accounting 1

Module 3 - Financial Management & Management Accounting 1

Page 81 © CMA Ontario, 2011

7. a. PV of university cost is $29,524: N I/Y PV PMT FV Enter 4 10 15000 Compute X =

47548

N I/Y PV PMT FV Enter 5 10 47548 Compute X The value of the investments for the first 5 years is $12,210 N I/Y PV PMT FV Enter 5 10 2000 Compute X Additional amount they need to save = $29,524 – 12,210 = $17,314 The additional annual savings required is $4,567 – 2,000 = 2,567. N I/Y PV PMT FV Enter 5 10 17314 Compute X =

4,567

b. The savings will have accumulated to $47,549 N I/Y PV PMT FV Enter 5 10 29524 Compute X After the wedding they have $47,549 – 7,000 = $40,549

PV of vacation is $9,947:

N I/Y PV PMT FV Enter 3 10 4000 Compute X Available for graduate school, $40,549 – 9,947 = $30,602 This will accumulate to $40,731 N I/Y PV PMT FV Enter 3 10 30602 Compute X

Page 82: MODULE 3 Financial Management and Management Accounting 1

Module 3 - Financial Management & Management Accounting 1

Page 82 © CMA Ontario, 2011

Problem 3

Cost of Debt = kb = k x (1 - T) = .08 x (1 - .40) = 4.8%

Cost of Preferred Stock = kp = Dp / Pp = $12 / $100 = 12%

Cost of Common Stock = ke = D1/Pe + g = $5.75(1.04)/$50 + .04 = 15.96% We are provided with the book value of the long-term debt and the market values of the preferred and common stock. However, we are not given the number of common shares outstanding. We must therefore use book values as weights in the calculation of the WACC. Proportion of Debt = B/V = $20,000 / $ 100,000 = 20% Proportion of Preferred = P/V = $40,000 / $100,000 = 40% Proportion of Equity = E/V = ($35,000 + $5,000) /$100,000 = 40%

Weighted Average Cost of Capital

= B/V x kb + P/V x kp + E/V x ke = (.20 x.048) + (.40 x .12) + (.40 x .1596) = .0096 + .048 + .0638 = .1214 or 12.14%

The figure of 12.14% should be used to evaluate capital budgeting proposals for DGL providing that the capital structure of the firm remains stable and that the projects represent levels of risk which are consistent with those currently undertaken by the firm.

Problem 4 Cost of Debt = interest cost x (1 - tax rate) = 10% x (1 -.40) = 0.06

Cost of Preferred Shares = dividends / net proceeds from sale of a share = $12/$100 = 0.12

Cost of Equity = (dividends / market price of stock) + dividend growth rate = ($6.75(1.04) / $50) +.04 = 0. 18

Page 83: MODULE 3 Financial Management and Management Accounting 1

Module 3 - Financial Management & Management Accounting 1

Page 83 © CMA Ontario, 2011

In this problem we are given market values for the preferred shares only. We know the current market price for common shares, but are not told the number of common shares outstanding. Therefore, we have no choice but use book value weights.

= B/V x kb + P/V x kp + E/V x ke = (40/200) 6% + (80/200) 12% + (80/200) 18% = 13.2% Problem 5

Cost of debt: 7.5% (.6) = 4.5% Cost of preferred shares = $16.50 ÷ 170 = 9.7% Cost of common stock = D1/P0 + g = ((6.50 x 1.025) ÷ 70) + 2.5% = 12% Market value of capitalization: Current Market Value Proportion Debt $ 22,000,000* 20.4% Preferred shared (45,000,000 ÷ 150 x 170) 51,000,000 47.2% Common equity (500,000 x $70) 35,000,000 32.4% $108,000,000 WACC = (4.5% x 20.4%) + (9.7% x 47.2%) + (12% x 32.4%) = 9.4%

* although the current yield on debt (7.5%) is not equal the debt’s current interest rate of 8.5%, we are not given the debt’s maturity. Therefore, it is assumed that the market value of the debt is equal to its book value. Note that for debt, market values are generally relatively closely aligned to book values. Problem 6

N I/Y PV PMT FV Enter 12 -1220 60 1000 Compute X =

3.70

YTM = 3.7% x 2 = 7.4%

Page 84: MODULE 3 Financial Management and Management Accounting 1

Module 3 - Financial Management & Management Accounting 1

Page 84 © CMA Ontario, 2011

Problem 7

P0 = D1÷ (r - g) r = D1 / P0 + g = 1.10 / 27.50 + .1 = .04 + .1 = 14% Problem 8

Market value of bond issue = $8,620,136:

N I/Y PV PMT FV Enter 24 5 400000 10000000 Compute X

kd = 10%(1 - .35) = 6.5% Market value of preferred shares = 8,000 x $833 = $6,664,000 Kp = $80 / 833 = 9.6% Market value of common shares = 1,200,000 x $13 = $15,600,000 Ke = 4% + 1.3(12% - 4%) = 14.4% WACC = (8,620/30,884)(6.5%) + (6,664/30,884)(9.6%) + (15,600/30,884)(14.4%) = 11.16%

Page 85: MODULE 3 Financial Management and Management Accounting 1

Module 3 - Financial Management & Management Accounting 1

Page 85 © CMA Ontario, 2011

Problem 9

P3 =

D4

r − g=

D3(1+ g)

r − g=

3.00(1.05)

.18 − .05= $24.23

Using the multiple cash flows feature of your discounted at 18% you get: P0 = $18.86

CF1 = $1.00 CF2 = $2.00 CF3 = $3.00 + 24.23 = $27.23 P1 = $21.25

CF1 = $2.00 CF2 = $3.00 + 24.23 = $27.23 P2 = $23.08

CF1 = $3.00 + 24.23 = $27.23

P4 =

D5

r − g=

D4(1+ g)

r − g=

3.15(1.05)

.18 − .05= $25.44

Year

Dividend Yield

Capital Gain Yield

Total Yield

1

D1

P0

=1.0018.86

= 5.3%

P1 −P0

P0

=21.25−18.86

18.86=12.7%

18.0%

2

D2

P1

=2.0021.25

= 9.4%

P2 −P1

P1

=23.08 − 21.25

21.25= 8.6%

18.0%

3

D3

P2

=3.0023.08

=13.0%

P3 −P2

P2

=24.23− 23.08

23.08= 5.0%

18.0%

4

D4

P3

=3.1524.23

=13.0%

P4 −P3

P3

=25.44 − 24.23

24.23= 5.0%

18.0%

Page 86: MODULE 3 Financial Management and Management Accounting 1

Module 3 - Financial Management & Management Accounting 1

Page 86 © CMA Ontario, 2011

Problem 10 a) ke = Rf + B(Rm, - Rf) = .14 + 1.2 (.20 - .14) = 21.2%

b) The CAPM is limited by our inability to obtain accurate measures of various

parameters, particularly beta.

The conceptual accuracy of the model has not been proven. Therefore, it is difficult to be certain that the model actually provides a good description of reality on which investors and firms should base their decisions.

The assumptions underlying the model are most appropriate for short-term analysis.

The model seems to perform better in describing and predicting risk-return relationships for portfolios of stocks rather than for individual securities.

c) It is appropriate to use the weighted average cost of capital for investment

considerations under the following conditions: i) neither investments nor financing methods represent a drastic departure from

past policies. ii) the project's risk is close to the firm's overall operating risk.

Page 87: MODULE 3 Financial Management and Management Accounting 1

Module 3 - Financial Management & Management Accounting 1

Page 87 © CMA Ontario, 2011

Problem 11 a) re = rf + B (rm – rf) = .10 + 1.2 (.16 - .10) = 17.2% b) Cost of Debt = 13% (1-.4) = 7.8% Cost of Preferred Shares = Dp / Pp = .80 / 8.50 = 9.412% Cost of Equity = 18% WACC = .4(7.8%) + .1(9.412%) + .5(18%) = 13.06% c) Market value weights are better than book value weights and hence should be

used since they represent marginal costs and hence are more theoretically correct. Problem 12

End of Present Year Dividend Value @ 12%

1 $2(1.1) = $2.20 $1.964 2 2(1.1)2 = 2.42 1.929 3 2(1.1)3 = 2.66 1.893 4 2(1.1)4 = 2.93 1.862 $7.6480

Dividend at the end of year 5 = $2.93(1.06) = $3.11 Market value at the end of year 4 = D5 = 3.11 = $51.83 k - g .12 - .06 Present value of $51.83:

N = 4, I = 12, FV = 51.83 Solve for PV = 32.94

Present value of one share of Western Oils Ltd. = $7.65 + $32.96 = $40.59 The maximum price that an investor should be willing to pay is $40.59/share.

Page 88: MODULE 3 Financial Management and Management Accounting 1

Module 3 - Financial Management & Management Accounting 1

Page 88 © CMA Ontario, 2011

Problem 13 Current receivables: (1,000,000) x 30/365 = 82,192 Expected receivables outstanding at the end of next year before adjustment to credit policy on sales of $1,100,000: (82,192)(1.10) = 90,411 Change to Policy A-60 days B-90 days C-120 days Incremental sales $300,000 400,000 600,000 Incremental profits (25%) 75,000 100,000 150,000 Level of receivables* 230,137 369,863 558,904 Additional receivables 139,726 279,452 468,493 Opportunity cost (20%) 27,945 55,890 93,699 Excess of incremental profit over opportunity cost 47,055 44,110 56,301 * ($1,100,000 + Incremental Sales) / 365 x Number of Days

The best policy would be C b) Consideration of even longer than 120 days credit terms since the marginal revenue curve is rising beyond policy C.

Consideration of a more liberal credit granting policy since bad debts are zero, indicating possibly too stringent a policy.

Consideration of offering a discount if the benefit is greater than cost.

Page 89: MODULE 3 Financial Management and Management Accounting 1

Module 3 - Financial Management & Management Accounting 1

Page 89 © CMA Ontario, 2011

Problem 14 a. Order

Size Average

Inventory Number of Orders/yr.

Carrying Costs

Ordering Cost

Total Costs

50 25 50.00 $12.38 $337.50 $349.88 75 37.5 33.33 18.56 225.00 243.56 100 50 25.00 24.75 168.75 193.50 150 75 16.67 37.13 112.50 149.63 250 125 10.00 61.88 67.50 129.38 400 200 6.25 99.00 42.19 141.19 600 300 4.17 148.50 28.13 176.63 Average inventory = order size / 2 Number of orders per year = annual demand of 2,500 / order size Carrying costs = average inventory x purchase price of $4.50 x 11% Ordering costs = number of orders per year x $6.75 b. 2(2,500)(6.75)

(4.50)(.11)= 261

Ordering costs = 2,500 / 261 x $6.75 $64.66 Carrying costs = 261 / 2 x $4.50 x 11% 64.60 129.26 Problem 15 a. 2(6,500)(7.50)

0.30= 570

b. Order point = lead time x inventory usage rate = 3 weeks x 125 units a week = 375 units c. Normal order point 375 Safety stock - Usage over increased lead time: 125 units x 2 weeks 250 Usage increase: (200 - 125) x 5 weeks 375 625 Revised order point 1,000

Page 90: MODULE 3 Financial Management and Management Accounting 1

Module 3 - Financial Management & Management Accounting 1

Page 90 © CMA Ontario, 2011

Problem 16 a)

Current Projected Debt (10%) $50,000 $50,000 Equity $50,000 $50,000 Revenue $28,000 $33,600 Variable Costs (50%) 14,000 16,800 Contribution Margin 14,000 16,800 Fixed Costs 7,000 7,000 EBIT 7,000 9,800 Interest ($50,000 x .10) 5,000 5,000 Net Income Before Taxes 2,000 4,800 Taxes (40%) 800 1,920 Net Income After Taxes $1,200 $2,880 Earnings per Share for 5,000 shares $ 0.24 $ 0.58

b) DOL = CM / EBIT = 14,000 / 7,000 = 2.0 DFL = EBIT / (EBIT – I) = 7,000 / (7,000 – 5,000) = 3.5 DTL = 2.0 x 3.5 = 7.0 c) Increase in revenue = 20% Increase in net income after taxes should equal: 7 x 20% = 140% Check: 2,880/1,200 - 1 = 140%

Page 91: MODULE 3 Financial Management and Management Accounting 1

Module 3 - Financial Management & Management Accounting 1

Page 91 © CMA Ontario, 2011

Problem 17 a. DOL = CM / EBIT = (60,000 x .4) / [60,000 x .4) – 12,000] = 24,000 / 12,000 = 2.0 b DFL = EBIT / (EBIT – I) = 12,000 / (12,000 – 6,000) = 2.0 c. DTL = 2.0 x 2.0 = 4.0 d)

Current Projected Debt (12%) $50,000 $50,000 Equity $100,000 $100,000 Revenue $60,000 $90,000 Variable Costs (60%) 36,000 54,000 Contribution Margin 24,000 36,000 Fixed Costs 12,000 12,000 Net Income Before Interest & Taxes 12,000 24,000 Interest Expense (12% x $50,000) 6,000 6,000 Net Income Before Taxes 6,000 18,000 Taxes (35%) 2,100 6,300 Net Income After Taxes $3,900 $11,700 Earnings per Share (10,000 shares) $0.39 $1.17

Problem 18 Alternative 1: Equity EPS = .6EBIT ÷ 1,400,000 Alternative 2: Debt EPS = [(EBIT - 288,000) x .6] ÷ 1,000,000 = (.6 EBIT - 172,800) ÷ 1,000,000 Equate both equations and solve for EBIT: 0.6EBIT = 0.6EBIT – 172,800 1,400,000 1,000,000 600,000EBIT = 840,000EBIT – 241,920,000,000 EBIT = 1,008,000

Page 92: MODULE 3 Financial Management and Management Accounting 1

Module 3 - Financial Management & Management Accounting 1

Page 92 © CMA Ontario, 2011

Problem 19 (a) Cost of Debt = kb = k x (1 - T) = .09 x (1 - .45) = 4.95%

Cost of Preferred Stock = kp = $7/$50 = 14% Cost of Common Stock = ke = D1/NPe + g = $9/$60 +.03 = 18%

Proportion of Debt = B/V = $70,000 / $200,000 = 35% Proportion of Preferred = P/V = $30,000 / $200,000 = 15% Proportion of Equity = E/V = ($80,000 + $20,000) / $200,000 = 50%

Weighted Average Cost of Capital = B/V x kb + P/V x kp + E/V x ke =.35 x .0495 + .15 x .14 + .50 x .18 =.0173 + .021 + .09 =.1283 or 13%

(b)

Initial

Investment

Annual Cash

Inflow

Project

Life

PV of Inflows @ 13%

Net Present

Value Project A $50,000 $10,000 10 $54,262 $4,262 Project B $150,000 $30,000 8 $143,963 $(6,037) Project C $100,000 $30,000 6 $119,926 $19,926

(c) Because the NPV of I Projects A and C are positive, CML should invest in these

projects. If capital is restricted to under $150,000, they should invest in C only. If less than $100,000 can be raised, project A should be undertaken.

Page 93: MODULE 3 Financial Management and Management Accounting 1

Module 3 - Financial Management & Management Accounting 1

Page 93 © CMA Ontario, 2011

Problem 20 (a)

Current Projected Debt (8%) $90,000 $90,000 Equity $110,000 $110,000

Revenue $216,667 $250,000 Variable Costs (40%) 86,667 100,000 Contribution Margin 130,000 150,000 Fixed Costs 70,000 70,000 EBIT 60,000 80,000

Interest ($90,000, x .08) 7,200 7,200 NI Before Taxes 52,800 72,800 Taxes (50%) 26,400 36,400 NI After Taxes $ 26,400 $36,400

Earnings per share for 11,000 shares $2.40 $3.31

DOL = CM / EBIT = 130,000 / 60,000 = 2.17 DFL = EBIT / (EBIT – I) = 60,000 / (60,000 – 7,200) = 1.14 DTL = 2.17 x 1.14 = 2.47

(b) If Dunwood replaced some of its debt with equity, its operating leverage would

remain unchanged as there is no change in the cost structure. With less debt in the capital structure, the degree of financial leverage would fall and so too would the total leverage figure. This would reduce the firm's return to shareholders when profits rose, but offer more protection if sales fell.

Page 94: MODULE 3 Financial Management and Management Accounting 1

Module 3 - Financial Management & Management Accounting 1

Page 94 © CMA Ontario, 2011

Problem 21 The first step is to prepare a partial income statement and cash budget for the upcoming year as follows: Pro Forma Income Statement To determine taxes payable For the Upcoming Year Sales Revenue: Regular sales $50,000,000 MMI contract ($25,000,000 / 100,000 x 40,000) 10,000,000 Total Revenue 60,000,000

Variable Costs: Production (50% x $60,000,000) $30,000,000 Selling & Admin. (7% x $60,000,000) 4,200,000 Total Variable Costs 34,200,000

Contribution Margin 25,800,000

Fixed Costs: Fixed manufacturing overhead

(Total overhead - depreciation) x 1.05 $8,400,000 New overhead related to MMI 750,000 Capital cost allowance 6,950,000 Fixed selling & administration

(4,500,000 x 1.05) + $300,000 5,025,000 Bank Loan Interest

($30,000,000 + $5,000,000) x . 12 4,200,000

Total Fixed Costs 25,325,000

Taxable Income 475,000

Taxes 40% $ 190 000

Page 95: MODULE 3 Financial Management and Management Accounting 1

Module 3 - Financial Management & Management Accounting 1

Page 95 © CMA Ontario, 2011

Cash Budget For the Upcoming Year Opening Balance $ 200,000 Cash Receipts: Collections from previous Accounts Receivable 10,000,000 Collections from current sales ($60,000,000 x 10/1 2) 50,000,000 Bank Loan for new equipment 5,000,000 Total Cash Receipts $65,000,000 Total Cash Available $65,200,000 Cash Disbursements: Purchase of new equipment $ 5,000,000 Accounts payable from previous year 2,736,000 Accounts payable this year ($34,200,000 x .92) 31,464,000 Fixed manufacturing overhead ($8,400,000 + $750,000) 9,150,000 Fixed selling & administrative 5,025,000 Bank loan interest 4,200,000 Original bank loan principal repayment 5,000,000 Dividends 500,000 Income tax expense (see pro forma income statement) 190,000

Total disbursements 63,265,000 Minimum balance required 200,000

Total Cash Needed $63,465,000 Surplus / (Deficit) $1,735,000 Therefore, the total amount available to repay principal on the new $5,000,000 loan would be $1,735,000. This is significant, especially given that 60% of the MMI contract will not be filled until the following year. Some alternative methods and sources of raising cash are as follows: • Decrease the level of inventory. $10 million in inventory is high, given sales of only

$60 million for the year. • Change the credit policy. Englehart should examine the impact of decreasing its

credit terms to 45 or even 30 days. They could also investigate offering a discount for early payment such as "2%, 10 days".

• Stretch accounts payable. 8% of variable cost in accounts payable implies that

Englehart receives credit terms of 30 days (1/12 of a year) from its suppliers. More

Page 96: MODULE 3 Financial Management and Management Accounting 1

Module 3 - Financial Management & Management Accounting 1

Page 96 © CMA Ontario, 2011

cash would be available if the credit terms on accounts payable matched those for accounts receivable.

• Forego paying dividends until the company is more liquid. This may not be a popular

option from the point of view of the shareholders and may cause the market price of Englehart's stock to fall. One alternative would be to reduce the level for the next year or so and explain why to the shareholders.

• Issue more long-term debt. Because the new equipment will presumably last for a

long time, it makes more sense to finance its acquisition with long-term rather than short-term debt.

• Raise additional equity capital by issuing new shares (common or preferred). This

would improve the firm's debt/equity ratio and improve Englehart's future ability to raise more money from the bank. The impact on the current shareholders and their reactions must be considered as well as the current state of the new equity market.

Problem 22 Stretching payments on accounts payable to 60 days: Expected accounts payable : (2 x $250,000) $500,000 Current accounts payable 250,000 Increase in accounts payable $250,000 Expected annual savings from stretching accounts payable 12% x $250,000 $ 30,000 Accounts payable subject to penalty 10% x $500,000 $ 50,000 Expected cost for late payment penalty $50,000 x 24% $ 12,000 Net benefit of stretching accounts payable $30,000 - 12,000 $ 18,000 Therefore, the controller's policy of stretching payments on accounts payable to 60 days is better than the present policy.

Page 97: MODULE 3 Financial Management and Management Accounting 1

Module 3 - Financial Management & Management Accounting 1

Page 97 © CMA Ontario, 2011

However, the company must consider the alternative policy of taking discounts rather than not taking discounts. Amount of payables under the present policy subject to the discount ($250,000)(80%) $200,000 Expected savings from discounts - ($200,000)(2%)(12) $ 48,000 Increased borrowing required - $200,000 x 20/30 $133,333 Cost of increased borrowing - ($133,333)(12%) $ 16,000 Expected benefit from taking discounts : $48,000 - 16,000 $ 32,000 Therefore, taking discounts is preferable to stretching accounts payable or to maintaining the present policy. Problem 23 Incremental CM: $1,000,000 x 25% $250,000 Incremental bad debts: $1,000,000 x 6% (60,000) Incremental discount: Before 0 After: $9,000,000 x 50% x 1% 45,000 (45,000) Incremental working capital - Accounts receivable After: $9,000,000/365 x 45 1,109,589 Before: $8,000,000/365 x 30 657,534 452,055 Inventory 25,000 477,055 x 15% (71,558) Incremental income if change is made $73,442 Recommend that the change be made.

Page 98: MODULE 3 Financial Management and Management Accounting 1

Module 3 - Financial Management & Management Accounting 1

Page 98 © CMA Ontario, 2011

Problem 24 Assume that there are 5 shares outstanding before the rights issue… Firm Value Number of Shares Value per Share Before $200 5 $40.00 After 225 6 $37.50 Value of a right = $40.00 – 37.50 = $2.50 Problem 25 Numbers in bold are given. Firm Value Number of Shares Value per Share Before $21,000,000 (2) 500,000 $42.00 (3) After 24,000,000 (1) 600,000 $40.00

1 600,000 x $40.00 = $24,000,000 2 $24,000,000 – (100,000 x $30) = $24,000,000 – 3,000,000 = $21,000,000 3 $21,000,000 / 500,000 = $42.00

Page 99: MODULE 3 Financial Management and Management Accounting 1

1. Role of the Management Accountant

Learning Objectives

The objective of this lesson is to provide a framework for your management accounting studies in this program. We will explore and exploit this framework in subsequent lessons; therefore, the content of this lesson is primarily definitional. In some ways, this lesson serves as a blueprint of what is coming, not only in your management accounting studies, but also in your other studies as you follow the path to certification.

After completing this lesson you will understand:

1. The scope of management accounting, 2. The role management accounting plays in strategy and decision making, 3. How a well known motivation theory provides a structure for management

accountants to think about performance measurement and designing incentive systems.

4. The meaning of planning and control and how they play a primary role in management accounting.

The Evolution of Management Accounting

The role of management accounting is to support decision making within organizations. This role has three components:

1. Recording and evaluating costs, 2. Developing information to support planning and control systems, 3. Developing, implementing and operating performance measurement and

evaluation systems.

The earliest known applications of management accounting practice involved recording the costs of products or activities. For example, scribes were part of the intellectual class in ancient Egypt and their duties included, among other things, recording wages and materials costs associated with large building projects, particularly the Pharaoh’s. Reminiscent of practice many millennia later, the scribes not only recorded cost information, but provided a commentary on what the costs implied about how well the project was being managed.

Management accounting historians continue to find and document management accounting practice used in organizations in the Early Modern

Period (1600 – 1850). A significant role of the cost accountant in craft operations in England during this period was to record the

cost of raw materials as they were often used strategically as a representation of product quality.

Page 100: MODULE 3 Financial Management and Management Accounting 1

Module 3 - Financial Management & Management Accounting 1

Page 100 © CMA Ontario, 2011

Management accounting practice flourished from 1850 - 1930 as industrialization created the need for information about costs and decision makers recognized the importance of cost control for competitive purposes. For example, Andrew Carnegie, the steel magnate, was famous for focusing on costs reports arguing that profits would take care of themselves if costs were controlled.

The period 1930 – 1980 witnessed the stagnation of management accounting practice as accounting for internal decision makers was subsumed to the process of developing information for financial statements and the evolving regulations regarding external financial reporting practice. The stock market collapse on October 29, 1929 was attributed, in part, to investors making bad investments based on misleading and often fraudulent financial information. This led governments, through enacting security acts, to create the legal requirement for regulated reporting for organizations selling their securities on the exchanges. This focus on rules based accounting for outsiders such as shareholders had the effect of diverting accounting efforts inside organizations from providing information for internal decision making to focusing accounting efforts on meeting new external financial reporting standards.

Increasing global competition in the 1980s created the need for improved decision making information and this, in turn, revived the evolution of management accounting toward its current role. Since the 1980s, management accounting practice has made huge improvements in the areas of cost measurement, aligning performance measurement with strategy and performance evaluation. This period also saw the integration of non-financial information into management accounting tools and practice.

What is evident from its history is the progress of management accounting has been driven by the needs and expectations of decision makers within organizations. Management accounting is only relevant when it is supporting management’s strategic and operational needs.

Management Accounting and Financial Accounting – The Odd Couple

Management accounting serves the needs of decision makers inside organizations, whereas, financial accounting serves a fiduciary role by reporting to shareholders and other interested parties outside the organization. Although both management and financial accounting rely on financial information management accounting and financial accounting have different perspectives as illustrated in the following table:

Management Accounting Financial Accounting Focused on internal decision makers Focused on external stakeholders Driven by decision making requirements within the organization

Driven by generally accepted accounting principles

Few rules Rules (principles) driven Generally forward looking Generally backward looking Generally prospective in nature Usually based on historical costs Includes both financial and non-financial info. Includes only financial information

Page 101: MODULE 3 Financial Management and Management Accounting 1

Module 3 - Financial Management & Management Accounting 1

Page 101 © CMA Ontario, 2011

Given their different perspectives and priorities, it was inevitable the focus on meeting external reporting standards following the stock market failure of 1929 would lead to the decline of management accounting relevance in supporting internal decision making. As previously mentioned, this decline was reversed in the 1980s as increasing global competition required organizations to recognize the importance of having management accounting practice driven by the strategic needs of internal decision makers.

The Mid-20th Century Roles of Management Accounting

In 1954, Herb Simon and two colleagues published a highly influential Controllership Foundation monograph. This monograph identified three primary roles of management accounting evident at that time:

1. Scorekeeping,

2. Attention-directing, and

3. Problem solving.

Scorekeeping refers to the task of recording relevant accounting information. Recording the cost of production is an example of scorekeeping.

Attention-directing refers to organizing recorded data into a format useful in supporting decision making. Producing a report that identifies results and where results differed from plan is an example of attention-directing.

Problem solving

Page 102: MODULE 3 Financial Management and Management Accounting 1

Module 3 - Financial Management & Management Accounting 1

Page 102 © CMA Ontario, 2011

refers to identifying possible causes of measured results and offering suggestions for improvement. Identifying the reason why production costs exceed planned costs and proposing a solution to avoid the cause in the future is an example of problem solving.

This view provides a good reflection of how people saw the role of management accounting between 1950 and 1980.

The problem solving role for management accounting has proved to be controversial, since many people believe it intrudes on management’s decision making responsibilities. However, this problem solving role anticipated the increased part management accountants are playing in supporting decision making in modern organizations.

The Mid-20th Century Roles of Management Accounting in Action

The job cost sheet is possibility the oldest known management accounting tool. The job cost sheet summarizes either the prospective costs of a job (when a bid is being prepared) or the actual costs of the job when the price charged will be a function of the cost. This is the scorekeeping role.

A job cost sheet that is used for an attention-directing role will compare the expected costs on a job with the actual job costs and highlight cost differences (which management accountants call variances).

Finally, the management accountant might identify the suspected cause of a variance and a recommended action, which illustrates the problem solving role.

The Modern Territory of Management Accounting

Recognizing the modern role for management accounting, the Society of Management Accountants of Canada defined the territory of the certified management accountant as:

Certified Management Accountants (CMAs) are professionals who contribute to the greater public good through the efficient and effective utilization of financial and non-financial resources. Each member possesses a unique blend of expertise in the interrelated areas of accounting, management and strategy – the three pillars of the profession. Integration across these three dimensions defines the differentiated territory for CMAs as strategic management accounting professionals.

What is both important and progressive about this definition is it envisions the management accountant as possessing a nexus of skills and insights based on financial, management and strategic knowledge that support informed decision making at all levels of the organization.

Management Accounting and Cost Accounting

Practitioners still use the terms cost accounting and management accounting when referring to management accounting practice. When used this way, cost accounting refers to approaches and methods for developing cost information, such as the cost of a product or unit of service. Management accounting has a broader connotation, which

Page 103: MODULE 3 Financial Management and Management Accounting 1

Module 3 - Financial Management & Management Accounting 1

Page 103 © CMA Ontario, 2011

includes cost accounting and planning and control practices that management accountants have developed. Examples of planning and control tools include: budgets, operations reports and performance measurement systems, all of which you will encounter on your path to certification.

Key Concepts Used in Management Accounting

Accountability:

Accountability is the process of assigning responsibility to managers for the consequences of their decisions. The management accounting approach to accountability is to hold decision makers responsible for the measured results of their decisions.

Decision makers are appropriately held accountable for their actions if the following conditions are met:

1. The decision maker has reasonable (but not necessarily total) control over the outcome of decisions.

2. The decision maker has been given and has accepted responsibility for the outcome of decisions.

3. The decision maker agrees that the measured outcomes reasonably reflect the effect of decisions on the organization.

4. The decision maker has the required skill and knowledge to accept responsibility for the outcome of decisions.

Accountability plays a crucial role in how management accountants think about management and control, as will be seen in subsequent lessons.

Motivation:

Organization psychologists have proposed a number of different theories to explain and predict the behaviour of organization decision makers. The most widely referenced of these theories in management accounting is Victor Vroom’s expectancy theory.

Expectancy theory argues that people are motivated to pursue a result if they believe:

1. There is a predictable relationship between their decisions and measured performance – this is called expectancy in expectancy theory.

2. There is a predictable relationship between measured performance and rewards – this is called instrumentality in expectancy theory.

3. Favourable measured performance will result in a valued reward – this is called valence in expectancy theory.

Vroom argued that motivation is a function of expectancy, instrumentality and valence.

Page 104: MODULE 3 Financial Management and Management Accounting 1

Module 3 - Financial Management & Management Accounting 1

Page 104 © CMA Ontario, 2011

Motivational force = expectancy * instrumentality * valence

The reason for the popularity of expectancy theory in management accounting is it articulates closely with the accountability model and, therefore, provides an important perspective from which management accountants can evaluate the critical management accounting role of developing performance measurement and reward systems.

Two management accounting activities have a critical effect on expectancy:

1. Management accountants must ensure the decision maker’s measured performance reflects all relevant facets of that person’s responsibilities in the organization and the decision maker clearly understands their defined responsibilities in the organization. For example, if production supervisors are told their performance will be evaluated based on their ability to meet production schedules, they might incur unreasonable overtime costs or ship products known to be defective in order to meet production deadlines. All facets of the production decision maker’s responsibility (such as controlling costs, ensuring quality production and meeting shipping deadlines) should be part of the performance measurement and reward structure.

2. Management accountants must ensure the decision maker believes that measured performance is something over which they control. For example, if there are factors beyond the decision maker’s control that can affect the measured consequences of their decision (for example rewards are based on group performance over which the individual decision maker has a small effect), expectancy associated with individual actions is lost. This means management

Page 105: MODULE 3 Financial Management and Management Accounting 1

Module 3 - Financial Management & Management Accounting 1

Page 105 © CMA Ontario, 2011

accountants must take steps to tie performance measures as closely as possible to individual decisions.

Missing these two conditions, expectancy is lost and, therefore, the motivational effectiveness of the performance measurement and reward system.

Management accounting practice has a significant effect on instrumentality. The decision maker must have a clear understanding of how measured performance translates into rewards or instrumentality is lost. For example, if the performance measurement system uses many performance measures and a complicated equation that translates measured performance into a reward, instrumentality will tend to be lost since the individual will be unable to identify the reward consequence of a measured level of performance. Without this instrumentality and, therefore, the motivational effectives of the performance, the measurement and reward system is lost.

Valence is something over which management accounting practice has little control. First, the valence of a particular reward will vary across different people. Some people will value public acknowledgement of a job well done and a certain amount of financial rewards, while others may not. Second, the choice of the types and level of rewards is a management decision and not the decision of the management accountant.

Aligning Individual and Organization Goals

With a theory of motivation in hand, management accountants can consider how the performance measurement and reward systems they design can be used to align organization and individual interests. The key is to reward decisions that create desired organization outcomes. Achieving this alignment through the use of rewarding desired

Page 106: MODULE 3 Financial Management and Management Accounting 1

Module 3 - Financial Management & Management Accounting 1

Page 106 © CMA Ontario, 2011

organization outcomes created by the decision maker is the primary task in the design of performance measurement and reward systems.

Poor incentive systems plague many organizations. For example, sales personnel are often rewarded based on the level of sales they generate. This causes sales personnel to focus on increasing sales without considering which products are the most profitable. Consequences of not focusing on sales profitability include the sales force incurring excessive sales costs to close the sale and promising costly service levels to customers.

The Planning and Control Cycle

Planning refers to the process of identifying the organization’s objectives and then developing and implementing a strategy to achieve those objectives. Control refers to measuring the results of actions associated with the chosen strategy and developing and implementing any action intended to move the organization back onto the trajectory needed to achieve its objectives.

Management accountants usually conjoin the two terms and refer to the process of planning and control, which is usually defined on the organization-wide scale as:

1. Identifying objectives,

2. Developing and implementing a course of action to achieve those objectives,

3. Measuring the progress toward achieving objectives, and

4. Evaluating any variances between planned and actual results and taking corrective actions when the organization is believed to be off course in achieving its objectives.

The management accounting approach to planning and control is reminiscent of the plan, do, check and act process known as the Deming Cycle in quality control literature. Many organizations have used the Deming Cycle to improve quality and reduce quality costs. Like the Deming Cycle, the management accounting approach to planning and control reflects a recursive process, wherein plans, and the strategies underlying those plans, are constantly revisited and revised when deemed necessary, in light of the realized results of the decisions implemented in pursuit of the current strategy. This idea is

central in the CMA Strategic Management model, which you will encounter later in your certification studies.

Page 107: MODULE 3 Financial Management and Management Accounting 1

Module 3 - Financial Management & Management Accounting 1

Page 107 © CMA Ontario, 2011

This planning and control cycle is similar to the notion of closed loop control in engineering. A room thermostat provides a good example of closed loop control. The desired temperature is set (plan) and the temperature management system (heating or air conditioning) moves the room temperature to the desired target (do). The thermometer in the thermostat constantly monitors the room temperature (check) and turns the temperature control system off or on (act) when the target room temperature is achieved or moves a pre-set distance from the target.

If the thermometer fails to record the temperature accurately or the temperature control system fails to respond correctly to temperature or measures, the system will fail to achieve its objectives as will an organization if performance measurement is inaccurate or incomplete and if decision makers fail to respond appropriately to measured performance.

To illustrate the effect of a performance measurement system using this analogy, assume it is important to control not only temperature in the room, but also humidity. If the room control unit fails to monitor humidity (for example large scale baking operations require humidity control systems), control over the environment of the room will be lost. This illustrates the importance of having a performance measurement system that can monitor all elements needed to achieve the control objectives.

Budgets and Variances

Historically, budgets are the most important tools management accountants have used to effect organization control.

Page 108: MODULE 3 Financial Management and Management Accounting 1

Module 3 - Financial Management & Management Accounting 1

Page 108 © CMA Ontario, 2011

Budgets lay out the expected financial consequences of business decisions. Management accountants compare the actual results for the planning period with the budget amounts and focus on the differences, which management accountants call variances, between budget amounts and actual amounts. Variances, therefore, are a form of exception reporting rooted in the accountability perspective wherein the focus is on deviations from budgeted results.

Page 109: MODULE 3 Financial Management and Management Accounting 1

Module 3 - Financial Management & Management Accounting 1

Page 109 © CMA Ontario, 2011

The Planning and Control Cycle and Management Accounting

Kaplan and Norton have proposed an integrated model of the planning and control cycle and management accounting. The interesting feature of this model is it provides a clear vision of how management accounting both supports and enables strategic management in organizations.

The five stages on the boundary of the circle in the attached diagram are steps in the planning and control cycle. Note that these steps reflect the plan (stages 1-3), do (the box at the bottom of the diagram), check (stage 4) and act (stage 5) sequence in the Deming Cycle. The items inside the circle (such as budgets and strategy maps) are the management accounting tools that enable and coordinate the elements of the control cycle. The items listed in the stages boxes of the diagrams (such as improve key processes and conduct profitability analysis) reflect management accounting reports intended to support and identify opportunities for improving performance on the organization’s objectives. We will develop each of these components during your progress toward the CMA designation, so it will be useful for you to keep this diagram in mind as you develop these concepts in order to relate them to their supporting role and place in the planning and control cycle.

Page 110: MODULE 3 Financial Management and Management Accounting 1

Module 3 - Financial Management & Management Accounting 1

Page 110 © CMA Ontario, 2011

Ethics in Government Costing Contracts In September 2008m the United States Department of Justice filed a civil lawsuit against the Boeing Company. The lawsuit states that Boeing failed to disclose it would outsource the manufacturing of the majority of parts needed to create the Towed Decoy System, a tool for intercepting missiles fired at the B-1. The company claimed, according to the suit, it would build the parts in-house at a greater cost than required for outsourcing. Consequently, Boeing charged more for something they outsourced for less and, in effect, defrauded the government for the price difference. An accounting person within Boeing prepared the cost statement for reimbursement by Boeing.

Ethics and the Management Accountant

As a candidate in the various programs leading to the CMA designation, you are bound by the Society of Management Accountants of Ontario Code of Professional Ethics. (http://www.cma-ontario.org/multimedia/Ontario/attachments/ProfessionalMisconductAndCodeOfProfessionalEthics.pdf). For this reason, it is important you read and understand the scope and implications of this Code of Ethics.

There are three critical elements in the code of ethics regulating member behaviour:

1. The duty to respect and maintain the confidence of the member’s employer (or client).

2. The member’s duty to broaden society.

3. The member will only prepare reports the member is competent to prepare and any positions represented reflects the member’s objective and unbiased view.

Management accountants are often subject to pressure to undertake courses of action that violate the Code of Professional Ethics. Some examples are:

1. Book the owner’s personal expenses as organization expenses (violates the broader duty to society in that it creates a tax related fraud).

2. Approve a superior’s travel claim that violates corporate policy (violates the duty to the employer).

3. Overstate expected benefits in a significant capital budgeting exercise being proposed by the accountant’s supervisor (violates the duty to the employer and the requirement to produce an independent and unbiased report)

4. Develop a segment income statement that overstates the profitability of a product or business that is being sold off (violates the duty to society and the requirement to produce an unbiased report).

5. Develop false or misleading information that will be used in financial statements being prepared for an outside entity (violates the duty to society and the requirement to produce an

Page 111: MODULE 3 Financial Management and Management Accounting 1

Module 3 - Financial Management & Management Accounting 1

Page 111 © CMA Ontario, 2011

unbiased report).

6. Prepare a report that overstates the income of an organization or a division to support a higher bonus payment for a superior (violates the duty to the employer and the requirement to produce an independent and unbiased report).

7. Fraudulently add costs to a cost reimbursement contract wherein the contract is reimbursed for costs plus a profit margin. This is one of the most commonly reported episodes of costing fraud and, invariably, relates to costing on government contracts.

Management accountants are often confronted with these requests and the pros and cons of acceding to these requires can be summarized as:

Pros Cons Ingratiate the management accountant with superior(s) in the organization hierarchy Keeping the management accountant’s job Be considered part of the team and worthy of consideration for more senior positions Avoid conflict with the boss

Violates what is felt to be the morally right thing to do Violating the CMA Code of Ethics and the possible loss of designation May result in legal action May lead to a slippery slope where the requests become more egregious May result in blame for the behaviour being deflected from the party proposing the behaviour to the management accountant

Page 112: MODULE 3 Financial Management and Management Accounting 1

Module 3 - Financial Management & Management Accounting 1

Page 112 © CMA Ontario, 2011

Fear as a Motivation for Accounting Fraud The judge in the Livent fraud trial found that Maria Messina, a Chartered Accountant, participated in fraudulent accounting manipulations while she was chief financial officer at Livent. The following summary, which appeared in Canadian Business, June 22, 2008, provides an insight on why Messina, who by all accounts knew what she was doing, was improper, participated in this fraud. Of the cast of characters who have appeared at the criminal fraud trial of Livent Inc. co-founders Garth Drabinsky and Myron Gottlieb, none is as complex or baffling as former chief financial officer Maria Messina. Prosecutors have portrayed her as a courageous whistle-blower who, after finding herself an unwitting participant in the alleged widespread accounting fraud at Livent between 1993 and 1998, spent months struggling to find the fortitude to stand up to Drabinsky and Gottlieb — men Messina describes as “ruthless” and “zealots” in their pursuit of what they wanted. “I did not have the strength to go up against Mr. Drabinsky and Mr. Gottlieb,” Messina testified. “They were men of money, power and influence. I was nobody.”

These represent hard choices since they relate to the management accountant’s viability in the organization.

There are many systems of ethics all of which prescribe either the right thing to do or how to determine the right thing to do. The problem is that we often observe ethical violations such as the ones described above. The question is why these ethical violations occur when what is morally right is obvious. The answer of course is that management accountants are conflicted between the need to remain employed and doing what they know is

ethically right.

Conclusion:

This lesson provides a foundation for topics and concepts you will cover in the remaining management accounting lessons and the strategic management topics you will cover later in your path to your CMA designation. You should keep these foundation ideas in mind, particularly the Kaplan and Norton diagram viewed above, as they will provide a good way to integrate what you will be studying as you move toward accreditation.

Page 113: MODULE 3 Financial Management and Management Accounting 1

Module 3 - Financial Management & Management Accounting 1

Page 113 © CMA Ontario, 2011

Multiple Choice Questions with Solutions:

1. Which of the following highlights a major difference between management accounting and financial accounting? a. Both use accounting information b. Both focus on future costs and revenues c. Both provide information relevant to an organization’s stakeholder d. Both require attention to detail

2. Which of the following is not an example of management accounting scorekeeping,

attention-directing or problem solving activity? a. Recording a marketing cost b. Identifying the amount of a cost variance c. Proposing a solution to reduce excessive marketing costs d. Preparing job cost sheet e. Developing an incentive plan rewarding effective cost control

3. Which of the following is an example of a management accounting problem solving

activity? a. Recording a marketing cost b. Identifying the amount of a cost variance c. Proposing a solution to reduce excessive marketing costs d. Preparing a job cost sheet e. Developing an incentive plan rewarding effective cost control

4. Which of the following is not one of the three pillars of the management accounting

profession as defined by CMA Canada? a. Strategy b. Accounting c. Auditing d. Management

5. Which of the following is not one of the six functional competencies of management

accounting as defined by CMA Canada? a. Strategic management b. Risk management and governance c. Financial management d. Operations management e. Financial reporting

Page 114: MODULE 3 Financial Management and Management Accounting 1

Module 3 - Financial Management & Management Accounting 1

Page 114 © CMA Ontario, 2011

6. Which of the following is not one of the four requirements mentioned in the notes to establish a system of accountability?

a. The decision maker has reasonable (but not necessarily total) control over the outcome of decisions.

b. The decision maker has been given and has accepted responsibility for the outcome of decisions.

c. The decision maker is in a supervisory position d. The decision maker agrees that measured outcomes reasonably reflect the

effect of their decisions on the organization. e. The decision maker has the required skill and knowledge to accept

responsibility for the outcome of their decisions. 7. The relationship between measured performance and rewards in expectancy theory is

called. a. Accountability b. Valence c. Expectancy d. Instrumentality e. Motivational force

8. The key to aligning individual and organization goals is:

a. Ensuring pay equals or exceeds outside opportunities b. Ensuring each person in the organization has a challenging job c. Hiring the right people d. Rewarding performance that promotes organization goal achievement e. Ensuring the incentive plan has a high degree of accountability

9. Management accountants call the difference between a budget amount and an actual

amount: a. An exception b. An accountability failure c. A management issue d. A plan deviation e. None of the above

10. A sufficient reason for a management accountant to depart from the CMA Code of

Professional Ethics is a. Direction by a superior that the course of action is acceptable b. The judgment that the Code of Professional Ethics does not deal adequately

with the circumstances of the situation c. Advice from a colleague d. None of the above e. All of the above

Page 115: MODULE 3 Financial Management and Management Accounting 1

Module 3 - Financial Management & Management Accounting 1

Page 115 © CMA Ontario, 2011

Problems with Solutions:

Problem 1

A production decision maker in a company that produces ice cream was evaluated primarily on his ability to control production costs. The production decision maker argued he should not be responsible for the cost of raw materials, particularly milk and sugar, because the cost of these raw materials, which constitute a significant portion of the total cost of the ice cream, were determined by market factors and, therefore, beyond his control. Was the decision maker correct? Would evaluating the production decision maker based on his ability to control production costs violate expectancy in the Vroom theory? Should the decision maker be accountable for all costs?

Problem 2

In the late 1920s, General Motors instituted a bonus plan. The bonus plan established a bonus pool for senior executives that equalled 10% of profit in excess of a 7% return on capital. The bonus pool share was based primarily on the participant’s annual salary. What do you think of this bonus plan?

Problem 3

The Scanlon plan, developed in the 1930s, provided a bonus pool for blue collar employees. The plan calculation began by computing the difference between actual labour costs and planned labour costs. If positive, this preliminary amount was allocated by formula to the company and to the workers. The amount assigned to workers became the bonus pool. Eligible plan members usually shared equally in the bonus pool. What do you think of this plan?

Problem 4

Many organizations routinely report employee initiatives in the organization newsletter. In effect, the organization rewards superior performance by recognition. What do you think of this as a motivational device?

Page 116: MODULE 3 Financial Management and Management Accounting 1

Module 3 - Financial Management & Management Accounting 1

Page 116 © CMA Ontario, 2011

Problem 5

Bonus rewards paid to production decision makers in a large organization that supplies parts to the automotive industry are based on their ability to control manufacturing costs. The major manufacturing cost is the cost of plastic pellets used to make the parts. The organization has decided all purchasing must be done by the central purchasing department. Because of the increase in the price of crude oil, the cost of plastic pellets has increased dramatically causing manufacturing costs to increase far about budgeted levels. As a consequence, few production decision makers are eligible for bonuses. In response to complaints from the production decision makers about the situation, a senior company executive has argued “these are tough times and everyone has to share the pain of increased costs”. What do you think of this argument?

Problem 6

At the heart of the accountability model is the comparison of a result with a predetermined budget or plan. The resulting difference or variance is used as a measure of the performance of the person charged with responsibility for that result. What do you like and dislike about this approach?

Problem 7

In the period leading up to the banking collapse in 2008, United States mortgage brokers were rewarded based on the value of the mortgages they arranged for clients. Many people have argued that this behaviour led, in part, to the collapse. What might be the basis for this argument?

Problem 8

Many organizations are observing a phenomenon called margin drift. This is a situation where profits are consistently smaller than planned or predicted levels. One of the culprits appears to be sales force personnel who offer price discounts, high after sales service levels, liberal warranty and return options, and expedited or special delivery terms.

Problem 9

You commute to work in a large and congested city by personal automobile. Use the plan, do, check and act sequence to identify a strategy that minimizes your commute time to work, assuming you are committed to commuting by personal automobile.

Page 117: MODULE 3 Financial Management and Management Accounting 1

Module 3 - Financial Management & Management Accounting 1

Page 117 © CMA Ontario, 2011

Problem 10

Consider the planning and control cycle in the context of your personal or family budgeting. Identify each of the considerations you would undertake in the plan, do, check and act planning and control cycle.

Problem 11

You have been instructed by a senior executive in your organization to participate in preparing an accounting report which you believe, without problem, is fraudulent. You also believe that failing to cooperate will result in dismissal. You have contacted a lawyer who believes you will have cause for legal action if you are dismissed, but the likelihood is you will be unemployed for at least six months until you find another job. Because of personal circumstances, you believe being out of work for six months will force you into bankruptcy. What would you do?

Problem 12

Find a description of an accountant who participated in an accounting fraud. Identify the circumstances of the fraud and why you believe the accountant was involved.

Page 118: MODULE 3 Financial Management and Management Accounting 1

Module 3 - Financial Management & Management Accounting 1

Page 118 © CMA Ontario, 2011

Solutions: Multiple Choice Questions 1. b Financial accounting deals primarily (but not exclusively) with historical

costs, management accounting often deals with future or prospective costs.

2. e While developing an incentive plan rewarding effective cost control is an

important management function, it is not one of the three activities envisioned by the taxonomy developed by Simon and his colleagues.

3. c 4. c 5. d 6. c The notion of a decision maker is independent of the person’s level or job

classification in the organization 7. d 8. d While the other items in this list may be important, the key is aligning

individual and organization interests through the incentive plan. 9. e Management accountants call the difference between a budget amount

and an actual amount a variance. 10. d The Code of Professional Ethics does not provide for situational ethics

wherein the management accountant can choose whether or not to follow the Code.

Page 119: MODULE 3 Financial Management and Management Accounting 1

Module 3 - Financial Management & Management Accounting 1

Page 119 © CMA Ontario, 2011

Problem 1

On the surface the decision maker appears to make a good point. Since these costs are based on market forces, the decision maker cannot control the spot price of these resources. Therefore, the expectancy criterion appears to be violated with a consequential loss in motivation. However, some organizations have argued that holding decision makers responsible for these costs will motivate them to take longer term approaches to managing costs through the use of commodity contracts or long-term purchase agreements. The counter argument is it may be too risky to have production decision makers speculating on commodity markets.

Problem 2

This bonus plan and its variations have been used in many organizations for a number of years. On the plus side, it is based on measured performance and rewards increase in measured performance. Therefore, instrumentality seems fine here. The problem is expectancy. The individual decision maker will perceive they contribute only slightly to overall performance and, worse, good performance by some decision makers can be wiped out by poor performance by other decision makers. Overall, many people feel that while measures based on group performance may promote group oriented behaviour, some plan members might relax and let others do the work, increasing the bonus pool in which they share. The counter argument is the assumption that senior level people who participate in the plan are hard workers or they would not have risen to a senior level in the organization. The problem with basing the bonus pool share on salary is it does not reflect whether the individual did a good job. A low performing high wage employee will receive a higher bonus than a high performing lower wage employee.

Problem 3

While this plan and its variations have been used for many years, it suffers the same limitations as does the plan identified in Problem 2. The argument against the individual shirking and relying on others to do the work to create the bonus pool is the self-regulating behaviour of groups in that peer pressure will be exerted on bonus pool members who do not do their fair share in creating the performance resulting in a bonus pool.

Problem 4

This problem addresses the issue of valence in the expectancy theory. Some people will be motivated by non-monetary rewards such as this, while others may not. However, if there is peer pressure not to perform at high levels (this may raise the performance level expected from all employees), this approach may be counterproductive. Overall, it is the

Page 120: MODULE 3 Financial Management and Management Accounting 1

Module 3 - Financial Management & Management Accounting 1

Page 120 © CMA Ontario, 2011

culture of the organization that will be a significant factor in deciding whether this approach will motivate employees.

Problem 5

On the one hand, the argument is the organization cannot afford to pay bonuses in hard times. On the other hand, the argument is if people do a hard job and they cannot control the price of a raw material, they should not suffer from the consequences. Both of these elements are negative in that good performance in other areas of cost control can be wiped out by increased plastics costs, which, by senior management decision, production decision makers cannot control. This has the potential of being a major demotivating force in the organization.

Problem 6

On the one hand, this approach relies on a measurable performance. On the other hand, if the predetermined budget is not deemed reasonable by the employee or if there are factors beyond the employee’s control that can materially affect performance, expectancy is lost.

Problem 7

The culprit is a poor alignment between employee rewards and good organization outcomes. By basing rewards on the dollar value of mortgages with no consideration of risk, mortgage brokers took on customers whose incomes could not support the mortgages they sought. Moreover, there was considerable evidence of mortgage broker fraud relating to inflating the earnings of mortgage applicants and inflating values of homes in order to quality higher mortgages. Overall, the system promoted inappropriate risk taking and fraudulent mortgage application documents.

Problem 8

Sales people are usually rewarded based on the level of sales they generate. Because of this, they are not concerned sales profitability. Offering inducements to customers to buy often creates excessive costs for the organization. These inducements, by increasing sales, provide additional rewards to the sales force since costs do not affect their measured performance.

Page 121: MODULE 3 Financial Management and Management Accounting 1

Module 3 - Financial Management & Management Accounting 1

Page 121 © CMA Ontario, 2011

Problem 9

Presumably, planning would involve identifying alternative routes and the time-based traffic patterns on each route. Doing would involve choosing a route based on the data developed during the planning stage. Checking would involve comparing commute times on the chosen route with the expected times and identifying whether transit time expectations were realized and whether some factors, such as accident rates not considered in the initial plan, are relevant. Finally, act would involve comparing realized results with information about other routes and identifying whether a route change should be implemented.

Problem 10

The approach here would be similar to that in Problem 9 and would relate primarily to estimates of uncontrollable costs such as food or utilities and how these risk elements could or should be incorporated into budgeting.

Problem 11

This is an absolutely personal decision that reflects how each individual would balance the pros and cons of refusing to undertake the required fraud. Anecdotal evidence suggests that most people would delay as long as possible while they were job hunting, but would eventually undertake the fraud if alternative employment was not found, provided the detection risk or detection penalty is not too great.

Problem 12

The purpose of this problem is to provide a basis for discussion of the proposition that most accountants do not want to be dishonest but are forced by the nature of their situation to be dishonest.

Page 122: MODULE 3 Financial Management and Management Accounting 1

Module 3 - Financial Management & Management Accounting 1

Page 122 © CMA Ontario, 2011

2. Cost Classifications

Learning Objectives

Like all professions, management accounting has important and specialized terms that make communications among management accountants about costs more efficient and precise. For management accountants, the term cost is ambiguous. Management accountants have a saying “different costs for different purposes”. This reflects the perspective that management accounting serves decision making needs, which often call for different types of cost calculations.

The objective of this lesson is to equip you with many items in the cost vocabulary that management accountants use. Many of the cost terms this lesson introduces have a special meaning for management accountants that differ from common use. Therefore, a clear understanding of how management accountants use these cost terms provides an important foundation for the lessons that follow.

After completing this lesson you will understand and be able to interpret most of the commonly used management accounting cost terms.

Cost Distinctions Important to Financial Accounting

Financial accountants find the following two cost definitions important:

Product Costs:

Both management accountants and financial accountants call estimates of a product’s costs a product cost. However, there are important differences.

For financial accountants, product cost includes only the cost of manufacturing or acquiring a product.

Management accountants will start with the financial accountants’ definition of product cost and add the cost of promoting, selling, distributing and servicing the product in the customer’s hands if the cost can reasonably be traced to the product.

Period Costs:

Period costs are costs accountants, financial or management do not count as product costs.

Financial accountants treat all non-manufacturing costs such as selling, administrative and research and development as period costs. The important consequence of this is only manufacturing costs are associated with an organization’s inventory.

Management accountants will treat any cost they feel cannot be reasonably traced to a product as a period cost. For example, a management accountant would treat advertising

Page 123: MODULE 3 Financial Management and Management Accounting 1

Module 3 - Financial Management & Management Accounting 1

Page 123 © CMA Ontario, 2011

Cost Classification Matters

Whether a cost is treated as direct or indirect can have important implications in contracts where the supplier is reimbursed for cost plus a profit margin. Contractors frequently argue to have what is clearly an indirect resource, such as a piece of general purpose equipment, treated as a direct resource to the project so that all the resource costs can be claimed for reimbursement.

designed to promote a company’s good name as a period cost, but advertising designed to promote a specific product as a product cost.

For the purpose of the Entrance Examination, you should use the financial accounting definition of product cost since it is the most common in practice.

Costing Terms Critical in Defining the Costing System Design

Cost Object:

A cost object is something the management accountant wishes to estimate a cost. Cost objects include: products (goods or services), product lines, departments, divisions and even entire organizations.

Cost Pool:

A cost pool is an account that accumulates costs.

Direct Cost:

A direct cost is a product cost that can be uniquely and unambiguously associated with a cost object. For example, the cost of glass to make a bottle is part of the direct cost of making the bottle. The cost of a warehouse acquired and used for the exclusive use of one product line is a direct cost to the product line.

A material cost uniquely associated with a cost object is called a direct materials cost. The cost of wood to make a piece of furniture is part of the direct materials cost of the piece of furniture.

Given this definition of direct materials, it would seem the cost of items like glue and stain that are consumed to make the piece of furniture

would also qualify as direct materials costs. However, it would be prohibitively expensive to track the amount of glue and stain used to make each piece of furniture, so management accountants put these costs that tend to vary with some underlying level of activity such as units produced, into a large cost pool and call them variable overhead costs. Management accountants then allocate costs from this cost pool using some activity measure such units produced, machine hours or labour hours. Lesson 3, which discusses job order costing, will deal with the issue of allocating indirect costs in more detail.

Labour uniquely associated with a cost object is called a direct labour cost. For example, the wages of an hourly paid worker operating a piece of production equipment is a direct

Page 124: MODULE 3 Financial Management and Management Accounting 1

Module 3 - Financial Management & Management Accounting 1

Page 124 © CMA Ontario, 2011

cost of the products produced on that equipment. As in the case of difficult to track direct materials costs, difficult to track direct labour costs are assigned to the variable overhead cost pool and allocated to cost objects using some activity measure.

The following is a good two step direct cost test. If the resource to which the cost relates did not exist, or be consumed if the cost object did not exist, and if the cost is material relative to the total direct cost of the product, then the cost is a direct cost.

Indirect Cost:

Any product cost that fails the direct cost test is treated as an indirect cost. The salary of a sales manager is an indirect cost of all products the sales manager handles.

The classification of a cost as direct or indirect will depend on the cost object. If the cost object is a product made in a multi-product factory, the factory supervisor’s salary will be an indirect cost. However, if the cost object is the factory itself, the factory supervisor’s salary will be a direct cost.

Assigned and Allocated Costs:

Management accountants assign direct costs to a cost object and allocate the appropriate indirect costs to a cost object. Lesson 6 will consider rules management accountants use to decide how to allocate indirect costs to cost objects.

Page 125: MODULE 3 Financial Management and Management Accounting 1

Module 3 - Financial Management & Management Accounting 1

Page 125 © CMA Ontario, 2011

Costing System Architecture

With the terms introduced above, the management accountant can describe the

architecture of any cost accounting system. Exhibit 1 illustrates the process.

A cost is first classified as direct or indirect. If the cost is classified as direct, it is assigned (charged) directly to the appropriate cost object. If the cost is classified as indirect, it is assigned to the appropriate indirect cost pool. For example, a cost incurred during a set up might be assigned to an indirect cost pool accumulating all set up related costs. Finally, indirect costs are allocated to the cost objects using some allocation rule. For example, set up costs might be assigned to a cost object based on the number of set ups done to produce the cost object.

Cost Terms Used to Describe and Predict Cost Behaviour

Fixed Cost:

Management accountants call a cost that does not change over the contemplated range of activity, a fixed cost. Most capacity related costs (such as depreciation on factory equipment) and supervisory costs are fixed costs. Fixed costs can be direct or indirect costs. Fixed manufacturing costs are usually called fixed manufacturing overhead.

Page 126: MODULE 3 Financial Management and Management Accounting 1

Module 3 - Financial Management & Management Accounting 1

Page 126 © CMA Ontario, 2011

There are many non-manufacturing fixed costs such as selling and administration fixed costs.

The amount of capacity acquired determines the level of fixed costs associated with a resource. Management accountants call the amount of capacity acquired the driver for the resources fixed cost.

Variable Cost:

A variable cost increases in constant proportion with changes in activity level. The formula for a variable cost is:

Variable cost = variable cost per unit of activity * number of units of activity

Management accountants call the “number of units of activity” the cost driver. Sample variable costs and their cost drivers are:

Variable Cost Item Cost Driver Set up Number of set ups

Factory supplies Number of machine hours Materials Number of units made

Materials handling Number of material moves Manufacturing labour Number of hours worked

Product support Number of products Hospital linen costs Days beds occupied

Page 127: MODULE 3 Financial Management and Management Accounting 1

Module 3 - Financial Management & Management Accounting 1

Page 127 © CMA Ontario, 2011

When management accountants use the term variable cost, they usually mean total variable cost. When they are referring to the variable cost per unit of the cost driver, they usually say variable cost per unit. So, if labour cost increases by $5 for each unit made, the management accountant will usually refer to the $5 as the variable cost per unit.

Note that this equation for variable cost assumes the variable cost per unit of activity remains constant. Therefore, the variable cost chart will always be a straight-line coming out of the origin with a constant slope. So, if the materials cost per unit is $250, the variable cost graph will look as shown in Exhibit 2.

Relevant Range:

Most fixed costs will change if the underlying cost driver changes enough. For example, suppose an organization uses a piece of production equipment that is rented at a cost of $500,000 per year. This production equipment has a capacity of 250,000 units. The cost driver for rental costs is the amount of the production equipment rented, which,

in turn, depends on the planned level of production. So, we can treat the planned level of production as the cost driver for rental costs. Exhibit 3 shows the graph for this machinery’s rental costs for production between 0 and 1,000,000 units. Management accountants call a rental cost like this a step cost.

Page 128: MODULE 3 Financial Management and Management Accounting 1

Module 3 - Financial Management & Management Accounting 1

Page 128 © CMA Ontario, 2011

Suppose management is planning for a production level between 350,000 and 450,000 units. In this case, management would rent two machines and the rental cost would be $1,000,000, which is a fixed cost over the planned range of production of 350,000 to 450,000 units. Management accountants refer to the planned range of the cost driver activity level as the relevant range.

Management accountants use relevant range when they are thinking about variable costs as well. Relevant range is used in settings where the variable cost per unit is not constant.

Suppose, for example, there is a learning effect such that the labour cost per unit begins at $300 and steadily declines at the rate of .5% per unit for the first 150 units and then remains constant as shown by the solid line in Exhibit 4. This cost is curvilinear. What management accountants often do in this situation is restrict their attention to a relevant (planned) range of activity, such as between 100 and 150 units, and make the assumption the variable cost per unit in this range is constant resulting in the variable cost line shown as the dotted line in Exhibit 4.

Page 129: MODULE 3 Financial Management and Management Accounting 1

Module 3 - Financial Management & Management Accounting 1

Page 129 © CMA Ontario, 2011

Cost Definitions Needed in Manufacturing Costing Systems

Cost Flows in Manufacturing Systems:

Exhibit 5 illustrates the flow of costs in manufacturing systems.

The costing process begins when raw materials are entered into production and become work-in-process. The accountant increases the work-in-process inventory account by the costs of the raw materials transferred in and decreases the balance of the raw materials inventory account by the corresponding amount.

As the product is produced more raw materials may be consumed – and their costs are added to the work-in-process account and deducted from the raw materials inventory account. Direct labour costs are added to work-in-process as incurred and variable and fixed manufacturing overhead costs are allocated to the work-in-process increasing the balance of the work-in-process account with corresponding accounting entries to appropriate liability or asset adjustment accounts. When the manufacturing process is complete the management accountant transfers the accumulated costs of the work-in-process to finished goods inventory and reduces the work-in-process by the corresponding amount. When the product is sold the product’s cost is reflected as an expense on the income statement and deducted from the finished goods inventory account.

Page 130: MODULE 3 Financial Management and Management Accounting 1

Module 3 - Financial Management & Management Accounting 1

Page 130 © CMA Ontario, 2011

Prime Costs:

Accountants refer to the direct costs used to manufacture a product as its prime costs. Therefore for most manufacturing operations, prime costs include direct materials and direct labour. Prime costs are particularly important in organizations where a major component of a product’s total cost is material and labour – for example in a craft operation requiring expensive raw materials or in food processing industries. If the organization chooses to track other resources that a product consumes during production – such as factory supplies and electricity – these would also be treated as prime costs.

Conversion Costs:

Accountants refer to all manufacturing costs, other than direct materials costs, as conversion costs since these costs are required to transform direct materials cost into the final product.

Page 131: MODULE 3 Financial Management and Management Accounting 1

Module 3 - Financial Management & Management Accounting 1

Page 131 © CMA Ontario, 2011

Schedule of Costs of Goods Manufactured:

Accountants summarize manufacturing costs with the schedule of cost of goods manufactured. The value of this statement is it provides a comprehensive view of the flow of costs in a manufacturing organization. Exhibit 6 illustrates the format of this statement:

Exhibit 6 Schedule of Costs of Goods Manufactured

Direct materials

Beginning direct materials inventory $150,000 Add: purchases of direct materials 1,450,000

Raw materials available for sale 1,600,000 Deduct: ending direct materials inventory 225,000

Direct materials transferred to work-in-process $1,375,000

Direct labour 750,000

Manufacturing overhead Indirect labour 85,000

Supervisory labour 510,000 Consumables (lubricants, drill bits, etc.) 175,000

Utilities (heat and power) 1,125,000 Depreciation of factory equipment 735,000

Taxes on factory property 125,000

2,755,000

Total manufacturing costs 4,880,000 Add: beginning work-in-process inventory 575,000

5,455,000 Deduct: ending work-in-process inventory 424,000

Cost of goods manufactured $5,031,000

Since the schedule of costs of goods manufactured summarizes manufacturing costs, it is a summary of work-in-process activities. The sum of direct materials, direct labour and various overhead items equals the amount of manufacturing costs incurred this period. These costs are added to the opening balance of work-in-process and ending inventory is deducted to yield the cost of the work-in-process completed and transferred to finished goods inventory this period. For competitive reasons, organizations rarely share the schedule of cost of goods manufactured statement with outsiders.

Page 132: MODULE 3 Financial Management and Management Accounting 1

Module 3 - Financial Management & Management Accounting 1

Page 132 © CMA Ontario, 2011

Note that the sum of the totals in the green and blue areas equals prime cost, in this case $2,125,000 (1,375,000 + 750,000). The sum of the totals in the blue and yellow areas equals conversion costs, in this case $3,505,000 (750,000+2,755,000).

The cost of goods manufactured is not the cost of goods sold. Exhibit 7 identifies how the cost of goods manufactured is used to compute the cost of goods sold.

Exhibit 7 Statement of Costs of Goods Sold

Finished inventory Beginning finished goods inventory inventory $1,175,000 Add: cost of goods manufactured 5,031,000 Goods available for sale 6,206,000 Deduct: ending finished goods inventory 945,000 Cost of goods sold $5,261,000

The cost of goods sold is the amount that is deducted (matched) with the sales for the period to determine the gross margin, which is the amount of revenue left to cover non-manufacturing expenses and provide a profit.

When organizations charge actual costs to work-in-process, they are said to use actual costing. Many organizations simplify the statement of cost of goods manufactured by charging overhead to work-in-process using a predetermined rate, an approach called normal costing. For example, overhead might be charged to work-in-process at the rate of $100 per machine hour worked. Lesson 3

will discuss the normal costing approach in more detail.

Page 133: MODULE 3 Financial Management and Management Accounting 1

Module 3 - Financial Management & Management Accounting 1

Page 133 © CMA Ontario, 2011

Cost Definitions Needed in Retail Costing Systems:

For retail organizations, the process is comparable to what is described above for manufacturing, except the products that are sold are purchased rather than manufactured and, therefore, the accounting is much simpler. Exhibit 8 illustrates cost flow in a retail organization.

For retail organizations, there is no statement that is comparable to the statement of cost of goods manufactured in a manufacturing organization. However, the cost of goods statement would look as shown in Exhibit 9.

Exhibit 9 Cost of Goods Sold Statement

Beginning inventory $2,300,000 Cost of goods purchased Merchandise cost $4,500,000 Transportation and freight-in cost 75,000 Cost of goods purchased 4,575,000 Cost of goods available for sale $6,875,000 Less: cost of goods in ending inventory 1,850,000 Cost of goods sold $5,025,000

Page 134: MODULE 3 Financial Management and Management Accounting 1

Module 3 - Financial Management & Management Accounting 1

Page 134 © CMA Ontario, 2011

Reporting Cost of Goods Sold:

The following is the income statement reported by Research in Motion for the year ended February 28, 2009. Note that the income statement discloses only the total amount of cost of goods sold (labelled as cost of sales) with no details other than showing amounts for hardware and software.

Page 135: MODULE 3 Financial Management and Management Accounting 1

Module 3 - Financial Management & Management Accounting 1

Page 135 © CMA Ontario, 2011

The following is the income statement reported by Canadian Tire for the year ended January 3, 2009. Note in this case that cost of goods sold is not disclosed as a separate item, it is included with the company’s operating expenses.

Cost Definitions Used in Planning and Control

Controllable vs. Non-Controllable Costs:

Accountants have developed the terms controllable and non-controllable costs, which they use when evaluating performance.

These cost terms appear in the context of responsibility accounting wherein a manager is assigned accountability for controlling the costs in an organization unit. An important concept in responsibility accounting is the controllability principle, which argues that managers should not be held accountable for costs they do not control.

For example, the cost of fuel, which in turn is affected by global demand and supply for oil, is a major component of cost in a courier company. Therefore, a manager in a courier company might agree that the cost of fuel is non-controllable and should not be part of the evaluation of how well they control costs.

Some people have argued against the controllability principle, believing it causes people to be reactive rather than proactive in controlling costs. For example, with the expectation of a run-up in fuel prices, the manager might have signed a contract fixing the price of fuel.

Page 136: MODULE 3 Financial Management and Management Accounting 1

Module 3 - Financial Management & Management Accounting 1

Page 136 © CMA Ontario, 2011

The Sunk Cost Fallacy The fallacy that sunk costs are relevant is sometimes referred to as the Concorde fallacy, reflecting the way the supersonic transport jet jointly created by the governments of France and Britain was funded. Even though it was apparent the Concorde would never be profitable, France and England kept pouring more money into the project arguing continued investment was appropriated because they had already invested a great deal of money. The sunk cost phenomenon is widely observed and the irrelevance of a sunk cost is frequently a point of dispute between managers and management accountants.

Discretionary vs. Engineered Costs:

Accountants refer to costs driven by production levels, such as materials costs, labour costs and the cost of machinery as engineered costs. They are engineered in the sense the organization cannot avoid these costs when undertaking production.

Accountants refer to costs subject to periodic budget allocations as discretionary costs. Advertising and research and development costs are two good examples of discretionary costs.

Since there is a cause and effect relationship between some activity level and an engineered cost, for example, we expect materials costs and machinery costs to increase in proportion to the production level, it is relatively straight forward to evaluate how well the organization has controlled engineered costs. The accountant can compare the actual amount of an engineered cost with an estimate of what it should be and decide whether the cost appears to be under control.

The problem with discretionary costs is the cause and effect relationship is hard to estimate. Management accountants struggle with how to evaluate the efficacy of spending on discretionary items such as advertising, maintenance, research and development, employee training and performance rewards since it is difficult to evaluate the effect expenditures in these areas have on organization performance.

Important Cost Concept Used in Decision Making

Opportunity Cost:

An opportunity cost is the benefit forgone when a resource is used for some purpose.

For example, suppose there are 100 hours of machine time available. This machine time can be used to produce 50 units of Product A, which would produce an increment to profit of $1,000 or 75 units of Product B, which would produce an increment to profit of $1,200. The opportunity cost of using the machine time to produce product A is $1,200

and the opportunity cost of using the machine time to produce product B is $1,000. A resource should always be allocated to the opportunity that has the lowest opportunity cost, which in this case, is product B. You can see from this example, and it is true, that maximizing profit or minimizing opportunity cost will result in the same decision.

If there is more capacity of a resource than there are productive opportunities, the

Page 137: MODULE 3 Financial Management and Management Accounting 1

Module 3 - Financial Management & Management Accounting 1

Page 137 © CMA Ontario, 2011

opportunity cost of the resource is zero.

Sunk Cost:

A sunk cost is a previous cost incurred for some resource. Examples of sunk costs include: the price paid for stock, the amount invested to develop a new product, the amount spent on a piece of production equipment and the amount of life lost pursuing a military objective.

Sunk costs are important because they cannot be changed by any decision subsequent to the acquisition. As will be seen in Lesson 10, sunk costs should play no role in evaluating current decisions because they cannot be reversed.

Committed Cost:

A committed cost is cost that is unavoidable for a certain period for time. A contractual lease payment is an example of a committed cost as is depreciation on buildings and equipment. By their nature, decisions creating committed costs create organization risk since they cannot be avoided in periods of financial distress.

Page 138: MODULE 3 Financial Management and Management Accounting 1

Module 3 - Financial Management & Management Accounting 1

Page 138 © CMA Ontario, 2011

Problems with Solutions Multiple Choice Questions 1. Consider a single hard copy of this study guide as a cost object. What would be the

best three labels to classify the relation between this cost object and the following two costs of producing the cost object respectively: (1) the paper and (2) the one-time fee paid to the authors (i.e. not royalties)? a. Direct cost; (2) Variable cost b. Variable cost; (2) Unavoidable cost c. Fixed cost; (2) Variable cost d. Direct cost; (2) Fixed cost e. Avoidable cost; (2) Variable cost

2. Given that a cost has been identified as a variable cost, which of the following

additional descriptions of that cost is incompatible with that identification? a. The cost, in total, does not change with changes in the volume of the cost driver. b. The cost can be traced directly to the cost object. c. The cost, in total, does change with changes in the volume of the cost driver. d. The cost cannot be traced directly to the cost object. e. The cost is not a prime cost.

3. The total direct labour cost of producing 100 units of Product X is $50. The direct

material cost of producing 100 units is perfectly variable and the cost driver is the number of units produced. The cost of the direct material traced to each unit is $1.25. Indirect costs are completely fixed at $75 for the production of 100 units. What are the total conversion costs for 100 units of Product X? a. $275 b. $250 c. $200 d. $175 e. $125

Page 139: MODULE 3 Financial Management and Management Accounting 1

Module 3 - Financial Management & Management Accounting 1

Page 139 © CMA Ontario, 2011

Problem 1

Label each of the following costs as product or period from the perspective of the financial accountant and the management accountant:

Cost Item

Financial Accountant

Management Accountant

1 Factory supplies

2 Raw materials used in production

3 Head office design costs for a specific product

4 Factory supervisory costs

5 Cost to repair a product under warranty

6 Factory machine maintenance

7 Cost of shipping a product to a customer

8 Production worker overtime costs

9 Cost to repair product defects before they are shipped

10 Factory utility costs

Problem 2

Bill is a faculty member in the accounting department in the Faculty of Business at a large university. Bill teaches five courses each year, all in the accounting department. Indicate whether Bill’s salary would be classified as direct or indirect for each of the cost objects.

Cost Object Classification of Bill’s Salary

The university

The faculty

The accounting department

One of the courses Bill teaches

Page 140: MODULE 3 Financial Management and Management Accounting 1

Module 3 - Financial Management & Management Accounting 1

Page 140 © CMA Ontario, 2011

Problem 3 Classify each of the following costs as direct or indirect assuming the cost object is a product. You should make any assumption you feel is necessary to classify the cost.

Cost Item Classification

1 Materials consumed in production

2 Salaried labour

3 Supplies consumed in production

4 Depreciation on factory equipment

5 Cost of packaging

6 Shipping a trailer load of various products

7 Invoicing a customer

8 Cost of a salesperson visiting a customer

9 Cost of repairing a piece of production equipment

10 Supervisory salary

Problem 4

Buddy Company manufactures two products. The following table summarizes the per unit information for each of the two products:

Product A Product B Direct materials used (kilograms) 12 7 Cost per kilogram of direct materials $40 $65 Direct labour hours used 4 6 Cost per direct labour hour $25 $30 Machine hours used 1.5 .8

a. Assume Buddy Company assigns factory overhead costs to the two products at the rate of $90 for each machine hour used to make the product. Use this information to compute the manufacturing cost of each product.

b. Now assume Buddy Company assigns factory overhead costs to the two products at the rate of $25 for each labour hour used to make the product. Use this information to compute the manufacturing cost of each product.

c. What conclusion do you draw from your responses to a) and b)?

Page 141: MODULE 3 Financial Management and Management Accounting 1

Module 3 - Financial Management & Management Accounting 1

Page 141 © CMA Ontario, 2011

Problem 5 Classify each of the following costs as fixed or variable:

Cost Item Classification

Depreciation on factory equipment

Computer costs for an Internet services provider

Supervisory costs in a factory

Electricity in a manufacturing plant

Employee wages in a consultancy

Aviation fuel costs for an airline

Advertising expenditures

Maintenance costs

Factory employee training costs

Shipping costs

Research and development costs

Printing costs for a book publisher

Problem 6

Identify the likely cost driver for each of the following costs.

Cost Cost Driver

Lubricants in an automobile service station

Set up costs in a factory

Travel costs in a consultancy

Airport landing fees for an airline

Variable overhead costs in a highly automated factory

Research and development costs

Variable overhead costs in a diamond cutting factory

Advertising costs

Selling costs

Page 142: MODULE 3 Financial Management and Management Accounting 1

Module 3 - Financial Management & Management Accounting 1

Page 142 © CMA Ontario, 2011

Problem 7

Classify each of the following costs as prime, conversion or other

Cost Classification Commissions paid to the sales force Wages paid to workers Rent paid for factory equipment Steel used in a steel fabrication plant Lubricants for production machines Plant manager salary Wages paid to factory accountant Quality control costs in factory Freight paid for materials purchases

Problem 8

Danny Company manufactures plumbing supplies. The following data has been gleaned from the accounting records for the past year:

1. Beginning raw materials inventory $ 45,000 2. Beginning work-in-process inventory $ 58,500 3. Materials purchased $756,000 4. President’s salary $150,000 5. Head office salaries $650,000 6. Wages paid to direct labour $456,000 7. Wages paid to factory supervisors $234,000 8. Wages paid to indirect labour $ 23,450 9. Advertising and marketing expenses $356,000 10. Electricity used in plant $130,000 11. Overhead supplies used in plant $ 23,500 12. Wages paid to marketing manager $125,000 13. Fuel purchased for delivery trucks $ 35,600 14. Oil to heat factory $ 87,000 15. Depreciation on factory equipment $ 86,000 16. Property taxes for factory $ 32,000 17. Rent paid for head office equipment $ 75,000 18. Rent paid for factory equipment $ 34,000 19. Ending raw materials inventory $ 24,500 20. Ending work-in-process inventory $102,000 Required:

a. Compute the total prime costs for this year b. Compute the total conversion costs for this year c. Prepare a schedule of cost of goods manufactured for this year

Page 143: MODULE 3 Financial Management and Management Accounting 1

Module 3 - Financial Management & Management Accounting 1

Page 143 © CMA Ontario, 2011

Problem 9

Classify each of the following costs as discretionary or engineered:

Cost Classification

Advertising

Maintenance

Tires in an automobile assembly plant

Safety related activities

Quality control in a factory

Commissions paid to sales force

Electricity in an aluminum plant

Factory worker training

Supervisory salaries

Problem 10

Maggie Company sells used cars and currently has 120 cars costing a total of $750,000.

a. If Maggie Company could invest money to earn 15% in a project of comparable risk to its used car operation, what is the opportunity cost of the funds tied up in inventory?

b. Maggie Company recently had an offer to sell the business, cars included, for $1,250,000. What is the opportunity cost of staying in this business?

c. How might the owner of Maggie Company use the number you computed in part b)?

Problem 11

Bill, the accountant mentioned in Question 2, and his wife have owned a home for 30 years, for which they paid $75,000. Recently, Bill discovered the market value of their home, which is mortgage free, is $1,200,000. When he heard this Bill commented: “We better move to the suburbs where homes are much less expensive. The opportunity cost of living here is eating us alive”. What does Bill mean by this?

Page 144: MODULE 3 Financial Management and Management Accounting 1

Module 3 - Financial Management & Management Accounting 1

Page 144 © CMA Ontario, 2011

Problem 12 Which of the following is a sunk or not sunk cost?

Cost Sunk or Not sunk

Season tickets to the opera

Depreciation on factory equipment

Taxes on the factory property

Costs to make warranty repairs

Lease payments

Purchase price of a stock

Prepaid cost for an airline ticket which is non-refundable

Problem 13

A portion of costs incurred by business organizations is designated as direct labour cost. As used in practice, the term ‘direct labour cost’ has a variety of meanings. Unless the meaning intended in a given context is clear, misunderstanding and confusion are likely to ensue. If a user does not understand the elements included in direct labour cost, erroneous interpretations of the numbers may occur and could result in poor management decisions.

The National Association of Accountants has issued a Statement on Management Accounting (SMA) Number 4C, 'Definition and Measurement of Direct Labour Cost,’ to assist management accountants in dealing with problems that may arise in interpreting and understanding direct labour costs. Along with providing a conceptual definition of direct labour cost, this statement describes how direct labour costs should be measured. Measurement of direct labour costs involves two aspects: (1) the quantity of labour effort to be included, that is, the types of hours or other units of time to be counted; and (2) the unit price by which each of these quantities is multiplied to arrive at a monetary cost.

Required:

a. Distinguish between direct labour and indirect labour.

b. Explain why some non-productive labour (e.g. coffee breaks, personal time) is treated as direct labour, while other non-productive labour (e.g. down time, training) is treated as indirect labour.

Page 145: MODULE 3 Financial Management and Management Accounting 1

Module 3 - Financial Management & Management Accounting 1

Page 145 © CMA Ontario, 2011

Problem 14 The following information is available for the Crites Manufacturing Company for the year ended December 31, 2009:

Sales $325,000 Work-in-process, January 1 19,000 Work-in-process, December 31 15,600 Direct materials inventory, January 1 21,400 Direct materials inventory, December 31 19,800 Finished goods inventory, January 36,700 Finished goods inventory, December 31 13,600 Direct materials purchased 32,400 Direct labour 52,000 Supervisory and indirect labour 27,300 Administrative salaries 43,000 Supplies and indirect materials 5,400 Heat, light and power (70% for manufacturing plant) 31,600 Amortization (80% for manufacturing plant) 37,500 Property taxes (75% for manufacturing plant) 9,900 Administrative costs 10,950 Marketing costs 42,600

Required:

Prepare an income statement with a supporting schedule of cost of goods manufactured.

Page 146: MODULE 3 Financial Management and Management Accounting 1

Module 3 - Financial Management & Management Accounting 1

Page 146 © CMA Ontario, 2011

Solutions Multiple Choice Questions 1. d 2. a 3. e

Problem 1

Cost Item

Financial Accountant

Management Accountant

1 Factory supplies Product Product 2 Raw materials used in production Product Product 3 Head office design costs for a specific product Period Product 4 Factory supervisory costs Product Product 5 Cost to repair a product under warranty Period Product 6 Factory machine maintenance Product Product 7 Cost of shipping a product to a customer Period Product 8 Production worker overtime costs Product Product 9 Cost to repair product defects before it is shipped Product Product

10 Factory utility costs Product Product

Problem 2

Cost Object Classification of Bill’s Salary The university Direct The faculty Direct The accounting department Direct One of the courses Bill teaches Indirect

Page 147: MODULE 3 Financial Management and Management Accounting 1

Module 3 - Financial Management & Management Accounting 1

Page 147 © CMA Ontario, 2011

Problem 3 Cost Item Classification 1 Materials consumed in production Direct 2 Salaried labour Direct, if traced to one cost

object, indirect otherwise 3 Supplies consumed in production Direct, if traced to one cost

object, indirect otherwise 4 Depreciation on factory equipment Direct, if the equipment was

acquired for the exclusive use of one cost object, indirect otherwise

5 Cost of packaging Direct, if traced to one cost object, indirect otherwise

6 Shipping a trailer load of various products Indirect 7 Invoicing a customer Direct, if the invoice relates

to one cost object, indirect otherwise

8 Cost of a salesperson visiting a customer Direct, if the visit relates to one cost object, indirect otherwise

9 Cost of repairing a piece of production equipment

Direct, if the production equipment is used only by one cost object, indirect otherwise

10 Supervisory salary Direct, if the supervisor only works for one cost object, indirect otherwise

Page 148: MODULE 3 Financial Management and Management Accounting 1

Module 3 - Financial Management & Management Accounting 1

Page 148 © CMA Ontario, 2011

Problem 4

a) and b)

Buddy Company Product Cost

Materials Labour Overhead

Used (Units)

Cost per unit

Total Used (Hours)

Cost per hour

Total Used (MH)

Rate per MH

Total Total Product

Cost

Product A 12 $40 $480 4 $25 $100 1.5 $90 $135 $715

Product B 7 $65 $455 6 $30 $180 0.8 $90 $72 $707

Materials Labour Overhead

Used (Units)

Cost per unit

Total Used (Hours)

Cost per hour

Total Used (LH)

Rate per LH

Total Total Product

Cost

Product A 12 $40 $480 4 $25 $100 4 $25 $100 $680

Product B 7 $65 $455 6 $30 $180 6 $25 $150 $785

The allocation base (cost driver) used to allocate the overhead cost will have an important effect on the resulting cost calculations, including changing the order of which product costs the most.

Problem 5

Cost Item Classification Depreciation on factory equipment Fixed, unless the depreciation is based

on hours the equipment is used Computer costs for an Internet services provider Fixed Supervisory costs in a factory Variable, if they are based on the

number of direct labour workers, fixed otherwise

Electricity in a manufacturing plant Variable Employee wages in a consultancy Fixed Aviation fuel costs for an airline Mostly fixed once routes have been set Advertising expenditures Fixed Maintenance costs Fixed unless they are based on number

of hours worked Factory employee training costs Fixed Shipping costs Variable Research and development costs Fixed Printing costs for a book publisher Variable based on pages printed

Page 149: MODULE 3 Financial Management and Management Accounting 1

Module 3 - Financial Management & Management Accounting 1

Page 149 © CMA Ontario, 2011

Problem 6 Cost Cost Driver

Lubricants in an automobile service station Mechanic hours worked Set up costs in a factory Number of set ups

Time spent setting up Travel costs in a consultancy Number of jobs

Number of trips Airport landing fees for an airline Number of landings Variable overhead costs in a highly automated factory Machine hours

Number of units produced Research and development costs Number of research hours

Number of researchers Number of research projects

Variable overhead costs in a diamond cutting factory Labour hours Number of diamonds cut

Advertising costs Number of advertisements placed

Selling costs Number of customers Number of visits to customers

Problem 7

Cost Classification Commissions paid to the sales force Other Wages paid to workers Prime, if direct, labour conversion otherwise Rent paid for factory equipment Conversion Steel used in a steel fabrication plant Prime Lubricants for production machines Conversion Plant manager salary Conversion Wages paid to factory accountant Conversion Quality control costs in factory Conversion Freight paid for materials purchases Prime

Page 150: MODULE 3 Financial Management and Management Accounting 1

Module 3 - Financial Management & Management Accounting 1

Page 150 © CMA Ontario, 2011

Problem 8 a. Prime costs = $776,500 + $456,000 = $1,232,500

b. Conversion costs = $456,000 + $649,950 = $1,105,950

c.

Schedule of Costs of Goods Manufactured

Direct materials

Beginning raw materials inventory $45,000

Add: purchases of raw materials 756,000

Raw materials available for sale 801,000

Deduct: ending raw materials inventory 24,500

Raw materials transferred to work-in-process $776,500

Direct labour 456,000

Manufacturing overhead

Indirect labour 23,450

Production supervisor wages 234,000

Plant electricity 130,000

Depreciation on factory equipment 86,000

Overhead supplies 23,500

Factory heating oil 87,000

Rent of factory equipment 34,000

Taxes on factory property 32,000

649,950

Total manufacturing costs 1,882,450

Add: beginning work-in-process inventory 58,500

1,940,950

Deduct: ending work-in-process inventory 102,000

Cost of goods manufactured $1,838,950

Page 151: MODULE 3 Financial Management and Management Accounting 1

Module 3 - Financial Management & Management Accounting 1

Page 151 © CMA Ontario, 2011

Problem 9

Cost Classification Supervisory salaries Advertising

Engineered, if based on constant ratio of supervisor to direct labour, discretionary otherwise Discretionary

Maintenance Engineered, if based on hours worked, discretionary otherwise

Tires in an automobile assembly plant

Engineered

Safety related activities Discretionary Quality control in a factory Engineered, if based on units produced,

discretionary otherwise Commissions paid to sales force Engineered Electricity in an aluminum plant Engineered Factory worker training Discretionary Supervisory salaries Engineered, if based on a constant ratio of

supervisor to direct labour, discretionary otherwise

Problem 10

a. The opportunity cost of the funds tied up in inventory would be $112,500 ($750,000 * 15%). The manager would have to compare this opportunity cost to the selling value of having this amount of inventory on the lot.

b. If the business were sold, the owner could generate a return of $187,500 (1,250,000 * 15%) from the alternative investment, which is the opportunity cost of continuing the investment in this business.

c. The business should provide an annual return of at least $187,500, which is the opportunity cost of the funds tied up in the business.

Problem 11

Suppose Bill could invest the equity he and his wife have in their home at a risk free rate of 10%. The opportunity cost of living in the home is $120,000 (1,200,000 * 10%). This opportunity cost would have to be compared with the cost of living in an alternative accommodation that provides at least the same satisfaction to Bill and his wife.

Page 152: MODULE 3 Financial Management and Management Accounting 1

Module 3 - Financial Management & Management Accounting 1

Page 152 © CMA Ontario, 2011

Problem 12

Cost Sunk or Not sunk Season tickets to the opera Sunk Depreciation on factory equipment Not sunk – the original cost of the

equipment upon which the depreciation is based is sunk depreciation is committed

Taxes on the factory property Not sunk – since if the factory is sold the cost can be avoided

Costs to make warranty repairs Sunk – but unknown at the time of sale since the commitment is to make the repair. Also called a committed cost

Lease payments Sunk, also called a committed cost Purchase price of stock Sunk cost Prepaid cost for an airline ticket, which is non-refundable

Sunk cost

Problem 13

a. Direct labour can be directly traced to a cost object such as a product or service This is accomplished through a source document, such as a time ticket or through automated tracking systems such as touch screens or bar code scanners at a work station. An example of direct labour is the hours spent by an artist in creating a painting.

Indirect labour cannot be easily traced to a particular product or service (or management chooses not to trace directly because of the cost involved in attempting direct tracing and/or the insignificance of the labour cost involved). Often, indirect labour is a common cost applicable to many products. For example, the time involved by an artist in obtaining supplies is an indirect labour cost because supplies are used on more than one painting.

b. Non-productive labour included in direct labour:

Some types of non-productive labour are normal, unavoidable and/or required by law. Examples include break and bathroom times. Thus, the amount of time budgeted (and must be paid for) as direct labour includes productive time directly working on a product or service plus an allocation of this non-productive time.

Non-productive labour classified as indirect labour:

Other types of labour time, such as down and training time, are classified as indirect labour costs because these activities are (potentially) avoidable and not attributable to an individual product.

Page 153: MODULE 3 Financial Management and Management Accounting 1

Module 3 - Financial Management & Management Accounting 1

Page 153 © CMA Ontario, 2011

Problem 14 Crites Manufacturing Company

Schedule of Cost of Goods Manufactured for the year ended December 31, 2005

Direct materials used Direct materials inventory, January 1, 2005 $21,400 Direct materials purchased 32,400 Direct materials inventory, December 31, 2005 (19,800) $34,000 Direct labour 52,000 Manufacturing overhead Supervisory and indirect labour 27,300 Supplies and indirect materials 5,400 Heat light and power ($31,600 x 70%) 22,120 Amortization ($37,500 x 80%) 30,000 Property taxes ($9,900 x 75%) 7,425 92,245 Total manufacturing costs 178,245 Work-in-process, January 1, 2005 19,000 Work-in-process, December 31, 2005 (15,600) Cost of goods manufactured $181,645

Crites Manufacturing Company

Income Statement for the year ended December 31, 2005

Sales $325,000 Cost of goods sold Finished goods inventory, January 1, 2005 $36,700 Cost of goods manufactured 181,645 Finished goods inventory, December 31, 2005 (13,600) 204,745 Gross margin 120,255 Operating expenses Administrative costs 10,950 Administrative salaries 43,000 Marketing costs 42,600 Heat light and power ($31,600 x 30%) 9,480 Amortization ($37,500 x 20%) 7,500 Property taxes ($9,900 x 25%) 2,475 116,005 Operating income $4,250

Page 154: MODULE 3 Financial Management and Management Accounting 1

Module 3 - Financial Management & Management Accounting 1

Page 154 © CMA Ontario, 2011

3. Job Order Costing Learning Objectives This chapter resumes the discussion of job order costing mentioned briefly in Chapter 1. After completing this chapter you will:

1. Be able to describe and explain the components of a job order costing system. 2. Understand the basic issues confronted in designing indirect cost pools and

recognize some of the important considerations in choosing an appropriate design. 3. Understand the role of a cost driver and explain the issues to be considered in

choosing an appropriate cost driver. 4. Understand the role and purpose of actual, normal and standard costing systems. 5. Understand the reason management accountants use variable manufacturing

overhead cost pools and the components of those pools. Overview of job order costing As mentioned in Chapter 1, management accountants use job order costing to accumulate costs associated with a unique cost object. Examples of job order costing systems you may have encountered in practice include:

1. An invoice prepared by your dentist 2. An invoice for an automobile repair 3. An invoice for a custom made apparel 4. A bill prepared in a restaurant when you order from a menu

In a manufacturing environment, a job order costing system is used to accumulate the cost of a custom made product such as an aircraft or a piece of speciality equipment. Contractors to governments will use a job order costing system to accumulate costs and they are reimbursed their costs, plus a profit margin, for the work they do.

In a service environment, such as a consulting engagement, the service provider will accumulate the costs of the engagement, which are primarily labour related costs, as part of the invoice submitted to the client.

When there is a fixed fee contract for a particular job, the contractor will use a job order costing system to accumulate the costs of the job to compare the costs against the revenue received for the job.

Therefore, management accountants use job order costing systems for two purposes:

1. To provide documentation to serve as a basis for charging the customer.

2. To compare the costs of a fixed price contract with the revenues received from the customer in order to evaluate the efficiency in executing the contract.

Page 155: MODULE 3 Financial Management and Management Accounting 1

Module 3 - Financial Management & Management Accounting 1

Page 155 © CMA Ontario, 2011

Components of a Job Costing System:

Chapter 1 provided a sample job order cost sheet, which summarized the elements of a job order costing system. In a manufacturing environment, these elements include:

1. The direct materials and direct labour costs assigned to the order.

2. The manufacturing overhead allocated to the order.

In a manufacturing environment costs are marked-up or incremented to provide a margin to cover administrative costs and provide a return to capital. So, for example, the customer might be billed manufacturing costs plus a mark-up of 20%.

Note that while actual direct materials and actual direct labour costs are assigned (accountants often say traced) to the job, manufacturing overhead is handled differently. Below, we will return to the topic of how management accountants handle manufacturing overhead.

An automobile repair facility provides a good example of a service environment. In an automobile repair facility, the job order cost sheet will include:

1. The direct materials consumed by the repair usually shown on the invoice at the retail price, which includes all costs of acquiring and handling the materials plus a profit margin.

2. The labour hours either allowed for the job or consumed by the job is multiplied by the shop rate for labour. This shop rate will provide for recovery of the mechanic’s wages, overhead in the repair facility and an additional amount to cover administrative costs and a return to capital (profit margin).

The service environment costs flows will be similar to the cost flow in a manufacturing environment except the labour charge and the overhead allocation will be combined. In fact, in a consultancy only, the labour amounts and rates may be shown meaning the labour rates will include a recovery of the costs of materials, labour and overhead. Summary of Job Order Costing Steps:

To summarize, the steps in job order costing include:

1. Identify the cost object. The cost object will be a unique order or service.

2. Assign the direct materials and direct labour costs to the cost object.

3. Allocate manufacturing or service related overhead to the cost object either by using a separate rate or including a component for overhead in the cost of materials or labour.

The idea in job order costing is to estimate the actual cost of completing a unique job to provide a basis for setting the price or to compare the revenue received for a job with the job’s estimated cost.

Page 156: MODULE 3 Financial Management and Management Accounting 1

Module 3 - Financial Management & Management Accounting 1

Page 156 © CMA Ontario, 2011

Actual and Normal Overhead Rates:

As you might suspect from the discussion above, a critical element in job order costing systems is how the management accountant handles manufacturing or service related overhead.

The following discussion relates to manufacturing systems. However, the process for handling service related overhead in service systems is comparable.

The management accountant will accumulate manufacturing overhead costs in various ledger accounts such as: machine depreciation, supervisory salaries, factory property taxes and repairs and maintenance. These costs are then traced to appropriate manufacturing overhead accounts. Finally, the management accountant will allocate manufacturing overhead costs to cost objects using some rate. The following is how the management accountant determines the rate:

Manufacturing overhead rate = budgeted manufacturing overhead costs/denominator activity

What is important to note in the following discussion is the amount of manufacturing overhead charged to a job will equal the manufacturing overhead rate * the denominator activity for the job. Both the numerator and the denominator on the right hand side of the manufacturing overhead rate equation reflect important choices the management accountant makes. It should be noted that in some settings, the rate at which manufacturing overhead will be allocated to a job is specified in the contract between the buyer and seller and, therefore, is not determined in this manner. For example, the industrial technologies office, a department of Industry Canada suggests a manufacturing overhead rate of 65% of allowed direct labour costs on a project. Choosing the Denominator Activity and its Level:

The choice of the denominator activity also reflects the management accountant’s judgment. First, the management accountant must decide on what to use as the denominator activity. Second, the management accountant must decide on the level of the activity.

Management accountants refer to the denominator activity as the cost driver and will choose a cost driver they think best reflects the underlying cost behaviour of manufacturing overhead costs. Therefore, in a craft operation where the production rate is primarily driven by labour, as in an automobile repair facility, the management accountant will choose labour hours as the cost driver. In a machine paced operation, such as producing plastic parts using an injection moulding machine, the management accountant will choose machine hours as the cost driver.

Page 157: MODULE 3 Financial Management and Management Accounting 1

Module 3 - Financial Management & Management Accounting 1

Page 157 © CMA Ontario, 2011

Once the cost driver is chosen, the management accountant must determine the activity level to use in the denominator. If the intended overhead rate is an actual rate, the management accountant will use the actual activity level of the cost driver and the actual level of overhead cost. Using actual values in the numerator and denominator results in a rate that management accountants call an actual rate. Any other combination of numerator and denominator results in what is called a normal or normalized rate.

There are four common choices for the cost driver activity level:

1. The actual activity level of the cost driver

2. The estimated activity level of the cost driver

3. The average activity of the cost driver (across different accounting periods), also called the normal activity level

4. The practical capacity of the cost driver (this is the theoretical capacity level less ordinary losses due to breaks and maintenance and an allowance for breakdowns)

In the past, the most common choice of the cost driver activity level has been the estimated hours of the cost driver. The reason for this choice is it aims to estimate the actual overhead cost incurred for the period. However, more recently, many organizations are starting to use the practical capacity of the cost driver as the cost driver activity level.

An illustration:

Suppose Benny Company, a manufacturer of custom wooden chairs, has estimated its total manufacturing overhead as $800,000 for the upcoming period. Much of the factory is hand paced so the management accountant at Benny Company plans to use direct labour hours as the cost driver for manufacturing overhead costs and has developed the following estimates and their associated overhead rates:

Benny Company

Cost Driver Labour Hours Estimated

Manufacturing Overhead

Manufacturing Overhead Rate

Expected direct labour hours 20,000 $800,000 $40.00

Average direct labour hours 23,000 $800,000 $34.78

Practical capacity direct labour hours 26,000 $800,000 $30.77

Note the important effect the choice of the level of the cost driver has on the manufacturing overhead rate. The chart below shows the effect each of these rates would have on the total amount of manufacturing overhead applied for different levels of direct labour hours actually worked, assuming the actual manufacturing overhead is $80,000.

Page 158: MODULE 3 Financial Management and Management Accounting 1

Module 3 - Financial Management & Management Accounting 1

Page 158 © CMA Ontario, 2011

Each line in this chart is computed by multiplying manufacturing rate computed for that cost driver by the actual hours worked.

Any area between the actual rate line and the other two lines reflects a difference between actual manufacturing overhead and the manufacturing overhead applied.

Arguments Supporting Different Choices of the Cost Driver Activity Level:

The argument for using the estimated value of the cost driver activity level is the intent to allocate the total estimated manufacturing overhead to all jobs. The problem with this approach is it leads to the counter-intuitive idea that costs go up as demand goes down (recall that most manufacturing overhead costs are fixed). This is particularly problematic when pricing is based on the costs allocated to the job since it results in a so-called death spiral since as demand continues to fall, the cost per unit continues to increase, causing the price to increase if pricing is cost based. The increased price causes demand to fall further and the cycle repeats.

The argument for using the average value of the cost driver activity level is that this is likely the amount used in the capital budgeting exercise supporting the acquisition of total capacity. The problem with this approach is it does not reflect periodic demand and supply conditions. However, many management accountants feel this choice provides a good basis for costing since it provides a steady estimate of the cost of a capacity resource, which, if factored into pricing, reflects a best estimate of the resource cost that needs to be recovered over the long-term.

The argument for using the practical capacity is that manufacturing overhead cost is primarily fixed and, therefore, the cost driver for fixed costs is the amount of capacity required and not the amount of capacity used. While this approach provides a constant cost estimate, it does not provide a basis for the expected recovery of fixed costs if pricing is cost based. Advocates of this approach argue that the important insight of this approach is it provides a constant reminder to managers that there is idle capacity and,

Page 159: MODULE 3 Financial Management and Management Accounting 1

Module 3 - Financial Management & Management Accounting 1

Page 159 © CMA Ontario, 2011

therefore, a motivation to find additional work and would argue that this approach to costing should not be used as a basis for pricing decisions.

The choice of the cost driver activity level will reflect the management accountant’s costing objectives and is, therefore, context specific.

Continuing the discussion of Benny Company, suppose the management accountant has decided to use the average activity rate of $34.78 to allocate the manufacturing overhead costs to each product.

The following is the data relating to the last three chairs that were made:

Chair Item Cutting

Department Assembly

Department Finishing

Department

Job 2945

Material cost $95.00 $3.00 $6.00

Labour hourly rate $18.00 $12.00 $15.00

Labour hours 3 5 1

Machine hours 3 1 0.5

Job 2946

Material cost $60.00 $3.00 $4.00

Labour hourly rate $18.00 $12.00 $15.00

Labour hours 2 4.5 1

Machine hours 1.5 1 0.5

Job 2947

Material cost $45.00 $3.00 $4.00

Labour hourly rate $18.00 $12.00 $15.00

Labour hours 2 4 1

Machine hours 1 1 0.5

With this information, we can compute the total manufacturing cost of the three chairs using the single plant wide rate of $34.78 per direct labour hour. When all manufacturing overhead costs are accumulated in one overhead cost pool for allocation to the cost objects, in this case jobs, the organization is said to be using a plant wide rate. The following are cost estimates for each job using the plant wide rate of $34.787 per direct labour hour.

Page 160: MODULE 3 Financial Management and Management Accounting 1

Module 3 - Financial Management & Management Accounting 1

Page 160 © CMA Ontario, 2011

Chair Item Cutting

Department Assembly

Department Finishing

Department Total

Job 2945

Material cost $95.00 $3.00 $6.00

Total material cost $104.00

Labour hourly rate $18.00 $12.00 $15.00

Direct labour hours 3 5 1

Total direct labour cost $129.00

Machine hours 3 1 0.5

Manufacturing overhead allocation

Cost driver quantity 3 5 1

Cost driver rate $34.78 $34.78 $34.78

Total $313.04

Total manufacturing costs $546.04

Job 2946

Material cost $60.00 $3.00 $4.00

Total material cost $67.00

Labour hourly rate $18.00 $12.00 $15.00

Labour hours 2 4.5 1

Total direct labour cost $105.00

Machine hours 1.5 1 0.5

Manufacturing overhead allocation

Cost driver quantity 2 4.5 1

Cost driver rate $34.78 $34.78 $34.78

Total $260.87

Total manufacturing costs $432.87

Job 2947

Material cost $45.00 $3.00 $4.00

Total material cost $52.00

Labour hourly rate $18.00 $12.00 $15.00

Labour hours 2 4 1

Total direct labour cost $99.00

Machine hours 1 1 0.5

Manufacturing overhead allocation

Cost driver quantity 2 4 1

Cost driver rate $34.78 $34.78 $34.78

Total $243.48

Total manufacturing costs $394.48

Multiple Overhead Cost Pools and System Design:

To this point, we assumed the management accountant uses a single cost pool from which costs are allocated to jobs. In fact, most organizations use multiple manufacturing overhead costs pools when allocating manufacturing overhead costs to jobs.

Suppose there are three departments in Benny Company: cutting, assembly and finishing. The management accountant has decided to use the average activity level in each of the departments to choose the cost driver activity level.

In the cutting department, pieces of wood are cut or shaped to the requirements for each job using computer controlled saws and lathes. The estimated manufacturing overhead

Page 161: MODULE 3 Financial Management and Management Accounting 1

Module 3 - Financial Management & Management Accounting 1

Page 161 © CMA Ontario, 2011

costs in the cutting department are $400,000. Since manufacturing overhead costs in the cutting department are thought to be driven by machine capacity, the management accountant plans to use machine hours as the cost driver to choose the manufacturing overhead rate for this department. The average activity level for this department is 5,000 machine hours.

The assembly department assembles pieces from the cutting department to build the furniture. The estimated manufacturing overhead costs in the assembly department are $250,000. Since manufacturing overhead costs in the assembly department are thought to be driven by direct labour hours, the management accountant plans to use direct labour hours as the cost driver to choose the manufacturing overhead rate for this department. The average activity level for this department is 12,000 direct labour hours.

The finishing department sands the chairs and provides a finish. The estimated manufacturing overhead costs in the finishing department are $150,000. While the chairs vary widely in terms of design intricacy, they are all thought to require the same amount of finishing, therefore, the management account plans to use the number of chairs as the cost driver to choose the manufacturing overhead rate for this department. The average activity level for this department is 7,000 chairs.

The following is a summary of the attributes of the three departments:

Benny Company

Department Cost Driver Cost Driver

Average Activity Level

Estimated Manufacturing

Overhead

Manufacturing Overhead Rate

Cutting Machine hours 5,000 $400,000 $80.00

Assembly Direct labour hours 12,000 $250,000 $20.83

Finishing Number of chairs 7,000 $150,000 $21.43

$800,000

With this information we can compute the total manufacturing cost of the three jobs using each of the three departmental rates and their associated cost drivers. When the management accountant uses an overhead cost pool for each department and then uses an overhead rate for each department, the costing system is said to be using departmental rates to allocate manufacturing overhead to cost objects, which, in this case, are the individual jobs.

Page 162: MODULE 3 Financial Management and Management Accounting 1

Module 3 - Financial Management & Management Accounting 1

Page 162 © CMA Ontario, 2011

Chair Item Cutting

Department Assembly

Department Finishing

Department Total

Job 2945

Material cost $95.00 $3.00 $6.00

Total material cost $104.00

Labour hourly rate $18.00 $12.00 $15.00

Direct labour hours 3 5 1

Total direct labour cost $129.00

Machine hours 3 1 0.5

Manufacturing overhead allocation

Cost driver quantity 3 5 1

Cost driver rate $80.00 $20.83 $21.43

Total $365.60

Total manufacturing costs $598.60

Job 2946

Material cost $60.00 $3.00 $4.00

Total material cost $67.00

Labour hourly rate $18.00 $12.00 $15.00

Labour hours 2 4.5 1

Total direct labour cost $105.00

Machine hours 1.5 1 0.5

Manufacturing overhead allocation

Cost driver quantity 1.5 4.5 1

Cost driver rate $80.00 $20.83 $21.43

Total $235.18

Total manufacturing costs $407.18

Job 2947

Material cost $45.00 $3.00 $4.00

Total material cost $52.00

Labour hourly rate $18.00 $12.00 $15.00

Labour hours 2 4 1

Total direct labour cost $99.00

Machine hours 1 1 0.5

Manufacturing overhead allocation

Cost driver quantity 1 4 1

Cost driver rate $80.00 $20.83 $21.43

Total $184.76

Total manufacturing costs $335.76

The following is a summary of what we have done so far:

Job 2945 Job 2946 Job 2947

Cost Driver Rate

Cost Driver Units

Overhead Cost Allocated

Cost Driver Units

Overhead Cost Allocated

Cost Driver Units

Overhead Cost Allocated

Using a plant wide rate From cutting $34.78 3.00 $104.35 2.00 $69.57 2.00 $69.57 From assembly $34.78 5.00 173.91 4.50 156.52 4.00 139.13 From finishing $34.78 1.00 34.78 1.00 34.78 1.00 34.78 Total $313.04 $260.87 $243.48 Using department rates From cutting $80.00 3.00 $240.00 1.50 $120.00 1.00 $80.00 From assembly $20.83 5.00 104.17 4.50 93.75 4.00 83.33 From finishing $21.43 1.00 21.43 1.00 21.43 1.00 21.43 Total $365.60 $235.18 $184.76

Page 163: MODULE 3 Financial Management and Management Accounting 1

Module 3 - Financial Management & Management Accounting 1

Page 163 © CMA Ontario, 2011

Note the large differences in manufacturing overhead cost allocations from the three departments to the three jobs under the plant wide rate and departmental rate approaches. A major part of the distortion occurs in the allocation of the cutting department manufacturing overhead, which is an expensive cost item. The overhead cost allocations from the cutting department to Job 2945 more than doubles and to Job 2946 almost doubles. This type of costing distortion occurs when a high cost resource, in the case the cutting department, is used in different proportions by different jobs and the overhead cost pool design does not reflect this difference, which is what happens with the plant wide rate. In fact, what has happened is when the plant wide rate is used, the two jobs that use relatively less time in the expensive cutting department are picking up more than their share of the costs in the cutting department.

The following table summarizes the calculation of the total job costs under the two systems:

Chair Item Plant Wide Rate Departmental

Rates

Job 2945 Total material cost $104.00 $104.00 Total direct labour cost 129.00 129.00 Manufacturing overhead allocation 313.04 365.60

Total manufacturing costs $546.04 $598.60

Job 2946 Total material cost $67.00 $67.00 Total direct labour cost 105.00 105.00 Manufacturing overhead allocation 260.87 235.18

Total manufacturing costs $432.87 $407.18

Job 2947 Total material cost $52.00 $52.00 Total direct labour cost 99.00 99.00 Manufacturing overhead allocation 243.48 184.76

Total manufacturing costs $394.48 $335.76

You should study this example carefully since it illustrates the goal management accountants are seeking when they design manufacturing overhead cost pools, which is the cost allocations should, to the extent possible, reflect cause and effect relationships between the cost objects and manufacturing overhead.

This insight leads to two principles that management accountants seek to apply when designing the system of overhead cost pools.

Page 164: MODULE 3 Financial Management and Management Accounting 1

Module 3 - Financial Management & Management Accounting 1

Page 164 © CMA Ontario, 2011

1. To the extent possible, avoid mixing manufacturing overhead costs that have different cost drivers in the same cost pool.

2. Use the activity base thought to explain the long-term behaviour of costs in the cost pool as the cost driver to allocate the costs in that pool to cost objects.

We will return to these two principles and discuss them in more detail in Chapter 6 where we will consider activity based costing.

The costs of maintaining overhead costing systems are thought to rise exponentially in the amount of cost pools used. Therefore, the management accountant must ensure the potential benefits from the expected increase in costing accuracy by increasing the number of overhead cost pools exceed the additional costs of the more complex overhead costing systems.

Over and Under Applied Manufacturing Overhead:

We now turn to consider a few underlying details of manufacturing overhead accounting. Initially, all factory-related expenses are recorded in ledger accounts that reflect the nature of the expenses such as supervisory salaries, incidental supplies, electricity, heating, real estate taxes and depreciation on the factory building and factory equipment. These accounts will have debit balances.

These costs are then transferred to the manufacturing overhead cost pools that reflect the costing system design. In the example above, there would be three manufacturing overhead cost pools each containing the actual manufacturing overhead costs for each of the three departments. The actual accounting transaction is to debit the manufacturing overhead account and credit the expense account for the expense reflecting the tracing of the expense to the appropriate manufacturing overhead cost pool.

To illustrate, suppose Benny Company was billed $110,000 for electricity. The relevant journal entry would be to credit accounts payable for $110,000 and debit the electricity expense account $110,000. Suppose the factory accountant determined that $56,000 of the electricity was consumed by the cutting department. The appropriate journal entry would be to credit the electricity expense account by $56,000 and debit the cutting department overhead account by $56,000. The same procedure would be followed for other manufacturing overhead items in order to accumulate total manufacturing overhead costs for each department.

As manufacturing overhead is allocated to each job, the manufacturing overhead account is credited by the allocated amount and debited to the job account, which is an element of the total work-in-process account.

If there is a credit (debit) balance in a manufacturing overhead account at the end of the accounting period it means the amount of manufacturing overhead allocated to jobs exceeds (is less) the amount of actual manufacturing overhead incurred and manufacturing overhead is said to be over- or under-applied.

Page 165: MODULE 3 Financial Management and Management Accounting 1

Module 3 - Financial Management & Management Accounting 1

Page 165 © CMA Ontario, 2011

Management accountants have two approaches for dealing with over- or under-applied overhead for financial reporting purposes. If the amount is material relative to total overhead costs the over- or under-applied overhead is prorated among work-in-process, finished goods inventory and cost of goods sold in an attempt to estimate the actual manufacturing costs incurred during the period. Otherwise, of the amount of over- or under-applied overhead is immaterial, it is simply charged to cost of goods sold. Under- or over-applied overhead will increase (decrease) the reported cost of goods sold.

Dealing with Over/Under Applied Overhead – An Example:

We will illustrate three ways a management accountant can handle over-applied manufacturing overhead using the following example. The treatment of under-applied overhead would be comparable.

At the beginning of the current period Jack Company showed the following account balances:

Work-in-process $50,000 Finished goods $100,000

Jack Company planned to allocate manufacturing overhead using direct labour costs and used the following formula to compute the manufacturing overhead rate per direct labour dollar: Budgeted manufacturing overhead/budgeted labour cost = $300,000/$150,000 = $2 per direct labour dollar The following summarizes cost activities during the current period:

Materials added to work-in-process $100,000 Labour applied to work-in-process $150,000 Manufacturing overhead applied to work-in-process $300,000 Actual manufacturing overhead $280,000 Cost of goods completed and transferred to finished goods $500,000 Cost of goods sold $450,000

Manufacturing overhead has been over-applied by $20,000 ($300,000 - $280,000). This means the manufacturing overhead account would have a credit balance of $20,000. Recalling that over-applied overhead means costs have been overstated, the following are the three approaches the management accountant might follow to handle the over-applied overhead. Method 1 – direct charge to cost of goods sold: In this approach, cost of goods sold would be reduced by $20,000. Therefore, the cost of goods sold for the period would be reported as $430,000. The journal entry the

Page 166: MODULE 3 Financial Management and Management Accounting 1

Module 3 - Financial Management & Management Accounting 1

Page 166 © CMA Ontario, 2011

management accountant would use to accomplish this would be to debit the manufacturing overhead account by $20,000 and credit cost of goods sold by $20,000. Note that if the entity is IFRS compliant, this is the only method that is acceptable for financial reporting purposes. Method 2 – proration based on ending balances: In this approach, the management accountant adjusts the work-in-process, finished goods inventory and cost of goods sold on a prorated basis for the over-applied overhead. The following is a summary of how this approach would work and the resulting balances in each account. Recall that a positive balance in these accounts is a debit balance.

Method 3 – adjustment of applied manufacturing overhead: This approach traces the distribution of applied manufacturing overhead and bases the proration on the balance of applied manufacturing overhead in the three accounts. So the key in this approach is to follow the applied manufacturing overhead. The following exhibit shows the process of the analysis.

Page 167: MODULE 3 Financial Management and Management Accounting 1

Module 3 - Financial Management & Management Accounting 1

Page 167 © CMA Ontario, 2011

Begin by identifying the distribution of current costs

Item Amount % of Total Materials costs 100,000 18.18% Labour costs 150,000 27.27% Manufacturing overhead applied 300,000 54.55% 550,000 100.00% Use these proportions to infer the distribution of costs in each account after reducing cost of goods sold by all opening inventories Ending balances Work-in-process Materials 18,181.82 Labour 27,272.73 Manufacturing overhead applied 54,545.45 100,000 Finished goods Materials 27,272.73 Labour 40,909.09 Manufacturing overhead applied 81,818.18 150,000 Cost of goods sold Materials 54,545.45 Labour 81,818.18 Manufacturing overhead applied 163,636.36 300,000 550,000 Now prorate the overapplied overhead to the initial balances of manufacturing overhead applied

Manufacturing Overhead Initial

Balance % of Total Proration

Ending Balance

Work-in-process 54,545.45 18.18% 3,636.36 50,909.09 Finished goods 81,818.18 27.27% 5,454.55 76,363.64 Cost of goods sold 163,636.36 54.55% 10,909.09 152,727.27 Total 300,000.00 20,000.00 280,000.00 Now recast the balances in the three accounts with the adjusted applied manufacturing overhead and add back the opening inventories to cost of goods sold Ending balances Work-in-process Materials 18,181.82 Labour 27,272.73 Manufacturing overhead applied 50,909.09 96,363.64 Finished goods Materials 27,272.73 Labour 40,909.09 Manufacturing overhead applied 76,363.64 144,545.45 Cost of goods sold Opening inventories 150,000.00 Materials 54,545.45 Labour 81,818.18 Manufacturing overhead applied 152,727.27 439,090.91 680,000.00

Page 168: MODULE 3 Financial Management and Management Accounting 1

Module 3 - Financial Management & Management Accounting 1

Page 168 © CMA Ontario, 2011

Highlighting Idle Capacity Costs:

For internal reporting purposes, the management accountant may take an intermediate step in reporting under- or over-applied overhead. Recall that if the management accountant uses practical capacity as the activity level for the cost driver, manufacturing overhead costs will always be under-applied. Management accountants following the approach of using the practical capacity as the denominator activity level will report the under-allocated cost internally as the cost of idle capacity before prorating the under-applied manufacturing overhead to the work-in-process, finished goods inventory and cost of goods sold.

To illustrate, return to the cutting department at Benny Company. Recall that the estimated manufacturing overhead was $400,000 and suppose the practical capacity of the cutting department is 6,000 machine hours. If practical capacity were used to compute the manufacturing overhead rate, the rate would be $66.67 (400,000/6,000) per machine hour. The following is a summary of what might be reported if the actual machine hours recorded were 4,500 hours and the actual manufacturing overhead was $425,000.

Cutting Department Manufacturing Overhead Cost Summary

Actual manufacturing overhead cost $425,000.00

Planned operations

Expected manufacturing overhead cost $400,000.00 Practical capacity 6,000

Manufacturing overhead rate $66.67 Expected capacity use 5,000

Planned manufacturing overhead allocation 333,333.33

Planned idle capacity cost $66,666.67 Actual capacity use 4,500

Actual manufacturing overhead allocation 300,000.00

Under-applied manufacturing overhead $125,000.00

Summary of under-applied overhead

Planned idle capacity cost $66,666.67 Unplanned idle capacity cost 33,333.33

Manufacturing overhead excess spending 25,000.00

Total under-applied manufacturing overhead $125,000.00

Variable Manufacturing Overhead:

The remaining topic for this chapter is the source and nature of variable manufacturing overhead.

Page 169: MODULE 3 Financial Management and Management Accounting 1

Module 3 - Financial Management & Management Accounting 1

Page 169 © CMA Ontario, 2011

You may recall that Chapter 2 mentioned some costs, not easily traceable to cost objects and not material, were treated as variable overhead costs.

We will return to Benny Company to illustrate this idea. Suppose a study was undertaken of a group of workers in the assembly department to identify items that were too difficult to track as direct costs, but appeared to increase in proportion to the number of direct labour hours worked. This study comprised 50 worker hours and identified the following items that were used by the workers during the 50 hours.

Benny Company

Assembly Department Study

Item Units Used Cost per Unit Total Cost

Indirect labour (helpers) 23 $12.22 $281.06 Electricity 235 5.56 1,306.60

Wood screws 57 8.55 487.35 Other fasteners 12 14.55 174.60

Glue 45 8.99 404.55 Drill attachments 154 3.12 480.48

Sandpaper 445 0.02 8.90 Knife blades 122 0.05 6.10

$3,149.64 Direct labour hours worked 50

Variable overhead cost per direct labour hour $62.99

This information would form the basis for identifying variable manufacturing overhead, which was discussed in Chapter 2. The costs for these items would be added to a general manufacturing overhead cost pool and would be treated as indirect rather than direct costs. This information would be used only indirectly in job order costing, but would be important in modelling cost behaviour, a topic discussed in Chapter 6.

The Accounting Entries Underlying Job Order Costing:

For the most part, we have ignored discussion of accounting journal entries needed to support manufacturing costing in general and job order costing in particular. This section provides a summary of the required accounting needed to support manufacturing costing by way of the following example.

Valani Corporation The Valani Corporation manufactures made-to-order widgets and uses a normal job order costing system. During the month of August, the following jobs were worked on and the following costs incurred:

Page 170: MODULE 3 Financial Management and Management Accounting 1

Module 3 - Financial Management & Management Accounting 1

Page 170 © CMA Ontario, 2011

Job A4 Job A5 Job A6 Job A7 Total Balance in opening Work-in-process

$5,600

$7,800

-

-

$13,400

Costs added during August Direct materials 10,000 20,000 $16,000 $4,000 50,000 Direct labour 5,000 4,000 3,000 1,500 13,500

Job status at end of August Sold Complete In FG

Inventory

Complete In FG

Inventory

In WIP Inventory

Valani’s controller budgeted total overhead costs to be $600,000 and budgeted direct labour cost to be $150,000. Overhead is allocated on the basis of direct labour cost. The company marks up their jobs at 40% over cost. There were no opening finished goods inventories.

Required:

Prepare all journal entries relative to these jobs for the month of August.

Page 171: MODULE 3 Financial Management and Management Accounting 1

Module 3 - Financial Management & Management Accounting 1

Page 171 © CMA Ontario, 2011

Solution:

WIP $50,000 Direct materials inventory $50,000 To record the issue of direct materials to production. WIP 13,500 Wages payable or Cash 13,500 To record direct labour. WIP ($13,500 x 4) 54,000 Manufacturing overhead 54,000 To record the application of manufacturing overhead. POR = $600,000 / $150,000 = 4 Finished goods inventory 119,400 WIP 119,400 To record the completion of Jobs A4, A5 and A6. Job A4: $5,600 OP WIP + 10,000 DM + 5,000 DL + 5,000*4 OH $40,600 Job A5: $7,800 OP WIP + 20,000 DM + 4,000 DL + 4,000*4 OH 47,800 Job A6: $16,000 DM + 3,000 DL + 3,000*4 OH 31,000 $119,400

Cost of goods sold 40,600 Finished goods inventory 40,600 To record the sale of Job A4 Accounts receivable (or Cash) 56,840 Sales 56,840 To record the sale of Job A4: $40,600 x 1.4 Note: |The ending WIP balance should be: Opening balance $13,400 Direct materials 50,000 Direct labour 13,500 Manufacturing overhead applied 54,000 Less: cost of goods manufactured (119,400) $11,500

Proof: Job A7: $4,000 DM + 1,500 DL + 1,500*4 OH $11,500

Page 172: MODULE 3 Financial Management and Management Accounting 1

Module 3 - Financial Management & Management Accounting 1

Page 172 © CMA Ontario, 2011

Problems with Solutions

Multiple Choice Questions

1. Which of the following situations would be appropriate for the application of a job order cost system?

a. Packaging cereal in a breakfast foods company b. Refining a barrel of crude oil c. Building a frigate for the Canadian Navy d. None of the above.

2. Which of the following is not a purpose of job order costing?

a. Developing cost information to evaluate the efficiency of operations. b. To provide primary support for the preparation of the organization’s financial

statements. c. Understanding/predicting prospective costs for the purpose of preparing a bid. d. To provide a basis for recovering costs from a customer.

3. Pepper Company uses a single plan wide rate to allocate manufacturing overhead to jobs. The rate is computed by dividing expected manufacturing overhead by the expected denominator activity level. Expected manufacturing cost was $6,000,000 and the expected denominator activity level was 15,000. The actual manufacturing overhead was $5,600,000 and the actual denominator activity level was 14,000. Which of the following is the manufacturing overhead result for this period?

a. $200,000 over-applied b. $0 c. $400,000 under-applied d. None of the above

4. Which of the following items would be a component of variable manufacturing

overhead in a facility printing business cards?

a. Paper b. Ink c. Wages of machine operator d. Machine depreciation

Page 173: MODULE 3 Financial Management and Management Accounting 1

Module 3 - Financial Management & Management Accounting 1

Page 173 © CMA Ontario, 2011

The following information relates to questions 5 and 6:

Solids Company uses a normal cost accounting system. Budgeted costs for the year were $4 per kilogram of raw materials, $15 per direct labour hour and $20 per machine hour. Budgeted production activity was 20,000 machine hours for the year. During the year, the following were actually incurred for manufacturing:

Kilograms of raw materials used 65,000 Direct labour hours incurred 15,000 Machine hours incurred 19,000 Raw materials costs $200,000 Direct labour costs $247,000 Factory overhead costs $358,000 5. How much was overhead over-applied or under-applied during the year?

a) $22,000 under-applied b) $20,000 under-applied c) $20,000 over-applied d) $22,000 over-applied e) $42,000 over-applied 6. For a job that used 250 kilograms of direct materials, 45 direct labour hours and 50

machine hours, what was the cost of the job under normal costing?

a) $2,452 b) $2,510 c) $2,617 d) $2,675 e) $2,683

Page 174: MODULE 3 Financial Management and Management Accounting 1

Module 3 - Financial Management & Management Accounting 1

Page 174 © CMA Ontario, 2011

The following information relates to questions 7 and 8:

Markham Company uses a normal job order costing system. Overhead rates are applied on the basis of machine hours in Department 1 and direct labour dollars in Department 2. Budgeted data for the two departments are:

Department 1 Department 2 Budgeted overhead $150,000 $300,000 Budgeted activity: Direct labour hours (DLH) 50,000 100,000 Machine hours (MH) 30,000 50,000 Direct labour wage rate $20 per DLH $30 per DLH

7. The following data pertains to Job 94-669 which was completed during the year:

Direct materials $350,000 Direct labour hours (DLH): Department 1 1,000 Department 2 2,000 Machine hours (MH): Department 1 5,000 Department 2 2,000 The total factory overhead applied to Job 94-669 is: a) $ 9,000 b) $15,000 c) $31,000 d) $37,000 e) $80,000 8. During the year, actual direct labour wage rates and factory overhead incurred in

Departments 1 and 2 were as budgeted. Also, 52,000 direct labour hours and 35,000 machine hours were used in Department 1 for production, whereas 98,000 DLH and 46,000 MH were used in Department 2. Overhead for Markham Company is

a) $19,000 under-applied b) $1,000 under-applied c) $1,000 over-applied d) $19,000 over-applied e) $31,000 over-applied

Page 175: MODULE 3 Financial Management and Management Accounting 1

Module 3 - Financial Management & Management Accounting 1

Page 175 © CMA Ontario, 2011

9. Zoom Corporation uses standard absorption costing based on machine hours to account for overhead costs. The company bases its standards on estimated annual sales of $5,000,000, annual overhead costs of $1,300,000 and annual machine hour usage of 500,000 hours. During 2004, however, Zoom Corporation experienced $5,800,000 in sales and used 600,000 machine hours and actual overhead costs amounted to $1,400,000. For 2004, overhead is:

a) $108,000 over-applied b) $100,000 under-applied c) $160,000 over-applied d) $260,000 over-applied e) $208,000 over-applied 10. SME Ltd. manufactures electronic components through two processes: fabrication

and assembly. It has been determined the appropriate cost driver to use for allocating overhead costs is direct machine hours for the fabrication process and direct labour hours for the assembly process. Budgeted overhead costs and activity for the upcoming year are:

Fabrication Assembly

Total overhead costs $300,000 $180,000 Direct machine hours 10,000 3,000 Direct labour hours 8,000 9,000

The actual production data for Job 111, which was completed during the year, were:

Fabrication Assembly

Direct machine hours 10 50 Direct labour hours 40 25

What amount of overhead costs would have been applied to Job 111?

a) $3,300 b) $2,000 c) $4,500 d) $2,200 e) $800

Page 176: MODULE 3 Financial Management and Management Accounting 1

Module 3 - Financial Management & Management Accounting 1

Page 176 © CMA Ontario, 2011

Problem 1

Pepper Company uses a plant wide rate for manufacturing overhead. Pepper Company allocates manufacturing overhead to jobs using direct labour hours. The following information applies to the most recent year:

Planned Actual Direct labour hours 3,400 3,900 Manufacturing overhead $459,000 $475,000

Job 1254-2 was completed during the year with direct materials cost of $5,300 and 45 direct labour hours. Labour was paid $20 per hour during the year.

Required:

a. Compute the over- or under-applied manufacturing overhead for this year assuming normal costing.

b. Compute the over- or under-applied manufacturing overhead for this year assuming actual costing.

c. Compute the revenue associated with Job 1254-2 assuming the actual costing rate was used to allocate manufacturing overhead to the job and the policy at Pepper Company is to charge cost plus 15%.

d. Pepper Company is operating at 60% of capacity and has found it harder to get jobs recently. Does any of the above data provide a possible explanation?

Page 177: MODULE 3 Financial Management and Management Accounting 1

Module 3 - Financial Management & Management Accounting 1

Page 177 © CMA Ontario, 2011

Problem 2

Fyfe Company manufactures plastic novelties used for advertising purposes. The process involves three steps. In the moulding department hot plastic is injected into a mould, cooled and then ejected. This creates the basic product. The products are then moved to the trimming department where excess plastic (flash) is trimmed off the product and inspected for faults. Any products found to be defective are recycled. The products are then moved to the painting department where they are painted with the customer’s message and logo.

The following information was reported for the most recent period:

Department Moulding Trimming Painting Planned manufacturing overhead $1,656,000 $645,000 $854,000 Actual manufacturing overhead $1,687,000 $664,000 $822,000 Average machine hours 12,000 2,100 4,500 Actual machine hours 12,750 2,250 4,100 Average direct labour hours 5,400 8,750 9,000 Actual direct labour hours 5,500 9,200 8,700

Required:

a. Compute the manufacturing overhead rate assuming Fyfe Company uses a normal plant wide rate based on average manufacturing overhead and average direct labour hours.

b. Compute the manufacturing overheard rate assuming Fyfe Company uses an actual plant wide rate base.

c. Compute the departmental manufacturing overhead rates assuming Fyfe Company uses normal department rates based on average manufacturing overhead and the average level of the cost driver activity. Fyfe Company plans to use machine hours as the cost driver in the moulding department and direct labour hours in the trimming and painting departments.

d. Compute the manufacturing overhead rate assuming Fyfe Company uses actual department rates. Fyfe Company plans to use machine hours as the cost driver in the moulding department and direct labour hours in the trimming and painting departments.

Page 178: MODULE 3 Financial Management and Management Accounting 1

Module 3 - Financial Management & Management Accounting 1

Page 178 © CMA Ontario, 2011

Problem 3

Return to the data in Question 2. Fyfe Company recently completed a job for Maggie Company for which the following data was accumulated:

Department Moulding Trimming Painting Direct materials $750 $25 $43 Direct labour cost $175 $90 $120 Direct labour hours 5 2 3 Machine hours 5 1 0

Required:

a. Compute the cost of this job using the four manufacturing overhead rates you developed in Question 1.

b. If you were Maggie Company and assuming the contracted price to be paid for this job is the total cost plus 20%, which manufacturing overhead rate would you prefer? What argument would you use to support the choice of this rate?

c. If you were Fyfe Company and assuming the contracted price to be paid for this job is the total cost plus 20%, which manufacturing overhead rate would you prefer? What argument would you use to support the choice of this rate?

d. As an independent observer, which rate do you think is the most reasonable?

Page 179: MODULE 3 Financial Management and Management Accounting 1

Module 3 - Financial Management & Management Accounting 1

Page 179 © CMA Ontario, 2011

Problem 4

The strategic services group at Angus Company, which consists of nine consultants and five clerical staff, provides advice on strategic matters to its clients. For the upcoming year the strategic services group is targeting $2,000,000 of profits, $3,000,000 of consultant salaries and $12,000,000 of other costs related to clerical salaries and all office related costs. Angus Company treats all costs other than consultant salaries as service overhead and allocates the service overhead costs to jobs based on consultant hours charged to the job. On average, each consultant works nine hours a day, 190 days per year.

Required:

The strategic services group uses a normal overhead rate computed by dividing expected service overhead costs by the average number of consultant hours. Angus Company computes a blended charge rate for consultants that include:

1. The salary per hour plus 2. The service overhead rate plus 3. A target mark-up to achieve the target revenue.

a. Given the information provided, what charge rate will the strategic services group need to use for each hour of consultant work? Show the individual components of the charge rate.

b. If you had profit responsibility for the strategic services group what would you be most interested in monitoring during the year to ensure the group was on track to achieve its profit target?

Problem 5

Danny Company provides excise tax advice to its clients. Although each type of job is unique, Buddy Threadgood, the Danny Company comptroller, identifies three broad groups of consulting jobs: simple, moderate and complex. There are four classifications of employees at Danny Company: partner, senior analyst, junior analyst and clerical. The following table summarizes the average number of hours consumed by each type of excise tax job last year.

Employee Simple Moderate Complex Non-participating partner 4 15 25 Senior analyst 3 15 28 Junior analyst 8 6 12 Clerical 4 7 14

Page 180: MODULE 3 Financial Management and Management Accounting 1

Module 3 - Financial Management & Management Accounting 1

Page 180 © CMA Ontario, 2011

The following table identifies the average salary paid annually to each employee group and the number of hours they are available annually (capacity), as well as the average number of days billed annually to clients in the past (average activity level). (All employees average nine hour work days except the clerical staff, which averages seven hour work days).

Employee Salary Capacity (days) Average (days) Non-participating partner $250,000 190 180 Senior analyst $105,000 210 190 Junior analyst $ 65,000 180 165 Clerical $ 50,000 180 150

A study of overhead, which consists of supplies, computers, Internet, office occupancy, telephone and other related office costs at Danny Company suggests that overhead appears to be driven primarily by the amount of time each type of employee work. The following table summarizes the amount of variable overhead estimated to be driven by an hour of each employee type.

Employee Overhead Per Employee Hour Non-participating partner $145 Senior analyst $185 Junior analyst $120 Clerical $60

Required:

a. Draw a diagram of the costing system that the strategic services group uses.

b. Danny Company allocates employee salaries to jobs using a normal rate computed by dividing each employee’s annual salary by the average activity level. Danny Company follows the approach of marking up variable costs (salary plus variable overhead) by 35% (to cover fixed overhead and provide a profit margin). Given this information, what would be the charge rate for each type of employee hour?

c. Given the charge rates you computed in Part b), what would be the price Danny Company would charge for each job?

d. What do you feel are the major financial risks if Danny Company sets a contract price for each job based on the expected distribution of costs and hours worked?

Page 181: MODULE 3 Financial Management and Management Accounting 1

Module 3 - Financial Management & Management Accounting 1

Page 181 © CMA Ontario, 2011

Problem 6 The Devoe Company uses a job order costing system and started the 2005 year with the following inventory balances:

Raw materials inventory $160,000 Work-in-process inventory 87,000 Finished goods inventory 265,000

The following transactions took place during 2005: 1. Raw material purchases amounted to $450,000. 2. Raw materials issued to production were:

Direct materials $475,000 Indirect materials 20,000

3. Direct labour hours incurred amounted to 19,500 at an average rate of $30. The

company estimated total overhead to be $500,000 and direct labour hours incurred to be 20,000. The company allocates overhead on the basis of direct labour hours.

4. Total manufacturing overhead (excluding indirect materials) incurred was

$485,000. 5. The work-in-process at December 31, 2005 consisted of the following two jobs:

Job A650 Job A652 Direct materials $45,000 $26,500 Direct labour hours 560 125

6. Ending finished goods inventory totalled $247,000. Required: a. Prepare journal entries to record the above transactions. b. Prepare a schedule of cost of goods manufactured and cost of goods sold. c. Prepare the journal entry to dispose of over-/under-applied overhead on the

following assumptions: i. The balance is written-off against cost of goods sold, and ii. The balance is prorated between the balances containing applied

overhead

Page 182: MODULE 3 Financial Management and Management Accounting 1

Module 3 - Financial Management & Management Accounting 1

Page 182 © CMA Ontario, 2011

Problem 7

Duench Inc., is a manufacturer of furnishings for infants and children. The company uses a normal job order costing system. Duench's WIP inventory at April 30, 2004, consisted of the following jobs:

Job Number Items Units Accumulated Cost CBS1O2 Cribs 20,000 $ 900,000 PLPO86 Playpens 15,000 420,000 DRS 114 Dressers 25,000 250,000 $1,570,000

The company's finished goods inventory, using the FIFO method, consisted of five items:

Item Quantity and Unit Cost Accumulated Cost Cribs 7,500 units @ $64 each $ 480,000 Strollers 13,000 units @ $23 each 299,000 Carriages 11,200 units @ $102 each 1,142,400 Dressers 21,000 units @ $55 each 1,155,000 Playpens 19,400 units @ $35 each 679,000 $3,755,400

Duench applies factory overhead on the basis of direct labour hours. The company's factory overhead budget for the fiscal year ending May 31, 2004, totalled $4,500,000 and the company plans to expend 600,000 direct labour hours during this period. Through the first 11 months of the year, a total of 555,000 direct labour hours were worked and total factory overhead amounted to $4,273,500. At the end of April, the balance in Duench's raw materials inventory account, which includes both raw materials and purchased parts, was $668,000. Additions to and requisitions from raw materials inventory during the month of May included:

Raw Materials

Purchased Parts

Additions $242,000 $396,000 Requisitions: Job CBS102 51,000 104,000 Job PLP086 3,000 10,800 Job DRS 114 124,000 87,000 Job STRO77 (10,000 strollers) 2,000 81,000 Job CRGO98 (5,000 carriages) 65,000 187,000

Page 183: MODULE 3 Financial Management and Management Accounting 1

Module 3 - Financial Management & Management Accounting 1

Page 183 © CMA Ontario, 2011

During the month of May, Duench's factory payroll consisted of:

Account Hours Cost CBS102 12,000 $122,400 PLP086 4,400 43,200 DRS114 19,500 200,500 STR077 3,500 30,000 CRG098 14,000 138,000 Indirect 3,000 29,400 Supervision 57,600 $621,100

The following are jobs that were completed and unit sales for the month of May:

Job Number Items Quantity CBS102 Cribs 20,000 PLPO86 Playpens 15,000 STRO77 Strollers 10,000 CRGO98 Carriages 5,000

Items

Quantity Shipped

Cribs 17,500 Playpens 21,000 Strollers 14,000 Dressers 18,000 Carriages 6,000

Required: a. Describe when it is appropriate for a company to use a job order cost system. b. Calculate the dollar balance in Duench's WIP inventory account as of May

31, 2004. c. Calculate the dollar amount related to the playpens in Duench's finished

goods inventory as of May 31, 2004. d. Explain the proper accounting treatment for over- or under-applied overhead

balances when using a job order cost system.

Page 184: MODULE 3 Financial Management and Management Accounting 1

Module 3 - Financial Management & Management Accounting 1

Page 184 © CMA Ontario, 2011

Problem 8

Eagleson Company employs a normal job order costing system. Manufacturing overhead is applied on the basis of machine hours using estimated manufacturing overhead costs of $1,200,000 and an estimated activity level of 80,000 machine hours. Eagleson's policy is to close the over-/under-applied application of manufacturing overhead to cost of goods sold. Operations for the year ended November 30, 2005, have been completed and all accounting entries have been made for the year except the application of manufacturing overhead to the jobs worked on during November, the transfer of costs from WIP to finished goods for the jobs completed in November and the transfer of costs from finished goods to cost of goods sold for the jobs sold during November. Jobs N11-007, N11-013 and N11-015 were completed during November 2005. All completed jobs except job N11-013 had been turned over to customers by the close of business on November 30, 2005. Summarized data accumulated from the accounting records as of October 31, 2005 and for November 2005, are: WIP November 2005 Activity Balance Direct Direct Machine Job # Oct. 3l 2005 Materials Labour Hours N11-007 $ 87,000 $ 1,500 $ 4,500 300 N11-013 55,000 4,000 12,000 1,000 N11-015 0 25,600 26,700 1,400 D12-002 0 37,900 20,000 2,500 D12-003 0 26,000 16,800 800 Totals $142,000 $95,000 80,000 6,000

Activity November Through 2005 Operating Activity Oct 31, 2005 Activity Manufacturing overhead incurred Indirect materials $ 125,000 $ 9,000 Indirect labour 345,000 30,000 Utilities 245,000 22,000 Depreciation 385,000 35,000 Total incurred overhead $1,100,000 $96,000

Other items Material purchases* $965,000 $98,000 Direct labour costs $845,000 $80,000 Machine hours 73,000 6,000

Page 185: MODULE 3 Financial Management and Management Accounting 1

Module 3 - Financial Management & Management Accounting 1

Page 185 © CMA Ontario, 2011

Account balances at beginning of fiscal year: Materials inventory* $105,000 Work-in-process inventory 60,000 Finished goods inventory 125,000

* Material purchases and materials inventory consist of both direct and indirect materials. The balance of the materials inventory account as of November 30, 2005 is $85,000. Required: a. Eagleson Company uses a predetermined overhead rate to apply manufacturing

overhead to its jobs. When overhead is accounted for in this manner, there may be over- or under-applied overhead. 1. Explain why a business uses a predetermined overhead rate to apply

manufacturing overhead to its jobs. 2. How much manufacturing overhead would Eagleson have applied to jobs

through October 31, 2005? 3. How much manufacturing overhead would Eagleson apply to jobs during

November 2005? 4. Determine the amount by which the manufacturing overhead is over- or

under-applied as of November 30, 2005. 5. Over- or under-applied overhead must be eliminated at the end of the

accounting period. Explain why Eagleson's method of closing over- or under-applied overhead to the cost of goods sold is acceptable in this case.

b. Determine the balance in Eagleson Company's finished goods inventory at November 30, 2005.

c. Prepare a schedule of cost of goods manufactured for Eagleson Company for the year ended November 30, 2005.

Page 186: MODULE 3 Financial Management and Management Accounting 1

Module 3 - Financial Management & Management Accounting 1

Page 186 © CMA Ontario, 2011

Solutions

Multiple Choice Questions

1. c This is a unique project and, therefore, is an ideal candidate for job order costing.

2. b Job order costing is not required to support the preparation of the

organization’s financial statements. 3. b The manufacturing overhead rate would be $400 ($6,000,000/15,000).

Therefore, the amount applied to jobs would be $5,600,000 ($400 * 14,000). Under-/over-applied manufacturing overhead equals actual manufacturing overhead minus applied manufacturing overhead, which is $0 ($5,600,000 - $5,600,000).

4. b It would likely be ink. The paper costs would probably be material in terms

of the total cost of the business cards, so would be direct costs. Wages would be direct costs. Depreciation is part of fixed factory overhead. The cost of ink used to print a batch of business cards would likely be prohibitively expensive to track and, therefore, would appropriately be treated as an overhead cost.

5. d Actual overhead applied = 19,000 x $20 = $380,000 Therefore, overhead

was over-applied by $380,000 - $358,000 = $22,000. 6. b Under normal costing, the cost of the job = actual direct costs + budgeted

indirect costs: (250 x $200,000/65,000) + (45 x $247,000/15,000) + (50 x $20) = $769 + $741 + $1,000 = $2,510.

7. c Department 1 overhead rate = $150,000/30,000 MH = $5/MH

Department 2 overhead rate = $300,000/100,000 = $3/DLH or $.10/DLH Job 94-669 overhead = ($5 x 5,000 MH) + ($3 x 2,000 DLH) = $25,000 + 6,000 = $31,000

8. d Department 1 overhead applied = $5 x 35,000 MH = $175,000

Department 2 overhead applied = $3 x 98,000 DLH = $294,000 Actual - applied = ($150,000 + $300,000) - ($175,000 + $294,000) = $19,000 over-applied

9. c ($1,300,000 . 500,000 x 600,000) - $1,400,000 = $160,000 over-applied

Page 187: MODULE 3 Financial Management and Management Accounting 1

Module 3 - Financial Management & Management Accounting 1

Page 187 © CMA Ontario, 2011

10. e Overhead rates: Fabrication = $300,000 ÷ 10,000 hours = $30 per direct machine hour Assembly = $180,000 ÷ 9,000 hours = $20 per direct labour hour Overhead applied to Job 111: ($30 x 10 hours of fabrication) + ($20 x 25 hours of assembly) = $800

Problem 1

a. The normal rate Pepper Company would use to allocate manufacturing overhead to jobs would be $135 per direct labour hour (459,000/3,400).

The manufacturing overhead applied would be actual hours * normal rate or $526,500 (3,900 * 135). Therefore, manufacturing overhead would be $51,500 over-applied (526,500 – 475,000).

b. Since actual costing applies the actual manufacturing overhead to jobs manufacturing overhead is never under- or over-applied.

c. The actual manufacturing overhead rate would be $121.79 (475,000 / 3,900).

Pepper Company

Job Cost Sheet for Job 1242-2

Direct materials $5,300.00

Direct labour 900.00 = 45 * 20

Manufacturing overhead applied 5,480.77 = 45 * (475000/3900)

$11,680.77

Pepper Company would charge $13,432.88 (11,680.77*1.15) for this job.

d. The problem with this approach is Pepper Company will charge more for jobs as demand decreases since as actual hours goes down, the manufacturing overhead rate will go up. Given that Pepper Company is operating at 60%, this means it is quite possible its manufacturing overhead rate will be considerably higher than its competitors, which might explain the problem of decreasing demand.

Page 188: MODULE 3 Financial Management and Management Accounting 1

Module 3 - Financial Management & Management Accounting 1

Page 188 © CMA Ontario, 2011

Problem 2

Department Moulding Trimming Painting Total

Planned manufacturing overhead $1,656,000 $645,000 $854,000 $3,155,000

Actual manufacturing overhead $1,687,000 $664,000 $822,000 $3,173,000

Average machine hours 12,000 2,100 4,500 18,600

Actual machine hours 12,750 2,250 4,100 19,100

Average direct labour hours 5,400 8,750 9,000 23,150

Actual direct labour hours 5,500 9,200 8,700 23,400

(a) Normal rate (planned manufacturing overhead/average activity level)

$136.29

(b) Actual rate = (actual manufacturing overhead/actual activity level)

$135.60

(c) Normal rates (planned manufacturing overhead/average activity level)

$138.00 $73.71 $94.89

(d) Actual rates (actual manufacturing overhead/actual activity level)

$132.31 $72.17 $94.48

Page 189: MODULE 3 Financial Management and Management Accounting 1

Module 3 - Financial Management & Management Accounting 1

Page 189 © CMA Ontario, 2011

Problem 3

a. The following is a summary of the costs that would be accumulated and the prospective price under each of the four manufacturing overhead costing alternatives:

Fyfe Company

Job Cost Sheet for Maggie Company Job

Department Moulding Trimming Painting Total Direct materials cost $750.00 $25.00 $43.00 $818.00 Direct labour cost $175.00 $90.00 $120.00 $385.00 Total direct costs $1,203.00 Machine hours 5 1 0 6 Labour hours 5 2 3 10

Manufacturing overhead using normal plant wide rate

$1,362.85

Total job cost $2,565.85 Price charged $3,079.02

Manufacturing overhead using actual plant wide rate

$1,355.98

Total job cost $2,558.98 Price charged $3,070.78

Manufacturing overhead using normal departmental rates

$1,122.10

Total job cost $2,325.10 Price charged $2,790.11

Manufacturing overhead using actual departmental rates

$1,089.36

Total job cost $2,292.36 Price charged $2,750.84

Plant wide normal rate $136.29

Plant wide actual rate $135.60

Departmental normal rates $138.00 $73.71 $94.89

Departmental actual rates $132.31 $72.17 $94.48

Page 190: MODULE 3 Financial Management and Management Accounting 1

Module 3 - Financial Management & Management Accounting 1

Page 190 © CMA Ontario, 2011

b. Fyfe Company would want to use the approach that produced the largest price, which would be to use a plant wide normal rate. Fyfe Company might argue this is the most practical approach since it is based on expected overhead costs at the start of the period. Fyfe Company might also mention this approach is most commonly used.

c. Maggie Company would prefer the approach that would use the actual departmental rates since this yields the lowest price. Maggie Company would argue that a departmental rate approach provides a better estimate of the cost of the overhead resources its job used and using the actual cost rate reflects the actual manufacturing overhead rate during the period.

d. It is difficult to argue against the departmental rate approach since this approach has a greater potential of estimating the cost and effect relationship between the job and its use of overhead resources. So the choice should really come down to choosing between normal and actual departmental rates. Certainly arguments can be made in favour of each; however, the argument might be stronger for the normal rate since this reflects expectations when the capacity was acquired and really does reflect the cost driver for capacity. As an aside, the possibility for a legitimate disagreement over rates is the reason many cost based contracts will specify the manufacturing overhead rate that will be used.

Page 191: MODULE 3 Financial Management and Management Accounting 1

Module 3 - Financial Management & Management Accounting 1

Page 191 © CMA Ontario, 2011

Problem 4

a.

Angus Company

Strategic Services Group

Blended Charge Rate Per Consultant Hour Work Up

Blended rate salary component

Number of consultants 9

Total salaries $3,000,000.00

Salary per consultant $333,333.33 Total average hours worked per consultant 1,710

Salary per consultant hour $194.93

Blended rate service overhead component

Service overhead amount $12,000,000.00

Total average hours worked all consultants 15,390

Service overhead per consultant hour $779.73

Target mark-up component

Target profit $2,000,000.00

Total average hours worked all consultants 15,390

Profit component per consultant hour $129.95

Total blended rate per consultant hour $1,104.61

b. There would be a number of things that deserved monitoring including the actual amounts of consultant salaries and the costs treated as overhead. However, the major source of attention would be the number of consultant hours billed. If that amount fell below the average number of hours worked, then the profit target would not be achieved.

Page 192: MODULE 3 Financial Management and Management Accounting 1

Module 3 - Financial Management & Management Accounting 1

Page 192 © CMA Ontario, 2011

Problem 5

a.

Page 193: MODULE 3 Financial Management and Management Accounting 1

Module 3 - Financial Management & Management Accounting 1

Page 193 © CMA Ontario, 2011

b. and c.

Danny Company

Job Prices

Employee Salary Average (Days per

Year)

Average (Hours

per Year)

Normal Rate Per

Hour

Variable Overhead Per Hour

Total Rate Before

Mark-Up

Marked-Up Rate

Non-participating partner

$250,000 180 1,620 $154.32 $145 $299.32 $404.08

Senior analyst $105,000 190 1,710 $61.40 $185 $246.40 $332.64

Junior analyst $65,000 165 1,485 $43.77 $120 $163.77 $221.09

Clerical $50,000 150 1,050 $47.62 $60 $107.62 $145.29

Simple Moderate Complex

Employee Hours Price Hours Price Hours Price

Non-participating partner

4 $1,616.33 15 $6,061.25 25 $10,102.08

Senior analyst 3 997.93 15 4,989.67 28 9,314.05

Junior analyst 8 1,768.73 6 1,326.55 12 2,653.09

Clerical 4 581.14 7 1,017.00 14 2,034.00

Total job price $4,964.14 $13,394.47 $24,103.23

d. Remember, all employee salaries are guaranteed and, therefore, do not vary with the actual amount of work done. The only items that vary with the amount of work done are the variable overhead costs created when the employees work. Since amounts charged for these contracts are far in excess of the variable overheads per hour, it is highly unlikely the incremental cost of a particular job will exceed its revenue. The true financial risk is that excess hours worked on a particular contract may result in those hours not being available to work on another contract if the organization is operating at capacity.

Page 194: MODULE 3 Financial Management and Management Accounting 1

Module 3 - Financial Management & Management Accounting 1

Page 194 © CMA Ontario, 2011

Problem 6 a. Raw materials inventory $450,000 Cash or accounts payable $450,000 To record the purchase of raw materials. WIP 475,000 Manufacturing overhead 20,000 Raw materials inventory 495,000 To record the issue of direct materials to production

and the use of indirect materials.

WIP (19,500 x $30) 585,000 Wages payable or cash 585,000 To record direct labour. WIP (19,500 x $25*) 487,500 Manufacturing overhead 487,500 To record the application of manufacturing

overhead.

* POR = $500,000 / 20,000 = $25 Manufacturing overhead 485,000 Accounts payable or cash 485,000 To record the incurrence of manufacturing

overhead costs.

Finished goods inventory 1,525,325 WIP 1,525,325 To record cost of goods manufactured (see Section b) Cost of goods sold 1,543,325 Finished goods inventory 1,543,325 To record the cost of goods sold (see Section b)

Page 195: MODULE 3 Financial Management and Management Accounting 1

Module 3 - Financial Management & Management Accounting 1

Page 195 © CMA Ontario, 2011

b. Devoe Company Schedule of Cost of Goods Manufactured

for the year ended December 2005 Direct materials used $475,000 Direct labour 585,000 Manufacturing overhead applied 487,500 Total manufacturing costs 1,547,500 WIP – beginning 87,000 WIP – end (Schedule) (109,175) Cost of goods manufactured $1,525,325

Devoe Company

Schedule of Cost of Goods Sold for the year ended December 2005

Finished goods inventory, beginning $ 265,000 Cost of goods manufactured 1,525,325 Finished goods inventory, ending (247,000) Cost of goods sold $1,543,325

Schedule – ending WIP Job A650 Job A652 Direct materials $45,000 $26,500 Direct labour: 560 x $30 | 125 x $30 16,800 3,750 Manufacturing overhead: 560 x $25 125 x $25 14,000 3,125 $75,800 $33,375

c. Actual manufacturing overhead ($20,000 indirect materiaIs + 485,000 other) $505,000 Manufacturing overhead applied 487,500 Under-applied overhead $ 17,500

i. Cost of goods sold 17,500 Manufacturing overhead $17,500 ii. Balance % Allocation COGS ($1,543,325 - 87,000 WIP-OP - 265,000 FG - OP) $1,191,325 77% $13,475 WIP 109,175 7% 1,225 FG 247,000 16% 2,800 $1,547,500 $17,500

Page 196: MODULE 3 Financial Management and Management Accounting 1

Module 3 - Financial Management & Management Accounting 1

Page 196 © CMA Ontario, 2011

Cost of goods sold 13,475 Work-in-process 1,225 Finished goods inventory 2,800 Manufacturing overhead 17,500 Problem 7 a. A job order cost system is appropriate for manufacturing, merchandising and

service companies that provide individual products, or services or products or services in unique batches. The process characteristic that makes a job order costing desirable is the ability to trace materials and labour directly to the unit or batch.

b. POR = $4,500,000 I 600,000 direct labour hours = $7.50 per DLH The only job in process at May 31, 2004 is Job DRS114: Costs in opening WIP $250,000 Direct materials added in May 124,000 Purchased parts added in May 87,000 Direct labour added in May 200,500 Manufacturing overhead applied: 19,500 hours x $7.50 146,250 $807,750

c. Cost

/Unit

Units

Total Beginning inventory, finished goods $35 19,400 $679,000 Playpens manufactured in May WIP – opening 420,000 Direct materials added in May 3,000 Purchased parts added in May 10,800 Direct labour added in May 43,200 Manufacturing overhead applied: 4,400 hours x $7.50 33,000 34 15,000 510,000 Less: sales Beginning inventory 35 19,400 (679,000) Manufactured in May 34 1,600 (54,400) Ending inventory, finished goods $34 13,400 $455,600

d. If the ending overhead balance at the end of the year is significant, it should be

prorated over COGS and the balances in WIP, FG and COGS. If it is insignificant, the ending balance can be closed to COGS or simply carried forward to the following year.

Page 197: MODULE 3 Financial Management and Management Accounting 1

Module 3 - Financial Management & Management Accounting 1

Page 197 © CMA Ontario, 2011

Problem 8

a. 1. PORs are used rather than actual overhead rates for four related purposes:

• To calculate an appropriate amount of overhead to be included in the

cost of products or services. • To enable overhead allocation to be made in a timely manner, rather

than waiting until total actual overhead costs and hours worked become known at the end of the year.

• To provide timely product cost information for journal entries

transferring completed products from WIP to FG and from FG to COGS.

• To normalize the overhead charge: PORs smooth out uncontrollable

fluctuations in actual overhead costs that are unrelated to activity or volume levels. Actual overhead rates, if calculated monthly for example, often vary due to seasonal factors.

2. POR = $1,200,000 / 80,000 = $15 per MH Applied overhead to October 31 = 73,000 MH x $15 = $1,095,000 3. 6,000 MH x $15 = $90,000 4. Actual manufacturing overhead - $1,100,000 + 96,000 $1,196,000 Manufacturing overhead applied - $1,095,000 + 90,000 1,185,000 Under-applied overhead $ 11,000

5. The ending overhead balance at the end of the fiscal year is $11,000 under-

applied. This is less than 1% of the actual overhead costs incurred. Theoretically, proper product costing requires any ending balance, regardless of its significance, to be prorated over all jobs worked on throughout the year. Insignificant amounts such as this will not distort net income or the ending WIP and FG balances. Thus, from a practical perspective, Eagleson Company is justified in closing the ending overhead balance to COGS.

Page 198: MODULE 3 Financial Management and Management Accounting 1

Module 3 - Financial Management & Management Accounting 1

Page 198 © CMA Ontario, 2011

b. Job N11-013

WIP Balance, October 31, 2005 $55,000 Costs added in November Direct materials 4,000 Direct labour 12,000 Manufacturing overhead: 1,000 MH x $15 15,000 $86,000

c. Ending WIP: Job

D12-002 Job

D12-003 Direct materials $37,900 $26,000 Direct labour 20,000 16,800 Manufacturing overhead:

2,500 MH x $15 | 800 MH x $15

37,500

12,000 $95,400 $54,800

Eagleson Company

Schedule of Cost of Goods Manufactured for the year ended November 30, 2005

Direct materials used: Raw materials inventory, beginning $105,000 Raw material purchases 1,063,000 Raw materials inventory, ending (85,000) Raw materials used 1,083,000 Less indirect materials used (134,000) $ 949,000

Direct labour 925,000 Manufacturing overhead applied 1,185,000

Total manufacturing cost 3,059,000 WIP, beginning 60,000 WIP, end (150,200)

Cost of goods manufactured $2,968,800

Page 199: MODULE 3 Financial Management and Management Accounting 1

Module 3 - Financial Management & Management Accounting 1

Page 199 © CMA Ontario, 2011

4. Process Costing and Spoilage

Learning Objectives

After completing this chapter, you will be able to:

1. Identify settings where process costing is appropriate and describe the managerial insights provided by process costing.

2. Understand the role and significance of identifying product flow, equivalent units and cost per equivalent unit in producing a process costing report.

3. Produce a process costing report using the FIFO and weighted average approaches.

4. Produce a process costing report that deals with situations involving transferred in costs and spoilage.

Process Costing Systems

Process costing is a costing system suitable for costing the mass production of identical products such as packaging soft drinks and manufacturing plastic bottles. When an organization is making a large volume of identical problems, the cost per unit calculation is trivial. The management accountant will simply divide the total manufacturing costs by the number of units produced to compute the cost per unit.

However, most production lines involve the product passing through numerous steps and process costing focuses on identifying the cost per unit that is incurred at each step; hence, the focus is on identifying the cost of each of the steps (processes) needed to produce the final product.

In this chapter, we will deal with simple settings involving a small number of steps and departments. However, the ideas and concepts of process costing can be fully developed in simple environments and applied to more complex situations.

Why Use Process Costing? A Process Flow Example

The following is a description of the process of making chocolate bars that has been adapted from material at www.hersheys.com.

Cacao trees grow melon-like fruit, which is harvested by hand. Inside each pod are about 20-40 seeds or cocoa beans. After the beans are removed from the pods, they are placed in large heaps or piles. This is called fermentation and takes about a week. Railroad cars carry the cocoa beans from the docks to the chocolate factory where they are cleaned and stored. Cocoa beans

are roasted in large, revolving roasters at high

Page 200: MODULE 3 Financial Management and Management Accounting 1

Module 3 - Financial Management & Management Accounting 1

Page 200 © CMA Ontario, 2011

temperatures. A special hulling machine then takes the dry, roasted cocoa beans and separates the shell from the inside of the bean, called the “nib.” Milling is a grinding process, which turns the nibs into liquid called chocolate liquor. Tanker trucks bring fresh milk to the factory every day where it is tested, pasteurized and then mixed with sugar. The milk-sugar mixture is slowly dried until it turns into a thick, taffy-like material. At the heart of the chocolate factory is the central blending operation where the chocolate liquor is combined with the milk and sugar. This new mixture is dried into a coarse, brown powder called chocolate crumb. The chocolate crumb powder is used to make milk chocolate. Hershey adds cocoa butter to the crumb, which brings out the rich taste and creamy texture of the chocolate. The crumb travels through special steel rollers, which grind and refine the mixture, making it smoother. The crumb becomes a thick liquid called chocolate paste. The paste is poured into huge vats called conches. Once inside the conche, large granite rollers smooth out the gritty particles from the crumb. This process can take anywhere from 24 to 72 hours to complete. The paste is tempered or cooled in a controlled manner to the right texture and consistency. Other ingredients, like almonds or peanuts, can be mixed into the paste during tempering or added directly to the moulds. Most chocolate bars are made by pouring the liquid chocolate paste into moulds. The filled moulds then take a bumpy, vibrating ride to remove air bubbles and allow the chocolate to settle evenly. Finally, they wind their way through a long cooling tunnel where the liquid chocolate is gently chilled into a solid candy bar. Now it’s ready to wrap.

You can take a Hershey plant tour by going to http://www.hersheys.com/discover/tour_video.asp

As you can see, there are many steps in the process of making a Hershey’s chocolate bar. Each of these steps involves a process where costs can be accumulated. The cost of each process along the way of making chocolate bars can be accumulated and used to determine the total cost of the product. This information allows managers to identify opportunities for improving cost performance. This is the insight and value of process costing.

Page 201: MODULE 3 Financial Management and Management Accounting 1

Module 3 - Financial Management & Management Accounting 1

Page 201 © CMA Ontario, 2011

Physical Flow and Equivalent Units

The Pepper Bat Company manufactures wooden baseball bats in two departments: forming and finishing. Forming department employees remove lumber from stores and cut the lumber into pieces measuring approximately 1.5 metres by 10 centimetres by 10 centimetres. Each unit of lumber yields approximately five pieces. The five pieces are then put on a lathe, which forms the bat. The finishing department employees sand the bat, stamp the bat and apply a finish.

The following table summarizes information for the forming department in this period:

Item Number/Cost Units of lumber started (pieces equivalent) 1,000 (5,000)

Pieces completed 5,000 Lumber cost $65,000 Labour cost $23,000

Manufacturing overhead $46,000

••

•••

Page 202: MODULE 3 Financial Management and Management Accounting 1

Module 3 - Financial Management & Management Accounting 1

Page 202 © CMA Ontario, 2011

Process Costing Steps

Step 1:

The first step in process costing is to identify the physical flow. The following is an exhibit summarizing the physical flow:

Forming Department

Physical Flow Physical

Units

Opening work-in-process 0 Started this period 5,000

Units to account for 5,000 Completed and transferred out 5,000 Ending work-in-process 0

Units accounted for 5,000

Step 2

The second step in process costing is to identify the equivalent units of production for each of the cost elements. In this case, we have two cost elements, materials (the cost of the wood) and conversion costs (the cost of labour and manufacturing overhead). The equivalent unit concept is used when there is partially completed work-in-process at the end of an accounting period. For example, if there were 1,000 physical units in ending work-in-process and 25% of the conversion was completed for those units, the management accountant would say the ending work-in-process is equivalent to conversion work on 250 (25% of 1,000) complete units. Since there is no ending work-in-process in this example, there are no equivalent units in ending inventory. Since the units completed and transferred out are complete, the equivalent units equal the physical units. The management accountant would produce the following table to show this:

Equivalent Units

Materials Conversion Costs

Units transferred out 5,000 5,000 Equivalent unit in ending WIP 0 0

5,000 5,000

Page 203: MODULE 3 Financial Management and Management Accounting 1

Module 3 - Financial Management & Management Accounting 1

Page 203 © CMA Ontario, 2011

Step 3:

In this step, the management accountant introduces costs into the process costing calculation and computes the cost per equivalent with the following result for our example:

Materials Conversion Costs

Costs incurred this period $65,000 $69,000

Divide by equivalent units 5,000 5,000

Cost per unit $13.00 $13.80

Total cost per unit transferred out = $13.00 + 13.80 = $26.80

Step 4:

In step 4, the final step the management accountant allocates the total costs to account for to ending work-in-process and work completed and transferred out.

The following is what the completed process costing statement would look like for our example:

Costs of units transferred out

5,000 units x $26.80 $134,000

Note that this is equal to the total costs incurred: $65,000 Lumber + 23,000 Labour

+ $46,000 Manufacturing Overhead = $134,000

Page 204: MODULE 3 Financial Management and Management Accounting 1

Module 3 - Financial Management & Management Accounting 1

Page 204 © CMA Ontario, 2011

Costing Approaches The two most widely used process costing approaches in Canada are the weighted average method and the first-in first-out (FIFO) method. We will continue our example using the weighted average method and return later to illustrate the FIFO method.

Second period:

Continuing our example, suppose the following information summarizes the activities in the forming department in the second period:

Item Number/Cost Units of lumber started (pieces equivalent) 1,200 (6,000)

Pieces completed 5,500

Lumber cost $73,000

Labour cost $28,000

Manufacturing overhead $49,000

Materials % completion 100%

Conversion % completion 45%

Note that we now have an ending inventory in work-in-process. In the forming department, all materials are entered at the start of production so work-in-process will always be 100% complete with respect to materials costs. However, conversion takes place uniformly in this department, so the work on the ending inventory is deemed (by estimate) to be 45% complete. Using the weighted average method, the management accountant would produce the following process costing report for period 2.

Page 205: MODULE 3 Financial Management and Management Accounting 1

Module 3 - Financial Management & Management Accounting 1

Page 205 © CMA Ontario, 2011

Forming Department Period 2 - Weighted Average

Equivalent Units

Materials Conversion Costs

Units transferred out 5,500 5,500 Equivalent units in ending WIP: 500 x 100% | 500 x 45% 500 225 6,000 5,725

Costs incurred during the period $73,000 $77,000 Cost per equivalent unit (total = $25.6165) $12.1667 $13.4498

Cost of units transferred out 5,500 x $25.6165 $140,891

Cost of ending WIP Materials: 500 x $12.1667 $6,083 Conversion costs: 225 x $13.4498 3,026 $9,109

The T-Account view of the process is as follows:

Work-in-Process Finished Goods

B $ 0 140,891 TO 140,891

CA 150,000

E 9,109

Where B = Beginning Inventory CA = costs added E = Ending Inventory TO = costs transferred to finished goods

Page 206: MODULE 3 Financial Management and Management Accounting 1

Module 3 - Financial Management & Management Accounting 1

Page 206 © CMA Ontario, 2011

Continuing our example, assume the following table reports the results for period 3:

Item Number/Cost Units of lumber started (pieces equivalent) 900 (4,500) Pieces completed 4,900 Lumber cost $62,000 Labour cost $21,000 Manufacturing overhead $44,000 Materials % completion 100% Conversion % completion 85%

Forming Department Period 3 - Weighted Average

Equivalent Units

Materials Conversion Costs

Units transferred out 4,900 4,900 Equivalent units in ending WIP (End WIP = 500 Beginning WIP + 4,500 Started – 4,900 Completed = 100 units)

100 x 100% | 100 x 85% 100 85 5,000 4,985 Costs in opening WIP $ 6,083 $ 3,026 Costs incurred during the period 62,000 65,000 $68,083 $68,026 Cost per equivalent unit (total = $27.2627) $13.6166 $13.6461 Cost of units transferred out 4,900 x $27.2627 $133,587 Cost of ending WIP Materials: 100 x $13.6166 $1,362 Conversion costs: 85 x $13.6461 1,160 $2,522

Page 207: MODULE 3 Financial Management and Management Accounting 1

Module 3 - Financial Management & Management Accounting 1

Page 207 © CMA Ontario, 2011

Work-in-Process Finished Goods

B $9,109 133,587 TO 133,587

CA 127,000

E 2,522

Spoilage Inherent in any manufacturing process is the potential for spoilage. The process costing model has a provision for handling spoilage related to defective units.

Continuing our example at the Pepper Bat Company, suppose there is a certain amount of spoilage expected due to faults in the raw material. Bats are inspected at the end of the forming process and units found to be defective (splits or knots in the wood) are rejected and cut up for firewood. Management has established an allowance for normal spoilage equal to 5% of good units produced.

Return to our continuing example and assume the results for period 4 are:

Item Number/Cost Units of lumber started (pieces equivalent) 1,100 (5,500) Pieces completed 5,100 Pieces found to be defective 300 Lumber cost $74,000 Labour cost $31,000 Manufacturing overhead $56,000 Materials % completion 100% Conversion % completion 25%

Since inspection occurs at the end of production in the forming department we know the number of good units produced and transferred out must have been 4,800 (5,100 - 300). Since the normal spoilage allowance is 5% of good production, the normal spoilage allowance for this period would be 240 units (5% * 4,800). Since total spoilage was 300 units, this means abnormal spoilage would be recorded as 60 units (300 – 240). Finally, since there were 100 units in opening inventory, 5,500 units started and 5,100 units completed, the ending work-in-process must be 500 units. With this information, the following is the process costing report the management accountant would produce for period 4:

Page 208: MODULE 3 Financial Management and Management Accounting 1

Module 3 - Financial Management & Management Accounting 1

Page 208 © CMA Ontario, 2011

Forming Department Period 4 - Weighted Average

Equivalent Units

Materials Conversion Costs

Units transferred out (5,100 – 300 Spoiled) 4,800 4,800 Normal spoilage 240 240 Abnormal Apoilage 60 60 Equivalent units in ending WIP (End WIP = 100 Beginning WIP + 5,500 Started – 4,800 Ttansferred Out – 300 Spoiled = 500 units)

500 x 100% | 500 x 25% 500 125 5,600 5,225

Costs in opening WIP $ 1,362 $ 1,160 Costs incurred during the period 74,000 87,000 $75,362 $88,160

Cost per equivalent unit (total = $30.3302) $13.4575 $16.8727

Cost of good units transferred out: 4,800 x $30.3302 $145,585 Normal Spoilage: 240 x $30.3302 7,279 $152,864

Cost of Abnormal Spoilage: 60 x $30.3302 $1,820

Cost of ending WIP Materials: 500 x $13.4575 $6,729 Conversion costs: 125 x $16.8727 2,109 $8,838

Work-in-Process Finished Goods

B $2,522 152,864 TO 152,864

CA 161,000 1,820 AS

E 8,838

AS = Abnormal Spoilage

Page 209: MODULE 3 Financial Management and Management Accounting 1

Module 3 - Financial Management & Management Accounting 1

Page 209 © CMA Ontario, 2011

The cost of abnormal spoilage would be charged to the cost of goods sold for the period. Note that if inspection occurs before the forming process is complete, the equivalent units associated with conversion costs would be less.

Transferred in Costs

So far, we have focused exclusively on the forming department. But what about the finishing department? The finishing department takes the partially completed bats from the forming department and completes the work on the bats by sanding them, stamping them with the company logo and applying a finish. A small amount of materials costs are incurred for each bat at the end of the production process when conversion costs are complete. Therefore there are no materials costs associated with work-in-process.

Item Number/Cost Bats in opening inventory 800 Bats transferred in from forming department 4,800 Transferred in costs in opening inventory $20,728 Conversion costs in opening inventory $1,127 Costs transferred in from forming department $152,864 Bats completed 5000 Materials cost $1400 Labour cost $9,000 Manufacturing overhead $11,000 Conversion % completion 65%

Page 210: MODULE 3 Financial Management and Management Accounting 1

Module 3 - Financial Management & Management Accounting 1

Page 210 © CMA Ontario, 2011

Finishing Department Period 4 - Weighted Average

Equivalent Units

Transferre

din Costs Materials

Conversion Costs

Units transferred out 5,000 5,000 5,000 Equivalent units in ending WIP (End WIP = 800 Beginning WIP + 4,800 Started – 5,000 Ttansferred Out = 600 units)

600 x 100% | 600 x 0% | 600 x 65% 600 0 390 5,600 5,000 5,390

Costs in opening WIP $20,728 $ - $1,127 Costs incurred during the period 152,864 1,400 20,000 $173,592 $1,400 21,127

Cost per equivalent unit (total = $35.1983) $30.9986 $0.2800 $3.9197

Cost of good units transferred out: 5,000 x $35.1983

$175,992

Cost of ending WIP Transferred-in Costs: 600 x $30.9986 $18,599 Conversion costs: 390 x $3.9197 1,529 $20,128

Work-in-Process Finished Goods

B $21,855 175,992 TO 175,992

CA 21,400

TI 152,864

E 20,127 Difference due to rounding

TI = transferred-in costs

Page 211: MODULE 3 Financial Management and Management Accounting 1

Module 3 - Financial Management & Management Accounting 1

Page 211 © CMA Ontario, 2011

First-In First-Out

As mentioned previously, the first-in first-out (FIFO) method of process costing (and, therefore, inventory valuation) is commonly used in Canada. We will now repeat the process costing exercise using the same data for the Pepper Bat Company using FIFO costing.

The following exhibit displays the process costing report the management accountant would prepare for period 3. The key in FIFO costing is to keep track of the costs associated with the items in opening work-in-process and to ensure those costs are part of the costs transferred out and are not, as in the case of weighted average costing, mixed in with the current period costs.

As the exhibit illustrates, this is done by allocating all the costs in opening inventory ($9,110) to the cost of units completed and transferred out. Then the cost of the work done this period ($3,755) to complete the opening work-in-process is added to the opening balance of the cost of opening inventory to compute the total cost associated with the opening inventory completed and transferred out. The remaining costs are allocated between units started and completed in the current period and the ending work-in-process inventory.

Recall that at the end of the second period, we had 500 units in ending inventory that were 100% complete with regards to materials and 45% complete with regards to conversion costs. The costs of these ending inventories was calculated as $6,083 for materials and $3,026 for conversion costs.

Page 212: MODULE 3 Financial Management and Management Accounting 1

Module 3 - Financial Management & Management Accounting 1

Page 212 © CMA Ontario, 2011

Forming Department Period 3 - FIFO

Equivalent Units

Materials Conversion Costs

Equivalent units of work done to complete opening WIP: 500 x 0 | 500 x 55% 0 275 Units started and completed: 4,900 – 500 Units in Opening WIP 4,400 4,400 Equivalent units in ending WIP (End WIP = 500 Beginning WIP + 4,500 Started – 4,900 Completed = 100 units)

100 x 100% | 100 x 85% 100 85 4,500 4,760

Costs incurred during the period $62,000 $65,000

Cost per equivalent unit (total = $27.4333) $13.7778 $13.6555

Cost of units transferred out: Costs in opening WIP ($6,083 + 3,026) $9,109 Costs to complete opening WIP: Conversion costs: $275 x 13.6555 3,755 Costs of units strated and completed: 4,400 x $27.4333 120,707 $133,571

Cost of ending WIP Materials: 100 x $13.7778 $1,378 Conversion costs: 85 x $13.6555 1,161 $2,539

Work-in-Process Finished Goods

B $9,109 133,571 TO 133,571

CA 127,000

E 2,539 Difference due to rounding

Page 213: MODULE 3 Financial Management and Management Accounting 1

Module 3 - Financial Management & Management Accounting 1

Page 213 © CMA Ontario, 2011

Continuing the example and using the same data that was used for period 4 for the weighted average costing illustration, the following is the process costing report the management accountant would produce for period 4. A simplifying assumption management accountants make is to assume all spoilage is costed using the costs incurred in the current period. (A more accurate but complicated way to handle spoilage would be to prorate the costs of spoilage based on costs in opening inventory and current costs.)

Page 214: MODULE 3 Financial Management and Management Accounting 1

Module 3 - Financial Management & Management Accounting 1

Page 214 © CMA Ontario, 2011

Forming Department Period 4 - FIFO

Equivalent Units

Materials Conversion Costs

Equivalent units to complete the opening WIP:

100 x 0% | 100 x 15% 0 15

Units started and completed: 4,800 – 100 Units in Opening WIP 4,700 4,700 Normal spoilage 240 240 Abnormal Apoilage 60 60 Equivalent units in ending WIP (End WIP = 100 Beginning WIP + 5,500 Started – 4,800 Ttansferred Out – 300 Spoiled = 500 units)

500 x 100% | 500 x 25% 500 125 5,500 5,140

Costs incurred during the period $74,000 $87,000

Cost per equivalent unit (total = $30.3807) $13.4546 $16.9261

Cost of units transferred out: Costs in opening WIP $2,522 Costs to complete opening WIP Conversion costs: 15 x 16.9261 254 Cost of units started and completed: 4,700 x 30.3807 142,789 Normal Spoilage: 240 x $30.3807 7,291 $152,856

Cost of Abnormal Spoilage: 60 x $30.3807 $1,823

Cost of ending WIP Materials: 500 x $13.4546 $6,727 Conversion costs: 125 x $16.9261 2,116 $8,843

Work-in-Process Finished Goods

B $2,522 152,856 TO 152,856

CA 161,000 1,823 AS

E 8,843

Page 215: MODULE 3 Financial Management and Management Accounting 1

Module 3 - Financial Management & Management Accounting 1

Page 215 © CMA Ontario, 2011

Conclusion:

Process costing provides important cost details about the individual components in the production process creating a product. Managers can use process costing information to find opportunities to reduce costs, thereby, making the organization more profitable and competitive.

Page 216: MODULE 3 Financial Management and Management Accounting 1

Module 3 - Financial Management & Management Accounting 1

Page 216 © CMA Ontario, 2011

Problems with Solutions

Multiple Choice Questions

The following information pertains to question 1 – 3: Deejay Company uses an actual process costing system. In Department 2, direct

materials are added at the 50% stage of completion of the process and conversion costs are added uniformly throughout the process. On March 1, there were 6,000 units in work-in-process and they were 60% completed as to conversion cost. During March, 42,000 units were transferred into Department 2 for processing. On March 31, there were 8,000 units in work-in-process: 3,000 units were 40% complete as to conversion costs and the other 5,000 units were 80% complete as to conversion costs. No spoilages were reported during March.

1. Using the FIFO method, the number of equivalent units of production for the transferred-in costs would be:

a) 8,000 higher than the weighted average method. b) 6,000 higher than the weighted average method. c) The same as the weighted average method. d) 6,000 lower than the weighted average method e) 8,000 lower than the weighted average method. 2. Using the FIFO method, the number of equivalent units of production for direct

materials for March would be: a) 39,000 b) 40,000 c) 42,000 d) 45,000 e) 48,000 3. Using the weighted average method, the number of equivalent units of production

for conversion costs would be: a) 39,200 b) 40,000 c) 41,600 d) 44,000 e) 45,200

Page 217: MODULE 3 Financial Management and Management Accounting 1

Module 3 - Financial Management & Management Accounting 1

Page 217 © CMA Ontario, 2011

The following apply to questions 4 - 6: Marlan Manufacturing produces a product that passes through two departments. The units from the moulding department are completed in the assembly department. The units are completed in assembly by adding the remaining direct materials when the units are 60% complete with respect to conversion costs. Conversion costs are added proportionately in assembly. The production activity in the assembly department for the current month is presented below. Marlan uses the FIFO (first-in, first-out) inventory method in its process cost system. The ending work-in-process inventory was 40% complete with regards to conversion costs. Beginning inventory units (25% complete with respect to conversion costs) 8,000 Units transferred in from the moulding department during the month 42,000 Units completed and transferred to finished goods inventory 38,000 4. The equivalent units transferred from the moulding department to the assembly

department for the current month would be: a) 30,000 units b) 38,000 units c) 40,800 units d) 42,000 units e) 50,000 units 5. The equivalent units in the assembly department for direct materials for the current

month would be: a) 30,000 units b) 38,000 units c) 40,800 units d) 42,000 units e) 50,000 units 6. The equivalent units in the assembly department for conversion costs for the current

month would be: a) 36,800 units b) 40,800 units c) 42,800 units d) 43,200 units e) 45,200 units

Page 218: MODULE 3 Financial Management and Management Accounting 1

Module 3 - Financial Management & Management Accounting 1

Page 218 © CMA Ontario, 2011

7. In computing the current period's manufacturing cost per equivalent unit, the

FIFO method of process costing considers current period costs: a) Only b) Plus cost of beginning work-in-process inventory c) Less cost of beginning work-in-process inventory d) Plus cost of ending work-in-process inventory 8. The cost of defective units that normally arise under efficient operating conditions

should be: a) Charged to administrative costs b) Prorated to cost of goods sold and ending inventory based on total actual units

produced c) Written-off against owners' equity d) Prorated to cost of goods sold and ending inventory based on good units

produced e) None of the above

Page 219: MODULE 3 Financial Management and Management Accounting 1

Module 3 - Financial Management & Management Accounting 1

Page 219 © CMA Ontario, 2011

The following information pertains to questions 9 – 10: Ex Company, which produces a single product, began operations on January 1, Year 1. Material A is added at the start of the production process and packaging material B is added at the end of the process. Conversion costs are incurred uniformly throughout the process. Inspection takes place when manufacturing is completed, but before packaging material B is added. Spoiled units are discarded. Normal spoilage for this production process is 4% of good output. Production data for the first quarter of Year 1 was: Units started 18,000 units Good units completed and transferred-out 15,000 units Ending work-in-process inventory 2,000 units Using a first-in, first-out (FIFO) process costing system, Ex Company incurred the following costs per equivalent unit during the first quarter:

Material A $11.00 Material B $.80 Conversion costs $15.00

The cost of ending work-in-process inventory using FIFO process costing was $34,000. 9. The loss from abnormal spoilage for the first quarter was a) $16,080 b) $10,720 c) $10,400 d) $15,600 e) $26,800 10. In terms of conversion, what was the percentage of completion of the ending work-

in-process inventory? a) 65.4% complete b) 34.7% complete c) 54.5% complete d) 40.0% complete e) 63.4% complete

Page 220: MODULE 3 Financial Management and Management Accounting 1

Module 3 - Financial Management & Management Accounting 1

Page 220 © CMA Ontario, 2011

11. Mesa Verde Co. had the following production for the month of June: Work-in-process at June 1 20,000 units Units Started during June 80,000 Completed and transferred to finished goods 66,000 Normal spoilage incurred 4,000 Work-in-process at June 30 30,000 Materials are added at the beginning of the process. As to conversion costs, the

beginning work-in-process was 70% complete and the ending work-in-process was 60% complete. Spoilage is detected at the end of the process. Using the weighted average method, what were the equivalent units for June with respect to conversion costs?

a) 84,000 b) 88,000 c) 90,000 d) 100,000

Page 221: MODULE 3 Financial Management and Management Accounting 1

Module 3 - Financial Management & Management Accounting 1

Page 221 © CMA Ontario, 2011

Problem 1 Horton Plastic Works manufactures plastic milk cartons. In the manufacturing process, all materials are added at the start of work-in-process and conversion costs are incurred uniformly through the manufacturing process. The following data was reported for the most recent period:

Units in opening inventory 15,000 Units started this period 85,000 Units completed and transferred out 90,000 Spoilage 0 Cost of materials in opening inventory $19,000 Conversion costs in opening inventory $25,000 Materials costs incurred this period $62,000 Conversion costs incurred this period $465,000 Percentage completion opening inventory 40% Percentage completion ending inventory 80%

Required:

Prepare a process costing report for the current period assuming Horton Plastic Works uses weighted average costing.

Problem 2

Return to the data for Question 1. Prepare a process costing report for the current period assuming Horton Plastic Works uses FIFO costing.

Problem 3

Return to the data for Question 1 but make the following changes. An audit of the manufacturing process reveals the information provided in Question 1 is incorrect. Machine time is incurred at a steady rate during the process and because the work is machine paced, percentage completion is based on the completion of the total machine time. However, materials are introduced at various points of the conversion process as follows:

At the start 40% When machine time is 60% complete 40% When machine time is complete 20%

Also labour costs and manufacturing overhead are incurred at different rates during the production process. Labour is incurred 70% at the start of production and 30% when

Page 222: MODULE 3 Financial Management and Management Accounting 1

Module 3 - Financial Management & Management Accounting 1

Page 222 © CMA Ontario, 2011

machine time is 60% complete. Manufacturing overhead is applied uniformly through the manufacturing process. The components of conversion costs provided in the original data for Question 1 are:

Inventory Item

Conversion Costs

Labour Costs

Manufacturing Overhead

Opening $25,000 $8,000 $17,000 Costs added $465,000 $225,000 $240,000

With this information, and assuming Horton Plastic Works uses weighted average costing, recast the process costing report.

Problem 4

Repeat the analysis you did in Question 3 assuming Horton Plastic Works uses FIFO costing.

Problem 5 Suppose the audit also discovered the production personnel neglected to advise the management accountant that inspection occurs when the manufacturing process is complete. Horton Plastic Works has an allowance for normal spoilage equal to 2% of good production passing the inspection point. During the period, 2,100 units were found to be defective. The ending work-in-process count of 10,000 units is confirmed as correct. With this information and the additional information in Question 3, complete a process costing report assuming Horton Plastic Works uses weighted average costing.

Problem 6

Use the information provided in Question 5 to recast the FIFO report you prepared in Question 4.

Problem 7 More news... A machine operator has just told you that in order to isolate defective units as quickly as possible thereby saving money, at the beginning of the period just completed the inspection point was moved from when the production process is complete to a point where machine time is 59% complete. Recast your analysis in Question 5 to reflect this new information.

Page 223: MODULE 3 Financial Management and Management Accounting 1

Module 3 - Financial Management & Management Accounting 1

Page 223 © CMA Ontario, 2011

Problem 8 Sartarelli Corporation is a manufacturer that uses process costing to account for costs of production. Sartarelli manufactures a product in three separate departments: moulding, assembly and finishing. The following information was obtained for the assembling department for the month of June: Work-in-process, June 1 – 1,000 units made up of the following: Degree of Amount Completion Moulding department costs transferred in $32,000 100% Costs added by the assembly department Direct materials 20,000 100 Direct labour 7,200 60 Manufacturing overhead 5,500 50 Work-in-process, June 1 $64,700 During the month of June, 5,000 units were transferred in from the moulding department at a cost of $160,000. The assembly department added the following $150,000 of costs:

Direct materials $ 96,000 Direct labour 36,000 Manufacturing overhead 18,000 $150,000

Four thousand units were completed and transferred to the finishing department. At June 30, 2,000 units were still in WIP. The degree of completion of WIP at June 30 was:

Direct materials 90% Direct labour 70 Manufacturing overhead 35

Required: Calculate the cost of the units transferred out to the finishing department and the value of the ending WIP assuming the Sartarelli Company used (a) the weighted average method, and (b) the FIFO method.

Page 224: MODULE 3 Financial Management and Management Accounting 1

Module 3 - Financial Management & Management Accounting 1

Page 224 © CMA Ontario, 2011

Problem 9 The Gagnon Manufacturing Company uses a process costing system for its three production departments: initiation, assembly and finishing. The following data is available for the assembly department for the month of September: Opening WIP Units 15,000 Costs: Transferred in $97,650 Direct materials 186,450 Conversion costs (60% complete) 77,670 Units transferred in from the initiation department 46,000 Units transferred out to the finishing department 48,500 Costs added during month Transferred in $298,540 Direct materials 460,650 Conversion costs 382,956 Units in ending WIP (25% completed as to conversion costs) 9,000 The assembly department adds direct materials when the process if 40% complete. Inspection takes place when the process is 75% complete, at which time all spoiled units are detected. The engineering department has concluded any spoiled units in excess of 1,500 for the month of September can be considered abnormal spoilage. Required: Calculate the cost of the units transferred out to finished goods inventory, the value of the ending WIP and the value of abnormal spoilage under (a) the FIFO method, and (b) the weighted average method.

Page 225: MODULE 3 Financial Management and Management Accounting 1

Module 3 - Financial Management & Management Accounting 1

Page 225 © CMA Ontario, 2011

Problem 10 Tsizis Corporation makes a product called Turbulence in one department of the company. Direct materials are added at the beginning of the process. Labour and overhead are added continuously throughout the process. Spoilage, if any, occurs at the beginning of the process, just after materials have been added, but before any conversion costs have been incurred. The following information relates to production during November: 1. Work-in-process inventory, November 1 (4,000 kg – 75% complete):

Direct materials $22,800 Direct labour 24,650 Manufacturing overhead 21,860

2. Direct materials:

Inventory, November 1 – 2,000 kg $10,000 Purchases, November 3 – 10,000 kg $51,000 Purchases, November 18 – 10,000 kg $51,500 Sent to production during November – 16,000 kg

3. Direct labour costs – $103,350 4. Manufacturing overhead costs – $93,340 5. Transferred out to finished goods inventory – 15,000 kg 6. Work-in-process inventory, November 30, 3,000 kg, 33 1/3% complete as to

conversion costs. The FIFO method is used for both materials inventory valuation and for WIP inventories. The controller has determined any spoilage in excess of 1,300 units is abnormal spoilage. Required: Calculate the cost of the units transferred out to finished goods inventory and the value of the ending WIP. Calculate the value of abnormal spoilage.

Page 226: MODULE 3 Financial Management and Management Accounting 1

Module 3 - Financial Management & Management Accounting 1

Page 226 © CMA Ontario, 2011

Solutions: Multiple Choice Questions 1. d The difference between the FIFO method and weighted average method of

process costing in computing equivalent units of production is attributed to the work done in the beginning work-in process inventory. Since there were 6,000 units in the beginning work-in-process inventory, the equivalent units of production for transferred in costs under the FIFO method would be 6,000 less than of the weighted average method of process costing.

2. a Direct materials equivalent units = 42,000 - 3,000 = 39,000.

Alternative calculation = 34,000 + 5,000 = 39,000 3. e Conversion costs equivalent units

= 6,000 + 42,000 - (3,000 x .6) - (5,000 x .2) = 6,000 + 42,000 - 1,800 - 1,000 = 45,200

4. d Units transferred in = 42,000 and are 100% complete with regards to

moulding department costs. 5. b Equivalent units to complete the opening WIP: 8,000 units x 100% 8,000 Units started and completed: 38,000 - 8,000 30,000 38,000

6 b Equivalent units to complete the opening WIP: 8,000 units x 75% 6,000 Units started and completed: 38,000 - 8,000 30,000 Equivalent units in ending WIP: 12,000 x 40% 4,800 40,800

7. a FIFO method only considers current period costs in the numerator. 8. d Spoilage expected to arise under efficient operating conditions is considered

normal spoilage. It is an inherent part of the selected production process. The costs of normal spoilage are typically viewed as a part of the costs of good units produced.

9. c Total spoilage = 18,000 - 15,000 - 2,000 = 1,000 units

Normal spoilage = 4% x 15,000 = 600 units Abnormal spoilage = 1,000 - 600 = 400 units Loss from abnormal spoilage = 400 x ($11 + $15) = $10,400

Page 227: MODULE 3 Financial Management and Management Accounting 1

Module 3 - Financial Management & Management Accounting 1

Page 227 © CMA Ontario, 2011

10. d Conversion costs = cost of ending work-in-process inventory - cost of

material A = $34,000 - ($11 x 2,000) = $34,000 - $22,000 = $12,000 Equivalent units of conversion costs = $12,000 ÷ $15 = 800 units Percentage of completion = 800 equivalent units ÷ 2,000 total units = 40% complete

11. c 66,000 + 4,000 + (30,000 x 60%) = 88,000

Page 228: MODULE 3 Financial Management and Management Accounting 1

Module 3 - Financial Management & Management Accounting 1

Page 228 © CMA Ontario, 2011

Problem 1

Direct Conversion Equivalent Units - Materials Costs Transferred out 90,000 90,000

WIP - end

10,000 x 100% | 10,000 x 80% 10,000 8,000

100,000 98,000

Costs

WIP – beginning $19,000 $25,000

Costs Added 62,000 465,000

$81,000 $490,000

Unit Cost $0.81 $5.00

Total unit cost = $5.81

Units transferred out: 90,000 x $5.81 $522,900

Cost of ending WIP

Direct materials: 10,000 x $0.81 $8,100

Conversion costs: 8,000 x $5.00 40,000

$48,100

Work-in-Process

Finished Goods

B $ 44,000 522,900 TO 522,900

CA 527,000

E 48,100

Page 229: MODULE 3 Financial Management and Management Accounting 1

Module 3 - Financial Management & Management Accounting 1

Page 229 © CMA Ontario, 2011

Problem 2

Direct Conversion Equivalent Units - Materials Costs To complete opening WIP -

15,000 x 0% | 60% - 9,000

Units started and completed

90,000 – 15,000 75,000 75,000

WIP - end

10,000 x 100% | 10,000 x 80% 10,000 8,000

85,000 92,000

Costs Added 62,000 465,000

Unit Cost $0.7294 $5.0543

Total unit cost = $5.7837

Cost of units transferred out -

Costs in opening WIP $44,000

Costs to complete opening WIP: 9,000 x $5.0543 45,488

Cost of units started and completed: 75,000 x $5.7837 433,778

$523,266

Cost of ending WIP

Direct materials: 10,000 x $0.7294 $7,294

Conversion costs: 8,000 x $5.0543 40,434

$47,728

Work-in-Process

Finished Goods

B $ 44,000 523,266 TO 523,266

CA 527,000

E 49,734

Difference due to rounding.

Page 230: MODULE 3 Financial Management and Management Accounting 1

Module 3 - Financial Management & Management Accounting 1

Page 230 © CMA Ontario, 2011

Problem 3 The following time chart is useful in organizing the production process:

With this information, we can compute the number of equivalent units in ending inventory. Since ending inventory is 80% complete with regard to machine time, materials costs will be 80% (40% plus 40%) complete, labour costs will be 100% (70% plus 30%) complete and manufacturing overhead will be 80% complete. With this information the following process costing report would result:

Page 231: MODULE 3 Financial Management and Management Accounting 1

Module 3 - Financial Management & Management Accounting 1

Page 231 © CMA Ontario, 2011

Direct Direct Equivalent Units - Materials Labour Overhead Transferred out 90,000 90,000 90,000

WIP - end

10,000 x 80% | 100% | 80% 8,000 10,000 8,000

98,000 100,000 98,000

Costs

WIP – beginning $19,000 $8,000 $17,000

Costs Added 62,000 225,000 240,000

$81,000 $233,000 $257,000

Unit Cost $0.8265 $2.3300 $2.6224

Total unit cost = $5.7789

Units transferred out: 90,000 x $5.7789 $520,101

Cost of ending WIP

Direct materials: 8,000 x $0.8265 $6,612

Direct labour: 10,000 x $2.33 23,300

Conversion costs: 8,000 x $2.6224 20,979

$50,891

Work-in-Process

Finished Goods

B $ 44,000 520,101 TO 522,900

CA 527,000

E 50,899

Difference due to rounding.

Page 232: MODULE 3 Financial Management and Management Accounting 1

Module 3 - Financial Management & Management Accounting 1

Page 232 © CMA Ontario, 2011

Problem 4 To complete the FIFO costing, we need the information developed in Question 3 relating to equivalent units in ending information and information about equivalent units in opening inventory. Since opening inventory is 40% complete with regard to machine time, materials costs will be 40% complete, labour costs will be 70% complete and manufacturing overhead will be 40% complete. With this information, the following process costing report would result:

Direct Direct Equivalent Units - Materials Labour Overhead To complete opening WIP -

15,000 x 60% | 30% | 60% 9,000 4,500 9,000

Units started and completed

90,000 – 15,000 75,000 75,000 75,000

WIP - end

10,000 x 80% | 100% | 80% 8,000 10,000 8,000

92,000 89,500 92,000

Costs Added $62,000 $225,000 $240,000

Unit Cost $0.6739 $2.5140 $2.6087

Total unit cost = $5.7966

Cost of units transferred out -

Costs in opening WIP $44,000 Costs to complete opening WIP: 9,000 x $0.6739 6,065

4,500 x $2.5140 11,313

9,000 x $2.6087 23,478 Cost of units started and completed: 75,000 x $5.7966 434,745

$519,601

Cost of ending WIP

Direct materials: 8,000 x $0.6739 $5,391

Direct labour: 10,000 x $2.5140 25,140

Conversion costs: 8,000 x $2.6087 20,869

$51,401

Page 233: MODULE 3 Financial Management and Management Accounting 1

Module 3 - Financial Management & Management Accounting 1

Page 233 © CMA Ontario, 2011

Work-in-Process

Finished Goods

B $ 44,000 519,601 TO 519,601

CA 527,000

E 51,399

Difference due to rounding.

Page 234: MODULE 3 Financial Management and Management Accounting 1

Module 3 - Financial Management & Management Accounting 1

Page 234 © CMA Ontario, 2011

Problem 5 90,000 units reach the inspection point less 2,100 defective units = 87,900 good units passing the inspection point Normal spoilage = 87,900 x 2% = 1,758 units Abnormal spoilage = 2,100 – 1,758 = 342 units

Direct Direct Equivalent Units - Materials Labour Overhead Transferred out 87,900 87,900 87,900

Normal spoilage 1,758 1,758 1,758

Abnormal spoilage 342 342 342

WIP - end

10,000 x 80% | 100% | 80% 8,000 10,000 8,000

98,000 100,000 98,000

Costs

WIP – beginning $19,000 $8,000 $17,000

Costs Added 62,000 225,000 240,000

$81,000 $233,000 $257,000

Unit Cost $0.8265 $2.3300 $2.6224

Total unit cost = $5.7789

Units transferred out: 87,900 x $5.7789 $507,965

Normal spoilage: 1,758 x $5.7789 10,159

$518,124

Abnormal spoilage: 342 units x $5.7789 $1,976

Cost of ending WIP

Direct materials: 8,000 x $0.8265 $6,612

Direct labour: 10,000 x $2.33 23,300

Conversion costs: 8,000 x $2.6224 20,979

$50,891

Page 235: MODULE 3 Financial Management and Management Accounting 1

Module 3 - Financial Management & Management Accounting 1

Page 235 © CMA Ontario, 2011

Work-in-Process

Finished Goods

B $ 44,000 518,124 TO 518,124

CA 527,000 1,976 AS

E 50,900

Difference due to rounding.

Page 236: MODULE 3 Financial Management and Management Accounting 1

Module 3 - Financial Management & Management Accounting 1

Page 236 © CMA Ontario, 2011

Problem 6

Direct Direct Equivalent Units - Materials Labour Overhead To complete opening WIP -

15,000 x 60% | 30% | 60% 9,000 4,500 9,000

Units started and completed

87,900 – 15,000 72,900 72,900 72,900

Normal spoilage 1,758 1,758 1,758

Abnormal Spoilage 342 342 342

WIP - end

10,000 x 80% | 100% | 80% 8,000 10,000 8,000

92,000 89,500 92,000

Costs Added $62,000 $225,000 $240,000

Unit Cost $0.6739 $2.5140 $2.6087

Total unit cost = $5.7966

Cost of units transferred out -

Costs in opening WIP $44,000 Costs to complete opening WIP: 9,000 x $0.6739 6,065

4,500 x $2.5140 11,313

9,000 x $2.6087 23,478 Cost of units started and completed: 72,900 x $5.7966 422,572

Cost of normal spoilage

1,758 x $5.7966 10,190

$517,618

Cost of abnormal spoilage

342 x $5.7966 $1,982

Cost of ending WIP

Direct materials: 8,000 x $0.6739 $5,391

Direct labour: 10,000 x $2.5140 25,140

Conversion costs: 8,000 x $2.6087 20,869

$51,401

Page 237: MODULE 3 Financial Management and Management Accounting 1

Module 3 - Financial Management & Management Accounting 1

Page 237 © CMA Ontario, 2011

Work-in-Process

Finished Goods

B $ 44,000 517,618 TO 517,618

CA 527,000 1,982 AS

E 51,400

Difference due to rounding.

Page 238: MODULE 3 Financial Management and Management Accounting 1

Module 3 - Financial Management & Management Accounting 1

Page 238 © CMA Ontario, 2011

Problem 7 Inspection takes place when machine time is 59% complete. Therefore, units failing inspection will be 40% complete with respect to materials costs, 70% complete with respect to labour costs and 59% complete with respect to machine time. With inspection occurring during the manufacturing process costing becomes more complex. Recall that units in opening inventory had not been through the inspection process but units in ending inventory have. Therefore, all units in the system passed through the inspection point this period. Therefore, 100,000 units were inspected. Since 2,100 were defective, the number of good units was 97,900 so normal spoilage would be 1,958 units (97,900 * 0.02) and abnormal spoilage would be 142 (2,100-1,958) units. Since there are inspected units in ending work-in-process, those units should be assigned their share of normal spoilage costs. That share will be in proportion to good units in ending inventory and good units transferred out. Therefore, normal spoilage will be allocated 10,000/87,900) to ending inventory and the balance (77,900/87,900) to work completed and transferred out. The following process costing report would result:

Direct Direct Equivalent Units - Materials Labour Overhead Transferred out 87,900.0 87,900.0 87,900.0

Normal spoilage: 1,958 x 40% | 70% | 59% 783.2 1,370.6 1,155.2

Abnormal spoilage: 142 x 40% | 70% | 59% 56.8 99.4 83.8

WIP - end

10,000 x 80% | 100% | 80% 8,000 10,000 8,000

96,740.1 99,370.0 97,139.0

Costs

WIP – beginning $19,000 $8,000 $17,000

Costs Added 62,000 225,000 240,000

$81,000 $233,000 $257,000

Unit Cost $0.8373 $2.3448 $2.6457

Total unit cost = $5.8278

Page 239: MODULE 3 Financial Management and Management Accounting 1

Module 3 - Financial Management & Management Accounting 1

Page 239 © CMA Ontario, 2011

Work-in-Process

Finished Goods

B $ 44,000 519,190 TO 518,124

CA 527,000 503 AS

E 51,307

Difference due to rounding.

Units transferred out: 87,900 x $5.8278 $512,264

Normal spoilage:

DM: 783.2 x $0.8373 656

DL: 1,370.6 x $2.3448 3,214

Overhead: 1,155.2 x $2.6457 3,056

$519,190

Abnormal spoilage:

DM: 56.8 x $0.8373 $48

DL: 99.4 x $2.3448 233

Overhead: 83.8 x $2.6457 222

$503

Cost of ending WIP

Direct materials: 8,000 x $0.8373 $6,698

Direct labour: 10,000 x $2.3448 23,448

Conversion costs: 8,000 x $2.6457 21,166

$51,312

Page 240: MODULE 3 Financial Management and Management Accounting 1

Module 3 - Financial Management & Management Accounting 1

Page 240 © CMA Ontario, 2011

Problem 8

a) Transferred

In Costs Direct

Materials Direct

Labour Mfg.

Overhead Equivalent units Units transferred out 4,000 4,000 4,000 4,000 Ending WIP 2,000 x 100% 2,000 2,000 x 90% 1,800 2,000 x 70% 1,400 2,000 x 35% 700 6,000 5,800 5,400 4,700

Costs in opening WIP $32,000 $ 20,000 $ 7,200 $ 5,500 Costs added 160,000 96,000 36,000 18,000 $192,000 $116,000 $43,200 $23,500

Cost per unit $32 $20 $8 $5

Total cost/unit = $65 Cost of goods transferred out Units transferred out: 4,000 x $65 $260,000

Cost of ending inventory Transferred in costs: 2,000 x $32 $64,000 Direct materials: 1,800 x $20 36,000 Direct labour: 1,400 x $8 11,200 Manufacturing overhead: 700 x $5 3,500 $114,700

T-account perspective: WIP – Assembly WIP – Finishing B $64,700 260,000 TO TI 260,000 TI 160,000 CA 150,000 E 114,700

Page 241: MODULE 3 Financial Management and Management Accounting 1

Module 3 - Financial Management & Management Accounting 1

Page 241 © CMA Ontario, 2011

b) Transferred In Costs

Direct Materials

Direct Labour

Mfg Overhead

Equivalent units Units in opening WIP 1,000 x 0% 0 1,000 x 0% 0 1,000 x 40% 400 1,000 x 50% 500 Units started and

completed 3,000 3,000 3,000 3,000

Ending WIP 2,000 x 100% 2,000 2,000 x 90% 1,800 2,000 x 70% 1,400 2,000 x 35% 700 5,000 4,800 4,800 4,200 Costs added $160,000 $96,000 $36,000 $18,000 Cost per unit $32 $20 $7.50 $4.2857

Total cost/unit = $63.7857 Cost of goods transferred out Costs in opening WIP $64,700 Cost to complete opening WIP Direct labour: 400 x $7.50 3,000 Manufacturing overhead: 500 x $4.2857 2,143 69,843 Units started and completed: 3,000 x $63.7857 191,357 $261,200

Cost of ending inventory Transferred in costs: 2,000 x $32 $64,000 Direct materials: 1,800 x $20 36,000 Direct labour: 1,400 x $7.50 10,500 Manufacturing overhead: 700 x $4.2857 3,000 $113,500

Page 242: MODULE 3 Financial Management and Management Accounting 1

Module 3 - Financial Management & Management Accounting 1

Page 242 © CMA Ontario, 2011

T-account perspective: WIP – Assembly WIP – Finishing B $64,700 261,200 TO TI 261,200 TI 160,000 CA 150,000 E 113,500

Page 243: MODULE 3 Financial Management and Management Accounting 1

Module 3 - Financial Management & Management Accounting 1

Page 243 © CMA Ontario, 2011

Problem 9 Part (a): Total spoilage WIP – beginning 15,000 Started 46,000 Transferred out (48,500) WIP – ending (9,000) 3,500 Less: normal spoilage 1,500 Abnormal spoilage 2,000

Transf-in

Costs Direct

Materials Conversion

Costs Equivalent Units To complete opening WIP: 15,000 x 40%

6,000

Good units started and completed 33,500 33,500 33,500 Normal spoilage 1,500 1,500 1,125 Abnormal spoilage 2,000 2,000 1,500 Ending WIP - 9,000 x 100% | 9,000 x 25% 9,000 - 2,250 46,000 37,000 44,375 Costs added $298,540 $460,650 $382,956 Cost per unit $6.49 $12.45 $8.63

Total cost = $27.57 Cost of units transferred out to finished goods inventory: Cost in opening WIP $361,770 Cost to complete opening WIP CC: 6,000 x $8.63 51,780 $413,550 Cost of units started and completed 33,500 x $27.57 923,595 Cost of normal spoilage TI: 1,500 x $6.49 $ 9,735 DM: 1,500 x $12.45 18,675 CC: 1,125 x $8.63 9,709 38,119 $1,375,264

Page 244: MODULE 3 Financial Management and Management Accounting 1

Module 3 - Financial Management & Management Accounting 1

Page 244 © CMA Ontario, 2011

Cost of ending WIP TI: 9,000 x $6.49 $58,410 CC: 2,250 x $8.63 19,417 $77,827

Cost of abnormal spoilage TI: 2,000 x $6.49 $12,980 DM: 2,000 x $12.45 24,900 CC: 1,500 x $8.63 12,945 $50,825

T-Account Perspective: WIP – Assembly WIP – Finishing B $361,770 1,375,264 TO TI 1,375,264 TI 298,540 50,825 AS DM 460,650 CC 382,956 E 77,826

Page 245: MODULE 3 Financial Management and Management Accounting 1

Module 3 - Financial Management & Management Accounting 1

Page 245 © CMA Ontario, 2011

Part (b): Transfer

In Costs Direct

Materials Conversion

Costs Equivalent units Units transferred out 48,500 48,500 48,500 Normal spoilage 1,500 1,500 1,125 Abnormal spoilage 2,000 2,000 1,500 Ending WIP - 9,000 x 100% | 9,000 x 25% 9,000 - 2,250 61,000 52,000 53,375 Costs in OP WIP + costs added $396,190 $647,100 $460,626 Cost per unit $6.49492 $12.44423 $8.63000

Total cost = $27.56915 Cost of units transferred out to finished goods inventory: Cost of units transferred out 48,500 x $27.56915 $1,337,104 Cost of normal spoilage TI: 1,500 x $6.49492 $ 9,742 DM: 1,500 x $12.44423 18,666 CC: 1,125 x $8.63000 9,709 38,117 $1,375,221

Cost of ending WIP TI: 9,000 x $6.49492 $58,455 CC: 2,250 x $8.63000 19,417 $77,872

Cost of abnormal spoilage TI: 2,000 x $6.49492 $12,990 DM: 2,000 x $12.44423 24,888 CC: 1,500 x $8.63000 12,945 $50,823

Page 246: MODULE 3 Financial Management and Management Accounting 1

Module 3 - Financial Management & Management Accounting 1

Page 246 © CMA Ontario, 2011

T-account perspective: WIP – Assembly WIP – Finishing B $361,770 1,375,221 TO TI 1,375,221 TI 298,540 50,823 AS DM 460,650 CC 382,956 E 77,872

Page 247: MODULE 3 Financial Management and Management Accounting 1

Module 3 - Financial Management & Management Accounting 1

Page 247 © CMA Ontario, 2011

Problem 10 Total spoilage: WIP – beginning 4,000 Started 16,000 Transferred out (15,000) WIP – ending (3,000) 2,000 Less: normal spoilage 1,300 Abnormal spoilage 700

Direct

Materials Conversion

Costs Equivalent Units: To complete opening WIP: 4,000 x 25% 1,000 Good units started and completed 11,000 11,000 Normal spoilage 1,300 -0- Abnormal spoilage 700 -0- Ending WIP: 3,000 x 100% | 3,000 x 33 1/3 % 3,000 1,000 16,000 13,000 Costs added* $81,600 $196,690 Cost per unit $5.10 $15.13

Total cost = $20.23 * Materials used: From opening inventory: 2,000 kg $10,000 From Nov. 3 purchase: 10,000 kg 51,000 From Nov. 18 purchase 4,000 kg x $51,500 / 10,000 20,600 $81,600

Cost of units transferred out to Finished Goods Inventory: Cost in opening WIP $69,310 Cost to complete opening WIP 1,000 x $15.13 15,130 $84,440 Cost of units started and completed 11,000 x $20.23 222,530 Cost of normal spoilage 1,300 x $5.10 6,630 $313,600

Page 248: MODULE 3 Financial Management and Management Accounting 1

Module 3 - Financial Management & Management Accounting 1

Page 248 © CMA Ontario, 2011

Cost of ending WIP Direct materials: 3,000 x $5.10 $15,300 Conversion costs: 1,000 x $15.13 15,130 $30,430

Cost of abnormal spoilage 700 x $5.10 $3,570

T-account perspective: WIP Finished Goods B $69,310 313,600 TO TI 313,600 DM 81,600 3,570 AS OH 196,690 E 30,430

Page 249: MODULE 3 Financial Management and Management Accounting 1

Module 3 - Financial Management & Management Accounting 1

Page 249 © CMA Ontario, 2011

5. Cost Behaviour Learning Objectives

After completing this chapter, you will:

1. Understand why knowing cost behaviour is important for management accountants

2. Understand the issues and limitations in using historical cost data to estimate cost behaviour

3. Understand and be able to apply the five tools management accountants use to estimate cost behaviour

4. Understand how to evaluate statistically based estimates of cost behaviour

The Importance of Good Cost Information

The role of good cost estimates is pervasive in management accounting. Managers use cost estimates to develop budgets, evaluate performance relative to plan, price new products and evaluate the profitability of existing products. In short, the importance of management accountants developing a solid understanding of cost behaviour and developing good estimates of costs is critical in supporting management control and strategic analysis.

Boston’s Central Artery/Tunnel Project (The Big Dig) Estimated to cost $2.9 billion in 1985 at the outset of planning for Boston’s central artery/tunnel project and to be completed in 1995, the eventual cost of the project was estimated to be $22 billion when the project was completed in 2007. Observers have attributed the cost overruns to management and contractor incompetence. Picture Source: Boston Globe

Canada’s Gun Registry Estimated to cost less than $2 million when it was introduced in 1995, in 2004, CBC news estimated the cost to date of Canada’s Gun Registry was $2 billion. Cost overruns were attributed mainly to unexpected computer systems and legal costs. Picture Source: CBC

Page 250: MODULE 3 Financial Management and Management Accounting 1

Module 3 - Financial Management & Management Accounting 1

Page 250 © CMA Ontario, 2011

Approaches to Cost Estimation

The cost estimation process begins by defining the cost behaviour the analyst wants to estimate/understand/predict. Examples include manufacturing overhead cost, materials costs, labour costs or even total product cost. The next step is to identify the cost driver or cost drivers the analyst believes drives or causes the cost behaviour the analyst wants to estimate. The analyst can then use various approaches to develop the equation that will be used to predict the cost behaviour.

There are five common approaches to estimating costs: two involve the basic application of process knowledge and judgment and three involve the use of historical data.

Judgment approaches:

The two judgement approaches exploit a good understanding of the process that creates the cost to develop a plausible model of explaining the cost behaviour.

Engineering estimates:

The engineering estimate approach involves the estimation of costs based on the analysis of product (good or service) specifications. This approach follows in the tradition of Frederick Taylor’s scientific management method and the industrial engineering ideas developed by Frank and Lilian Gilbreth. In the engineering estimate approach, all required material, labour time and machine time components of the product are laid out and the cost of each is estimated. Then, the costs of all the components are summed to estimate the product costs.

Account analysis:

Account analysis involves the evaluation of each of the costs in a ledger account and classifying each cost as fixed or variable with regard to a cost driver.

For example, a study of the items in the manufacturing overhead account might lead the cost analyst to classify supplies as variable, machine depreciation as fixed, supervisory salaries as fixed, taxes on factory land as fixed, power as variable and so on. The cost analyst would then accumulate the classified costs into two pools, variable and fixed costs to create an equation estimating cost behaviour.

Data approaches:

In the data, approaches the management accountant uses historical operations data to estimate cost behaviour. By considering the historical pattern of costs, the data approaches have the greatest potential to yield realistic estimates of cost behaviour. However, there are inevitable characteristics of historical cost data that can impair their relevance and usefulness in cost estimation.

Contaminants of historical cost data:

Page 251: MODULE 3 Financial Management and Management Accounting 1

Module 3 - Financial Management & Management Accounting 1

Page 251 © CMA Ontario, 2011

The maintained assumption when management accountants use historical cost data is there has been no underlying change in the manufacturing process that is not captured in the recorded data. Statisticians call this the omitted variable problem. The longer the period over which the recorded data stretches extends the possibility there has been some change (such as changes in technology, product design, learning and inflation) that can affect costs, but which has not been picked up in the measurement process.

Another potential contaminant of historical cost data is inaccurate cost accrual. The shorter the cost recording period, for example weekly or monthly, the greater the chance accruals are less reliable and, therefore, misleading in the cost estimation process. For example, costs driven by an activity in one period might be inaccurately assigned to the next period.

Generally, the management accountant is forced to rely on quarterly accruals to provide a minimum assurance of data integrity. This means only four observations can be made a year to develop the cost estimation database. Since the rule of thumb is a minimum of 20 observations are needed to provide a reliable cost estimate, this means five years of time series data is needed to provide a reasonable database for a cost estimate. The five year time period, however, introduces the possibility of important changes in the manufacturing environment, such as technological innovation that may have taken place and is not captured in the data, thereby, creating errors in cost estimation.

The way the accounting system treats costs can create issues of data integrity. For example, if fixed costs are allocated to production based on an activity level, such as units made, labour hours or machine hours, what is actually a fixed cost may appear to be variable.

Continuous growth creates issues when using regression analysis. Using an estimated cost equation to predict costs beyond the range of experience is inappropriate since this can result in significant estimation errors.

Observation Units Cost

1 812 145,300

2 586 139,650

3 845 151,125

4 863 151,575

5 852 146,300

6 698 147,450

7 610 135,250

8 614 145,350

9 642 136,050

10 836 140,900

11 759 143,975

12 693 137,325

13 553 133,825

14 512 127,800

15 707 147,675

16 802 145,050

17 635 135,875

18 629 145,725

19 773 139,325

Page 252: MODULE 3 Financial Management and Management Accounting 1

Module 3 - Financial Management & Management Accounting 1

Page 252 © CMA Ontario, 2011

Finally, there is the troubling issue of outliers, which are data points that are significantly different from the rest. These extreme points create issues in cost estimation, particularly if regression analysis is used. The usual treatment for an unusual item is to try to identify the cause of the observation. If it is determined the cause was a non-recurring item such as a natural disaster or a strike, the usual approach is to eliminate the outlier from the data set. Most analysts will avoid eliminating an outlier from a data set if its cause cannot be determined on the grounds that if the outlier cannot be explained, then the circumstances that created the outlier cannot be accommodated in cost estimation.

High-Low Method

Suppose the data in the accompanying table for Danny Company was recorded during the past 20 periods. Note that this data assumes the number of units made is the appropriate cost driver.

In the high-low method, the management accountant computes the formula for the line connecting the lowest and highest values of the cost driver.

Therefore, the pair of observations that would form the basis for estimating the cost relationship would be:

Highest level of cost driver/cost- observation 20 872/146,800

Lowest level of cost driver/cost – observation 14 512/127800

The slope of the line connecting the lowest and highest values of cost driver is computed using the following formula: Slope = cost associated with the highest level of cost driver - cost associated with the lowest level of cost driver highest value of cost driver - lowest value of cost driver

= (146,800 - 127,800) / (872 - 512) = 19,000 / 360 = $52.778 Substitute the slope value in either pair of observations to find the fixed cost estimate

1. High driver value observation fixed cost estimate = 146,800 – 52.778*872 = 100,778

2. Low cost observation fixed cost estimate = 127,800 – 52.778*512 = 100,778 The estimate of the cost function using the high low method is

Total cost = 100,778 + 52.778*number of units made

20 872 146,800

Page 253: MODULE 3 Financial Management and Management Accounting 1

Module 3 - Financial Management & Management Accounting 1

Page 253 © CMA Ontario, 2011

The following is a plot of the data showing the high low estimate of the cost function:

The primary advantage of the high low method is it is easy and straight forward to apply. The major disadvantage is it ignores all data except the two points used to develop the cost estimate. This creates the possibility that the end points are not representative of the data that has been gathered. A second disadvantage is this approach can only be used with a single cost driver, in this case the number of units made. More on cost estimation with multiple cost drivers follows below.

Visual fit:

The second approach using cost data is the visual fit method. In this approach, the analyst draws a line through the data that seems to reflect the relationship between cost and the cost driver. This approach is a particularly useful as an alternative to the high low method if the end points in the high low method do not appear to be representative of the other data. This approach is subjective and reflects the personal judgment of the analyst in fitting the line representing the cost equation to the data.

Page 254: MODULE 3 Financial Management and Management Accounting 1

Module 3 - Financial Management & Management Accounting 1

Page 254 © CMA Ontario, 2011

Suppose the analyst chooses the line shown in the figure above on the grounds it appears to pass through the centre of data after ignoring the observations associated with the lower levels of the cost driver on the grounds there may be cost inefficiencies at lower production levels. The driver value/cost coordinates of the end points of the line shown in the diagram below are 872/146,800 and 586/139,650. The cost estimation equation would be developed as follows:

Slope = cost associated with the highest units observed - cost associated with the lowest units observed highest units observed - lowest units observed

= (146,800 - 139,650) / (872 - 586) = 7,150 / 286 = $25

Substitute the slope value in either of the pair of observations to find the fixed cost estimate

1. High cost observation fixed cost estimate = 146800 – 25*872 = 125,000 2. Low cost observation fixed cost estimate = 139650 – 25*586 = 125,000

The estimate of the cost function using visual inspection is

Total cost = 125,000 + 25*number of units made Note the significant difference between the two cost estimates.

High-low estimate: total cost = 10,778 + 52.778*number of units made Visual inspection estimate: total cost = 125,000 + 25*number of units made

This shows how sensitive these approaches are to the assumptions made when generating them. Statistical/regression approach: We now consider the formal statistical approach to estimating the statistical relationship

Page 255: MODULE 3 Financial Management and Management Accounting 1

Module 3 - Financial Management & Management Accounting 1

Page 255 © CMA Ontario, 2011

appearing in a set of data, which is a way of considering all data (unlike the high-low method) and not relying on subjective judgment (unlike the visual inspection method). The regression approach fits an equation to the observed data using the criterion of minimizing the sum of the squared differences between the values predicted by the regression equation and the original data. This criterion is summarized by the statistic r-square (r2), which is called the goodness of fit measure. The first step in applying the regression model is to choose the cost to be estimated (called the dependent variable because it is assumed to be dependent upon the value of the cost driver(s)). The second step is to choose the cost driver(s), called the independent variable, which is/are assumed to cause the behaviour of the independent variable. (We will consider the approach of using multiple independent variables later) Note that when using the term cost driver, we are implying we believe there is a plausible causal relationship between the cost driver and the dependent variable. In this case, we have chosen the total cost as the dependent variable and number of units made as the independent variable. This is a plausible relationship. Plausibility is important because we cannot infer causality from a high goodness of fit (r2). A high goodness of fit only implies a statistical relationship between the dependent variable and the independent variable(s) and not causality. Statisticians use the term spurious correlation to refer to a situation where there is a strong statistical relationship between two variables, but no cause and effect relationship between those variables. To illustrate how this can happen, suppose event x causes both event y and event z to happen. As x moves, so will y and z and, therefore, y and z will move together. Someone might observe y and z moving together and infer that one is causing the other, which is not the case, it is x causing them to move.

Page 256: MODULE 3 Financial Management and Management Accounting 1

Module 3 - Financial Management & Management Accounting 1

Page 256 © CMA Ontario, 2011

The following is an example that is used to illustrate spurious correlation:

Someone observes that the more firefighters there are at a fire the more damage occurs and concludes the firefighters caused the damage. Of course, it is the severity of the fire that caused both more firefighters to be called to the scene and the amount of damage.

So when choosing cost drivers to develop a cost estimation equation, we must always be sure to pick one that makes sense and that should always be guided by experience of the process. The third step in using regression is to plot the dependent variable (by convention on the y or vertical axis) vs. the independent variable (by convention on the x or horizontal axis). We do this to determine where the relationship appears to be linear since the linear regression model assumes the relationship between the two variables is approximately linear. The following is a plot of data that was observed in a study of the labour cost required to

Page 257: MODULE 3 Financial Management and Management Accounting 1

Module 3 - Financial Management & Management Accounting 1

Page 257 © CMA Ontario, 2011

produce a complex piece of electronic equipment:

This relationship, which reflects a phenomenon called a learning effect, is common in labour-paced assembly operations. This relationship is clearly not linear and using a linear equation to estimate this relationship would lead to significant errors. Fortunately, there are straightforward ways of dealing with situations like this. (In fact, the equation estimating this relationship is shown on the graph.) However, discussion of dealing with non-linear relationships is beyond the scope of these notes. Returning to Danny Company, we can see by inspecting the plot of total cost to units produced when we used the high-low method, the relationship is approximately linear. The fourth step is to undertake the regression analysis. While there is specialized software for regression analysis, there are reasonable regression tools in most spreadsheet packages such as Microsoft’s Excel and Open Office’s Calc. If we enter the data for this problem into an Excel spreadsheet and use Excel to undertake a regression analysis, the following output will be produced:

Page 258: MODULE 3 Financial Management and Management Accounting 1

Module 3 - Financial Management & Management Accounting 1

Page 258 © CMA Ontario, 2011

The Estimated Coefficients: The regression model has estimated the following cost equation

Total cost = 113,520 + 40.014*units produced The fifth step is to interpret the regression results. There are two items in the above output we will interpret in these notes. Goodness of Fit: You will note there are three goodness of fit measures that Excel reports for each regression.

1. Multiple R, which is a measure of correlation between the independent variable and the dependent variable,

2. R-square, which is the correlation measure squared, and 3. Adjusted r-square, which is the r-square value adjusted for the number of

independent variables. Statisticians believe the adjusted r-square is the best measure of association between the independent variable and the dependent variable. Adjusted r-square is interpreted as the proportion of variability in the dependent variable that is explained by the independent variable. Following this convention, we will use adjusted r-square measure as the measure of goodness of fit in our discussions that follow.

Therefore, the goodness of fit measure for this regression is 49.52%.

Page 259: MODULE 3 Financial Management and Management Accounting 1

Module 3 - Financial Management & Management Accounting 1

Page 259 © CMA Ontario, 2011

The t-statistic:

The t-statistic is actually used in a formal statistical test of the hypothesis that the coefficient associated with the independent variable is zero. If the absolute value of the t-statistic is greater than a critical value, we reject the null hypothesis that the true value of the coefficient associated with the independent variable is zero.

We will not discuss the formal statistical tests in these notes. Instead, we will use the common approximation (rule of thumb) that if the absolute value of the t-statistic is two or greater, we conclude there is a statistically significant relationship between the independent variable and the dependent variable. In every day terms, what we are saying is the true value of the coefficient is not zero and there is a relationship between the dependent variable and the independent variable (the assumed cost driver).

In this case the value of the t-statistic associated with the estimated coefficient is greater than two, so we conclude there is a relationship between units produced and total cost.

The following is a summary diagram showing the estimated regression equation (solid line), the high low equation (dotted line) and the visual fit equation (dashed line):

We can see that the estimated relationships converge on the right hand side at higher values for the cost driver but diverge substantially at lower values of the cost driver.

Page 260: MODULE 3 Financial Management and Management Accounting 1

Module 3 - Financial Management & Management Accounting 1

Page 260 © CMA Ontario, 2011

Suppose, as suggested in the discussion of the visual fit, the analyst believes some other phenomenon was affecting costs at the two lowest levels of the cost driver. This situation is handled by regressions using multiple cost drivers.

Using Multiple Cost Drivers (Independent Variables):

Linear regression will accommodate multiple cost drivers when the cost analyst believes there is more than one cost driver affecting the value of the dependent variable. Simple regression is the term used when a single cost driver (independent variable) is used. Multiple regression is the term when more than one cost driver (independent variable) is used.

After viewing the results of the simple regression model reported for Danny Company above, the cost analyst interviewed production staff and determined that two low production periods occurred when the most recently hired production staff was attending a quality control course. More experienced staff managed the production process during these two periods, resulting in higher levels of efficiency and lower labour costs. The cost analyst decided to add a second independent variable to account for this situation and prepared a revised data table. As you can see from the table below, the second independent variable is coded with a zero for a non-training period and a one for the training period. If the information is correct, the coefficient associated with the training variable should be negative to account for increased productivity and lower costs during the training periods.

Observation Units Training Period Cost

1 812 0 145,300 2 586 0 139,650 3 845 0 151,125 4 863 0 151,575 5 852 0 146,300 6 698 0 147,450 7 610 0 135,250 8 614 0 145,350 9 642 0 136,050

10 836 0 140,900 11 759 0 143,975 12 693 0 137,325 13 553 1 133,825 14 512 1 127,800 15 707 0 147,675 16 802 0 145,050 17 635 0 135,875 18 629 0 145,725 19 773 0 139,325 20 872 0 146,800

The following is the regression result using these two independent variables:

Page 261: MODULE 3 Financial Management and Management Accounting 1

Module 3 - Financial Management & Management Accounting 1

Page 261 © CMA Ontario, 2011

Note that the regression has estimated the following relationship:

Total cost = 30.492*units made – 6,388.40*(1 if this is a training period, 0 otherwise) + 120,964

The adjusted r-square is 54.20% which is an improvement over the simple regression result of 49.52% we saw earlier in the simple regression.

The t-statistics associated with the estimated values of the coefficients associated with the number of units produced and the training period variable are 2.96 and -1.68, respectively. This means the estimated value of the units produced coefficient is statistically significant but the estimated coefficient for the training period is not. While we can use the regression to predict total costs, it is not reasonable to make inferences about the coefficient associated with the training period (that is, the effect of the training period on total costs). As a practical matter, it would be best to throw out this regression and use the regression using only the units variable.

Other Statistical Tests:

There are important tests that statisticians perform, mostly on the error terms (the difference between the values predicted by the regression equation and the actual values) to determine whether the estimated regression equation is reliable. While these tests are important for practicing management accountants, this discussion is beyond the scope of this review.

An Example – Estimating Route Time

Step 1 – determine the item of interest – dependent variable:

Always on Time Courier (ATC) Company is interested in estimating the time required for its couriers to complete their routes and then to identify the cost of delivering packages to different customers.

Page 262: MODULE 3 Financial Management and Management Accounting 1

Module 3 - Financial Management & Management Accounting 1

Page 262 © CMA Ontario, 2011

Step 2 – identify plausible time drivers:

After speaking with the drivers the management accountant believes the time required to complete the route is determined by (that is the time drivers are): number of packages to be delivered, number of stops on the route and the length of the route. This establishes the plausibility of the drivers in explaining the total route time – remember evaluative plausibility is done to avoid identifying a spurious relationship. The following data has been gathered by sampling what are considered to be the average routes.

Page 263: MODULE 3 Financial Management and Management Accounting 1

Module 3 - Financial Management & Management Accounting 1

Page 263 © CMA Ontario, 2011

Step 3 – plot the candidate drivers against the dependent variable:

The following are the plots of the total time against each of the three candidate time drivers:

Page 264: MODULE 3 Financial Management and Management Accounting 1

Module 3 - Financial Management & Management Accounting 1

Page 264 © CMA Ontario, 2011

The plots of route time vs. number of stops and route length appear to show linear relationships. The plot of route time vs. number of packages appears indeterminate or almost random.

Step 4 – run the regression:

The following are the results produced by Excel for this regression:

The relationship estimated by the regression is:

Route time in minutes = 11.39+1.423*number of packages

+14.761*number of stops+4.550*number of kilometres on route

In practice, the analyst would speak with the couriers to determine whether these values seem reasonable as test of the regression results use experience and process understanding.

Step 5 – evaluate the results:

Goodness of fit:

The adjusted r-square is 69.21%, which is good.

T-statistics

The t statistics of all the estimated time driver coefficients exceed the rule of thumb value of two.

Conclusion:

This is a model that would likely be helpful in planning for route times. Although approximately 30% of the time variability remains unexplained, this is likely the random traffic or customer effects on the total route time.

Page 265: MODULE 3 Financial Management and Management Accounting 1

Module 3 - Financial Management & Management Accounting 1

Page 265 © CMA Ontario, 2011

An Example – Estimating Overhead Cost

Princeton Metal Works manufactures copper and steel plumbing products. Products are made on machines in batches to the customers’ orders.

Steps 1 and 2

The plant accountant, wanting to develop a better estimate of overhead to support planning and product costing, asked plant personnel to help identify what they felt were the cost drivers for plant overhead. The plant personnel believed that units made, labour hours, machine hours, number of batches of products made and the number of products made during the period were all plausible candidates as cost drivers for plant manufacturing overhead. Over the next several months the following data was gathered:

Page 266: MODULE 3 Financial Management and Management Accounting 1

Module 3 - Financial Management & Management Accounting 1

Page 266 © CMA Ontario, 2011

Page 267: MODULE 3 Financial Management and Management Accounting 1

Module 3 - Financial Management & Management Accounting 1

Page 267 © CMA Ontario, 2011

Step 3:

The following are the plots of the variables identified as possible cost drivers (independent variables) for manufacturing overhead costs:

None these plots, except the plot of overhead vs. the number of products, appear to show violations of the linearity assumption.

Step 4 – run the regression:

The following is the regression result provided by Excel:

Page 268: MODULE 3 Financial Management and Management Accounting 1

Module 3 - Financial Management & Management Accounting 1

Page 268 © CMA Ontario, 2011

Step 5 – evaluate the results:

There are obvious problems evident in these results. The first and most glaring problem is the coefficient estimated for the number of units is negative. It is neither plausible nor sensible that as the number of units made goes up, the manufacturing overhead would go down. This is clear evidence there is a problem in this regression result. Moreover, the absolute value of the t-statistic estimated for number of units and labour hours are both less than two.

Usually, but not always, a counter intuitive result like an unexpected estimated coefficient sign occurs when one or more of the independent variables are highly correlated, which is true in this case. The following correlation matrix shows that variable units is highly correlated with both labour hours and number of products.

How to decide what variables to eliminate when there are correlated independent variables (the rule of thumb is the absolute value of the correlation should be less than .5) is beyond the scope of these notes. Assume that through a process of elimination, the analyst decides to use machine hours and number of products as the independent variables. The result follows:

Page 269: MODULE 3 Financial Management and Management Accounting 1

Module 3 - Financial Management & Management Accounting 1

Page 269 © CMA Ontario, 2011

We now have a statistically significant regression equation that explains about 43% of the variation in the manufacturing overhead. Our estimated equation is:

Manufacturing overhead = 123,702 + 153.31*machine hours + 5,594.56*number of products

This estimate would be shown to production personnel for their evaluation, based on their hands on experience with the production process, as to whether this estimate appears to be plausible.

Summary:

Good understanding of cost behaviour provides important input to many management activities including budgeting, pricing new products and performance evaluation. Management accountants have different tools to estimate costs one being the analysis of historical cost data. Using historical cost data can be problematic for a number of reasons including reliability of data and the confidence that users place in estimated cost behaviour. An important requirement is that the management accountant conveys to the users of cost behaviour estimates the amount of confidence and potential limitations of the estimates.

Page 270: MODULE 3 Financial Management and Management Accounting 1

Module 3 - Financial Management & Management Accounting 1

Page 270 © CMA Ontario, 2011

Problems with Solutions Multiple Choice Questions

(Note that for questions 1 – 6, there may be more than one correct answer)

1. Which of the following statements is/are true about the high low method of estimating cost behaviour?

a. Uses all the historical data available b. Can be affected by extreme points c. Requires a good knowledge of the production process d. Requires a good knowledge of product specifications

2. Which of the following statements is/are true about the account analysis method of

estimating cost behaviour?

a. Uses all the historical data available b. Can be affected by extreme points c. Requires a good knowledge of the production process d. Requires a good knowledge of product specifications

3. Which of the following statements is/are true about the engineering method of estimating cost behaviour?

a. Uses all the historical data available b. Can be affected by extreme points c. Requires detailed knowledge of the production process d. Requires a good knowledge of product specifications

4. Which of the following statements is/are true about the visual inspection method of estimating cost behaviour?

a. Uses all the historical data available b. Can be affected by extreme points c. Requires detailed knowledge of the production process d. Requires a good knowledge of product specifications

5. Which of the following statements is/are true about the proper method of handling data outliers when doing cost estimation?

a. Outliers should always be eliminated b. Outliers should never be eliminated c. Outliers should not be eliminated if their cause is known d. None of the above

Page 271: MODULE 3 Financial Management and Management Accounting 1

Module 3 - Financial Management & Management Accounting 1

Page 271 © CMA Ontario, 2011

6. Assuming that over- or under-applied overhead is charged to cost of goods sold, which of the following statements is/are true about the effect of allocating fixed manufacturing costs to production using a predetermined overhead rate on data used for cost estimation?

a. There will be no effect b. There will be an indeterminate effect c. The estimate of fixed cost will be affected d. The estimate of variable cost will be affected.

Questions 7 – 10 are based on the following information:

Beaver Fine Foods is developing a new product for its line of food entrees. The estimated variable costs for the new product are $.95 for materials and $.35 for labour. Based on experience with comparable products, management estimates the new product will consume .03 hours of machine time.

Management wants to determine the manufacturing overhead costs that should be estimated for this new product. Manufacturing overhead data was accumulated and, after some analysis, decided to use manufacturing overhead as the dependent variable and machine hours as the independent variable. Excel produced the following regression equation result:

7. Given these results, would you conclude?

a. The regression results are significant and should be accepted b. The regression results are not significant and should not be accepted c. The regression results should not be considered reliable because the sample size is

too small d. The regression results should not be used because only one independent variable

is used to estimate total manufacturing cost

Page 272: MODULE 3 Financial Management and Management Accounting 1

Module 3 - Financial Management & Management Accounting 1

Page 272 © CMA Ontario, 2011

8. What is the best measure of goodness of fit in the regression results?

a. 10.02 b. 15.50 c. 88.44% d. None of the above

9. Assume management estimates the total manufacturing overhead in the upcoming

period will be $650,000 and the expected number of machine hours will be 5,000. Based on these expectations, management has decided to allocate fixed manufacturing overhead to production at the rate of $130 ($650,000/5,000) per machine hour. If management follows this approach, what total cost per unit will it estimate for this new product?

a. $1.39 b. $2.89 c. $6.79 d. None

10. What variable cost would you estimate for the above product?

a. $1.39 b. $2.89 c. $6.79 d. None

Page 273: MODULE 3 Financial Management and Management Accounting 1

Module 3 - Financial Management & Management Accounting 1

Page 273 © CMA Ontario, 2011

Problem 1 Bev’s Trattoria serves a varied menu of Italian food. A particular favourite is the pasta meat sauce. The following ingredients are used to make 10 litres of the meat sauce:

2 kilograms of ground beef @ $4.95 per kilogram 2 large onions (approximately .6 kilograms each) @ $1.96 per kilogram 2 Green peppers (approximately .1 kilograms each) @ $2.19 per kilogram 2 Celery stalks (celery is purchased in a bunch that contains five stalks) @ $2.19 a

bunch 2 Garlic cloves (garlic is purchased in a bulb that usually contains eight cloves) @ $3

a bulb 1 large can tomato juice @ $2.29 1 large can tomato sauce @ $1.49 1 can tomato paste @ $1.19 Approximately $.35 for salt and pepper

It takes a part-time employee paid $8 per hour about 35 minutes to prepare the ingredients, which are put into a large pot and simmered for four hours. The gas burner is estimated to consume approximately $1.40 of natural gas per hour. Bev follows the common restaurant guideline that the food cost ingredients should be between 25% and 30% of the price of each menu item and has chosen the mid-point of 27.5% for pricing this sauce in any menu item it is used. Required: a. Use the engineering method to compute the variable cost of producing a serving of

the sauce if each serving amount is .2 litres. b. What is the target price of a serving the sauce given the pricing formula Bev uses?

Round to the nearest two decimal places. Problem 2 Jacques, an enterprising and long-time resident of Northern Ontario, owns and operates a thriving, but illegal, business called Jacques’ Discount Beverages, whose sole product is swish. Jacques makes swish in used Kentucky whiskey barrels (preferably oak), which he acquires in truck load lots from US distilleries. The production process consists of filling a barrel with one litre of grain alcohol, a cup of sugar and 20 litres of water. The brew is allowed to “age”, as Jacques calls it, for three to four weeks with the barrel being turned (swished) daily. (Swishing allows the mixture to extract some of the whiskey that was absorbed into the barrel.) The result is a potent beverage reputed to cause occasional blindness but, nevertheless, is in high demand from Jacques’ clients. After two uses, Jacques saws the barrel in half and sells the halves to a local gardening centre for their

Page 274: MODULE 3 Financial Management and Management Accounting 1

Module 3 - Financial Management & Management Accounting 1

Page 274 © CMA Ontario, 2011

customers to use as planters. The planters are in high demand because of their remarkable effect on producing healthy and robust plants. Although all swish is made the same way, variations in the barrels produce variations in the taste and the taste determines the product’s price. Jacques believes it is the source of the barrels and not simple variations in the barrels themselves that determines the taste. Following up on his hunch, Jacques gathered data on 100 batches of swish. Jacques treated the price received per litre of the swish as the dependent variable and the distillery where Jacques got the barrel as the independent variable. There were two distilleries, so Jacques coded a zero if the barrel was obtained from one distillery and one if the barrel was obtained from the other. The following is the Excel result run by a client of Jacques, who was promised several litres of product in exchange for his services:

Required: Do these results support Jacques’ hypothesis that it is the source of the barrel that affects the quality and, therefore, the price of the swish? Problem 3 The factory accountant at Scotland Manufacturing has developed data to help estimate the behaviour of manufacturing overhead. The accountant has developed two sources of data, an analysis of the manufacturing overhead account and activity and cost data over the past 10 periods. Since the production process at Scotland Manufacturing is labour driven, the accountant believes labour hours is the most appropriate driver to estimate manufacturing overhead.

Page 275: MODULE 3 Financial Management and Management Accounting 1

Module 3 - Financial Management & Management Accounting 1

Page 275 © CMA Ontario, 2011

The following is the data for the past 10 periods and the plot of the data:

Period Labour Hours Manufacturing Overhead

1 1,564 268,121 2 1,501 268,698 3 1,455 267,086 4 1,378 261,041 5 1,309 255,742 6 1,268 259,405 7 1,319 260,660 8 1,386 258,014 9 1,343 262,811 10 1,415 263,506

The following is the result of the account analysis for the most recent period:

Item Amount Cost Classification

Factory property taxes $37,456 Fixed

Machine depreciation $94,568 Fixed

Supervisory salaries $67,845 Fixed

Indirect labour $22,665 Variable

Indirect supplies $16,167 Variable

Power $24,805 Variable

$263,506

Required: a. Develop an estimate of manufacturing overhead using the high low method. b. Develop an estimate of manufacturing overhead using account analysis c. Would regression analysis be appropriate in this case? Explain.

Page 276: MODULE 3 Financial Management and Management Accounting 1

Module 3 - Financial Management & Management Accounting 1

Page 276 © CMA Ontario, 2011

d. Which of the estimates you developed in a) and b) do you think is more reasonable? Why?

Problem 4 Dora Manufacturing operates a number of plants all producing the same mix of mother boards for computers. The management accountant in head office wants to develop an equation to predict manufacturing data and believes labour hours is the primary cost driver, since production involves a lot of labour and is, therefore, labour paced. The management accountant has developed a data base of xx observations of labour hours and manufacturing overhead and has plotted the data with the following result:

Excel produced the following regression output:

Page 277: MODULE 3 Financial Management and Management Accounting 1

Module 3 - Financial Management & Management Accounting 1

Page 277 © CMA Ontario, 2011

The management accountant observed: “This regression analysis produced a good result with an estimated coefficient that has a t-statistic that is greater than two. However, it is clear from the plot there are seven observations that are outliers. I am going to rerun the regression with these seven observations omitted”. The following was the result of the second regression analysis:

Required: a. Which regression equation would you use? Explain why. b. Suppose the vice president manufacturing sees this data and observes that the seven

observations deleted in the second regression were seven observations taken at one plant. Would this change your response in part a)? Explain why.

Page 278: MODULE 3 Financial Management and Management Accounting 1

Module 3 - Financial Management & Management Accounting 1

Page 278 © CMA Ontario, 2011

Problem 5 A cost analyst at Downeast Engineering is using regression analysis to develop an equation for manufacturing overhead. The cost analyst has identified two possible cost drivers, units produced and machine hours. The analyst has run three regressions with the following Excel output:

Using units as the independent variable:

Page 279: MODULE 3 Financial Management and Management Accounting 1

Module 3 - Financial Management & Management Accounting 1

Page 279 © CMA Ontario, 2011

Using machine hours as the independent variable:

Required:

a. Choose the regression result you think is best and explain why.

b. What is the manufacturing overhead equation predicted by the regression equation you chose?

Page 280: MODULE 3 Financial Management and Management Accounting 1

Module 3 - Financial Management & Management Accounting 1

Page 280 © CMA Ontario, 2011

Problem 6

Panther Brewery produces a single product, a pilsner beer. Panther Brewery competes in the discount beer market, which is extremely competitive resulting in highly volatile sales.

The product is priced at $1 per bottle, which is the price in this market niche. The marketing manager believes sales volatility is affected by two factors, Panther’s advertising and the actions of competitors in this market niche.

The marketing manager believes advertising has the greatest effect in the current period, but also has residual effects in the two following periods.

The marketing manager has constructed a competitive index that reflects the advertising activities of competitors. The index value ranges from one, reflecting low competitor advertising activities, to nine, reflecting high competitor advertising activities.

Sales and advertising data were gathered for 30 periods and a regression was run. The following is the Excel result:

Required:

a. Based on these results, would you agree?

i. That advertising does have a residual (lagged) effect on sales

ii. That the index of competitor advertising activities is a useful variable to predict sales

b. Based on these results, what are the effects of advertising (both Panther and its competitors on sales?

Page 281: MODULE 3 Financial Management and Management Accounting 1

Module 3 - Financial Management & Management Accounting 1

Page 281 © CMA Ontario, 2011

Solutions - Multiple Choice Questions Question 1

a. Uses all the historical data available (false – only uses two points) b. Can be affected by extreme points (true – if either end is extreme) c. Requires a good knowledge of the production process (false – only looks at the data) d. Requires a good knowledge of product specifications.(false – only looks at the data) Question 2

a. Uses all the historical data available (false- only looks at current account balances) b. Can be affected by extreme points (possibly, but since all current data is considered,

the effect of extreme points will be mitigated) c. Requires a good knowledge of the production process (true – since this is the basis for

classifying the cost as fixed or variable) d. Requires a good knowledge of product specifications (false – the focus is only on

costs)

Question 3

a. Uses all the historical data available (false – since this approach usually prospective costs)

b. Can be affected by extreme points (false – since this is based on single point values) c. Requires detailed knowledge of the production process (true) d. Requires a good knowledge of product specifications (true)

Question 4

a. Uses all the historical data available (true) b. Can be affected by extreme points (possibly, but the visual method usually allows the

analysis to discount these) c. Requires detailed knowledge of the production process (false) d. Requires a good knowledge of product specification (false)

Question 5

a. Outliers should always be eliminated (false – only if a cause can be determined) b. Outliers should never be eliminated (false – can be if a cause can be determined) c. Outliers should not be eliminated if their cause is known (false – the opposite is true) d. None of the above (true)

Page 282: MODULE 3 Financial Management and Management Accounting 1

Module 3 - Financial Management & Management Accounting 1

Page 282 © CMA Ontario, 2011

Question 6

a. There will be no effect (false – see Part d)) b. There will be an indeterminate effect (false – see Part d)) c. The estimate of fixed cost will be affected (false – see Part d)) d. The estimate of variable cost will be affected (true – using a predetermined overhead

rate to allocate fixed manufacturing overhead to production will cause the estimated variable cost to increase by the amount of the rate. Moreover, if over- or under-applied overhead is charged directly to cost of goods sold, the values of manufacturing overhead each period will be miss-stated by the amount of the over- or under-applied overhead with the consequential effect on the estimated cost equation.)

Question 7

a. The regression results are significant and should be accepted (true – goodness of fit is reasonable and the t-statistic associated with the estimated coefficient is greater than two.

b. The regression results are not significant and should not be accepted (false – see a)) c. The regression results should not be considered reliable because the sample size is too

small (false – a sample size of 30 is reasonable) d. The regression results should not be used because only one independent variable is

used to estimate total manufacturing cost (false – the goodness of fit is reasonable and there is no other evident cost driver)

Question 8

The best measure is the r-square adjusted value, which is 77.44% Question 9

The cost will be .95 + .35 + (52.97 * 0.03) + (130 * 0.03) = $6.79 Question 10

The cost will be .95 + .35 + (52.97 * 0.03) = $2.89

Page 283: MODULE 3 Financial Management and Management Accounting 1

Module 3 - Financial Management & Management Accounting 1

Page 283 © CMA Ontario, 2011

Problem 1

Ingredient Calculation Amount Ground beef = 2 * $4.95 $9.90

Onions = 2 * 0.6 * $1.96 2.35

Green peppers = 2 * 0.1 * $2.19 0.44

Celery stalks = 2/5 * $2.19 0.88

Garlic cloves = 2/8 * $3.00 0.75

Tomato juice 2.29

Tomato sauce 1.49

Tomato paste 1.19

Salt and pepper 0.35

Food costs 19.64

Labour = 35/60 * $8.00 4.67

Natural gas = 4 * $1.40 5.60

(a) Total variable cost per batch $29.90

Servings per batch = 10/.2 50

Total variable cost per serving = $29.90/50 $0.60

(b) Target price = 19.64/27.5% /50 $1.43

Problem 2 The price equation estimated by the regression analysis is:

Price = $2.99 - $.26 * distillery code The expected price is $2.99 for the product made in the barrels obtained from the distillery coded as zero and $2.73 ($2.99 – .26) if the product is made in barrels obtained from the distillery coded as one. The estimated coefficient is significant implying a strong result and that Jacques’ intuition is correct. However, the estimated equation has an adjusted r-square of 38%, implying there are other factors, possibly the season or the activities of Jacques’ cousin, a competitor, that are affecting the price.

Page 284: MODULE 3 Financial Management and Management Accounting 1

Module 3 - Financial Management & Management Accounting 1

Page 284 © CMA Ontario, 2011

Problem 3 a. The observations with the highest and lowest values of the cost driver are,

respectively, observations one and six and the paired values are (1,564, 268,121) and (1,268, 259,405). Therefore, the estimate of the slope (variable cost) of the graph is:

Variable cost = cost at highest driver value - cost at lowest driver value highest driver value - lowest driver value = ($268,121 - 259,405) / (1,564 - 1,268) = $29.45 Substitute this value in either of the observations to get the fixed cost:

Fixed cost = 268,121 – 29.45*1,564 = $222,068, or Fixed cost = 259,405 – 29.45*1,268 = $222,068

The estimated cost equation is:

Manufacturing overhead = 222068 + 29.45 * number of labour hours b. Total variable costs $63,637

Cost driver activity level 1,415 Variable cost per LH $44.97 Total fixed costs $199,869

Therefore, the estimated manufacturing overhead equation using the account analysis method is:

Manufacturing overhead = 199,869 + 44.97*number of labour hours c. It would be unreasonable to use regression analysis in this case since there are too

few observations to develop a reliable estimate

d. Visual inspection suggests the high-low estimate may understate the slope of the cost line because the line would fall under many of the data points (that is the line should be rotated clockwise). In this case, the account analysis estimate seems more reasonable.

Page 285: MODULE 3 Financial Management and Management Accounting 1

Module 3 - Financial Management & Management Accounting 1

Page 285 © CMA Ontario, 2011

Problem 4 a. Judiciously pruning outlier data will always improve goodness of fit, but

improving goodness of fit is not a sufficient reason to eliminate data without an adequate explanation of what caused the outlier. Eliminating data without explanation creates a false confidence in the predictive ability of the equation, since the situation that produced the outliers could reoccur.

b. Identifying the source of the variation is sufficient reason to eliminate the data since the cause is understood. Another approach is to add a second independent variable with a value of one if the observation is from this plant that produces the outliers and zero otherwise. The excess cost will load on this new independent variable as shown in the following result:

This regression suggests the following equation for manufacturing overhead: Manufacturing overhead = 117,889 + 46.44 * labour hours + 12,120 (if the plant is coded as one)

Page 286: MODULE 3 Financial Management and Management Accounting 1

Module 3 - Financial Management & Management Accounting 1

Page 286 © CMA Ontario, 2011

Problem 5 a. Pick Result 3, the regression using only machine hours as the independent

variable. Although the regression with both units and machine hours has the highest adjusted r-square, the estimated coefficient associated with units produced is not significant. This means, although this regression equation has the strongest predictive power for total manufacturing overhead, the coefficient estimated for units produced would not produce reliable results if it were used separately to make inferences about how units produced affects total overhead.

The regression equation estimated using units produced has the lowest r-square and is inferior to the regression equation estimated using machine hours, although both regression equations have estimated coefficients with t-statistics greater than two. What might be happening is different units require different amounts of machine hours and it is the number of machine hours that is driving manufacturing overhead.

b. Manufacturing overheard = 264,299 + 116.80*machine hours Problem 6 a. i. The regression has estimated coefficients for all the advertising levels and

all are significantly (statistically) different from zero. Therefore, the manager’s hypothesis is supported.

ii. The estimated coefficient associated with the index of competitors’ advertising levels is statistically significant, which supports the relevance of the manager’s index in predicting sales.

b. Given these results the estimated effect on sales of the various variables are:

i. Current sales will increase by $20.94 for every dollar of advertising in the current period

ii. Current sales will increase by $10.36 for every dollar of advertising in the last period

iii. Current sales will increase by $1.04 for every dollar on advertising in the second previous period

iv. Current sales will decrease by $11,191,975.44 for each unit increase in the competitor index the marketing manager has created.

Page 287: MODULE 3 Financial Management and Management Accounting 1

Module 3 - Financial Management & Management Accounting 1

Page 287 © CMA Ontario, 2011

6. Activity Based Costing

Learning Objectives

After completing this chapter, you will understand and be able to explain:

1. The critical design choices management accountants make when designing costing systems.

2. The perspective of activity based costing (ABC) systems (a decision oriented approach) and how that differs from conventional costing systems (a responsibility oriented approach).

3. How and why activity based costing systems provide an improved potential for costing system accuracy.

4. The organization conditions under which activity based costing systems have the potential to improve costing accuracy.

Costing System Architecture

The following diagram provides a simple schematic of the critical design issues and choices in all costing systems:

Page 288: MODULE 3 Financial Management and Management Accounting 1

Module 3 - Financial Management & Management Accounting 1

Page 288 © CMA Ontario, 2011

Step 1 – identifying costs as direct or indirect:

The first step in costing system design is to develop a method that allows a cost to be identified as direct or indirect. To refresh your understanding of direct and indirect, you can return to the material in Chapter 2. As discussed in Chapter 2, if the cost is identified as direct to a cost object, it is assigned to that cost object. If the cost is indirect, it is assigned to an indirect cost pool.

Step 2 – designing indirect cost pools:

The second step in costing system design is to develop the system of indirect cost pools. Chapter 3 provided some initial insights into approaches to designing indirect cost pools and we will now develop those ideas in more detail.

We need to remember that indirect cost pools will be required for both manufacturing and non-manufacturing costs. In the past, many organizations used indirect cost pools only for manufacturing costs because their accounting systems were oriented to supporting financial accounting. In financial accounting, virtually all non-manufacturing costs are

Is the cost

direct?

Assign cost to the appropriate cost

object

Yes No

Indirect cost pool Indirect cost poolIndirect cost pool

Design indirect cost pool structure and

assign indirect cost to the appropriate cost

pool

The various cost objects

Choose the cost driver and the allocation rate for each indirect cost pool and allocate the costs from each indirect cost pool to the cost objects

Page 289: MODULE 3 Financial Management and Management Accounting 1

Module 3 - Financial Management & Management Accounting 1

Page 289 © CMA Ontario, 2011

charged to the income statement as current period costs. This promoted the design and use of single plant wide indirect cost pools to accumulate indirect manufacturing costs.

In the early part of the 20th century, as organizations became larger, decision making authority and responsibility were delegated down the organization hierarchy, creating a need for accountability. Within the factory, department managers were assigned the responsibility to control costs. This focus on accountability, in turn, led to an interest in designing cost centres and, therefore, indirect manufacturing cost pools around various departments within the plant such as receiving, machining, assembly, finishing and shipping. The focus was on accumulating cost by department so the accumulated costs could be compared to a budget or standard.

Driven by competitive issues, particularly beginning in the 1980s, managers began to demand better cost information, not only about the behaviour of indirect manufacturing costs, but also about other indirect costs such as product development and customer related costs, such as order taking and order processing. This led not only to an interest in developing different indirect manufacturing cost pool designs but also the design and use of indirect cost pools to accumulate non-manufacturing costs.

Objective in Indirect Cost System Design

The objective in indirect cost system design is to develop better cost information to support organization decision making. Chapter 5 introduced some of the major decision making support roles of cost information. The most widely touted roles being to improve accuracy in forecasting budgets for budgeting and product pricing and supporting efforts to identify opportunities for cost reduction.

Improved cost information allows the decision maker to avoid opportunity costs caused by faulty cost information. Examples of opportunity costs caused by bad cost information include the losses caused by:

1. Continuing to make a product that appears to be profitable, but is not.

2. Inappropriate pricing in cost plus environments, such as contract pricing.

3. Failing to recognize opportunities for process improvement.

4. Inappropriate budget projections creating cash flow problems

5. Inappropriate product mix choices when allocating constrained capacity.

The objective of costing system improvements is not to increase cost accuracy at any cost. The consensus among experts seems to be that the incremental cost of designing and operating a costing system increases

Page 290: MODULE 3 Financial Management and Management Accounting 1

Module 3 - Financial Management & Management Accounting 1

Page 290 © CMA Ontario, 2011

You know you need a new costing system when:

Vendor bids are lower than expected Functional managers want to drop seemingly

profitable lines Profit margins are hard to explain Hard-to-make products show big profits Departments have their own cost systems You have a high-margin niche all to yourself Competitors’ prices are unrealistically low Customers don’t mind price increases The results of bids are hard to explain

Source: Harvard Business Review, Jan.-Feb. 1989

exponentially in the system’s complexity and the marginal value provided by improved costing system accuracy declines logarithmically. The accompanying graph summarizes this conventional wisdom and the point at which the marginal benefit and marginal cost of increased complexity are equal.

Recall that costing issues by assigning inappropriate indirect cost pools since direct costs should always be assigned to the appropriate cost object. Therefore, we take as given in the following discussion that the focus is on improving the design of the indirect cost pools.

How Do We Know When the Costing System Needs Improvement?

Based on an extensive study of costing systems, Robin Cooper prepared a list of symptoms of a costing system that is producing inaccurate information, causing inappropriate decision making. This list is shown in the accompanying text box.

As an exercise, see if you can explain why each symptom in the text box suggests a costing system that has gone awry.

An example:

At this point, it is useful to develop an example to illustrate how costing systems can produce inaccurate and misleading cost information.

Drumbo Aerospace produces three products, all aircraft components. The following is a summary from a recent production period:

Page 291: MODULE 3 Financial Management and Management Accounting 1

Module 3 - Financial Management & Management Accounting 1

Page 291 © CMA Ontario, 2011

Product

Units

Produced

Machine Hours

1 8,600 800

2 4,700 600

3 2,300 400

15,600 1,800

Indirect manufacturing costs $586,000

Overhead rate = $586,000 / 1,800 $325.56

Product Unit MH

1 $37.56 $30.28

2 37.56 41.56

3 37.56 56.62

Initially, the factory accountant allocated indirect manufacturing costs equally to all units of product using a rate of $37.56 ($586,000/15,600) per unit. Product 1 sales manager complained that the indirect manufacturing costs should be allocated based on machine hours as that was a more relevant cost driver for indirect manufacturing costs.

Following up on this idea, the factory accountant computed a rate per machine hour as:

Total indirect manufacturing costs/total machine hours

The indirect manufacturing cost to be allocated to each unit of production was computed as: machine hour rate * machine hours used by product group number of units in the product group For example, the calculation for Product 3 was: ($325.56 * 400) / 2,300

Page 292: MODULE 3 Financial Management and Management Accounting 1

Module 3 - Financial Management & Management Accounting 1

Page 292 © CMA Ontario, 2011

= $56.62

Drumbo marked-up a product’s total cost by 50% to compute its selling price. Recently, several customers complained, arguing that prices seem too high based on what competitors are offering. After discussions with factory engineers, the factory accountant believed there were three major activities in the factory: 1. Batch related activities which included moving work-in-process about the plant

and setting up equipment for production runs. 2. Inspection activities, which involved subjecting parts to laser and x-ray

inspection. 3. Machining activities required to produce the products. With this insight the factory accountant developed the following information:

Product

Units Produced

Batches

Inspection Hours

Machine Hours

1 8,600 20 100 800 2 4,700 30 500 600 3 2,300 50 200 400

Cost driver units 15,600 100 800 1,800

Total cost $16,000 $120,000 $450,000

Cost/cost driver unit $160 $150 $250 This exhibit shows the cost driver and the rate per unit of the cost driver identified using this data. Finally, the indirect manufacturing cost to be allocated to each unit of production was computed and is shown in the following table as the activity rate. For comparison purposes, the rate using units produced and machine hours as the cost drivers are also shown.

Product Units MT Activity

1 $37.56 $30.28 $25.37

2 37.56 41.56 48.89

3 37.56 56.62 60.00

Page 293: MODULE 3 Financial Management and Management Accounting 1

Module 3 - Financial Management & Management Accounting 1

Page 293 © CMA Ontario, 2011

For example, the indirect manufacturing cost assigned to Product 2 is computed as follows: Total indirect manufacturing cost allocated to Product 2 =

Number of batches * cost driver rate per batch + number of inspection hours * cost driver rate per inspection hour + number of machine hours * cost driver rate per machine hour =

30*160 + 500*150 + 600*250 = $229,800 indirect manufacturing cost per unit of product 2 = 229,800/4,700 = $48.89

You should verify your understanding of this approach by verifying the indirect manufacturing costs shown for Products 1 and 3. This approach to costing reflects the recognition that it is activities the organization undertakes to produce goods and services that create costs and this focus if using activities as the cost driver for indirect costs led to this approach being called activity based costing. The underlying idea is that indirect cost allocations should be based on cause and effect relationships and it is the level of activities performed in an organization that determines, in the long-term, the level of indirect costs. This idea is readily illustrated in a diagram that has come to be known as the ABC cross.

Page 294: MODULE 3 Financial Management and Management Accounting 1

Module 3 - Financial Management & Management Accounting 1

Page 294 © CMA Ontario, 2011

This diagram summarizes the idea that production of a cost object creates the need to undertake activities, which, in turn, create a demand for resources. The cost driver reflects the rate at which a cost object consumes an activity and the resource driver reflects the rate at which an activity consumes a resource. This is called the cost view of what is going on in the organization and is shown as the vertical plane of the ABC cross. The horizontal plane reflects the process view, which reflects that cost drivers draw on activities and are summarized by performance measures that assess how well the work is being done. Therefore, the vertical view is the control or performance measurement view of the process.

The Drumbo example illustrates how cost distortions arise. The setting in the Drumbo example is that expensive indirect resources are used at different rates by different cost objects. If the costs of these indirect resources are grouped together into a single indirect cost pool and allocated in proportion to some quantity measure, such as units produced, machine hours, or labour hours, the high volume products will tend to pick up more than their fair share of costs if resource use is not proportional to volume.

To illustrate this important concept, consider an organization that produces 100,000 units of Product 1 and 100,000 units of Product 2. Product 1 is produced in batches of 1,000 units and Product 2 is produced in batches of 500 units. Therefore, 100 batches will be used to produce Product 1 (100,000/1,000) and 200 (100,000/500) batches will be used to produce Product 2. If the cost of processing a batch (set up costs and costs of moving work-in-process about the factory) is $400 per batch, Product 1 creates $40,000 ($400 * 100) of set up costs and Product 2 creates $80,000 ($400 * $200) of set up costs. The total batch related costs will be $120,000 (40,000 + 80,000). If these batch costs, which are indirect manufacturing costs, are allocated in proportion to units produced (or any volume measure such as labour hours or machine hours), each product will be allocated $60,000 of batch costs resulting in Product 1 being over-costed by $20,000 ($80,000 - $60,000) and Product 2 being under-costed ($60,000 - $80,000).

The ABC Cost Hierarchy

ABC proponents have developed what is called the ABC cost hierarchy, which consists of the following elements:

• Unit level (costs that vary directly with the level of production such as factory supplies, commonly called variable manufacturing costs)

• Batch level (examples include setting up machines for a production run and shipping a truckload of products)

• Product sustaining (examples include advertising to support a product and developing and implementing a product design change)

• Facility sustaining (examples include plant depreciation and the factory manager’s salary). These costs are not usually allocated, since there is no apparent cause and effect relationship with activity levels.

Page 295: MODULE 3 Financial Management and Management Accounting 1

Module 3 - Financial Management & Management Accounting 1

Page 295 © CMA Ontario, 2011

1031-2 05 Allocation of Indirect Costs

Indirect costs must be accumulated in appropriate indirect cost pools, reflecting a contractor's organizational or operational lines and these pools subsequently allocated to contracts in accordance with the following two principles:

The costs included in a particular indirect cost pool should have a similarity of relationship with each contract to which that indirect cost pool is subsequently distributed; further, the costs included in an indirect cost pool should be similar enough in their relationship to each other that the allocation of the total costs in the pool provides a result, which would be similar to that achieved if each cost within that pool were separately distributed;

The allocation basis for each indirect cost pool should reflect, as far as possible, the causal relationship of the pooled costs to the contracts to which these costs are distributed.

• Customer sustaining (examples include the cost of the sales representative visiting the customer and special customer shipping requirements)

• Organization sustaining (examples include the fees paid to the Board of Directors and depreciation on the administrative office suite). These costs are not usually allocated, since there is no apparent cause and effect relationship with activity levels.

The insight provided by this cost hierarchy is that most cost drivers for these costs are not volume related. Therefore, accumulating these costs in single cost pool and allocating these costs to products using a volume based driver will cause costing distortions.

In summary, indirect cost pools should be designed so all costs in an indirect cost pool have the same cost driver and costs should be allocated from that indirect cost pool to cost objects using that cost driver. The attached excerpt from the Government of Canada’s Contract Costing Principles 1031-2 summarizes this idea well.

Page 296: MODULE 3 Financial Management and Management Accounting 1

Module 3 - Financial Management & Management Accounting 1

Page 296 © CMA Ontario, 2011

Activity Based Costing Example

Danny Company produces two versions of a single product, standard and deluxe. The following is a summary of the most recent period’s activities:

Regular Deluxe

Units produced 6,000 1,000

Direct material costs $ 300,000 $ 75,000

Direct labour cost 150,000 50,000

Indurect manufacturing cost 1,875,000 625,000

Total manufacturing cost $2,325,000 $750,000

Manufacturing cost pert unit $387.50 $750.00

Investigation of these results establishes three facts:

1. Indirect manufacturing cost, which totals $2,500,000 for this organization, is currently being allocated to units using direct labour costs based on the observation that production in the plant involves a great deal of manual assembly and, therefore, is labour intensive. Since the regular product comprises 75% (150,000/(150,000+50,000) of total direct labour cost, the regular product is allocated 75% of total indirect manufacturing cost (75% * 2,500,000).

2. Prices are set by marking-up manufacturing cost per unit by 70%, which is standard practice in this industry.

3. While sales of the regular product continue to exceed expectations, sales staff is reporting difficulty in maintaining sales volumes of the deluxe product and have been pushing for a price decrease on the deluxe product.

The plant manager attended an ABC seminar, concluded that the current costing system may be deficient and ordered a study of plant costs. The study reported the following results:

Indirect Cost Pool Cost Cost Driver Regular Deluxe Total

Materials Handling $ 500,000 Material Move 800 200 1,000

Production Support 1,300,000 Production Runs 100 60 160

Shipping 700,000 Shipments 500 200 700

$2,500,000

Page 297: MODULE 3 Financial Management and Management Accounting 1

Module 3 - Financial Management & Management Accounting 1

Page 297 © CMA Ontario, 2011

The plant manager has asked you to recast the manufacturing cost to reflect the ABC perspective on costing.

The first step is to compute the indirect manufacturing cost per cost driver unit. To do this, divide the total cost in each indirect cost pool by the total number of cost driver activity units associated with that pool.

The following is the result:

Cost/Unit

Materials Handling: $500,000 / 1,000 $500

Production Support: $1,300,000 / 160 8,125

Shipping: $700,000 / 700 1,000

We now have the cost per unit for each cost driver. With this information, we can now reallocate the indirect manufacturing costs to the two products. The following is the allocation for the regular product:

Materials handling: 800 moves * $500 per move = $400,000

Production support: 100 production runs * $8,125 per production run = $812,500

Shipping: 500 shipments * $1,000 per shipment = $500,000

The following is the completed table for both products. You should take a few moments and verify the calculations for the deluxe product:

This example illustrates a common distortion where a high volume product that puts relatively fewer demands on production activities than a lower volume product ends up,

Page 298: MODULE 3 Financial Management and Management Accounting 1

Module 3 - Financial Management & Management Accounting 1

Page 298 © CMA Ontario, 2011

in a volume based allocation system like direct labour hours, being over-costed. The following are the percentages of activity units demanded by each product underlying this production schedule:

Regular Deluxe

Units produced 6,000 1,000

Direct materials costs $300,000 $75,000

Direct labour cost $150,000 $50,000

Indirect manufacturing overhead $1,875,000 $625,000

Total manufacturing cost $2,325,000 $750,000

Another Activity Based Costing Example

Gene’s Foods (GF) is a wholesale distributor of ethnic foods. GF has three groups of customers: supermarket chains, convenience store chains and individual high end food stores. The following is a summary of operations for the most recent period. Indirect overhead is allocated to each customer group based on cost of merchandise sold, which is taken as a good measure of the activities involved in handling customer orders.

For several years the sales manager has argued that GF should focus on the convenience store and high-end store segments and phase out its unprofitable supermarket business.

Patricia Conroy, the company president, is leery about this suggestion based on comments, primarily relating to good and bad customers, she has heard from various staff members.

Based on her intuition, Patricia ordered a study of the overhead costs allocated to the customer groups. The study produced the following results:

Page 299: MODULE 3 Financial Management and Management Accounting 1

Module 3 - Financial Management & Management Accounting 1

Page 299 © CMA Ontario, 2011

Given this information, was Patricia’s concern about the sales manager’s recommendation justified?

We can begin this analysis by developing an activity summary like the following:

This exhibit confirms what Patricia has been hearing from the staff. GF has been using cost of merchandise sold as a proxy for activities when allocating indirect costs, while the demand for activities by the various groups differs substantially from this proxy. The pattern is that smaller customers make smaller and more frequent orders and place relatively more demands on order picking, shipping and invoicing and adjustments.

It appears ABC will give us a different picture as we can calculate the cost per unit of the cost driver for each of the activities.

Page 300: MODULE 3 Financial Management and Management Accounting 1

Module 3 - Financial Management & Management Accounting 1

Page 300 © CMA Ontario, 2011

With this information, we can complete the following exhibit:

(You should take the time to check your understanding of the ABC calculations by verifying the overhead allocations. The calculations follow the same process used in the first example.)

For example, the overhead allocations to Supermarkets were determined as: (remember the numbers in the above table reflect Excel level significance)

Order taking: 500 * 5,833.33 = $2,916,667

Order picking: 100 * 8,510.64 = $851,064

Shipping: 300 * 10,476.19 = $3,142,857

Invoicing: 650 * 6,578.95 = $4,276,316

Page 301: MODULE 3 Financial Management and Management Accounting 1

Module 3 - Financial Management & Management Accounting 1

Page 301 © CMA Ontario, 2011

Patricia’s intuition is confirmed! The two smaller customer groups are placing excessive (relative to the supermarkets) and costly demands on the system. It is now up to Patricia to decide what to do. One approach is to charge customers for each of the activities to try to promote less costly behaviour on their parts.

Conclusion:

Activity based costing tries to model the cause and effect relationship between activities undertaken to produce a good or service and the cost of those activities. The objective is to provide improved cost information for decision making. However, these ABC costing systems are expensive to design and maintain so organizations must decide whether the costs of these systems are justified against the benefits provided by improved decisions resulting from the ABC costs.

Page 302: MODULE 3 Financial Management and Management Accounting 1

Module 3 - Financial Management & Management Accounting 1

Page 302 © CMA Ontario, 2011

Problems with Solutions Multiple Choice Questions

1. Which of the following represents a major difference between activity based costing (ABC) and traditional product based costing?

a. ABC accumulates indirect costs in many cost pools, whereas traditional product based costing accumulates indirect costs in one or a few cost pools.

b. ABC assigns overhead costs based on cost drivers, whereas traditional product based costing assigns overhead costs using cost application bases that are not cost drivers.

c. ABC assigns overhead costs based on both financial and non-financial cost drivers, whereas traditional product based costing assigns overhead costs based on financial cost drivers only

d. ABC uses indirect application bases as cost drivers, whereas traditional product based costing uses direct cost application bases as cost drivers.

e. None of the above

2. Activity based costing in service organizations should be considered if:

a. Operating personnel have little faith in the accuracy of the existing cost information

b. They have a widely diverse set of operating activities

c. Services offered change frequently over time

d. All of the above are true

e. Only a and c above are true

Page 303: MODULE 3 Financial Management and Management Accounting 1

Module 3 - Financial Management & Management Accounting 1

Page 303 © CMA Ontario, 2011

The following information is for questions 3 – 4:

Scissors Inc. is a manufacturer of scissors. The company has always used a plant wide rate for allocating manufacturing overhead to its products. The plant manager believes it is time to change to a better method of cost allocation. The accounting department has been able to establish new relationships between production activities and the manufacturing overhead. They are:

Activity Cost Driver Allocation Rate Material handling Number of parts $4 per part

Assembly Labour hours $40 per hour Inspection Time item is at inspection station $6 per minute

The previous allocation method is based upon direct manufacturing labour hours and, if that method is used, the rate is $400 per labour hour.

3. What are the indirect manufacturing costs per scissor assuming the traditional method is used and a batch of 1,000 scissors is produced? The batch requires 2,000 parts, 20 direct manufacturing labour hours and 30 minutes of inspection time.

a. $8

b. $9.80

c. $8,000

d. $8,980

e. None of the above

4. What are the indirect unit manufacturing costs of a batch of 100 scissors assuming the activity base method is used? The batch requires 200 parts, 12 direct manufacturing labour hours and five minutes of inspection time.

a. $8

b. $13.10

c. $48

d. $1,310

e. None of the above

Page 304: MODULE 3 Financial Management and Management Accounting 1

Module 3 - Financial Management & Management Accounting 1

Page 304 © CMA Ontario, 2011

5. Which costs are not allocated in an ABC costing system?

a. Product sustaining costs

b. Facility sustaining costs

c. Batch level costs

d. Unit level costs

6. Which one of the following is not one of the ABC cost hierarchy?

a. Customer sustaining

b. Facility sustaining

c. Fixed costs

d. Batch level costs

7. Which of the following would not be an example of an organization sustaining cost?

a. The chief executive officer’s salary

b. Insurance on the manufacturing facility

c. Legal department costs

d. Annual fees paid to the external auditors

8. Which of the following would not be a cost that would be allocated using ABC?

a. The cost of wood used in a factory making furniture.

b. Costs associated with handling customer complaints

c. The costs of heating a batch of steel

d. Product advertising

9. Which of the following statements about ABC is true?

a. ABC will always produce more accurate costs than a costing system using a single indirect cost pool.

b. ABC can be used for non-manufacturing costs.

c. ABC costing systems are only useful in manufacturing organizations.

d. The benefits of an ABC system will always outweigh its costs.

Page 305: MODULE 3 Financial Management and Management Accounting 1

Module 3 - Financial Management & Management Accounting 1

Page 305 © CMA Ontario, 2011

10 Which of the following would not be an appropriate cost driver for assigning batch related costs to a product?

a. The number of hours spent moving a batch around the factory floor

b. Number of hours spent setting up for the product

c. Number of set ups done for the product

d. The number of direct labour hours needed to make the product

Page 306: MODULE 3 Financial Management and Management Accounting 1

Module 3 - Financial Management & Management Accounting 1

Page 306 © CMA Ontario, 2011

Problem 1

There on Time (TOT) provides regional courier service in Eastern Canada. Although sales have been increasing, profits have not been following. The controller Dana Strict has ordered a cost study. An exhaustive cost study resulted in identifying the following major activities at TOT.

1. Pick-up activities 2. Sorting activities 3. Transportation activities 4. Delivery activities

Further investigation reveals the following cost estimates:

1. Cost to pick-up a single parcel at an urban location averaged $5 plus $.20 per parcel picked up. Cost behaviour in rural locations averaged $8 plus $.20 per parcel.

2. At the hub, parcels were sorted either automatically by machines or manually, depending on the nature of the parcel and any special handling requirements. The cost per parcel sorted automatically was $1 and the cost of a parcel that required manual sorting cost $6.

3. Parcels were transported either by ground or air. The average cost of an air shipment was estimated as $12 per unit and the average cost of a ground shipment was estimated as $1.50

4. The cost of delivering shipments averaged $2 per parcel in urban locations and $3 per unit in rural locations.

5. The parcel weight affects the costs of all activities. For convenience, parcels were organized into three groups: one kilogram or less, between one and five kilograms and more than five kilograms. The additional costs created by these three weight groups were: $.50, $2 and $9, respectively.

Required: With the above information, estimate the total costs of each of these shipments:

a. A shipment of 1,000 parcels was picked up at a warehouse in a rural location. 40% of the parcels required manual handling during sorting, 80% of the parcels went by ground and 90% of the parcels were delivered to urban locations. The distribution of one, one to five and over 5 kilogram parcels was 60%, 35% and 5%.

Page 307: MODULE 3 Financial Management and Management Accounting 1

Module 3 - Financial Management & Management Accounting 1

Page 307 © CMA Ontario, 2011

b. Two shipments, each of 500 parcels, were picked at two warehouses, one in an urban and the other in a rural location. 10% of the parcels required manual handling during sorting, 50% of the parcels went by ground and 60% of the parcels were delivered to urban locations. The distribution of one, one to five and over five kilogram parcels was 90%, 10% and 0%.

c. A shipment of 1,000 parcels was picked up at a warehouse in an urban location. 0% of the parcels required manual handling during sorting, 0% of the parcels went by ground and 90% of the parcels were delivered to urban locations. The distribution of one, one to five and over five kilogram parcels was 70%, 30% and 0%.

Problem 2

Down East Consulting (DEC) provides consulting services to its clients. For planning purposes, DEC divides consulting jobs into three groups: simple, medium and complex. DEC uses four staff classifications for people who work on these jobs: support, analyst, manager and non-participating partner. The following are average times (in days) required by each type of consulting job for each of the four staff classifications:

The following table summarizes the number of employees in each group, the total number of days of work available in that group and the salary paid to each employee in the group.

Required:

a. Suppose DEC develops a charge rate per day by summing all the salaries paid to all employees and dividing the total by the total days provided by all employees. For each job, the cost allocated equals the number of days worked in the job by all employees multiplied by the rate. Compute the cost allocated to each job using this approach.

Page 308: MODULE 3 Financial Management and Management Accounting 1

Module 3 - Financial Management & Management Accounting 1

Page 308 © CMA Ontario, 2011

b. Suppose DEC develops a charge rate per day for each staff group by dividing the total salaries paid to the group by the total number of days of work supplied by that group. The cost allocated to each job from each staff group will be the number of days worked on the job by the group multiplied by that group’s charge rate. Compute the cost allocated to each job using this approach.

c. If DEC develops a price for each job by taking the cost allocated to the job and multiplying the cost by 1.5, which is the convention in this industry, what would be the likely consequence of the approach in Part a) relative to pricing the jobs using the costs developed in the approach in Part b).

Problem 3

Elora Corporation manufactures several different types of printed circuit boards: however, two of the boards account for the majority of the company’s sales. The first of these boards, a TV circuit board, has been a standard in the industry for several years. The market for this type of board is competitive and, therefore, price sensitive. Elora plans to sell 65,000 of the TV boards in 2009 at a price of $150 per unit. The second highest volume product, a PC circuit board, is a recent addition to Elora’s product line. Because the PC board incorporates the latest technology, it can be sold at a premium price; the 2009 plans include the sale of 40,000 PC boards at $300 per unit.

Elora’s management group is meeting to discuss strategies for 2009 and the current topic of conversation is how to spend sales and promotion dollars for next year. The sales manager believes the market share for the TV board could be expanded by concentrating Elora’s promotional efforts in this area. In response to this suggestion, the production manager said, “Why don’t you go after a bigger market for the PC board? The cost sheets I get show the contribution margin from the PC board is more than double the contribution from the TV board. I know we get a premium price for the PC board; selling it should help overall profitability.”

Elora uses the following data in its current costing system:

Item TV Board PC Board

Direct materials $80 $140 Direct labour 1.5 hours 4 hours Machine time 0.5 hours 1.5 hours

Variable manufacturing costs are allocated based on direct labour hours. For 2009, variable manufacturing costs are budgeted at $1,120,000 and direct labour hours are estimated at 280,000. The hourly rates for machine time and direct labour are $10 and $14, respectively. Elora applies a materials handling charge of 10% of materials cost;

Page 309: MODULE 3 Financial Management and Management Accounting 1

Module 3 - Financial Management & Management Accounting 1

Page 309 © CMA Ontario, 2011

this materials handling charge is not included in variable manufacturing costs. Total 2009 expenditures for material are budgeted at $10,600,000.

Pat Toste, Elora’s controller, believes that before the management group proceeds with the discussion about allocating sales and promotional sales efforts to individual products, it may be worthwhile to look at these products based on the activities involved in their production. Toste has prepared the following schedule for the management group:

Cost Item Budgeted Cost Cost Driver Annual Activity for Cost Driver

Material support costs:

Procurement $400,000 Number of parts 4,000,000

Production scheduling 220,000 Number of boards 110,000

Packaging and shipping 440,000 Number of boards 110,000

Total costs $1,060,000

Variable support costs:

Machine set up $446,000 Number of setups 278,750

Hazardous waste disposal 48,000 Pounds of waste 16,000

Quality control 560,000 Number of inspections

160,000

General supplies 66,000 Number of boards 110,000

Total costs $1,120,000

Manufacturing support costs:

Machine insertion $1,200,000 Number of parts 3,000,000

Manual insertion 4,000,000 Number of parts 1,000,000

Wave soldering 132,000 Number of boards 110,000

Total costs $5,332,000

Required per Unit TV Board PC Board

Parts 25 55

Machine insertions 24 35

Manual insertions 1 20

Machine set ups 2 3

Hazardous waste 0.02 pounds 0.35 pounds

Inspections 1 2

“Using this information,” Toste explained, “we can calculate an activity based cost for each TV board and each PC board and then compare it to the cost computed using our current system. The only cost that remains the same for both systems is the cost of direct materials. The cost drivers will replace the direct labour, machine time and support costs in the existing costing system.

Page 310: MODULE 3 Financial Management and Management Accounting 1

Module 3 - Financial Management & Management Accounting 1

Page 310 © CMA Ontario, 2011

Required:

a. Compute the cost of each product using the current costing system.

b. Compute the cost of each product using the activity based costing system.

c. Explain how the new costing system might change the promotional decisions made by Elora’s management group.

Problem 4

Pepper Company, a growing mail order clothing and accessory company, is concerned about its growing marketing, distribution, selling and administration expenses. It, therefore, examined its customer ordering patterns for the past year and identified four different types of customers, as illustrated in the following table. Pepper sends catalogues and flyers to all its customers several times a year. Orders are taken by mail or over the phone. Pepper maintains a toll-free number for customers phone orders. Pepper prides itself on the personal attention it provides shoppers who order over the phone. All purchases are paid by cheque or credit card. Pepper has a generous return policy if customers are not satisfied with the merchandise. Customers must pay return shipping charges, but their purchase price is fully refunded.

Customer 1 Customer 2 Customer 3 Customer 4 Initial Sales $1000 $1000 $2,500 $3,000 Number of items returned 0 4 2 24 Dollar value of items returned 0 $200 $500 $1,500 Number of all orders per year 1 6 4 12 Number of phone orders per year 1 0 0 12 Time spent on phone placing orders .25 hours 0 0 1 hour Number of overnight deliveries 1 0 0 12 Number of regular deliveries 0 6 4 0

Prices are set so that cost of goods sold is, on average, about 75% of the sales price. Customers pay actual shipping charges, but extra processing is required for overnight deliveries. Pepper has developed the following activity cost driver rates for its support costs:

Activity Activity Cost Driver Rate Process mail orders $5 per order Process phone orders $80 per hour Process returns $5 per item returned Process overnight delivery requests $4 per request Maintain customer relations (send catalogues and respond to customer comments or complaints)

$50 per year

Page 311: MODULE 3 Financial Management and Management Accounting 1

Module 3 - Financial Management & Management Accounting 1

Page 311 © CMA Ontario, 2011

Required

a. Using activity based costing, determine the yearly profit associated with each of the four customers described.

b. Comment on which customers are most profitable and why.

c. What advice do you have for Pepper regarding managing customer relationships with the different types of customers represented?

Problem 5

Adrian Rose, senior vice president of sales for Oakville Shoes, Inc., noticed that the company had substantially increased its market share for the high quality boomer boots (BB) and lost market share for the lower quality lazy loafers (LL). Adrian found Oakville’s prices were lower than its competitors for BB but higher for LL. He did not understand the reasons for the price differences because all companies used the same production technology and were equally efficient.

The manufacturing process is relatively simple. Oakville’s manufacturing facility has a cutting department and an assembly department. The high quality BB is produced in small batches (1,000 pairs of shoes each) and the lower quality LL is produced in large batches (3,000 pairs each). Adrian has asked you, the company’s new controller, to analyze the product costing method to see if the product prices should be changed.

The company currently uses a plant wide cost driver rate based on direct labour hours. The rate is computed at the beginning of the year using the following budgeted data:

Total manufacturing support costs $1,200,000 Total direct labour hours 49,000 Total machine hours 49,400 Total set up hours 520

Your assistant has provided you with the following additional information about the production of batches of BB and LL:

Each Batch of BB: 1,000 Pairs Item Cutting Assembly Total

Direct labour hours 80 120 200 Machine hours 160 120 280 Set up hours 3 1 4 Direct costs $7,500 $6,000 $13,500

Page 312: MODULE 3 Financial Management and Management Accounting 1

Module 3 - Financial Management & Management Accounting 1

Page 312 © CMA Ontario, 2011

Each Batch of LL: 3,000 Pairs Item Cutting Assembly Total

Direct labour hours 150 180 330 Machine hours 150 120 270 Set up hours 1 1 2 Direct costs $9,000 $7,200 $16,200

Upon further inquiry, your assistant has been able to trace the support costs to the two service departments and the two production departments and to identify the following details for potential cost drivers for the service departments:

Item Maintenance Set Up Cutting Assembly Totals Support costs $160,000 $400,000 $440,000 $200,000 $1,200,000 Direct labour hours 0 0 21,400 27,600 49,000 Machine hours 0 0 27,800 21,600 49,400 Set up hours 0 0 340 180 520

Your assistant has also collected the following information on activities and their cost drivers:

Support Activities Cost Activity Category Cost Driver Maintenance $160,000 Product sustaining Machine hours

Set ups 400,000 Batch related Set up hours Cutting supervision 280,000 Batch related Set up hours Cutting depreciation 160,000 Business sustaining Machine hours

Assembly supervision 160,000 Unit related Direct labour hours Assembly depreciation 40,000 Business sustaining Machine hours

Required:

a. Using a single, plant wide cost driver rate based on direct labour hours, determine the costs per pair of BB and LL.

b. Determine the costs per pair of BB and LL using activity based costing.

Page 313: MODULE 3 Financial Management and Management Accounting 1

Module 3 - Financial Management & Management Accounting 1

Page 313 © CMA Ontario, 2011

Solutions -

Multiple Choice Questions

1. a Activity based costing focuses on activities as the fundamental cost objects and uses the costs of these activities as building blocks for compiling the costs of other cost objects. The final product costs are built up from the costs of the specific activities undergone in manufacturing the product. Because of the many activity areas, indirect costs are accumulated in many indirect cost pools. Traditional product based costing is much simpler and more general in that costs are accumulated in only one or a few cost pools. Costs may be allocated based on cost drivers, such as direct labour hours or machine hours, or a cost application base could be financial, such as direct labour costs or direct material costs.

2. d Organizations most likely to benefit from activity based costing have

operating personnel with little faith in the accuracy of existing cost information, a widely diverse set of operating activities and many changes in activities over time, among other characteristics.

3. a 20 hours x $400 / 1,000 scissors = $8 4. b (200 x $4) + (12 x $40) + (5 x $6) = $1,310 / 100 = $13.10 5. b Since these costs have no apparent cause and effect relationship with activity

levels in the organization 6. c 7. b Insurance on a factory would be properly treated as a facility sustaining

activity 8. a This is a direct materials cost – ABC only allocates indirect costs 9. b Only properly designed ABC systems have the potential to produce more

accurate estimates of product cost. ABC costing systems can be used in any type of organization including service, not-for-profit and government. ABC systems can be expensive and, in some cases, not cost effective.

10. d There is no apparent relationship between the number of direct labour hours

needed to make the product and the batch related costs.

Page 314: MODULE 3 Financial Management and Management Accounting 1

Module 3 - Financial Management & Management Accounting 1

Page 314 © CMA Ontario, 2011

Problem 1 The following is the solution with the calculation for the costs in part a) shown in the calc column:

A B C % Number Cost % Number Cost % Number Cost

Pick-up Rural 100% 1,000 $208 50% 500 $108 0% 0 $0 Urban 0% 0 $0 50% 500 $105 100% 1,000 $205

Sorting Automatic 60% 600 600 90% 900 900 100% 1,000 1000 Manual 40% 400 2400 10% 100 600 0% 0 0

Transportation Ground 80% 800 1200 50% 500 750 0% 0 0 Air 20% 200 2400 50% 500 6000 100% 1,000 12000

Delivering Rural 10% 100 300 40% 400 1200 10% 100 300 Urban 90% 900 1800 60% 600 1200 90% 900 1800

Weight Less than 1 60% 600 300 90% 900 450 70% 700 350 One to five 35% 350 700 10% 100 200 30% 300 600 More than five

5% 50 450 0% 0 0 0% 0 0

Total $10,358 $11,513 $16,255

Page 315: MODULE 3 Financial Management and Management Accounting 1

Module 3 - Financial Management & Management Accounting 1

Page 315 © CMA Ontario, 2011

Problem 2 a) The following is the formula for total salaries:

(20 * 50,000) + (35 * 75,000) + (13 * 130,000) + (8 * 275,000) = $7,515,000

The rate per hour is:

$7,515,000 / (4,000 + 7,350 + 2,925 + 1,840) = $7,515,000 / 16,115

= $466.34

The cost for each job will be computed by multiplying the rate per day by the total number of all staff members working on the job. The cost for Simple jobs will be: (8 + 15 + 5 + 2) x $466.34 = $13,990 Medium jobs: (15 + 28 + 15 + 5) x 466.34 = $29,379 Complex jobs: (23 + 44 + 22 + 8) x $466.34 = $45,235

b) Rates: Support: ($50,000 x 20) / 4,000 = $250 per day Analyst: ($75,000 x 35) / 7,350 = $357.14 per day Manager: ($130,000 x 13) / 2,925 = $577.78 per day NP Partner: ($275,000 x 8) / 1,840 = $1,195.65 per day

Simple Medium Complex Support: $250 x 8 | 15 | 23 $2,000 $3,750 $5,750 Analyst: $357.14 x 15 | 28 | 44 5,357 10,000 15,714 Manager: $577.78 x 5 | 15 | 22 2,889 8,667 12,711 NP Partner: $1,195.65 x 2 | 5 | 8 2,391 5,978 9,565

$12,637 $28,395 $43,740

c) If we take the costs computed in Part b) as more accurate the costing approach in

Part a) is over-costing all the jobs. If cost is marked-up by 50% and if DEC’s competitors have the same cost structure, DEC will be quoting higher prices for all its jobs and will lose business if the market is competitive (that is, it is unable to differentiate its services so that it can charge a higher price).

Page 316: MODULE 3 Financial Management and Management Accounting 1

Module 3 - Financial Management & Management Accounting 1

Page 316 © CMA Ontario, 2011

Problem 3 a)

Current Costing System Cost Item TV Board PC Board

Direct Materials $80.00 $140.00 Materials Handling 8.00 14.00 Direct Labour 21.00 56.00 Machine Time 5.00 15.00 Variable manufacturing costs

6.00 16.00

Total cost per unit $120.00 $241.00

b)

Activity Based Costing System Cost Item TV Board PC Board

Direct Materials $80.00 $140.00 Material Support Costs: - Procurement $2.50 $5.50 - Production Scheduling 2.00 2.00 - Packaging and Shipping 4.00 4.00

- Total costs $8.50 $11.50

Variable Support Costs: - Machine setup $3.20 $4.80 - Hazardous waste disposal 0.06 1.05 - Quality control 3.50 7.00 - General Supplies 0.60 0.60

- Total costs $7.36 $13.45

Manufacturing support costs: - Machine insertion $9.60 $14.00 - Manual insertion 4.00 80.00 - Wave soldering 1.20 1.20

- Total costs $14.80 $95.20

Total Cost $110.66 $260.15

Page 317: MODULE 3 Financial Management and Management Accounting 1

Module 3 - Financial Management & Management Accounting 1

Page 317 © CMA Ontario, 2011

c) The current system is over-costing the TV board and under-costing the PC board. The major culprit causing the distortions is the manual insertion cost. The consequence is the TV boards are more attractive than the current system would recognize and the PC boards are less attractive. Another managerial implication is the organization might consider developing a system for machine insertions on the PC boards.

Page 318: MODULE 3 Financial Management and Management Accounting 1

Module 3 - Financial Management & Management Accounting 1

Page 318 © CMA Ontario, 2011

Problem 4

a.

Cust 1 Cust 2 Cust 3 Cust 4 Initial sales $1,000.00 $1,000.00 $2,500.00 $3,000.00 Less: returns 0.00 200.00 500.00 1,500.00

Net sales $1,000.00 $800.00 $2,000.00 $1,500.00 Cost of merchandise sold 750.00 600.00 1,500.00 1,125.00

Gross margin $250.00 $200.00 $500.00 $375.00 Customer related costs Process mail orders $0.00 $30.00 $20.00 $0.00 Process phone orders 20.00 0.00 0.00 80.00 Process returns 0.00 20.00 10.00 120.00 Process overnight delivery requests 4.00 0.00 0.00 48.00 Maintain customer relations 50.00 50.00 50.00 50.00 Total customer related costs $74.00 $100.00 $80.00 $298.00 Customer profitability $176.00 $100.00 $420.00 $77.00 Profit divided by net sales 17.60% 12.50% 21.00% 5.13%

b. The profit divided by net sales measure identifies the effect of customer

demands on this organization’s costs. This measure identifies Customer 4 as, by far, the worst customer who inflicts significant customer-related costs on the organization.

c. While Customer 4 has the highest level of initial sales, the returns and other customer-related costs are so excessive for this customer, it becomes the least profitable. Clearly, the returns policy is promoting unreasonable behaviour from this customer. Pepper Company should consider levying a restocking fee when returns exceed a certain level, for example, 10% of sales.

Page 319: MODULE 3 Financial Management and Management Accounting 1

Module 3 - Financial Management & Management Accounting 1

Page 319 © CMA Ontario, 2011

Problem 5 a)

BB LL Direct Costs - Cutting $7,500 $9,000 - Assembly 6,000 7,200 Support Costs "= 200 * 24.49 4,898 8081.7 " = 330 * 24.49

Total Costs $18,398 $24,282 Units made 1,000 3,000 Cost per unit $18.40 $8.09

b) First compute the cost driver rates:

Support Activities Cost Activity Category Cost Driver Cost Driver Units Cost Driver Rate

Maintenance $160,000 Product sustaining Machine hours 49,400 $3.24

Setups 400,000 Batch related Setup hours 520 $769.23

Cutting supervision 280,000 Batch related Setup hours 340 $823.53

Cutting depreciation 160,000 Business sustaining Machine hours 27,800 $5.76

Assembly supervision 160,000 Unit related Direct labour hours

27,600 $5.80

Assembly depreciation

40,000 Business sustaining Machine hours 21,600 $1.85

Now, use these rates to compute the activity based costs:

Page 320: MODULE 3 Financial Management and Management Accounting 1

Module 3 - Financial Management & Management Accounting 1

Page 320 © CMA Ontario, 2011

BB LL Direct Cost $13,500.00 $16,200.00 - Maintenance = 280 * 3.24 906.88 = 270 * 3.24 874.49 - Setup = 4 * 769.23 3,076.92 = 2 * 769.23 1,538.46 - Cutting Supervision = 3 * 823.53 2,470.59 = 1 * 823.53 823.53 - Cutting Depreciation = 160 * 5.76 920.86 = 150 * 5.76 863.31 - Assembly Supervision =120 * 5.80 695.65 =180 * 5.80 1,043.48 - Assembly Depreciation = 120 * 1.85 222.22 = 120 * 1.85 222.22

Total Indirect Cost 8,293.13 5,365.49 Total Cost $21,793.13 $21,565.49 Units Made 1,000 3,000

Cost per unit $21.79 $7.19

Page 321: MODULE 3 Financial Management and Management Accounting 1

Module 3 - Financial Management & Management Accounting 1

Page 321 © CMA Ontario, 2011

7. Service Department Cost Allocation

Learning Objectives

After completing this chapter, you will:

1. Understand and be able to explain the nature of support departments and why organizations allocate support department costs to other departments.

2. Be able to undertake the calculations for the direct, step (sequential) and reciprocal methods of support department cost allocation.

3. Understand why variable and fixed support department costs should be allocated separately and how each should be allocated.

Support and Production Departments

All organizations have production departments that make goods or services and other departments, called support departments, which provide services the production departments need to do their work.

There are a number of reasons why organizations allocate support department costs to production departments:

1. Since these non-production departments provide the support the production departments need to complete their work, it is appropriate to allocate support costs to the production departments so the costs of products made in the production departments will reflect the needed support services. By including production and non-production costs in total product costs, organizations have a better understanding of the costs of all resources needed to produce a product (good or service) and, therefore, a better basis to evaluate product profitability.

2. Allocating non-production costs to the production departments and making the production department managers accountable for those costs, motivates production department managers to use support departments wisely.

3. In contractual situations, such as reimbursement costing, international transfer pricing and insurance claims, organizations may either be required or motivated by their own self-interest to include the costs of both manufacturing and non-manufacturing resources in the product cost.

The system structure

It is most common to organize non-manufacturing cost pools by function. Common examples of support departments include: accounting, legal, administrative, human resources and information technology.

The following diagram shows the structure of flow of costs:

Page 322: MODULE 3 Financial Management and Management Accounting 1

Module 3 - Financial Management & Management Accounting 1

Page 322 © CMA Ontario, 2011

Costs are initially traced to the departments that incurred the costs, which are the direct costs of each of these departments. The first stage of the cost allocation process is for support departments to allocate their direct costs to the production departments. Note that this first stage can involve the support departments allocating costs to themselves. The second stage of the cost allocation process is for the production departments to allocate their costs to the final cost objects.

An example:

The process of allocating support departments costs is best illustrated using an example. Brant Metal Industries (BMI) has the following four departments:

1. Production departments

a. Cutting

b. Assembly

2. Support departments

a. Maintenance

b. Engineering

Page 323: MODULE 3 Financial Management and Management Accounting 1

Module 3 - Financial Management & Management Accounting 1

Page 323 © CMA Ontario, 2011

The following is a summary of the most recent activities and costs in BMI. Costs incurred directly by the cutting and assembly departments totalled $7,800,000 and $6,700,000, respectively.

Direct Method of Support Cost Allocation

In the direct method of support cost allocation, the costs of the support departments are allocated directly to the production departments. All support services provided to support departments are ignored.

Cutting Assembly

Operating department's overhead $7,800,000 $6,700,000

Allocation from maintenance: $2,400,000 x 1,800 / 2,800 | 1,000 / 2,800 1,542,857 857,143 Allocation from engineering: $750,000 x 800 / 1,200 | 400 / 1,200 500,000 250,000

$9,842,857 7,807,143

The rate per unit of service under the direct method can be calculated as follows: Maintenance: $2,400,000 / 2,800 = $857.14 Engineering: $750,000 / 1,200 = $625

Page 324: MODULE 3 Financial Management and Management Accounting 1

Module 3 - Financial Management & Management Accounting 1

Page 324 © CMA Ontario, 2011

Step Method (also known as the step-down method and the sequential method) In the step method, the approach is to allocate the support department costs sequentially. One support department is chosen to allocate its direct costs first. That support department computes its cost driver rate by dividing its direct costs by the total of all support units provided to other departments, both support and production. Then that support department allocates its costs to the other departments. Each department is allocated an amount equal to the number of support units it receives multiplied by the cost driver rate. Then, the support department whose costs have been allocated is dropped from the process and the next support department repeats the same process by allocating its direct cost plus the cost it has been allocated by the previous support department(s).

Order makes a difference. The order in which the support departments allocate their costs will affect the final support department cost allocations to the production departments. For this reason, folklore has developed surrounding the appropriate order. Perhaps the most popular approach is to start with the support department that provides the highest proportion of its costs to the remaining support departments. Let’s illustrate this approach using the data for BMI.

The following is the distribution of services showing the proportions each support department provides to other support departments:

Maintenance provides 19.44% of its services to Engineering while Engineering provides 13.33% of its services to maintenance. So using the above rule for choosing order, we will allocate the maintenance department costs first. The following is the result:

Page 325: MODULE 3 Financial Management and Management Accounting 1

Module 3 - Financial Management & Management Accounting 1

Page 325 © CMA Ontario, 2011

Let’s study the calculations in this exhibit. We chose the maintenance department to go first. Note how the support units the maintenance supplies itself are ignored. The cost driver rate of $685.71 for maintenance is computed by dividing its direct cost of $2,400,000 by 3,500, the total number of cost driver units it provides to other departments. This rate is then used to allocate the maintenance department’s direct costs to all the other departments. For example, the $480,000 cost allocated to engineering equals the service units of 700 provided to engineering multiplied by the $685.71 rate. (Remember, the earlier warning that in these calculations, the amounts are rounded to two decimal places but, the spreadsheet accuracy is maintained.)

Now that maintenance department costs have been allocated, we act as if the maintenance department does not exist and turn to consider the engineering department to allocate the engineering department’s costs. The engineering department’s cost to be allocated is $1,230,000, which includes its direct costs of $750,000 and the $480,000 of costs allocated to it from the maintenance department. We divide the total of these costs by the total units of support provided to the remaining two departments (1,200) to compute the driver rate of $1,025 per unit of support. This rate is now used to allocate the engineering department costs to the two production departments where the cost allocated to each department equals the driver rate, multiplied by the support units provided to that department.

The allocation of support department costs is now complete. The production departments will add the costs allocated to them by the support departments to their direct costs to determine the costs they will each allocate to the final cost objects.

The calculation of the support department cost allocations if the engineering department goes first is left as an exercise at the end of this chapter.

Page 326: MODULE 3 Financial Management and Management Accounting 1

Module 3 - Financial Management & Management Accounting 1

Page 326 © CMA Ontario, 2011

Reciprocal Method

The reciprocal method of support department cost allocation recognizes all support department activities and, therefore, provides the most accurate modelling of the support patterns.

The first step in the reciprocal method is to compute what is called the reciprocal cost for each of the support departments. The reciprocal cost for each support department is its direct cost plus its share, based on the proportion of support units consumed, of the direct costs of the other support departments. We ignore any support department’s self-service units. (It turns out whether we ignore self-service units results or not in the same service department cost allocations.)

The following is what the three reciprocal cost equations would be for BMI where M is the reciprocal cost of the maintenance department and E is the reciprocal cost of the engineering department (remember we ignore self service units). M = 2,400,000 + (200/1400) E E = 750,000 + (700 / 3500) M We have two equations in two unknowns that we now have to solve to find M and E.

These equations can be quickly solved using Excel but they can also be solved using what is called the method of substitution. The following is how that works for this example. We substitute the second equation into the first equation to get the following: M = 2,400,000 + (200/1400)(750,000 + 700/3500*M) M = 2,400,000 + 107,143 + (14/490)M M = 2,507,143 + (1/35)M (34/35)M = 2,507,143 M = 2,507,143 (35/34) = $2,580,882 We can now substitute this value for RCM in either of the initial reciprocal cost equations to find E. Using the second equation, we have: E = 750,000 + (700 / 3500) M E = 750,000 + (700 / 3500) 2,580,882 E = 1,266,176 We now use these reciprocal costs to determine the allocations to the production departments. Each production department is assigned its share of the reciprocal cost of each support department based on its proportion consumed of the department’s support units:

Page 327: MODULE 3 Financial Management and Management Accounting 1

Module 3 - Financial Management & Management Accounting 1

Page 327 © CMA Ontario, 2011

Cutting Assembly

Operating department's overhead $7,800,000 $6,700,000

Allocation from maintenance: $2,580,882 x 1,800 / 3,500 | 1,000 / 3,500 1,327,311 737,395 Allocation from engineering: $1,266,176 x 800 / 1,400 | 400 / 1,400 723,529 361,765

$9,850,840 7,799,160

The rate per unit of service under the reciprocal method can be calculated as follows: Maintenance: $2,580,882 / 3,500 = $737.39 Engineering: $1,266,176 / 1,400 = $904.41 The following is the summary of the support cost allocations to the production departments for each of the three methods (recall that other allocation sequences in the step method will result in different cost allocations). Cutting Assembly Total

Direct method $9,842,857 $7,807,143 $17,650,000 Step method 9,854,286 7,795,714 $17,650,000 Reciprocal method 9,850,840 7,799,160 $17,650,000 At first glance, there does not appear to be much difference between the three methods. However, if we compare the rate per unit of service, the distortions become much clearer: Maintenance Engineering

Direct method $857.14 $625.00 Step method 685.71 1,025.00 Reciprocal method 737.39 904.41 Support Department Allocations – Variable and Fixed Costs To this point, we have discussed support department cost allocations without considering the nature of the support department costs. Many management accountants believe that each support department should maintain two support cost pools, one for variable costs and one for fixed costs. Variable costs should be allocated following the procedure we have already described, that is, variable costs should be allocated based on the number of cost driver units. This results in a so-called dual rate system of support department cost allocations.

Page 328: MODULE 3 Financial Management and Management Accounting 1

Module 3 - Financial Management & Management Accounting 1

Page 328 © CMA Ontario, 2011

Management accountants supporting the dual rate system of allocating support department costs argue that the cost driver for variable support costs is actual use, while the cost driver for fixed support cost is the amount of capacity acquired or the average long-term use of capacity by the various departments. Therefore, proponents of the dual rate system believe that fixed support costs should be allocated based on the planned use of support department capacity or long-term average use of the capacity. Note that if the relative use of support department capacity is stable then short-term use will approximate long-term average use and, once again, allocating both fixed and variable support costs together is deemed to be appropriate. The only other consideration is whether all fixed support costs should be allocated. Some management accountants argue that if average long-term use of capacity is less than practical capacity then not all fixed support costs should be allocated. For example, if the total fixed support costs of a support unit equal $1,000,000 and, on average, only 90% of the practical capacity of the support department is used, then management accountants advocating this approach would argue only $900,000 ($1,000,000 * 90%) of the fixed support costs should be allocated with the balance of the fixed support costs charged to the income statement each year as an idle capacity cost. The issue of whether fixed and variable support costs should be allocated separately requires the judgment of each management accountant. The overriding consideration is whether the resulting cost allocations reflect the decision making and motivational objectives of undertaking the cost allocations.

Page 329: MODULE 3 Financial Management and Management Accounting 1

Module 3 - Financial Management & Management Accounting 1

Page 329 © CMA Ontario, 2011

Problems with Solutions Multiple Choice Questions 1. B.F. Spurs Co. has two service Departments A and B and two production

departments. The two service departments render services to each other as well as to the production departments. The costs of service Department A are allocated to the other three departments based on their direct usage of Department A's services. The costs of service Department B are then allocated to the production departments. Which of the following cost allocation methods is being used to allocate the service departments' costs?

a) Dual rate b) Direct c) Step-down (sequential) d) Activity based e) Reciprocal (simultaneous equations) 2. A business uses the step-down method to allocate service department costs to the

manufacturing departments. Assume there are two service departments and two manufacturing departments, as shown below: Service

Departments Manufacturing Departments

Plant Admin.

Custodial Services

Cutting

Polishing

Costs $360,000 $90,000 $261,000 $689,000 Labour hours 25,000 6,000 18,000 30,000 Space occupied (m2) 10,000 1,000 5,000 45,000

Plant administration costs are allocated based on labour hours and custodial services costs are allocated based on space occupied. The total costs of the cutting and polishing departments (rounded to the closest .000), after allocating all the service department costs, starting with plant administration are:

a) Cutting - $396,000 and polishing - $914,000 b) Cutting - $405,000 and polishing - $995,000 c) Cutting - $381,000 and polishing - $889,000 d) Cutting - $394,000 and polishing - $1,006,000 e) Cutting - $380,000 and polishing - $892,000

Page 330: MODULE 3 Financial Management and Management Accounting 1

Module 3 - Financial Management & Management Accounting 1

Page 330 © CMA Ontario, 2011

The Following Data Apply to Items 3 – 4: The managers of Rochester Manufacturing are discussing ways to allocate the cost of service departments such as quality control and maintenance to the production departments. To aid them in this discussion, the controller has provided the following information: Quality

Control Maintenance Machining Assembly Total Budgeted overhead costs before allocation $350,000 $200,000 $400,000 $300,000 $1,250,000 Budgeted machine hours - - 50,000 - 50,000 Budgeted direct labour hours

- - - 25,000 25,000

Budgeted hours of service: Quality control - 7,000 21,000 7,000 35,000 Maintenance 10,000 - 18,000 12,000 40,000 3. Using the direct method, the total amount of overhead allocated to each machine

hour in the machining department would be: a) $2.40 b) $5.25 c) $8 d) $9.35 e) $15.65 4. If Rochester Manufacturing uses the step-down method of allocating service costs

beginning with quality control, the maintenance department costs allocated to the assembly department would be:

a) $70,000 b) $108,000 c) $162,000 d) $200,000 e) $210,000

Page 331: MODULE 3 Financial Management and Management Accounting 1

Module 3 - Financial Management & Management Accounting 1

Page 331 © CMA Ontario, 2011

Problem 1 Repeat the step allocation exercise for BMI but allocate the support department costs in the following order: engineering and then maintenance to illustrate that order in the step method does make a difference in terms of the final cost allocations. The following data is used in Problem 2 – 5: This data reflects the actual service units provided by the support departments to the two production departments.

Actual Service Units Provided To

Power Mtce Fabrication Finishing Total Variable

Cost

Su

pp

ort

Un

its

Pro

vid

ed

Fro

m Power 700 1100 4500 2700 9000 $1,200,000

Mtce 1400 100 1800 1200 4500 $600,000

This data reflects the long-term average planned service units provided by the support departments to the two production departments and the costs shown are the fixed costs for the two support departments.

Planned Service Units Provided To

Power Mtce Fabrication Finishing Total

Fixed Cost

Su

pp

ort

Un

its

Pro

vid

ed

Fro

m

Power 600 1200 4800 2900 9500 $1,500,000

Mtce 1700 200 1600 1500 5000 $300,000

Problem 2 Using the actual support units provided and the total costs, allocate the support department costs to the production departments using:

a. The direct method

b. The step method with power first and then maintenance

c. The step method with maintenance first and then power

d. The reciprocal method

Page 332: MODULE 3 Financial Management and Management Accounting 1

Module 3 - Financial Management & Management Accounting 1

Page 332 © CMA Ontario, 2011

Problem 3

Using the actual support units provided and the variable costs, allocate the support department costs to the production departments using:

a. The direct method

b. The step method with power first and then maintenance

c. The step method with maintenance and then power

d. The reciprocal method

Problem 4

Using the long-term average support units provided and the fixed costs, allocate the support department costs to the production departments using:

a. The direct method

b. The step method with power first and then maintenance

c. The step method with maintenance and then power

d. The reciprocal method.

Problem 5

Summarize your results in Questions 2 through 4 and discuss the implications of those results.

Page 333: MODULE 3 Financial Management and Management Accounting 1

Module 3 - Financial Management & Management Accounting 1

Page 333 © CMA Ontario, 2011

Solutions: Multiple Choice Questions 1. c The step-down allocation method allows partial recognition of services

rendered by service departments to other service departments. This method involves a sequence of allocations such as described for B.F. Spurs Co.

2. d Plant administration allocation

= $360,000 ÷ (6,000 + 18,000 + 30,000) = $6.667/hr. Custodial services allocation = [$90,000 + (6,000 x $6.667)] ÷ (5,000 + 45,000) = $2.60/m2 Cutting department costs = $261,000 + (18,000 x $6.667) + (5,000 x $2.60) = $394,000 Polishing department costs = $689,000 + (30,000 x $6.667) + (45,000 x $2.60) = $1,006,000

3. e Quality control: $350,000 x 21,000 / 28,000 $262,500 Maintenance: $200,000 x 18,000 / 30,000 120,000 Add: machining department's overhead costs 400,000 $782,500 /50,000 MH $15.65

4. b Quality Maintenance Assembly

$350,000 $200,000 Allocate quality: $350,000 x 7/35 70,000 $270,000 Allocate maintenance: $270,000 x 12/30 $108,000

Page 334: MODULE 3 Financial Management and Management Accounting 1

Module 3 - Financial Management & Management Accounting 1

Page 334 © CMA Ontario, 2011

Problem 1

Problem 2

a.

Page 335: MODULE 3 Financial Management and Management Accounting 1

Module 3 - Financial Management & Management Accounting 1

Page 335 © CMA Ontario, 2011

b.

c.

Page 336: MODULE 3 Financial Management and Management Accounting 1

Module 3 - Financial Management & Management Accounting 1

Page 336 © CMA Ontario, 2011

d. P = 2,700,000 + 1,400/4,400 M M = 900,000 + 1,100/8,300 P Simplifying the fractions, we get: P = 2,700,000 + 7/22 M M = 900,000 + 11/83 P Substituting the second equation into the first, we get: P = 2,700,000 + 7/22 (900,000 + 11/83 P) P = 2,700,000 + 286,364 + 77/1,826*P 1,749 / 1,826 *P = 2,986,364 P = 2,986,364 * 1,826 / 1,749 P = 3,117,839 M = 900,000 + 11/83(3,117,839) = 1,313,208 Fabrication Finishing

Allocation from power:

$3,117,839 x 4,500 / 8,300 | 2,700 / 8,300 $1,690,395 $1,014,237 Allocation from maintenance: $1,313,208 x 1,800 / 4,400 | 1,200 / 4,400 537,221 358,148

$2,227,616 $1,372,385

The rate per unit of service under the reciprocal method can be calculated as follows:

Power: $3,117,839 / 8,300 = $375.64 Maintenance: $1,313,208 / 4,400= $298.46

Page 337: MODULE 3 Financial Management and Management Accounting 1

Module 3 - Financial Management & Management Accounting 1

Page 337 © CMA Ontario, 2011

Problem 3

a.

b.

Page 338: MODULE 3 Financial Management and Management Accounting 1

Module 3 - Financial Management & Management Accounting 1

Page 338 © CMA Ontario, 2011

c.

Page 339: MODULE 3 Financial Management and Management Accounting 1

Module 3 - Financial Management & Management Accounting 1

Page 339 © CMA Ontario, 2011

d. P = 1,200,000 + 1,400/4,400 M M = 600,000 + 1,100/8,300 P Simplifying the fractions, we get: P = 1,200,000 + 7/22 M M = 600,000 + 11/83 P Substituting the second equation into the first, we get: P = 1,200,000 + 7/22 (600,000 + 11/83 P) P = 1,200,000 + 190,909 + 77/1,826*P 1,749 / 1,826 *P = 1,390,909 P = 1,390,909 * 1,826 / 1,749 P = 1,452,144 M = 600,000 + 11/83(1,452,144) = 792,453 Fabrication Finishing

Allocation from power:

$1,452,144 x 4,500 / 8,300 | 2,700 / 8,300 $787,307 $472,384 Allocation from maintenance: $792,453 x 1,800 / 4,400 | 1,200 / 4,400 324,185 216,124

$1,111,492 $688,508

The rate per unit of service under the reciprocal method can be calculated as follows:

Maintenance: $1,452,144 / 8,300 = $174.96 Engineering: $792,453 / 4,400= $180.10

Page 340: MODULE 3 Financial Management and Management Accounting 1

Module 3 - Financial Management & Management Accounting 1

Page 340 © CMA Ontario, 2011

Problem 4

a.

b.

Page 341: MODULE 3 Financial Management and Management Accounting 1

Module 3 - Financial Management & Management Accounting 1

Page 341 © CMA Ontario, 2011

c.

Page 342: MODULE 3 Financial Management and Management Accounting 1

Module 3 - Financial Management & Management Accounting 1

Page 342 © CMA Ontario, 2011

d. P = 1,500,000 + 1,700/4,800 M M = 300,000 + 1,200/8,900 P Simplifying the fractions, we get: P = 1,500,000 + 17/48 M M = 300,000 + 12/89 P Substituting the second equation into the first, we get: P = 1,500,000 + 17/48 (300,000 + 12/89 P) P = 1,500,000 + 106,250 + 204/4,272*P 4,068 / 4,272 *P = 1,606,250 P = 1,606,250* 4,272 / 4,068 P = 1,686,799 M = 300,000 + 1,200/8,900 (1,686,799) = 527,434 Fabrication Finishing

Allocation from power:

$1,686,799 x 4,800 / 8,900 | 2,900 / 8,900 $909,734 $549,631 Allocation from maintenance: $527,434 x 1,600 / 4,800 | 1,500 / 4,800 175,811 164,823

$1,085,545 $714,454

The rate per unit of service under the reciprocal method can be calculated as follows:

Maintenance: $1,686,799 / 8,900 = $189.53 Engineering: $527,434 / 4,800= $109.88

Page 343: MODULE 3 Financial Management and Management Accounting 1

Module 3 - Financial Management & Management Accounting 1

Page 343 © CMA Ontario, 2011

Question 5

One of the important principles in management accounting is that an appropriate cost driver should be used to allocate any indirect cost. If we accept that planned or average use is the cost driver for fixed costs and actual use is the driver for variable costs, then we would conclude that doing separate allocations for fixed and variable costs will result in the most appropriate cost allocations. So we can take the cost allocations in Questions 3 and 4 as the most reasonable and compare the costs allocations in Question 2 to those. Within Questions 3 and 4, the reciprocal allocation method provides the cost allocations that track the actual support use patterns. Note that the direct cost allocations come close to the values produced by the reciprocal method when fixed costs and variable costs are allocated separately.

Page 344: MODULE 3 Financial Management and Management Accounting 1

Module 3 - Financial Management & Management Accounting 1

Page 344 © CMA Ontario, 2011

•••

8. Joint and By-Product Costing

Learning Objectives

After completing this chapter, you will understand:

1. The nature of joint products, joint costs and by-products

2. Why management accountants allocate joint costs

3. Be able to apply the five major approaches to joint cost

4. The insights provided by joint costs in decision making.

Joint Costs, Joint Products and By-Products:

In some production environments, a single raw material produces multiple products. For example, refining a barrel of crude oil will produce (among other products) gasoline, diesel fuel, gas, naphtha and kerosene. Products that are jointly produced from a single raw material, in this case crude oil, are called joint products. A by-product is a joint product that has a relatively low (or negative value). The point at which the single raw material is rendered into the joint and by-products is

called the split-off point and the cost incurred up to the split-off point is called the joint cost.

Why Allocate Joint Costs

As we will discuss below there is no decision making need to allocate joint costs. Any economic decisions can be made without allocating joint costs. In fact, basing economic decisions on joint cost allocations will almost always lead to incorrect decisions. Moreover, it is important to remember, all joint cost allocations are arbitrary and are, therefore, subject to debate and criticism.

There are many practical reasons to allocate joint costs, almost all of which relate to contractual reasons:

1. Joint cost allocations are required to prorate joint costs between inventory and cost of goods sold for external reporting or taxation purposes.

Page 345: MODULE 3 Financial Management and Management Accounting 1

Module 3 - Financial Management & Management Accounting 1

Page 345 © CMA Ontario, 2011

2. Joint cost allocations may be used for transfer pricing purposes within organizations. We will discuss transfer pricing in Chapter 20.

3. Joint cost allocations may be required for cost based contracts wherein the price paid is a mark-up on reported costs.

4. Joint cost allocations may be required for insurance purposes when a joint product is lost, damaged or stolen.

5. Joint cost allocations may be required by regulators when one of the joint products is sold in a regulated industry and the allowed product price is cost based.

6. Joint cost allocations may be required in litigation, for example, in a predatory pricing environment to determine whether a product is being sold below its costs.

Again, remember that joint cost allocations are not required for any economic based decision making.

Joint Cost Allocation Methods

While there are variations of the basic methods, there are four basic methods for allocating joint costs to joint products. These are:

1. Physical output

2. Value at split-off

3. Net realizable value

4. Constant gross margin percent

The following example will be used to illustrate these four methods. Jack Chemical Company manufactures chemicals used for industrial applications requiring bleaching and cleaning. Chemicals are made in batches of 1,000 litres. The process begins by blending 400 litres of alpha costing $30 per litre with 600 litres of beta costing $50 per litre. The mixture is then subjected to pressure and heated to produce 300 litres of Product A, 600 litres of Product B and 100 litres of Product C. This manufacturing process costs (exclusive of the raw materials cost) $15,000 per batch.

Products A, B and C can be sold for $90, $60 and $2 per litre, respectively. Product A can be sold as is or refined in batches of 300 litres at a cost of $8,000 per batch to produce product D that can be sold for $120 per litre. In what follows, we will assume that Jack Chemical Company treats all the products as joint products.

The Decision

To underline the point that economic decision making does not require any cost allocations, consider the decision relating to whether or not to process a batch of

Page 346: MODULE 3 Financial Management and Management Accounting 1

Module 3 - Financial Management & Management Accounting 1

Page 346 © CMA Ontario, 2011

chemicals. The following diagram and table summarize the process and the financial facets of this decision:

Begin by noting that selling Product A at the split-off point provides a contribution of $27,000, while refining Product A to produce Product D provides an incremental profit contribution of $28,000. Therefore, processing Product A further increases the batch contribution.

The following table summarizes the contribution provided by each batch. Note that neither the decision of whether to process a batch of chemicals nor the decision of whether to sell Product A or process it further to produce Product D required an allocation of joint costs.

Page 347: MODULE 3 Financial Management and Management Accounting 1

Module 3 - Financial Management & Management Accounting 1

Page 347 © CMA Ontario, 2011

Product Units Price Additional

Cost Net Realizable

Revenue Formula

Revenues

B 600 60 0 36,000 =(60*600)-0

C 100 2 0 200 =(2*100)-0

D 300 120 8,000 28,000 =(300*120)-8000

Total revenue $64,200

Batch costs

Alpha 400 30 0 $12,000 =(30*400)-0

2 Beta 600 50 0 30,000 =(50*600)-0

Processing cost 15,000 15,000 =15000

Joint cost $57,000

Batch contribution

$7,200

Physical Output Method

The physical output method allocates joint costs based on a physical measure such as litres or kilograms. In this setting, the physical output method would allocate the joint costs of $57,000 based on the volume of joint products produced. The following is a table summarizing the joint cost allocation that would result following the physical output method:

Product Units % of Total Joint Cost Allocation

Formula

A 300 30% $17,100 30% * 57,000

B 600 60% 34,200 60% * 57,000

C 100 10% 5,700 10% * 57,000

Total 1,000 100% $57,000

Page 348: MODULE 3 Financial Management and Management Accounting 1

Module 3 - Financial Management & Management Accounting 1

Page 348 © CMA Ontario, 2011

Value at Split-Off Method

The value at split-off method allocates joint costs based on the value of the joint products at the split-off point. The following table summarizes the joint cost allocations that would result from applying this method:

Product Units Price Revenue % of Total Revenue

Joint Cost Allocation

Formula

A 300 90 $27,000.00 42.72% $24,351.27 = 42.72% *

57,000

B 600 60 36,000.00 56.96% 32,468.35 = 56.96% *

57,000 C 100 2 200.00 0.32% 180.38 = 0.32% * 57,000

Total $63,200.00 100.00% $57,000

Net Realizable Value Method

The net realizable value method allocates the joint costs based on the net realizable value of each joint product. The net realizable value for each product is computed by subtracting any additional cost required to complete the product beyond the split-off point from the product’s revenue. From the above analysis we know that Product A will be refined to produce Product D. The following table summarizes the joint cost allocations that would result from using the net realizable value method:

Product

Units Price Revenue Additional

Cost

Net Realizable

Value

% of Total Net

Realizable Value

Joint Cost Allocation

Formula

B 600 60 36,000.00 0 36,000.00 56.07% 31,962.62 = 56.07% *

57,000

C 100 2 200.00 0 200.00 0.31% 177.57 = 0.31% *

57,000

D 300 120 36,000.00 8,000 28,000.00 43.61% 24,859.81 = 43.61% *

57,000

Total $64,200.00 100.00% $57,000.00

Constant Gross Margin Percent Method

In the constant gross margin method, the joint costs are allocated so each product reports the same gross margin percent where gross margin equals revenue minus total cost and gross margin percent equals (revenue – costs)/revenue. Again, recall that the best disposition of Product A is to refine it to produce Product D.

When applying this method, we begin by computing the overall or combined gross margin percentage for all the products

Overall gross margin percentage = batch contribution /total revenue

= 7,200 / 72,200 = 9.97%

Page 349: MODULE 3 Financial Management and Management Accounting 1

Module 3 - Financial Management & Management Accounting 1

Page 349 © CMA Ontario, 2011

This percentage is then multiplied by each product’s revenue to compute the gross margin for the product. The difference between each product’s revenue and gross margin is the total product cost. Finally, the separable cost is deducted from each product’s total cost to determine the joint cost that will be allocated to the product.

The following are the cost allocations that would result using the constant gross margin percent method:

Product Units Price Revenue Less

Gross Margin

Formula Total

Product Cost

Less Separable

Cost

Allocated Joint Cost

B 600 60 $36,000.00 $3,590.03 = 9.97% * 36000 $32,409.97 $0.00 $32,409.97

C 100 2 200.00 $19.94 = 9.97% * 200 180.06 0.00 180.06

D 300 120 36,000.00 $3,590.03 = 9.97% * 36000 32,409.97 8,000.00 24,409.97

Total $57,000.00

Page 350: MODULE 3 Financial Management and Management Accounting 1

Module 3 - Financial Management & Management Accounting 1

Page 350 © CMA Ontario, 2011

By-Products

As mentioned above, joint products that have small values are called by-products. In the above example, joint Product C would clearly qualify as a by-product.

Management accountants have developed different approaches to account for by-products. The following are the two most widely-used:

1. Recognize the by-product’s net realizable value at the point of split-off and net that value against the joint cost. Therefore, if the net realizable value of the product is positive, joint cost will be reduced. The journal entries to accomplish this would be to debit a by-product inventory account at the split-off point for the by-product’s net realizable value and credit work-in-process by the net realizable value. At the time of sale, the by-product inventory account would be credited and cash or accounts receivable debited for the sale price. If the net realizable value is negative, the journal entries described above would be adjusted in the opposite direction. Most management accountants agree this is the preferred treatment of by-products. Moreover, International Accounting Standard #2 requires the use of this method.

2. Recognize the by-product’s net realizable value at the time of sale. Between the split-off point and the point of sale, the by-product is not recognized on the income statement. At the time of sale, the by-product’s net realizable value is recorded either as other income (debit cash or accounts receivable and credit other income for the sales revenue) or as a reduction of cost of goods sold (debit cash or accounts receivable and credit cost of goods sold for the sales revenue). The objection to this approach is since the by-product inventory does not appear on the balance sheet it can be used by management to manipulate income by timing the sale of the by-product to a period when an income increase is wanted. For this reason, recognizing the by-product’s net realizable value at the time of sale is not illustrated in what follows.

Returning to the Jack Chemical Company example above, it would certainly be appropriate to recognize Product C as a by-product. The following are the joint cost allocations that would result under each of the methods described earlier if Product C were treated as a by-product and its expected net realizable value recognized at the split-off point. Recall in what follows that Product C’s estimated net realizable value is $200, so the joint cost to be allocated now is $56,800 ($57,000 - $200).

Physical Output Method:

Product

Units

% Total

Joint Cost Allocation

A 300 33% $18,933 B 600 67% 37,867

900 $56,800

Page 351: MODULE 3 Financial Management and Management Accounting 1

Module 3 - Financial Management & Management Accounting 1

Page 351 © CMA Ontario, 2011

Value at Split-Off Method:

Product

Revenue

% Total

Joint Cost Allocation

A - 300 x $90 $27,000 42.86% $24,343 B - 600 x $60 36,000 57.14% 32,457

$63,000 $56,800

Net Realizable Value Method:

Product

Net Realizable Value

% Total

Joint Cost Allocation

B - 600 x $60 $36,000 56.25% $31,950 D - (300 x $120) - 8,000 28,000 43.75% 24,850

$64,000 $56,800

Constant Gross Margin Percent Method:

As above, we begin by computing the overall or combined gross margin percentage for all the products:

Overall gross margin percentage = batch contribution /total revenue

= 7,200 / 72,000 = 10%

The following are the cost allocations that would result using the constant gross margin percent method:

Product

Units

Price

Revenue Less

Gross Margin

Formula Total

Product Cost

Less Separable Cost

Allocated Joint Cost

B 600 60 $36,000.0

0 $3,600.0

0 = 10% *

36000 $32,400.0

0 $0.00 $32,400.0

0

D 300 120 36,000.00 $3,600.0

0 = 10% *

36000 32,400.00 8,000.00 24,400.00

Total $56,800.0

0

Conclusion:

We have reviewed four approaches management accountants use to allocate joint costs to joint products. A remaining question is what are the advantages and disadvantages of each of these methods. Note that three of the four methods are revenue based and,

Page 352: MODULE 3 Financial Management and Management Accounting 1

Module 3 - Financial Management & Management Accounting 1

Page 352 © CMA Ontario, 2011

therefore, require estimates of future revenues when they used. Two of the revenue methods (net realizable value method and constant gross margin percent method) also require estimates of future processing costs. A desire to avoid the subjectivity of estimating future costs and revenues may be a strong motivation for using the physical output method. However, as a counterpoint, the physical output method takes no account of the value of the individual joint products and using this approach might lead to a situation where the joint cost allocated to a joint product could exceed any reasonable estimate of the product’s eventual selling price – an inventory valuation consideration that would sway the choice toward using a value base approach to cost allocation.

Within the revenue methods management accountants might object to the value at split-off method because it ignores potentially significant value adding activities beyond the split-off point, as we saw with Products A and D in the Jack Chemical Company. A mitigating factor supporting the value at split-off method is it does not require estimates of future processing costs.

With regard to the choice between the net realizable value method and the constant gross margin percent method, the choice appears to be one based on a form of equity consideration. The argument is given that all joint cost allocations are inherently arbitrary one might impose a fairness or equity consideration by requiring the cost allocations result in the same gross margin percent for all joint products.

Management accountants will exercise judgment to determine which approach suits the decision making and the external reporting objectives of their respective organizations.

Page 353: MODULE 3 Financial Management and Management Accounting 1

Module 3 - Financial Management & Management Accounting 1

Page 353 © CMA Ontario, 2011

Problems with Solutions

Multiple Choice Questions

The following information pertains to questions 1 – 2: Omega Company manufactures three chemicals in a joint process. The

manufacturing costs of the joint process include $25,000 of direct materials and $35,000 of conversion costs. All three chemicals are then processed further before they are sold. Other pertinent data is:

Chemicals Sales Value at Split-off Separable Costs Final Sales Value A $50,000 $28,000 $100,000 B 40,000 10,000 60,000 C 30,000 12,000 40,000 1. Using the estimated net realizable value method, the joint costs allocated to Product

A would be: a) $16,800 b) $25,000 c) $28,800 d) $30,000 e) $33,600 2. The decision to process all three chemicals beyond the split-off point is sub-

optimal. If the optimal decision had been made, the income of Omega Company would have improved by:

a) $2,000 b) $10,000 c) $30,000 d) $60,000 e) An amount different from the above 3. The main purpose of allocating joint costs to products is to: a) Determine prices for the products b) Make decisions about dropping the products c) Rationalize economic decisions concerning the products d) Determine the cost of the products e) Determine budgets for the production of the products

Page 354: MODULE 3 Financial Management and Management Accounting 1

Module 3 - Financial Management & Management Accounting 1

Page 354 © CMA Ontario, 2011

The following information pertains to questions 4 – 5: SMT Ltd. manufactures three products. Production begins with a joint process and the three outputs of the joint process are processed further to produce products L, M and N. The outputs at split-off have no market value. Last year, the joint costs amounted to $600,000. Other data for last year is: Product L Product M Product N Selling price per unit $160 $300 $400 Costs per unit after split-off to complete and sell $100 $200 $350 Total output at split-off used in production 16,400 kg 10,000 kg 8,400 kg Production in units 20,000 10,000 7,000 Sales in units 18,000 8,000 7,000 4. Using the physical quantities method (also called the physical measure method or

the average method) of joint costing, the total joint costs allocated to the production of product L last year was:

a) $200,000 b) $327,273 c) $282,759 d) $324,324 e) $282,353 5. Using the estimated (approximate) net realizable value method of joint costing, the

inventory cost per unit of product M is:

a) $226.91 b) $223.53 c) $221.52 d) $220.00 e) $217.24

Page 355: MODULE 3 Financial Management and Management Accounting 1

Module 3 - Financial Management & Management Accounting 1

Page 355 © CMA Ontario, 2011

6. Raynor Manufacturing purchases rough cut trees and processes them up to the split-off point, where two products (paper and pencil casings) are obtained. The products are then sold to an independent company that markets and distributes them to retail outlets. In October, Raynor processed 50 trees (yield is 30,000 sheets of paper and 30,000 pencil casings and no scrap) and had the following production and sales:

Production: 30,000 sheets of paper and 30,000 pencil casings Sales: 29,000 sheets of paper at $.04 per page 30,000 pencil casings at $.10 per casing

Cost of purchasing 50 trees and processing them up to the split-off point to yield 30,000 sheets of paper and 30,000 pencil casings is $1,500. Opening inventories are nil. What are the approximate joint costs assigned to the paper ending inventory if joint costs are allocated using the sales value at split-off method?

a) $14.29 b) $50 c) $435 d) $750 e) $915

Page 356: MODULE 3 Financial Management and Management Accounting 1

Module 3 - Financial Management & Management Accounting 1

Page 356 © CMA Ontario, 2011

7. Beverage Drink Company processes direct materials up to the split-off point, where two products, A and B, are obtained. The following information was collected for the month of July: Direct materials processed: 2,500 litres (with 20% shrinkage)

Production: A 1,500 litres B 500 litres Sales: A $15 per litre B $10 per litre

Cost of purchasing 2,500 litres of direct materials and processing it up to the split-off point to yield a total of 2,000 litres of good products was $4,500. There were no inventory balances of A and B. Product A may be processed further to yield 1,375 litres of Product Z5 for an additional processing cost of $150. Product Z5 is sold for $25 per litre. There was no beginning inventory and ending inventory was 125 litres. Product B may be processed further to yield 375 litres of Product W3 for an additional processing cost of $275. Product W3 is sold for $30 per litre. There was no beginning inventory and ending inventory was 25 litres. What is Product Z5's estimated net realizable value at the split-off:

a) $11,100 b) $22,350 c) $24,225 d) $29,375 e) $34,225

Page 357: MODULE 3 Financial Management and Management Accounting 1

Module 3 - Financial Management & Management Accounting 1

Page 357 © CMA Ontario, 2011

Problem 1

Downeast Fisheries operates a fish processing plant. Fish is processed in batches of 10,000 kilograms. Processing fish yields three products:

1. Fillets, which are sold fresh or quick frozen

2. Fish pieces, which are sold to another fish products plant to be used in breaded fish cakes.

3. Offal (unused parts of the fish), which is transported to a fertilizer plant.

The following table summarizes the yield and financial characteristics of processing a batch of fish:

Fresh Fish Pieces Offal Yield (kilograms) 2,000 1,000 7000 Revenue (disposal cost) per kilogram $6.00 $3.00 -$.20 Separable processing and selling cost per kilogram $1.50 $1.00 0

Fresh fish can be quick frozen with an associated separable processing and selling cost of $.50 per kilogram and sold for $5.50 per kilogram. Quick freezing also incurs a fixed cost of $500 per batch. If the fresh fish is frozen, the $1.50 separable cost of processing the fresh fish would be avoided.

a. Should the fillets be sold as fresh fish or quick frozen.

b. Assume the joint cost, which includes the cost of purchasing and processing the fish, is $7,000 per batch and the offal is treated as a by-product that is recognized at the split-off point, allocate the joint cost to each of the products using:

i. Physical output method

ii. Net realizable value method

iii. Constant gross margin

Page 358: MODULE 3 Financial Management and Management Accounting 1

Module 3 - Financial Management & Management Accounting 1

Page 358 © CMA Ontario, 2011

Problem 2

Jacques (see Chapter 5, Question 2) has been selling used barrels to customers for $25 when they have outlived their usefulness making swish and has decided he has a better use for his barrels. Jacques is now cutting up the barrels to produce wood, which he sells to barbecue pit enthusiasts to impart, what Jacques calls “a unique flavour to your barbecuing”. Jacques will use a price of $25 to transfer each barrel from his drinks business to his barrels business. Jacques pays his cousin Clementine $5 to disassemble each barrel.

The barrels render three joint products:

1. Staves that Jacques packages into bundles and sells to barbecue enthusiasts,

2. Solid white oak barrel tops and bottoms Jacques sands and sells to craft enthusiasts, and

3. Steel barrel hoops Jacques sells to the local scrap yard.

The following table summarizes the yield from the average barrel:

Product Top/Bottom Hoops Staves

Volume (weight) 5 1 40 Sales value at split-off $15 $.10 $20

a. Assuming the tops and bottoms and the staves are treated as joint products and the hoops as by-products use each of the following methods to allocate the joint cost of each barrel. Assume Jacques recognizes the by-products at the time production is completed:

i. Physical output method

ii. Sales value at split-off

iii. Constant gross margin

b. Clementine advises Jacques that her brother Philip has been buying staves, rendering them into charcoal and selling them as “Philip’s Fantastic BBQ Charcoal”. Upon investigation, Jacques discovers the batch of staves from each barrel could be rendered into charcoal for $6 a batch, packaged for $1 per batch and sold for $35. Should Jacques get into the charcoal business and, if so, recalculate the joint cost allocations assuming the allocation alternatives are:

i. Physical output method

ii. Net realizable value

iii. Constant gross margin

Page 359: MODULE 3 Financial Management and Management Accounting 1

Module 3 - Financial Management & Management Accounting 1

Page 359 © CMA Ontario, 2011

Problem 3

a. Return to the data in Question 2 part b). Suppose Fred, cut off from his supply of staves, retaliated by burning Jacques’ inventory of staves. Miraculously, Jacques has insured his inventory. The insurance policy covers the cost of the lost inventory. The quantity lost is the staves from 20 barrels. Assuming Jacques reports honestly, what value will he report as his loss?

b. Comment on the accounting implications of your result in part a).

Problem 4

Multi Product Company processes a raw material with a joint cost of $25,000 to produce joint Product A and joint Product B with a new realizable value of $15,000 and $25,000 respectively. The processing also produces a by-product, Product C, with a net realizable value of $4,000. All Product A and Product B have been sold this period. However, none of Product C has been sold. Consider each of the following questions separately:

a. The president of Multi Product Company is going to retire at the end of this year and would like to make his bonus, which is computed as a percentage of current profits, as high as possible. What treatment of Product C will the president prefer?

b. Suppose Multi Product Company has had a bad year overall. Profits are expected to be so low that the president will not earn a bonus. What treatment of Product C will the president prefer?

Problem 5

Return to Question 1 and suppose Downeast Fisheries can sell a maximum of 1,500 kilograms of fresh fish or quick frozen fish. How would this change your allocations in Question 1?

Page 360: MODULE 3 Financial Management and Management Accounting 1

Module 3 - Financial Management & Management Accounting 1

Page 360 © CMA Ontario, 2011

Solutions – Multiple Choice Questions 1. c Net Realizable Value A $ 72,000 B 50,000 C 28,000 $150,000 $60,000 x (72 ÷ 150) = $28,800 2. a Only C should not be processed further – improvement in income by not

processing C further = $30,000 - 28,000 = $2,000 3. d Allocating joint product costs is useful for pricing and production decisions;

however, the main purpose of allocating joint costs is to determine inventory cost and cost of goods sold for internal and external reporting

4. c $600,000 x [16,400 ÷ (16,400 + 10,000 + 8,400)] = $282,759 5. b Joint costs are allocated on the basis of the relative estimated net realizable

value (i.e. expected final sales value minus expected separable costs to complete and sell the total production). Total net realizable value of all products = ($160 - $100) x 20,000 units of L produced + ($300 - $200) x 10,000 units of M produced + ($400 - $350) x 7,000 units of N produced = $1,200,000 + $1,000,000 + $350,000 = $2,550,000 Inventory cost per unit of Product M = $200 + [$600,000 x ($1,000,000/$2,550,000) ÷ 10,000] = $200 separable costs + $23.53 joint costs = $223.53

6. a Sales value at split-off: Paper: 30,000 x $.04 = $1,200 Pencil casings: 30,000 x $.10 = 3,000 Joint costs allocated to paper = $1,500 x (1,200/4,200) = $428.57 Allocated to remaining inventory: $428.57 x 1,000/30,000 = $14.29 7. e Net realizable value of Product A: (1,375 x $25) - 150 = $34,225

Page 361: MODULE 3 Financial Management and Management Accounting 1

Module 3 - Financial Management & Management Accounting 1

Page 361 © CMA Ontario, 2011

Problem 1

a.

Fresh Fish Quick Frozen Unit Total Unit Total

Yield (kilograms) 2,000 2,000 Revenue (disposal cost) per kilogram $6.00 $12,000.00 $5.50 $11,000.00 Separable variable processing cost per kilogram

1.50 3,000.00 0.50 1,000.00

Variable contribution $4.50 $9,000.00 $5.00 $10,000.00 Separable fixed cost per batch 0.00 500.00

Total net realizable value $9,000.00 $9,500.00

Return is increased by $500 if fresh fish is sold as frozen.

b.

The following is a summary of the data that is useful in answering the following questions:

Quick Frozen Pieces Offal

Yield (kilograms) 2,000 1,000 7,000 Revenue (disposal cost) per kilogram $5.50 $3.00 -$0.20 Separable processing cost per kilogram 0.50 1.00 0.00

Variable contribution per kilogram $5.00 $2.00 -$0.20 Total variable contribution $10,000.00 $2,000.00 -$1,400.00 Less batch fixed cost 500.00 0.00 0.00

Net realizable value $9,500.00 $2,000.00 -$1,400.00 Joint cost 7,000.00

Total joint cost to be allocated $8,400.00

Total net realizable value $11,500.00 Total joint cost 8,400.00

Gross margin $3,100.00 Gross margin as a % of total net realizable value

26.96%

i.

Quick Frozen Pieces Total

Yield (kilograms) 2,000 1,000 3,000 Percent of total 66.67% 33.33%

Page 362: MODULE 3 Financial Management and Management Accounting 1

Module 3 - Financial Management & Management Accounting 1

Page 362 © CMA Ontario, 2011

Joint cost allocation $5,600.00 $2,800.00 $8,400.00

ii.

Quick Frozen Pieces Total

Yield (kilograms) 2,000 1,000 Revenue (disposal cost) per kilogram $5.50 $3.00 Separable processing cost per kilogram 0.50 1.00

Variable contribution per kilogram $5.00 $2.00 Total variable contribution $10,000.00 $2,000.00 Less: batch fixed cost 500.00 0.00

Net realizable value $9,500.00 $2,000.00 $11,500.00 % total 82.61% 17.39% Joint cost allocation $6,939.13 $1,460.87 $8,400.00

iii.

Quick Frozen Pieces

Total revenue $11,000.00 $3,000.00 $14,000.00 Less gross margin (@22.14% of revenue) 2,435.71 664.29 3,100.00

Total cost $8,564.29 $2,335.71 Less: separable variable cost (2000 * 0.50)

1,000.00 1,000.00

Less: separable fixed cost 500 0

Equals joint cost allocation $7,064.29 $1,335.71 $8,400.00

Total batch separable contribution $11,500.00 Total joint cost 8,400.00

Total batch contribution $3,100.00 Gross margin % 22.14%

Page 363: MODULE 3 Financial Management and Management Accounting 1

Module 3 - Financial Management & Management Accounting 1

Page 363 © CMA Ontario, 2011

Problem 2

a.

i.

Transfer in cost $25.00 Processing cost 5.00 Less: hoop Sales 0.10

Joint cost $29.90

Product Units % of Total Joint Cost Allocation

Formula

Top/bottom 5 11.11% $3.32 11.11% * $29.90 Staves 40 88.89% 26.58 88.89% * $29.90

Total 45 100.00% $29.90

ii.

Acquisition Cost $25.00

Processing Cost 5.00 Less: hoop sales 0.10

Joint cost $29.90

iii.

Batch revenue $35.00 Acquisition cost $25.00 Processing cost 5.00 Less: hoop sales 0.10 29.90

Gross margin $5.10 Gross margin % 14.57%

Product Sales Value

Less Gross

Margin

Joint Cost Allocation

Top/bottom $15.00 $2.19 $12.81 Staves 20.00 2.91 17.09

Total $35.00 $29.90

Page 364: MODULE 3 Financial Management and Management Accounting 1

Module 3 - Financial Management & Management Accounting 1

Page 364 © CMA Ontario, 2011

b. The value of the staves at split-off is $20 the incremental value of processing further is $35 - $6 - $1 - $20 or $8. Therefore, it is to Jacques advantage to process further and the batch revenue will be $43 comprised of $28 for the staves ($35-6-1) and $15 for the top and bottom.

i. Transfer in cost $25.00 Processing cost 5.00 Less: hoop sales 0.10 Joint cost to be allocated $29.90

Units % of Total

Joint cost Allocation

Top/bottom 5 11.11% $ 3.32 Charcoal (staves) 40 88.89% 26.58 45 $29.90 ii

NRV

% of Total

Joint cost Allocation

Top/bottom $15 34.88% $10.43 Charcoal (staves) 28 65.12% 19.47 $43 $29.90

iii.

Batch revenue $50.00 Acquisition cost $25.00 Processing cost 5.00 Rendering cost 6.00 Packaging cost 1.00 Less: hoop sales 0.10 36.90

Separable cost 7.00 Gross margin $13.10 Gross margin % 26.20%

Product Sales Value

Less Gross Margin

Total Cost Less

Separable Cost

Joint Cost Allocation

Top/bottom $15.00 $3.93 $11.07 $0.00 $11.07 Staves 35.00 9.17 25.83 7.00 18.83

Total $50.00 $36.90 $29.90

Page 365: MODULE 3 Financial Management and Management Accounting 1

Module 3 - Financial Management & Management Accounting 1

Page 365 © CMA Ontario, 2011

Problem 3

a. The cost assigned to the staves inventory is highest under the physical output method and equals $26.58 for the staves from each barrel. Therefore, Jacques would choose the physical output method to allocate the joint costs and would report a loss of $531.60 ($26.58 * 20).

b. The variation in cost allocations under the three different approaches raises the possibility of accounting manipulation. Note that Jacques is not committing any fraud in choosing the accounting method that best supports his goal of maximizing reimbursement from the insurance company, this is pure accounting system manipulation.

Problem 4

a. The president would prefer the by-product value be recognized at the time of production since this will reduce costs by $4,000 for this year.

b. The president would prefer the by-product value be recognized at the time of sale. All the joint costs of the batch will be recognized in the current year when the president is not going to earn a bonus anyway. The $4,000 of by-product will be available to increase sales in the next year.

Problem 5

The following is a summary of the data that is useful in answering the following questions

Quick Frozen Fresh Pieces

Yield (kilograms) 1,500 500 1,000 Revenue (disposal cost) per kilogram $5.50 $6.00 $3.00 Separable processing cost per kilogram 0.50 1.50 1.00

Variable contribution per kilogram $5.00 $4.50 $2.00 Total variable contribution $7,500.00 $2,250.00 $2,000.00 Less: batch fixed cost 500.00 0.00 0.00

Net realizable value $7,000.00 $2,250.00 $2,000.00 Joint cost Total joint cost to be allocated

Total net realizable value $11,250.00 Total joint cost 8,400.00

Gross margin $2,850.00 Gross margin % 25.33%

Page 366: MODULE 3 Financial Management and Management Accounting 1

Module 3 - Financial Management & Management Accounting 1

Page 366 © CMA Ontario, 2011

i.

Quick Frozen Fresh Pieces Total

Yield (kilograms) 1,500 500 1,000 3,000

Percent of total 50.00% 16.67% 33.33%

Joint cost allocation $4,200.00 $1,400.00 $2,800.00 $8,400.00

ii.

Quick Frozen Fresh Pieces Total

Yield (kilograms) 1,500 500 1,000 Revenue (disposal cost) per kilogram $5.50 $6.00 $3.00 Separable processing cost per kilogram 0.50 1.50 1.00

Variable contribution per kilogram $5.00 $4.50 $2.00 Total variable contribution $7,500.00 $2,250.00 $2,000.00 Less: batch fixed cost 500.00 0.00 0.00

Net realizable value $7,000.00 $2,250.00 $2,000.00 $11,250.00 % total 62.22% 20.00% 17.78% Joint cost allocation $5,226.67 $1,680.00 $1,493.33 $8,400.00

iii.

Quick Frozen Fresh Pieces Total revenue $8,250 $3,000 $3,000 Less gross margin 1,650 600 600 Total cost $6,600 $2,400 $2,400 Less: separable variable cost 750 750 1,000 Less: separable fixed cost 500 0 0 Equals joint cost allocation $5,350 $1,650 $1,400 Total batch separable contribution $11,250 Total joint cost 8,400 Total batch contribution 2850 Gross margin % 20%

Page 367: MODULE 3 Financial Management and Management Accounting 1

Module 3 - Financial Management & Management Accounting 1

Page 367 © CMA Ontario, 2011

CVP Analysis in the News

The automaker (Chrysler) can break even by selling one million cars annually in the US and make $1 billion for every additional 100,000 vehicles, two people familiar with its finances have said.

Source: http://www.bloomberg.com/apps/news?pid=20601087&sid=alTdWHyet7II&pos=1

9. Cost-Volume-Profit Analysis

Learning Objectives

After completing this chapter, you will:

1. Understand the role of cost volume profit analysis

2. Be able to apply cost volume profit analysis in both single product and multi-product settings

3. Be able to adapt and use cost volume profit analysis to evaluate the profit effect of management initiatives such as advertising and re-engineering

4. Understand and be able to adapt cost volume profit tools to more complex, spreadsheet-based models

The Cost Volume Profit Model

The basic cost volume profit (CVP) model makes the following important assumptions:

1. The volume of sales does not affect product price.

2. The volume of product does not affect variable cost per unit.

3. All costs are either variable or fixed.

4. All production is sold.

With these assumptions, we can use the profit equation to develop the CVP relationship.

Let:

• P = price per unit • V = variable cost per unit • F = total fixed cost • x = number of units produced and sold • OI = operating income

Revenues - costs = operating income

Px - Vx - F = OI

Page 368: MODULE 3 Financial Management and Management Accounting 1

Module 3 - Financial Management & Management Accounting 1

Page 368 © CMA Ontario, 2011

x (P - V) - F = OI

Accountants call price per unit minus variable cost per unit (P – V) the contribution margin per unit. The contribution margin per unit is the dollar amount that each unit made and sold contributes to covering fixed costs and provides a profit. We can now write the above equation as:

xCM - F = OI

x = (fixed costs + operating income) / contribution margin per unit

For convenience, we will refer to this equation as the units CVP equation. What the units CVP equation does is it allows the decision maker to identify the number of units (x) that must be made and sold to cover fixed costs and provide a target profit. Or, put another way, it predicts the profit (or loss) resulting from a given level of unit sales.

Using the Units Cost Volume Profit Model

The following is an example illustrating the use of the units CVP model. Russell Company sells a product that has a price of $4 per unit, variable manufacturing costs of $2 per unit and selling costs of $.50 per unit. Russell Company has fixed manufacturing costs of $125,000 and fixed general, selling and administrative expenses of $25,000. Assume management has two questions:

1. How many units does Russell Company have to sell to breakeven?

2. How many units does Russell Company have to sell to earn a pre-tax profit of $100,000?

The total variable cost of the company’s product is $2.50 ($2 + $.50). Therefore, the contribution margin per unit is $1.50 ($4 - $2.50). The company’s total fixed cost is $150,000.

Breakeven means zero profit, so putting this data in the units CVP equation we have:

x = 150,000 / 1.50 = 100,000 units

The following equation computes the unit sales needed to generate a target profit of $100,000

X = (100,000 + 150,000) / 1.50 = 166,667 units

We will follow the convention in these notes that when a target quantity is computed, it will be rounded up, when necessary, to calculate the solution.

A measure that management accountants often find useful as a means of expressing financial risk is the margin of safety measure. The margin of safety is computed as follows:

Margin of safety = expected units - breakeven units

Page 369: MODULE 3 Financial Management and Management Accounting 1

Module 3 - Financial Management & Management Accounting 1

Page 369 © CMA Ontario, 2011

Or as the margin of safety percentage:

Margin of safety percentage = (expected units - breakeven units) / expected units

Assume in the above example management believes the company will sell 150,000 units, the margin of safety is 150,000 - 100,000 = 50,000 units. The margin of safety percentage is 33% [(150,000 - 100,000) / 150,000]. The margin of safety percentage measure says that sales would have to drop at least 33% from expected levels before losses would occur.

Developing and Using the Revenue Cost Volume Profit Model

Often decision makers prefer that the required levels to achieve breakeven or a target profit be stated in revenue terms. We can easily adapt the units CVP model to adopt the revenue perspective.

Return to the units CVP equation. Note that we can convert this equation to a revenue equation by multiplying both sides of the equation by price per unit. We now have:

Revenue = Px = P(operating income + fixed costs) CM per unit Dividing the top and the bottom of the right hand side of the equation, we have:

Revenue = (operating income + fixed costs) / (CM/unit / P)

Management accountants call the ratio of contribution margin per unit divided by price per unit the contribution margin ratio. The contribution margin ratio reflects the proportion of each sales dollar that goes toward covering fixed costs and providing a profit.

So, we have the following equation, which we will call the revenue CVP equation:

Revenue = (operating income + fixed costs) / CM ratio

Returning to the Russell Company example, we can compute the product’s contribution margin ratio as follows:

$1.50 / 4 = 37.5%

We can now answer the following questions expressed in revenue terms for the above product:

1. What sales revenue must Russell Company achieve to breakeven?

2. What sales revenue must Russell Company achieve to earn a profit of $100,000?

Question one can be answered with the following calculation:

$150,000 / .375 = $400,000

Page 370: MODULE 3 Financial Management and Management Accounting 1

Module 3 - Financial Management & Management Accounting 1

Page 370 © CMA Ontario, 2011

Question two can be answered with the following calculation:

($150,000 + 100,000) / .375 = $666,667

The Cost Volume Profit Chart

Often, management accountants find it useful to express the CVP relationship in a CVP chart. The CVP chart for the product described above would look as follows:

The advantage of the CVP chart is it provides a visual representation of units, revenues, costs and profits, which often help communicate these relationships more effectively.

Taxes and the CVP Equations

Returning to the basic profit equation, we have:

After tax profit = (revenue – variable cost – fixed cost) * (1 – tax rate)

or, using the notation, we developed above:

After tax profit = ((P – V)x – f) * (1 – tax rate))

rearranging, and recalling that (p – v) is the unit contribution margin, we get the following unit CFP equation adapted for taxes:

x = [(after tax profit / 1 - t)] + fixed costs / CM per unit

and following the same procedure we used above, we can develop the following sales CVP equation:

Page 371: MODULE 3 Financial Management and Management Accounting 1

Module 3 - Financial Management & Management Accounting 1

Page 371 © CMA Ontario, 2011

Revenue = [(after tax profit / 1 - t)] + fixed costs / CM ratio

Returning to Russell Company, assume the company faces a tax rate of 30%, how many units must Russell Company sell to earn a post-tax profit of $100,000?

We have, in units:

[(100,000 / .7) + 150,000] / 1.5 = 195,339 units

or, in revenue:

[(100,000 / .7) + 150,000] / .375 = $780,953

Exploiting the Profit Equation

The profit equation provides, in effect, a financial model of the organization. Manipulating the profit equation allows management to undertake analyses to predict the effect of proposed decisions, a process called what-if analysis.

Assume, given the data noted above, Russell Company management expects unit sales of 160,000 for the upcoming period. The marketing manager believes that a 5% price decrease and a $25,000 advertising budget will increase sales to 175,000 units. Are these changes desirable?

We use incremental analysis to calculate the incremental income of the proposed change as follows:

Total contribution margin Before: 160,000 units x $1.50 $240,000 After: 175,000 x $1.30* 227,500 Incremental contribution margin 12,500 Advertising costs 25,000

Incremental income ($12,500)

* Sales price decrease = decrease in CM/Unit = $4 x 5% = $.20

New CM/unit = $1.50 - .20 = $1.30 Modelling Uncertainty

Even with limited computational power, decision makers were able to incorporate probabilistic concepts into CVP analysis. Again, the applications required considerable simplification.

For example, return to the original Russell Company example and assume demand in the forthcoming period is uncertain and has been estimated at between 80,000 and 180,000, with all units on this interval equally likely. In effect, sales have been estimated as following a uniform distribution.

Page 372: MODULE 3 Financial Management and Management Accounting 1

Module 3 - Financial Management & Management Accounting 1

Page 372 © CMA Ontario, 2011

Recall that breakeven sales were computed as 100,000 units. The probability of at least breaking even given this uniform distribution is computed as follows:

Probability = upper limit - breakeven quantity = 180,000 - 100,000 = 80% upper limit - lower limit 180,000 - 80,000 As an exercise you should verify that the probability that Russell Company will earn $100,000 or more is 13.3%. Similar statements could be developed for any other standard distribution whose properties are well known such as the normal distribution.

Multi-Product

Most of what we have discussed in this chapter was well known to, and practiced by, both managers and accountants in the early 1900s. In fact, there is evidence to suggest that CVP analysis was originally developed by managers and not accountants. This approach was, therefore, developed at a time when there were no calculators, let alone computers. For this reason assumptions needed to be made to make the analysis tractable. This is why assumptions like constant prices, variable costs and fixed costs were so important.

So, when decision makers decided they wanted to adapt the CVP models described above to multi-product organizations, they needed to make another assumption to make the CVP model work. The assumption made was as total sales levels increased or decreased, each product’s proportion of total sales would remain constant.

An example best illustrates the approach and the insights of multi-product CVP analysis.

Brant Consulting design costing and transfer pricing systems for its clients. The following is a summary of volume, revenue and cost expectations for the upcoming year:

Costing Projects Transfer Pricing Projects

Item Number 50 Number 75 Firm Total Per Job Total Per Job Total

Revenue $10,000 $500,000 $15,000 $1,125,000 $1,650,000 Variable cost 4,500 225,000 11,500 862,500 1,103,500

Contribution margin

$5,500 $275,000 $3,500 $262,500 $537,500

Fixed cost 475,000

Profit $62,500

The solution to the problem comprises of combining units in a 'bundle' based on their sales mix. In this case, the sales mix is 50 costing projects (CP) and 75 transfer pricing projects (TPP). At its lowest common denominator, we can say the sales mix is 2:3 (CP:TPP).

Page 373: MODULE 3 Financial Management and Management Accounting 1

Module 3 - Financial Management & Management Accounting 1

Page 373 © CMA Ontario, 2011

We then create a product bundle containing two CP and three TPP and calculate the contribution margin generated by that bundle: (2 x $5,500) + (3 x $3,500) = 21,500 The number of bundles needed to be sold to breakeven are: $475,000 / 21,500 = 22.1 We then break out the bundles into the individual products: Costing projects: 22.1 bundles x 2 = 45 Transfer pricing projects: 22.1 bundles x 3 = 67 The unit sales Brant Consulting needs to achieve an operating income of $60,000 is: ($475,000 + 60,000) / 21,500 = 24.88 Breaking out the bundles into the individual products: Costing projects: 24.88 bundles x 2 = 50 Transfer pricing projects: 24.88 bundles x 3 = 75

Spreadsheets

Today, with powerful computers and electronic spreadsheets, business modelling is done on computers. However, the basic modelling insights developed in CVP analysis underlies the financial models implemented on computers.

Page 374: MODULE 3 Financial Management and Management Accounting 1

Module 3 - Financial Management & Management Accounting 1

Page 374 © CMA Ontario, 2011

Problems with Solutions

Multiple Choice Questions

1. Spencer Company expects to sell 60,000 units of Product B next year. Variable

production costs are $4 per unit and variable selling costs are 10% of the selling price. Fixed costs are $115,000 per year and the company desires an after-tax profit of $30,000 next year. The company's tax rate is 40%. Based on this information, the unit selling price next year should be:

a) $7.00 b) $10.75 c) $7.50 d) $6.75 e) None of the above 2. Operating income is shown on a cost-volume-profit chart where the: a) Total variable cost line exceeds the total fixed cost line b) Total cost line exceeds the total sales revenue line c) Total sales revenue line exceeds the total fixed cost line d) Total sales revenue line exceeds the total cost line e) Total cost line intersects the total sales revenue line 3. The following information relates to a new product an organization plans to

introduce: Selling price $80 per unit Variable selling cost $5 per unit Direct materials $25 per unit Direct labour $10 per unit Variable overhead $20 per unit Fixed overhead $140,000 per year Fixed selling expense $60,000 per year How many units of this product must be sold each year to breakeven? a) 2,500 b) 10,000 c) 7,000 d) 8,000 e) 4,444

Page 375: MODULE 3 Financial Management and Management Accounting 1

Module 3 - Financial Management & Management Accounting 1

Page 375 © CMA Ontario, 2011

4. Which of the following statements is consistent with the conventional cost-volume-

profit model? a) All costs can be divided into fixed and variable elements b) Costs and sales revenues are linear over a relevant range c) Variable costs remain constant on a unit basis d) All of the above e) b) and c) only 5. Monnex Corp. sells three designs of all-weather vehicle tires: Radial, All Terrain

and Super Pro. The following represents the sales and cost information budgeted for 1997:

Radial All Terrain Super Pro Sales price per unit $50 $100 $200 Costs per unit Direct materials 25 50 70 Direct labour 15 15 15 Variable overhead 10 10 10 Fixed overhead* 5 5 5 55 80 100 Gross margin per unit -$5 $20 $100

* Fixed overhead per unit is based on the 1996 sales of 5,000 Radial tires,

20,000 A11 Terrain tires and 10,000 Super Pro tires. Fixed administration costs total $150,000 in the 1997 budget. Assuming the 1996

sales mix continues, what is the breakeven volume of Radial tires? a) Monnex Corp. cannot breakeven b) 6,500 units c) 351 units d) 7,338 units e) 1,049 units

Page 376: MODULE 3 Financial Management and Management Accounting 1

Module 3 - Financial Management & Management Accounting 1

Page 376 © CMA Ontario, 2011

The following information pertains to items 6 – 9: Tic Toc Ltd. produces two types of clocks: a digital model and an analog model.

Budgeted sales for next year are:

Digital Analog Total Sales volume 8,000 units 12,000 units 20,000 units Sales revenue $180,000 $140,000 $320,000 Total variable manufacturing costs, which are joint costs, are estimated to amount

to $160,000 next year and variable selling costs are estimated to amount to 5% of sales. Budgeted fixed costs for next year are $80,000 for manufacturing overhead and $24,000 for selling and administration. All manufacturing costs are allocated to the two models on the basis of sales revenue. The company's effective tax rate is 30%.

6. The budgeted contribution margin percentage of sales is: a) 45% for both models b) 59.4% for the digital model and 26.4% for the analog model c) 25% for both models d) 50% for both models e) 12.5% for both models 7. Assuming the budgeted contribution margin percentage is 40% of sales for both

models, the desired total sales required to be raised by Tic Toc Ltd. to earn an after-tax income of $70,000 is:

a) $354,286 b) $453,333 c) $487,500 d) $510,000 e) $582,857 8. Assuming the budgeted dollar sales mix is maintained during the year and the

contribution margin percentage of sales is 30% for the digital model and 50% for the analog model, how many units of each model must Tic Toc Ltd. sell during the year to make a contribution margin of $164,000?

a) Digital model – 15,449 units; Analog model – 10,332 units b) Digital model – 13,667 units; Analog model – 12,300 units c) Digital model – 10,581 units; Analog model – 15,871 units d) Digital model – 9,762 units; Analog model – 14,643 units e) Digital model – 9,719 units; Analog model – 16,869 units

Page 377: MODULE 3 Financial Management and Management Accounting 1

Module 3 - Financial Management & Management Accounting 1

Page 377 © CMA Ontario, 2011

9. Assume the budgeted dollar sales mix and the budgeted sales volume for each

model is maintained for next year. Also, assume the budgeted contribution margin is 40% of sales for both models and total fixed costs amount to $105,000 for next year. What is the minimum unit price for each model that should be set to earn a 7% after-tax return on sales next year?

a) Digital model – $17.50/unit; Analog model – $17.50/unit b) Digital model – $18.46/unit; Analog model – $9.57/unit c) Digital model – $20.51/unit; Analog model – $10.64/unit d) Digital model – $22.37/unit; Analog model – $11.60/unit e) Digital model – $24.61/unit; Analog model – $12.76/unit 10. When you undertake long-range cost-volume-profit analysis, step cost functions

with multiple steps may be treated as variable costs if: a) The range over which production volumes vary is wide b) The range over which production volumes vary is narrow c) The operating leverage is high d) The relevant range is fixed e) None of the above

Page 378: MODULE 3 Financial Management and Management Accounting 1

Module 3 - Financial Management & Management Accounting 1

Page 378 © CMA Ontario, 2011

The following information pertains to Questions 11 – 13: Joie Inc. produces Product X. Each unit of the product requires 0.2 hours of direct labour, two kilograms of Material A and one kilogram of Material B. The company has a production capacity of 30,000 units of Product X per year, but its current production and sales are 25,000 units per year. For the current year, costs and revenues are: Price per unit of Product X $13.50 Direct labour cost per hour $15.00 Material A cost per kilogram $.80 Material B cost per kilogram $2.40 Fixed factory overhead $50,000 Variable selling and administration costs $12,500 All other fixed expenses $37,500 11. At the current level of production, the contribution margin per unit of Product X is: a) $6.50 b) $4.50 c) $6.80 d) $4.00 e) $6.00 12. At the current level of production, the gross margin per unit of Product X is: a) $6.00 b) $4.50 c) $4.83 d) $3.00 e) $4.00 13. Assume that variable production costs for next year will be $8 per unit of Product X

and that all other costs will be the same as for the current year. If the selling price remains at $13.50 per unit, the breakeven volume for next year would be

a) 17,500. b) 10,000. c) 18,182. d) 15,909. e) 10,294.

Page 379: MODULE 3 Financial Management and Management Accounting 1

Module 3 - Financial Management & Management Accounting 1

Page 379 © CMA Ontario, 2011

14. Which of the following statements is true with regard to the profit-volume chart, where profit represents the vertical axis and sales volume represents the horizontal axis?

a) The slope of the profit line is affected by the product’s total fixed costs. b) The slope of the profit line is not affected by the product’s selling price. c) The slope of the profit line remains unchanged as the variable cost per unit

decreases, assuming the selling price and total fixed costs remain unaffected. d) The slope of the profit line is the product’s contribution margin per unit. e) None of the above.

Page 380: MODULE 3 Financial Management and Management Accounting 1

Module 3 - Financial Management & Management Accounting 1

Page 380 © CMA Ontario, 2011

Problem 1

Return to the CVP analysis in the news text box at the beginning of this chapter. Draw the CVP chart for Chrysler using the information provided for car sales between 700,000 and 1,200,000 units per year.

Problem 2

Russell Company manufactures a plastic novelty item. The revenue per unit is $2.50 and the variable manufacturing cost is $.75 per unit. Variable selling and distribution costs are $.05 per unit. The fixed manufacturing and selling and administrative costs for this product amount to $250,000 and $90,000, respectively.

Required: (consider each question separately)

a. What is the breakeven quantity in units?

b. What revenue is required to earn a target pre-tax profit of $200,000?

c. If the company tax rate is 30%, what is the required revenue to earn a target post-tax profit of $100,000?

d. At a sales level of $700,000, what is the degree of operating leverage?

e. If management estimates the margin of safety percentage as 20%, what is its expected level of sales in units?

f. Russell Company has signed a contract to deliver 210,000 units of this product at a price of $2.50 per unit. Total incremental fixed costs associated with this contract will be $370,000. Variable selling and distribution costs will be zero. However, variable manufacturing costs can vary between $.70 and $.85 per unit with all values on this interval equally likely. What is the probability of at least breaking even on this contract?

Problem 3

Burford Ball Company (BBC) manufactures soccer balls, which sell for $30 apiece and have direct materials cost of $12 each. The BBC production manager is considering acquiring a new computer-controlled sewing machine that sews together the 32 panels comprising the ball. The current cost per ball to sew the panels together is $5.50 and reflects the wage paid to a seamstress operating a manually controlled machine. The computer-controlled machine would reduce the cost to $4 per ball by reducing the required operator time.

The manufacturer of the new machine has offered to rent the machine to BBC for $75,000 per year.

a. What volume of annual production is required to make this machine attractive to BBC?

Page 381: MODULE 3 Financial Management and Management Accounting 1

Module 3 - Financial Management & Management Accounting 1

Page 381 © CMA Ontario, 2011

b. If BBC believes annual sales are uniformly distributed on the interval between 40,000 and 70,000 units, what is the probability that acquiring this machine will increase profits at BBC?

Problem 4

Drumbo Quality Safes (DQS) manufactures and sells three models of safes: home, office and banker.

The following is a summary of the three products and DQS’ fixed costs:

Home Office Bank DQS Total

Per unit Total Per unit Total Per unit Total

Units sold 8,000 4,000 3,000

Price $600 $4,800,000 $1,800 $7,200,000 $5,400 $16,200,000 $28,200,000

Variable cost 350 2,800,000 1,250 5,000,000 4,000 12,000,000 19,800,000

Contribution margin $250 $2,000,000 $550 $2,200,000 $1,400 $4,200,000 $8,400,000

Fixed manufacturing 4,500,000

Fixed selling and administrative

3,000,000

Income $900,000

Assume the sales mix remains constant in answering the following problems.

a. What is the breakeven level of sales?

b. What level of unit sales is required to earn a target income that is 20% of sales?

Problem 5

Refer to the data in Problem 4. The marketing manager wants to increase the sales of the bank product. The marketing manager believes a price cut of 10% accompanied by an advertising campaign of $100,000 to support the bank product will increase its sales. What is the minimum unit sales increase of the bank product that would be required to make this proposal financially attractive? Problem 6

Return to the data in Problem 4. The purchasing agent has announced that steel prices will increase in the current year. The effect will be to increase all variable costs by 12%.

a. If the current sales mix and quantities remain the same, what must DQS charge for each of its products to achieve the $900,000 originally planned?

b. Explain why the required price increase for each product is not 12%

Page 382: MODULE 3 Financial Management and Management Accounting 1

Module 3 - Financial Management & Management Accounting 1

Page 382 © CMA Ontario, 2011

Problem 7

The Scotland Jammers hockey club is considering offering Bud Kelly a hockey contract. As a star player, Kelly expects to be well paid. The general manager at Scotland Jammers believes the team will have to pay Kelly a bonus of $2,000,000 when he signs his contract and an annual salary of $5,000,000 per year. Kelly is demanding a five year contract.

The team plays 35 games a year. On average, the Jammers receive $75 per ticket for home games and sell $20 of food and merchandise to each customer. The variable cost per customer averages approximately $40.

Current ticket sales average about 15,000 per game and signing Kelly is expected to increase average ticket sales to 17,000 per game, which is the arena’s capacity.

Required:

a. What would this contract’s effect be on total five year income ignoring the time value of money?

b. The general manager believes that simply focusing on regular season games understates Kelly’s value to the club. On average, the Jammers play six home playoff games each year, which are all sold out. However, the general manager believes the number will increase to eight playoff games per year. Given this information, what is this contract’s effect on income?

Problem 8

Ayr Manufacturing pays taxes of 20% on all pre-tax income under $200,000. For pre-tax income above $200,000, the tax rate is 32%. If Ayr Manufacturing’s contribution margin ratio is 35% and its fixed costs are $300,000, what level of sales must it achieve to earn a target post-tax income of $300,000?

Page 383: MODULE 3 Financial Management and Management Accounting 1

Module 3 - Financial Management & Management Accounting 1

Page 383 © CMA Ontario, 2011

Problem 9 Excellent Text Book Company produces an accounting text that is used by many universities and colleges. The company sells the book to bookstores at a price of $43.50 each. The costs of manufacturing and marketing the text at the company’s normal volume of 3,000 units per month are:

Unit manufacturing costs: Variable materials $5.50 Variable labour 8.25 Variable overhead 4.20 Fixed overhead 6.60

Total unit manufacturing costs $24.55 Unit marketing cost: Variable 2.75 Fixed 7.70

Total unit marketing costs 10.45 Total unit costs $35.00

Required: a. What is the breakeven volume in units? In sales dollars? b. Market research indicates that monthly volume could increase to 3,500 units,

which is well within production capacity limitations if the price were cut from $43.50 to $38.50 per unit. Would you recommend this action be taken? Support your response by showing your calculations.

Page 384: MODULE 3 Financial Management and Management Accounting 1

Module 3 - Financial Management & Management Accounting 1

Page 384 © CMA Ontario, 2011

Problem 10 The estimates made for Nixon Company, a one-product company, are:

Nixon Company Projected Income Statement

for the year ended December 31, 1995

Sales revenue (100 units x $100 per unit) $10,000 Manufacturing cost of goods sold: Direct materials $1,400 Direct labour 1,500 Variable overhead 1,000 Fixed overhead 500 4,400 Gross margin 5,600 Selling and administrative expenses: Variable 1,100 Fixed 2,000 3,100 Operating income $ 2,500

Required: a. How many units of the product must Nixon sell to breakeven? b. What would be the operating income if projected units increased by 25%? c. What would dollar sales be at the breakeven point if fixed overhead increased by

$1,700.

Page 385: MODULE 3 Financial Management and Management Accounting 1

Module 3 - Financial Management & Management Accounting 1

Page 385 © CMA Ontario, 2011

Problem 11 Mistry Company manufactures a line of electric garden tools that are sold in general hardware stores. The company's controller, Sylvia Harlow, has just received the sales forecast for the coming year for Mistry's three products: weeders, hedge clippers and leaf blowers. Mistry has experienced considerable variations in sales volumes and variable costs over the past two years and Harlow believes the forecast should be carefully evaluated from a CVP viewpoint. The preliminary budget information for 2008 is: Hedge Leaf Weeders Clippers Blowers Unit sales 50,000 50,000 100,000 Unit selling price $28 $36 $48 Variable manufacturing cost per unit $13 $12 $25 Variable selling cost per unit $5 $4 $6 For 2008, Mistry's fixed factory overhead is budgeted at $2,000,000 and the company's fixed selling and administrative expenses are forecasted to be $600,000. Mistry has an effective tax rate of 40%. Required: a. Determine Mistry Company's budgeted net income for 2008. b. Assuming the sales mix remains as budgeted, determine how many units of each

product Mistry Company must sell in order to breakeven in 2008. c. Determine the total dollar sales Mistry Company must sell in 2008 in order to

earn an after-tax net income of $450,000. d. After preparing the original estimates, Mistry Company determined its variable

manufacturing cost of leaf blowers would increase 20% and the variable selling cost of hedge clippers could be expected to increase $1 per unit. However, Mistry has decided not to change the selling price of either product. In addition, Mistry has learned its leaf blower has been perceived as the best value on the market and it can expect to sell three times as many leaf blowers as any other product. Under these circumstances, determine how many units of each product Mistry Company would have to sell in order to breakeven in 2008.

Page 386: MODULE 3 Financial Management and Management Accounting 1

Module 3 - Financial Management & Management Accounting 1

Page 386 © CMA Ontario, 2011

Solutions: Multiple Choice Questions

1. c Let X = unit selling price

60,000X - 60,000(4) - 60,000(X)(.1) - 115,000 = 30,000 ÷ .6 54,000X = 405,000 X = $7.50

2. d A cost-volume-profit chart contains elements (lines, points, axes) that identify

variable cost, fixed cost, the breakeven point, total revenue, profit and volume in units. When the total sales revenue line rises above the total cost line, a company will have positive operating income.

3. b Breakeven = fixed cost / (unit price - variable cost per unit)

= $200,000 / ($80-$60) = 10,000 units. 4. d Radial All Terrain Super Pro 5. e Sales price per unit $50 $100 $200 Less: variable costs Direct materials 25 50 70 Direct labour 15 15 15 Variable overhead 10 10 10 Contribution margin/unit 0 $25 $105 Sales mix 1 4 2 CM x sales mix $0 $100 $210 Bundle CM $310 Total fixed overhead = (35,000 x $5) + $150,000 = $325,000 Breakeven bundles = $325,000 ÷ 310 = 1,048.39 Number of radial tires = 1,048 x 1 = 1,049 6. a Total contribution margin = ($320,000 x .95) - $160,000 = $144,000

Contribution margin percentage = $144,000 / $320,000 = 45%. The variable costs are allocated to the products based on sales revenue; therefore, both models have a contribution margin percentage of 45%.

7. d Total sales = [($70,000 / .7) + $80,000 + $24,000] ÷ 40% = $510,000.

Page 387: MODULE 3 Financial Management and Management Accounting 1

Module 3 - Financial Management & Management Accounting 1

Page 387 © CMA Ontario, 2011

8. c Dollar sales mix = $180,000 ÷ $320,000 = .5625 for the digital model and $140,000 ÷ $320,000 = .4375 for the analog model. $164,000 CM = (.5625 total sales x .3) + (.4375 total sales x .5) Total sales dollars = $423,226 Digital sales = ($423,226 x .5625) ÷ ($180,000 ÷ 8,000) = 10,581 units. Analog sales = ($423,226 x .4375) ÷ ($140,000 ÷ 12,000) = 15,871 units.

9. e Before-tax return on sales = 7% .7 = 10%.

10% of total sales = 40% of total sales - $105,000. Therefore, total sales = $350,000. Digital model price = $350,000 x .5625 ÷ 8,000 units = $24.61 per unit. Analog model price = $350,000 x .4375 ÷ 12,000 units = $12.76 per unit.

10. a The wider the range of production volumes, the more “steps” of activity and

costs are experienced. Therefore, the wider the range in production volumes, the more variable the costs.

11. e Selling price $13.50

Direct labour ($15 x .2) $3.00 Material A ($.80 x 2) 1.60 Material B 2.40 Variable selling and administration costs ($12,500 / 25,000) .50 7.50 Contribution margin per unit 6.00

12. b Selling price $13.50

Direct labour ($15 x .2) $3.00 Material A ($.80 x 2) 1.60 Material B 2.40 Fixed factory overhead ($50,000 / 25,000) 2.00 9.00 Gross margin per unit $4.50

Page 388: MODULE 3 Financial Management and Management Accounting 1

Module 3 - Financial Management & Management Accounting 1

Page 388 © CMA Ontario, 2011

13. a Fixed costs = $50,000 + $37,500 = $87,500 Contribution margin = $13.50 - $8 variable production costs - $.50 variable selling and administration costs = $5 per unit Breakeven volume = $87,500 ÷ $5 = 17,500 units Other choices: b) $50,000 ÷ $5 = 10,000 (did not include other fixed expenses) c) ($50,000 + $12,500 + $37,500) ÷ ($13.50 - $8) = $100,000 ÷ $5.50 = 18,182 (treated variable selling and administration as a fixed cost) d) $87,500 ÷ ($13.50 - $8) = $87,500 ÷ $5.50 = 15,909 (omitted variable selling and administration costs) e) $87,500 ÷ ($8 + $.50) = $87,500 ÷ $8.50 = 10,294 (used variable costs instead of contribution margin)

14. d In a profit-volume chart, profit = (selling price - variable cost per unit) x sales

volume - fixed costs. The slope of the profit-volume line represents the unit contribution margin (statement d). The intersection of this line and the horizontal axis is the breakeven sales volume. Changes to the fixed costs do not affect the slope of the line; therefore, statement a) is false. However, a change in selling price or in the variable cost per unit would change the unit contribution margin, so they would definitely affect the slope of the line. Therefore, statements b) and c) are false.

Page 389: MODULE 3 Financial Management and Management Accounting 1

Module 3 - Financial Management & Management Accounting 1

Page 389 © CMA Ontario, 2011

Problem 1 The quote implies the contribution per unit is $10,000 ($1,000,000,000 / 100,000). With that information and knowledge that breakeven is 1,000,000 units, we can estimate fixed costs as: Profit = contribution margin * units sold – fixed costs 0 = 1,000,000 * 10,000 – fixed costs Fixed costs = $10,000,000,000 The CVP chart is

Page 390: MODULE 3 Financial Management and Management Accounting 1

Module 3 - Financial Management & Management Accounting 1

Page 390 © CMA Ontario, 2011

Problem 2 a) CM/unit = $2.50 - .75 - .05 = $1.70 Fixed costs = $250,000 + 90,000 = $340,000 Breakeven quantity = $340,000 / 1.70 = 200,000 units b) CM ratio = 1.70 / 2.50 = .68 Revenue = ($340,000 + 200,000) / .68 = $794,118 c) Pre-tax income = $100,000 / .7 = $142,857 Revenue = ($340,000 + 142,857) / .68 = $710,084 d) CM = $700,000 x .68 = $476,000 Operating income $476,000 - 340,000 = $136,000 DOL = $476,000 / 136,000 = 3.50 e) Sales level - .20*sales level = 200,000 (the breakeven point) 0.8* sales level = 200,000 Sales level = 200,000 / .8 = 250,000 f) Lowest margin on contract = (2.5-.85)*210,000 – 370,000 = -$23,500 Highest margin on contract = (2.5-.70)*210,000 – 370,000 = $8,000

Since all values on this interval will be equally likely

Problem 3

a. Current labour cost = 5.50 * units made New labour cost with machine = 75,000 + 4 * units made Let x equal unit sales and setting the two costs together, we find the breakeven sales 5.5x = 75,000 + 4x x = 50,000 units If sales are above 50000 units the new machine will reduce costs.

b. The probability of at least breaking even on this rental is computed as follows: Upper limit - breakeven quantity = 70,000 - 50,000 = 66.67% Upper limit - lower limit 70,000 - 40,000

Page 391: MODULE 3 Financial Management and Management Accounting 1

Module 3 - Financial Management & Management Accounting 1

Page 391 © CMA Ontario, 2011

Problem 4

a. Sales mix = 8H : 4O : 3B Bundle sales price = (8 x $600) + (4 x $1,800) + (3 x $5,400) = $28,200 Bundle CM = (8 x $250) + (4 x $550) + (3 x $1,400) = $8,400 CM ratio = 8,400 / 28,200 Breakeven sales = $7,500,000 / (8,400 / 28,200) = $25,178,571 b. Let X = bundles 8,400X - 7,500,000 = .20(28,200)X 2,760X = 7,500,000 X = 2,717.39 Unbundling, we get: Home: 2,717.39 x 8 = 21,740 Office: 2,717.39 x 4 = 10,870 Bank: 2,717.39 x 3 = 8,153

Problem 5

Note that the current contribution margin provided by the bank product is $4,200,000. The proposed price would be $4,860 per unit ($5,400 * 90%) and fixed costs would increase by $100,000, the amount of the increased advertising.

The breakeven sales level (x) is given by the following equation:

4,200,000 = (4,860 – 4,000)x – 100,000

Solve find x = 5,000, which means sales would have to increase by at least 2,000 units (5,000 – 3,000) to justify this proposal.

Page 392: MODULE 3 Financial Management and Management Accounting 1

Module 3 - Financial Management & Management Accounting 1

Page 392 © CMA Ontario, 2011

Problem 6

a. Variable costs will increase by 12% to $22,176,000. Fixed costs remain the same at $7,500,000 as does the target profit at $900,000. The sum of these three items is the total revenue requirement, which is $30,576,000. Since the current revenue is $28,200,000, the required revenue increase will be 8.4255319% ((30,576,000 / 28,200,000) - 1). This is the required price increase for each product. The following table summarizes the results.

Home Office Bank DQS Total

Per Unit Total Per Unit Total Per Unit Total

Units sold 8,000 4,000 3,000

Price $651 $5,204,426 $1,952 $7,806,638 $5,855 $17,564,936 $30,576,000

Variable cost 392 3,136,000 1,400 5,600,000 4,480 13,440,000 22,176,000

Contribution margin $259 $2,068,426 $552 $2,206,638 $1,375 $4,124,936 $8,400,000

Fixed manufacturing 4,500,000

Fixed selling and administrative

3,000,000

Income $900,000

30,576,000/28,200,000 1.08426

b. The reason is the revenue covers not only the variable cost, but also the fixed cost and the target profit. Therefore, an increase in variable cost does not require a proportional increase in revenues since fixed costs and the target profit remain constant.

Problem 7

a. The incremental effect (x) on profit is computed as follows:

x = increase in ticket sales * contribution margin per customer * number of games per year * number of years – annual salary * number of years – signing bonus

x = (2,000 * (75+20 - 40) * 35 * 5) – (5,000,000 * 5) = 19,250,000 – 25,000,000 – 2,000,000 = -$7,750,000

b. The incremental contribution (y) from additional playoff games is computed as follows:

y = number of additional games * capacity * contribution margin per customer * number of years

y = 2 * 17,000 * 55 * 5 = $9,350,000

Contract contribution = -7,750,000 + 9,350,000 = $1,600,000

Page 393: MODULE 3 Financial Management and Management Accounting 1

Module 3 - Financial Management & Management Accounting 1

Page 393 © CMA Ontario, 2011

Problem 8

Computing the required pre-tax income (x):

x – (.2 * 200,000) – (.32 * (x - 200,000)) = 300,000

Solving to find x = $405,882.35

Letting y be the required sales revenue

y - .65y – 300,000 = 405,882.35

Solving find y = $2,016,806.72

Page 394: MODULE 3 Financial Management and Management Accounting 1

Module 3 - Financial Management & Management Accounting 1

Page 394 © CMA Ontario, 2011

Problem 9 a. Total fixed costs = (allocated fixed manufacturing costs per unit

*(normal volume) + (allocated fixed marketing costs per unit) *(normal volume)

= ($6.60 x 3,000) + ($7.70 x 3,000) = $42,900 CM/unit = (unit selling price) - (all unit variable costs) = $43.50 - ($5.50 + $8.25 + $4.20 + $2.75) = $43.50 - $20.70 = $22.80 Breakeven point in units = total fixed costs / CM per unit = $42,900 / $22.80 = 1,882 CM ratio = CM / sales = $22.80 / $43.50 = .52414 Breakeven point in sales = total fixed costs / CM ratio = $42,900 / .52414 = $81,848 or, breakeven units x selling price per unit = 1,882 x $43.50 = $81,867 (difference due to rounding of the CM ratio) b. New CM/unit = $38.50 - $20.70 = $17.80 Total contribution margin: Before: 3,000 x $22.80 $68,400 After: 3,500 x $17.80 62,300 Decrease in TCM $ 6,100

It is not recommended this action be taken as operating income would

drop by $6,100.

Page 395: MODULE 3 Financial Management and Management Accounting 1

Module 3 - Financial Management & Management Accounting 1

Page 395 © CMA Ontario, 2011

Problem 10 a. Variable costs per unit = ($1,400 + 1,500 + 1,000 + 1,100) / 100 units = $5,000 / 100 = $50 CM/unit = $100 - $50 = $50 Breakeven point in units = ($500 + $2,000) / $50 = $2,500 / $50 = 50 units b. DOL = CM / operating income = $5,000 / 2,500 = 2 Increase in operating income = 25% x 2 = 50% New operating income = $2,500 x 1.5 = $3,750 or, (125 units x $50 ) - $2,500 = $3,750 or, Increase operating income = 25 units x $50 = $1,250 New operating income = $2,500 + 1,250 = $3,750 c. CM ratio = $50 / $100 = 50% Breakeven sales = ($2,500 + 1,700) / .5 = $4,200 / .5 = $8,400

Page 396: MODULE 3 Financial Management and Management Accounting 1

Module 3 - Financial Management & Management Accounting 1

Page 396 © CMA Ontario, 2011

Problem 11 a. Weeder CM = $28 - 13 - 5 = $10 Hedge clippers CM = $36 - 12 - 4 = $20 Leaf blowers CM = $48 - 25 - 6 = $17 Budgeted net income = [(50,000*10) + (50,000*20) + (100,000*17)

– 2,600,000]*0.6 = [$500,000 + 1,000,000 + 1,700,000 - 2,600,000]*.6 = $600,000*.6 = $360,000 b. Let a bundle = 1 unit of weeder + 1 unit of hedge clipper + 2 units of leaf blowers

(i.e. at their standard mix) Then, the bundle contribution margin = $10 + $20 + ($17 x 2) = $64 Number of bundles to breakeven = $2,600,000 / $64 = 40,625 or, 40,625 weeders, 40,625 hedge clippers and 81,250 leaf blowers c. Operating income desired = $450,000 / .60 = $750,000 Bundle selling price = $28 + $36 + ($48 x 2) = $160 Bundle CM ratio = $64 / $160 = 40% Required sales = ($2,600,000 + $750,000) / .40 = $8,375,000 d. New contribution margins: Weeder CM = $28 - 13 - 5 = $10 Hedge clippers CM = $36 - 12 - 5 = $19 Leaf blowers CM = $48 - 30 - 6 = $12 New mix: 1:1:3 Bundle contribution margin = $10 + $19 + ($12 x 3) = $65 Number of bundles to breakeven = $2,600,000 / $65 = 40,000 or, 40,000 weeders, 40,000 hedge clippers and 120,000 leaf blowers

Page 397: MODULE 3 Financial Management and Management Accounting 1

Module 3 - Financial Management & Management Accounting 1

Page 397 © CMA Ontario, 2011

10. Pricing Learning Objectives

After completing this chapter, you will:

1. Understand the role that cost information plays in pricing decisions

2. Understand and be able to compute the appropriate cost based price in short-term and long-term pricing situations

3. Recognize and be able to interpret the scope and role of cost information in setting international transfer prices

4. Understand and be able to apply target costing when developing price for a new product

5. Understand and be able to apply basic analysis to evaluate an organization’s control over its pricing policy.

The Role of Cost Information in Pricing

The role of cost information in product pricing depends on the nature of the product market.

In highly competitive markets, the market sets the price and the role of cost information is to help management decide whether it can compete profitably at the market determined price.

If products can be differentiated or if the market is dominated by a one or a few large suppliers, the role of cost information is to help management identify a target price customers will find acceptable and will allow the organization to recover its costs and

provide an adequate return on invested capital.

This chapter will consider the role of cost information in each of these pricing environments.

Cost Information and Short- and Long-Term Pricing

Costs and Short-Term Pricing

Chapter 10 discussed the role of cost information in short-term pricing in the context of the special order problem. The idea in the special order problem is in the short-term, if

Management’s Pricing Role

Market Sets Price• Use cost information

to identify whether the market price provides an adequate return on investment given the organization’s cost and capital structure

Supplier Sets Price• Use cost information to

identify a product/price combination customers find acceptable and provides an adequate return on investment given the organization’s cost and capital structure

Page 398: MODULE 3 Financial Management and Management Accounting 1

Module 3 - Financial Management & Management Accounting 1

Page 398 © CMA Ontario, 2011

there are no long-term implications, the price offered should at least cover short-term variable costs plus any opportunity costs. Note that the special order problem is characterized by a non-competitive situation – the supplier is asked whether or not to accept a price offer.

But what about the role of cost information in the short-term pricing of ongoing products whose demand fluctuates across periods? Of particular interest in this context is pricing services that cannot be inventoried, for example, pricing airline tickets. Over many routes, the market for airline seats is reasonably competitive, which means that through intense competition the airlines, in effect, create a market price. The airline will supply seats in this market using the same criterion used in the special order problem. That is, supply a seat as long as the price at least meets the variable cost plus any opportunity cost of supplying the seat. The problem is since an airline’s variable costs comprise of such a tiny total of its total costs, short-term pricing results in cycles of airline bankruptcies, a topic covered below.

Costs and Long-Term Pricing

An organization can use cost information in several different ways when it sets the price for its products. However, the most widely used approach is cost plus a mark-up. The primary reason for the popularity of this approach is it provides a price based on a profit target. If the price is one the customers find reasonable and the profit target is based on the costs of an efficient supplier with a rate of return on invested capital that reflects market expectations, the price reflects a stable long-term equilibrium price that will support demand and supply.

An example:

Carr Company is developing an electric motor to sell to suppliers in the home heating industry. Customers regard Carr Company’s products as superior because of the company’s outstanding reputation for product quality and performance. For this reason, Carr Company can set the prices for its products.

Based on past experience developing and selling many types of electric motors, the finance group at Carr Company has developed the following prospective costs for this new motor:

Page 399: MODULE 3 Financial Management and Management Accounting 1

Module 3 - Financial Management & Management Accounting 1

Page 399 © CMA Ontario, 2011

Per Unit Annual (50,000

units) Total for Product Life (10 years)

Direct materials cost $105.00 Direct labour cost 145.00 Variable overhead cost 25.00 Variable S,G and A 22.00

Total variable cost $297.00 $14,850,000 $148,500,000 Fixed manufacturing 2,500,000 25,000,000 Fixed S,G and A 450,000 4,500,000 Cost of quality 150,000 1,500,000 Product development costs 10,000,000 Product abandonment costs 1,500,000

Total product related costs $191,000,000 Total lifetime unit sales 500,000 Total product cost per unit $382.00 Prospective selling price (20% mark-up) $458.40

The total lifetime product cost is estimated to be $382 per unit. If Carr Company adds 20% mark-up to the computed product cost to provide a return on invested capital, the prospective target price would be $458.40.

Varying Demand and Varying Prices

When demand is seasonal, prices will vary to reflect the fluctuations in demand and the actions of competitors. While prices may vary in the short-term, in the long-term, the average price will need to at least equal the target long-term price or the organization will fail to recover its total out of pocket costs and its capital costs, thereby, impairing owner’s wealth.

The Role of the Management Accountant in Supporting Revenue Control

Assume during the first five years of the project, Carr Corporation faces lower demand than planned. The sales force begins to engage in aggressive price cutting in order to meet the target sales levels in the original plan. The following results were observed over the first five years of the motor project’s life:

Page 400: MODULE 3 Financial Management and Management Accounting 1

Module 3 - Financial Management & Management Accounting 1

Page 400 © CMA Ontario, 2011

It is evident the revenue to date experience is tracking well below the projected target revenues required to justify this project.

This type of analysis illustrates the management accountant’s role in providing control information after the fact. Recall that fixed costs are sunk so, if demand is lower than expected and the only way to improve revenues is to decrease prices (that is, demand is elastic), then the sales force is doing what is required as long as the short-term price covers the short-term variable cost. However, if price cutting reflects undisciplined sales force activities merely designed to boost short-term unit sales then management intervention is required to restore target pricing discipline.

When variable costs are comprised of a small fraction of fixed costs, a situation is created where organizations are tempted to engage in short-term pricing that easily covers variable costs, but fails to achieve an average cost that meets the target long-term price. This is precisely what causes the cyclical pattern of airline bankruptcies.

The Management Accountant’s Role in Revenue Control

As part of the process of evaluating ongoing performance, senior management at Carr Company might ask the management accountant to investigate how prices were set during the first five years of the motor project. Assume the management accountant’s investigation produces the following result:

Target price $458.40 Average discretionary competitive discount - 5% -22.92

Adjusted price 435.48 Average quantity discount - 3% -13.06

Invoice price 422.42 Average prompt payment discount - 1.5% -6.34

Net price 416.08

Page 401: MODULE 3 Financial Management and Management Accounting 1

Module 3 - Financial Management & Management Accounting 1

Page 401 © CMA Ontario, 2011

Average take backs - .5% -2.08

Realized average price $414.00

The following, called a price waterfall, shows the sources of price erosion for the motor:

The sales force at Carr Company controls the discretionary competitive discount and the average take backs (the cost to recover unsold inventory from customers) through direct negotiations with customers, while management policy sets the quantity discount and the prompt payment discount. More than half of the price erosion is caused by concessions made by the sales force offering a level of discounts senior management might consider far too high and, therefore, control more tightly.

Page 402: MODULE 3 Financial Management and Management Accounting 1

Module 3 - Financial Management & Management Accounting 1

Page 402 © CMA Ontario, 2011

Contractual Settings Where Cost Information Determines the Price

There are five major contractual settings where cost information determines the price:

1. Cost plus contracts

2. Insurance claims

3. Legal settings

4. Bidding

5. International transfer pricing

Cost Plus Contracts

In cost plus contracts, the supplier is reimbursed costs plus a profit margin. In this sense, cost plus contracts follow the same approach as the Carr Company example illustrated above. The difference being that the parties to the contract must agree how the contract costs are to be computed. The key areas of negotiation will be:

1. How variable costs will be computed and whether standard or actual costs will be used for variable costs,

2. What fixed manufacturing costs will be deemed to be related to the contract and the basis for allocating fixed costs to the contract,

3. What selling, general and administrative costs will be deemed to be related to the contract,

4. What other costs such as product development and abandonment costs will be deemed to be allocated to the project,

5. The base upon which the mark-up percentage will be applied, and

6. The mark-up percentage.

Many cost plus contract disputes focus on failures to adequately specify what costs are to be included in computing the total contract cost and how these costs will be computed. (The other major dispute focuses on inadequate or potentially fraudulent record keeping by the supplier.) During contract costing disputes, appeals to generally accepted costing standards or conventional industry costing standards are invariably useless, since acceptable costing practices vary widely. A vital role for management accountants on both sides of cost plus contracting is to ensure all components of costs in the contract are fully specified.

Insurance Claims

Unless otherwise specified, insurance claims focus on the cost of the loss. Therefore, in effect, developing an insurance policy requires attention to the same details discussed in cost plus contracting with the exception that there is no mark-up.

Page 403: MODULE 3 Financial Management and Management Accounting 1

Module 3 - Financial Management & Management Accounting 1

Page 403 © CMA Ontario, 2011

Legal Settings

There are legal issues surrounding the relationship between costs and prices when an organization is charged with predatory pricing under the provisions of the Canadian Competition Act. A price is deemed unreasonable and, therefore, predatory under the Act if:

It is deemed to be either intended to drive or to have the effect of driving, competitors out of a market, and

It is followed by a price increase deemed to be intended to recover losses incurred during the predatory pricing period.

The role of pricing in claims of predatory pricing focuses on price relative to variable cost in general and average variable cost in particular. Prices that approach or fall beneath average variable costs are likely to be viewed as increasingly unreasonable and, therefore, regarded as potentially predatory.

Therefore, management accountant must ensure the accounting system can compute costs in a reasonably accurate way, not only for internal decision making regarding prices, but also to withstand charges, if they are laid, that the price charged for the organization’s products are below variable costs.

Bidding Settings

Competitive bidding situations are a form of cost plus pricing in environments where the expectation is the contract will be awarded to the lowest bidder. The tension is that as the bid price is increased, the profit is increased if the bid is successful, but the probability of the bid being successful decreases. Therefore, there are two major forms of uncertainty in the competitive bidding environment: costs are subject to uncertainty, and being awarded the contract is uncertain. The one certainty is the price if the bid is accepted.

Return to the data for Carr Company and assume all the information is the same, except the situation is now:

1. A single customer is asking for bids to supply 50,000 units of the motor each year for 10 years.

2. The computed total price per unit is subject to uncertainty. Based on the underlying cost data, the Carr Company cost analyst believes the total product cost per unit is now estimated to lie between $370 and $400 with all values on this interval equally likely.

3. The bid is to be expressed as a price per unit. Based on an analysis of past bidding situations, the marketing manager believes any bid equal to or more than $480 will lose with certainty and any bid less than or equal to $380 will win with certainty. The probability of a bid between $380 and $480 being accepted is (480 – bid amount) / 100. For example, a bid of $400 would have an 80% probability of being accepted.

Page 404: MODULE 3 Financial Management and Management Accounting 1

Module 3 - Financial Management & Management Accounting 1

Page 404 © CMA Ontario, 2011

Using the expected value criterion developed in Chapter 12, we can write the expected value of any bid being equal to the probability the bid will be accepted multiplied by the product of the difference between the bid amount and the expected cost per unit and the total number of units in the order. Recalling the properties of the uniform distribution discussed in Chapter 12, the expected cost per unit will be $385 ((400 + 370) / 2) and the expected value of any bid b will be:

Expected value of b = (480 - b) / 100 * (b - 385) * (50,000 * 10)

By trial and error or simple calculus, we find the bid amount that maximizes expected profit is $432.50 and the expected profit associated with this bid is $11,281,250.

Note that the expected profit margin range for this bid is a high of 16.89% ((432.50-370) / 370) to a low of 8.13% ((432.50-400) / 400) and an expected value of 12.34% ($432.50-385) / 385). Therefore, with this expected profit maximizing bid, Carr Company has no chance of achieving its target mark-up of 20% on costs.

The role of accurate cost information is critical in this exercise since it provides the basis for the best bid.

International Transfer Pricing

Whenever organizations transfer goods or services between two tax jurisdictions, they must price the transfer in order to determine the profit and, therefore, the tax liability in each tax jurisdiction.

The transfer price that international conventions (reflecting the OECD guidelines on international transfer pricing) is an arm’s length market price (called the comparable uncontrolled price or CUP). Unfortunately, there is seldom an agreement on a CUP and transfer prices are based one of the four subsidiary methods, one of which is cost plus. Since tax authorities are aware it is in an organization’s interests to use a transfer price to shift income to the lower tax jurisdiction, the affected tax authorities will carefully scrutinize how the organization computes the cost base for the cost plus based transfer price. Generally, the tax authorities will use the approach described in the cost plus contracts section above when evaluating a proposed cost base for a transfer price.

Cost Information and Price in Product Planning

As we have seen, cost plus pricing moves from cost toward establishing a market price. An alternative approach works backward from the market prices to determine what cost must be in order for the product to be successful. This approach is called target costing.

Target costing is a planning tool, rather than an operational tool, since it is used to determine the product’s design and the design of the process that will make the product. The rationale for target costing is the realization that, on average, 80% of a product’s costs are committed at the time the product and its manufacturing process are designed. Therefore, it is easier to reduce product costs during the planning stage rather than after the product is designed and its manufacturing system is put in place.

Page 405: MODULE 3 Financial Management and Management Accounting 1

Module 3 - Financial Management & Management Accounting 1

Page 405 © CMA Ontario, 2011

The target costing approach involves the following steps:

1. The price the market will set for a proposed product with specific attributes is estimated. This might be done by evaluating the price for similar products or by questioning prospective customers using customer panels. This price is called the target price.

2. The cost of developing and producing the proposed product is estimated. This is called the as-if cost.

3. The product and process design determine the required investment level in the product which, in turn, allows the organization to identify the target profit that will provide the required return on investment. The target profit is deducted from the target price to compute the target cost.

4. The product’s as-if cost is compared to the target cost. If the target cost is less than the as-if cost, the product development team reformulates the product and process design to reduce the product’s as-if cost. If the as-if cost reduction results in changing the target price the customer is willing to pay, this must be taken into account. The product and process redesign continues until the as-if cost is less than the target cost.

Page 406: MODULE 3 Financial Management and Management Accounting 1

Module 3 - Financial Management & Management Accounting 1

Page 406 © CMA Ontario, 2011

During the recursive planning process when a tentative product and process design are developed and their resulting target profits and target costs are computed, there is a delegation of the cost reduction targets to each of the product’s components. For example, assume a proposed product has 40 components that comprise 90% of the product’s estimated as-if cost. A significant planning consideration is what proportion of the as-if cost reduction will be assigned to each of the components. This delegation of cost reduction targets is particularly difficult, primarily because different components may have significantly different potential for cost reduction and, in many cases, the design of the components will not only affect their own costs, but the costs of other components and assembly costs. Common tools used at this stage include value engineering (compares the value to the cost of each component or each product feature) and design for manufacturing assembly (which considers the manufacturing cost of each design).

There is a significant role for the management accountant in target costing. Not only does the management accountant prepare the cost estimates that are crucial in guiding the entire target costing process, but often is the obvious choice to play the general management role on a team often comprised of functional specialists such as design, engineering and marketing.

Page 407: MODULE 3 Financial Management and Management Accounting 1

Module 3 - Financial Management & Management Accounting 1

Page 407 © CMA Ontario, 2011

Problems with Solutions Multiple Choice Questions Questions 1 – 5 inclusive refer to the data for Carr Company in the cost plus example in the chapter. 1. If sales are 40,000 per year the prospective selling price will be:

a. $403.25 b. $445.80 c. $483.90 d. None of the above

2. Return to the original data in the problem. If the project has an eight year life, the target selling price will be: a. $387.75 b. $403.20 c. $427.43 d. $372.24

3. Return to the original data in the problem. If the mark-up is 25%, the target selling price will be: a. $387.60 b. $445.00 c. $465.70 d. None of the above

4. Return to the original data in the problem. If the product abandonment costs are

$2,500,000, the target selling price will be: a. $390.50 b. $460.80 c. $480.00 d. None of the above

5. Return to the original data in the problem. If the fixed manufacturing costs each year are $3,000,000, the target selling price will be: a. $390.55 b. $410.00 c. $445.50 d. None of the above

Page 408: MODULE 3 Financial Management and Management Accounting 1

Module 3 - Financial Management & Management Accounting 1

Page 408 © CMA Ontario, 2011

Questions 6 – 8 refer to the following data: Brantford Ovens, a supplier of cooking ovens, is considering developing an oven that would be used in retail pizza shops. What is the target cost in each of the following questions? Question 6

1. Target price – $12,000 2. Required return – 25% margin on costs The target cost is:

a. $8,000 b. $9,600 c. $10,000 d. None of the above

Question 7 1. Target price – $12,000 2. Required return – 25% margin on sales The target cost is:

a. $8,000 b. $9,600 c. $10,000 d. None of the above

Question 8 1. Target price – $12,000 2. Annual sales – 500 units 3. Average inventory and capital asset investment level for this product during its

lifetime – $1,500,000 4. Required pre-tax return on invested capital – 25% The target cost is:

a. $10,900 b. $11,000 c. $11,750 d. None of the above

Page 409: MODULE 3 Financial Management and Management Accounting 1

Module 3 - Financial Management & Management Accounting 1

Page 409 © CMA Ontario, 2011

Problem 1

Amelia Hughes is a commercial helicopter pilot who uses her helicopter to fly employees of large organizations from the Hamilton area to airports in the Northern United States. The average commute time from Hamilton to these US airports is one hour, which cuts down travel time considerably.

Since Amelia is operating at approximately 60% of capacity, she is considering offering two hour helicopter tours to various venues from her home airport in Hamilton. Tours would focus primarily on Toronto and Niagara Falls.

Since she would have no competitors, Amelia plans to price the tours by the hour. The prices Amelia offers to current customers are based on hourly market rates, which average 120% of the total hourly cost of each flight.

Amelia has estimated flight related costs as:

• Variable operating costs per hour – $400

• Fixed ground related costs per flight – $50

• Allocation of the annual fixed cost (including depreciation on Amelia’s helicopter) of Amelia’s other flight related costs – $30 per hour based on operating at 60% of capacity and using the expected activity level as the denominator to compute the fixed overhead rate.

Required:

a. What hourly rate will Amelia charge her current customers?

b. If Amelia’s tour operation results in increasing operations to 80% of capacity and she continues to compute the hourly fixed cost overhead rate in the same way, what would be Amelia’s revised hourly rate?

c. What is the apparent relationship between the planned level of capacity use and the rate Amelia charges her customers?

d. Amelia has no control over the rates she charges business customers since they are set by the market. What hourly rate should she charge for her proposed tours? Since she would have no competition, there is no current market rate.

Page 410: MODULE 3 Financial Management and Management Accounting 1

Module 3 - Financial Management & Management Accounting 1

Page 410 © CMA Ontario, 2011

Problem 2

Ayr Company manufactures residential and commercial security alarms. The company has designed a new residential alarm that is able to connect to home Wi-Fi systems. This feature reduces the cost of installation and avoids the need for conventional telephone access to monitoring systems. Because the system is advanced and innovative it has no direct competitors and, therefore, Ayr Company plans to use a cost based price for this new product. The company policy is to mark-up total manufacturing cost by 30% to set the product price.

Ayr Company is organized into three departments: stores, production and assembly. The fixed cost structure in each department is:

Cost Driver

Department Cost Driver Fixed

Manufacturing Overhead

Practical Capacity

Average Activity

Level

Planned Activity Level

Stores Materials cost $200,000 $1,200,000 $900,000 $1,000,000 Production Machine hours $1,200,000 10,000 7,000 8,000 Assembly Labour hours $450,000 8,000 5,000 7,000

The costs in each department for this new product have been estimated as:

Stores: • Materials – $35 • Direct labour – .15 hours @ 18 per hour • Variable overhead – materials cost * 20%

Manufacturing: • Materials – $2.50 • Direct labour – .2 hours @$22 per hour • Machine hours – .15 • Variable overhead – machine hours $15

Assembly • Materials – $.75 • Direct labour – .9 hours @ $28 • Variable overhead – labour hours * $2

Required:

The current approach is to compute manufacturing costs as all variable costs plus an allocation of total plant fixed manufacturing overhead based on an hourly rate for direct labour. The current total labour hours worked in this plant is 12,000 hours. Compute the product price based on the current policy.

Page 411: MODULE 3 Financial Management and Management Accounting 1

Module 3 - Financial Management & Management Accounting 1

Page 411 © CMA Ontario, 2011

Problem 3

Return to the data in Problem 2.

Required:

Compute the product price under each of the following assumptions:

a. The fixed manufacturing overhead rate is calculated using departmental practical capacity.

b. The fixed manufacturing overhead rate is calculated using average departmental activity level.

c. The fixed manufacturing overhead rate is calculated using planned departmental activity rate.

Problem 4

Niagara Steelworks is engaged in a costing dispute with Public Works Canada. The company has a cost plus contract to supply speciality steel that has no evident market price. The contract calls for Niagara Steelworks to be reimbursed for its manufacturing costs plus 35%. An independent shipper hauls the steel from the Niagara Steelworks plant to the various construction sites.

The variable cost of manufacturing the steel is $200 per ton, which is not in dispute. The issue concerns the allocation of fixed manufacturing costs to this product. Current annual fixed manufacturing cost at Niagara Steelworks is $300,000,000. The plant is operating at 40% of practical capacity, which is measured in tons. Niagara Steelworks computes the fixed manufacturing overhead rate by dividing fixed manufacturing cost by the planned level of operations. This has resulted in charging this contract a rate of $120 per ton for fixed manufacturing costs.

Cost analysts at Public Works have objected citing industry evidence that, on average, steel companies are using 70% of their practical capacity.

Required:

a. Compute the contract price per ton using Niagara Steelwork’s approach.

b. Compute the contract price per ton if Niagara Steelwork’s uses average capacity use in the industry to compute the fixed manufacturing overhead rate.

c. If you were hired to arbitrate this dispute, how would you resolve it?

d. If you were a Public Works Canada auditor, what would you recommend based on this experience?

Page 412: MODULE 3 Financial Management and Management Accounting 1

Module 3 - Financial Management & Management Accounting 1

Page 412 © CMA Ontario, 2011

Problem 5

Nancy Ng is preparing a bid to supply and maintain plants for a year in the head office of a large organization. Nancy estimates the costs associated with this job will be $15,000 for the plants, $8,000 for direct labour and $3,000 for other variable costs related to the contract. Nancy’s only other costs are office costs that do not depend on the number or size of contracts she is awarded.

Nancy applies a mark-up on the estimated contract cost to determine a target bid price.

The income statement Nancy’s accountant prepared last year follows:

Sales $290,000 Contract related costs Direct materials 45,000 Direct labour 55,000 Variable overhead 24,500 124,500

Contract contribution to head office costs 165,500 Head office costs 80,000

Income before taxes $85,500

Required:

a. What average mark-up percentage is Nancy using to price jobs?

b. If Nancy uses the average mark-up percentage you found in part a), what bid would she submit on this job?

Page 413: MODULE 3 Financial Management and Management Accounting 1

Module 3 - Financial Management & Management Accounting 1

Page 413 © CMA Ontario, 2011

Problem 6

Grand River Company (GRC) manufactures hotel furniture. Recently, GRC received an order for 500 beds at $1,000 per bed. The production manager has developed a preliminary cost of making one of these beds as:

Direct materials – $400

Direct labour – $200

Variable overhead – $50

Fixed manufacturing overhead – 120% of direct labour cost

Required:

a. If GRC has a target of 20% on sales, what is the target cost?

b. If GRC has a target of 30% on total manufacturing cost, what is the target cost?

Problem 7

Eye Savers Company (ESC) manufactures a variety of eye care products. ESC is considering the introduction of a lens cleaner for anti-reflective plastic lenses.

Initial cost estimates, which include the product and packaging cost per 125 ml package of this product, were:

Direct materials – $1.45

Direct labour – $.35

Variable manufacturing overhead – $.20

ESC has a target margin of 100% of variable manufacturing costs.

Market research suggests the target price for this product would be $3.75

Required:

a. What is this product’s target variable cost?

b. Assume ESC will only offer products that meet its target margin. The plant manager has indicated a re-engineering project costing $1,000,000 could reduce the initial variable cost estimate to the target cost. Ignoring taxes and the time value of money, what is the breakeven level of this product’s lifetime sales that would justify this study?

Page 414: MODULE 3 Financial Management and Management Accounting 1

Module 3 - Financial Management & Management Accounting 1

Page 414 © CMA Ontario, 2011

Problem 8

Return to the data of Problem 7. Assume the results of a consumer panel testing a sample of the proposed product revealed the following information. The lifetime demand for this product follows this formula:

Lifetime demand in units = 1,000,000 – ((variable cost – 1.50) *1,000,000)

The target price for the product is a function of the appearance of the product and its packaging which, in turn, are directly affected by the product’s variable cost. The apparent relationship is:

Variable Cost Target Price

$1.50 $1.50 $1.55 $1.88 $1.60 $2.09 $1.65 $2.24 $1.70 $2.34 $1.75 $2.42 $1.80 $2.49 $1.85 $2.54 $1.90 $2.59 $1.95 $2.63 $2.00 $2.66

Required:

Given this information, what is best product and packaging design?

Page 415: MODULE 3 Financial Management and Management Accounting 1

Module 3 - Financial Management & Management Accounting 1

Page 415 © CMA Ontario, 2011

Solutions:

Multiple Choice Questions

Q 1 (c)

Per Unit Annual

Total for Product Life

Direct materials cost $105.00 Direct labour cost 145.00 Variable overhead cost 25.00 Variable S,G and A 22.00

Total variable cost $297.00 $11,880,000 $118,800,000 Fixed manufacturing 2,500,000 25,000,000 Fixed S,G and A 450,000 4,500,000 Cost of quality 150,000 1,500,000 Product development costs 10,000,000 Product abandonment costs 1,500,000

Total product related costs $161,300,000 Total lifetime unit sales 400,000 Total product cost per unit $403.25 Prospective selling price $483.90

Q 2 (d)

Per Unit Annual

Total for Product Life

Direct materials cost $105.00 Direct labour cost 145.00 Variable overhead cost 25.00 Variable S,G and A 22.00

Total variable cost $297.00 $14,850,000 $118,800,000 Fixed manufacturing 2,500,000 20,000,000 Fixed S,G and A 450,000 3,600,000 Cost of quality 150,000 1,200,000 Product Development costs 10,000,000 Product abandonment costs 1,500,000

Total product related costs $155,100,000 Total lifetime unit sales 500,000 Total product cost per unit $310.20 Prospective selling price $372.24

Page 416: MODULE 3 Financial Management and Management Accounting 1

Module 3 - Financial Management & Management Accounting 1

Page 416 © CMA Ontario, 2011

Q 3 (d)

Per Unit Annual

Total for Product Life

Direct materials cost $105.00 Direct labour cost 145.00 Variable overhead cost 25.00 Variable S,G and A 22.00

Total variable cost $297.00 $14,850,000 $148,500,000 Fixed manufacturing 2,500,000 25,000,000 Fixed S,G and A 450,000 4,500,000 Cost of quality 150,000 1,500,000 Product development costs 10,000,000 Product abandonment costs 1,500,000

Total product related costs $191,000,000 Total lifetime unit sales 500,000 Total product cost per unit $382.00 Prospective selling price $477.50

Q4 (b)

Per Unit Annual

Total for Product Life

Direct materials cost $105.00 Direct labour cost 145.00 Variable overhead cost 25.00 Variable S,G and A 22.00

Total variable cost $297.00 $14,850,000 $148,500,000 Fixed manufacturing 2,500,000 25,000,000 Fixed S,G and A 450,000 4,500,000 Cost of quality 150,000 1,500,000 Product development costs 10,000,000 Product abandonment costs 2,500,000

Total product related costs $192,000,000 Total lifetime unit sales 500,000 Total product cost per unit $384.00 Prospective selling price $460.80

Page 417: MODULE 3 Financial Management and Management Accounting 1

Module 3 - Financial Management & Management Accounting 1

Page 417 © CMA Ontario, 2011

Q 5 (d)

Per Unit Annual

Total for Product Life

Direct materials cost $105.00 Direct labour cost 145.00 Variable overhead cost 25.00 Variable S,G and A 22.00

Total variable cost $297.00 $14,850,000 $148,500,000 Fixed manufacturing 3,000,000 30,000,000 Fixed S,G and A 450,000 4,500,000 Cost of quality 150,000 1,500,000 Product development costs 10,000,000 Product abandonment costs 1,500,000

Total product related costs $196,000,000 Total lifetime unit sales 500,000 Total product cost per uit $392.00 Prospective selling price $470.40

Q 6 (b)

The formula is:

Target price - target margin = target cost

12,000 - .25* target cost = target cost

Target cost = $9,600

Q 7 (d)

The formula is:

Target price - target margin = target cost

12,000 - .25* target price = target cost

12,000 - .25 * 12000 = target cost

Target cost = $9,000

Page 418: MODULE 3 Financial Management and Management Accounting 1

Module 3 - Financial Management & Management Accounting 1

Page 418 © CMA Ontario, 2011

Q 8 (d)

The formula is:

Target price - target margin = target cost

12,000 - .25 * (1,500,000 / 500) = target cost

Target cost = $11,250

Page 419: MODULE 3 Financial Management and Management Accounting 1

Module 3 - Financial Management & Management Accounting 1

Page 419 © CMA Ontario, 2011

Problem 1

a)

Variable overhead cost per hour $400.00 Fixed flight cost ($50/1) 50.00 Other flight related fixed costs 30.00

Total hourly flight cost $480.00 Mark-up @ 20% 96.00

Hourly price $576.00

b)

Variable overhead cost per hour $400.00 Fixed flight cost ($50/2*) 25.00 Other flight related fixed costs - (80-20)/80*30 22.50

Total hourly flight cost $447.50 Mark-up @ 20% 89.50

Hourly price $537.00

*Because in a flight from A to B there are two landing fees and two take off fees. When the flight returns to its home base without landing anywhere else there is only one take off fee and one landing fee. c) As planned capacity use goes up, price goes down. Apparently as demand goes up, price will go down. Therefore, as demand drops price will go up. d) Since the objective is to use currently idle capacity, one approach would be to start with a higher price and then lower the price until desired capacity use is reached.

Page 420: MODULE 3 Financial Management and Management Accounting 1

Module 3 - Financial Management & Management Accounting 1

Page 420 © CMA Ontario, 2011

Problem 2

Note that the total labour hours per unit in all departments will be 1.25 (.15+.20+.90). Total fixed manufacturing cost in all departments will be $1,850,000 ($200,000 + $1,200,000 + $450,000). The rate per labour hour for fixed overhead will be $154.17 (1,850,000 / 12,000). Therefore, the fixed overhead allocated per unit will be $192.71 (1.25 * $154.17).

Stores Production Assembly Total

Direct materials $35.00 $2.50 $0.75 $38.25 Direct labour .15 hours @ $18 2.70 2.70 .20 hours @ $22 4.40 4.40 .90 hours @ $28 25.20 25.20 Variable overhead Materials cost * 20% 7.00 7.00 Machine hours * $15 2.25 2.25 Labour hours * $2 1.80 1.80 Fixed overhead Total labour hours * ($1,850,000/12,000)

192.71

Total manufacturing cost $274.31 Price mark-up @ 30% 82.29

Product price $356.60

Page 421: MODULE 3 Financial Management and Management Accounting 1

Module 3 - Financial Management & Management Accounting 1

Page 421 © CMA Ontario, 2011

Problem 3

a)

Stores Production Assembly Total

Direct materials $35.00 $2.50 $0.75 $38.25 Direct labour .15 hours @ $18 2.70 2.70 .20 hours @ $22 4.40 4.40 .90 hours @ $28 25.20 25.20 Variable overhead Materials cost * 20% 7.00 7.00 Machine hours * $15 2.25 2.25 Labour hours * $2 1.80 1.80 Fixed overhead $200,000/1,200,000 * $35 5.83 5.83 $1,200,000/10,000 * .15 18.00 18.00 $450,000/8,000 * .9 5.06 5.06

Total manufacturing cost $110.50 Price mark-up @ 30% 33.15

Product price $143.64

b)

Stores Production Assembly Total

Direct materials $35.00 $2.50 $0.75 $38.25 Direct labour .15 hours @ $18 2.70 2.70 .20 hours @ $22 4.40 4.40 .90 hours @ $28 25.20 25.20 Variable overhead Materials cost * 20% 7.00 7.00 Machine hours * $15 2.25 2.25 Labour hours * $2 1.80 1.80 Fixed overhead $200,000/900,000 * 35 7.78 7.78 $1,200,000/7,000 * .15 25.71 25.71 $450,000/5,000 * .9 81.00 81.00

Total manufacturing cost $196.09 Price mark-up @ 30% 58.83

Product price $254.92

Page 422: MODULE 3 Financial Management and Management Accounting 1

Module 3 - Financial Management & Management Accounting 1

Page 422 © CMA Ontario, 2011

c)

Stores Production Assembly Total

Direct materials $35.00 $2.50 $0.75 $38.25 Direct labour .15 hours @ $18 2.70 2.70 .20 hours @ $22 4.40 4.40 .90 hours @ $28 25.20 25.20 Variable overhead Materials cost * 20% 7.00 7.00 Machine hours * $15 2.25 2.25 Labour hours * $2 1.80 1.80 Fixed overhead $200,000/1,000,000 * 35 7.00 7.00 $1,200,000/8,000 * .15 22.50 22.50 $450,000/7,000 * .9 57.86 57.86

Total manufacturing cost $168.96 Price mark-up @ 30% 50.69

Product price $219.64

Page 423: MODULE 3 Financial Management and Management Accounting 1

Module 3 - Financial Management & Management Accounting 1

Page 423 © CMA Ontario, 2011

Problem 4

a)

Variable manufacturing cost per ton $200.00 Fixed manufacturing cost per ton 120.00

Total manufacturing cost per ton $320.00 Mark-up @ 35% 112.00

Contract price per ton $432.00

b)

Variable manufacturing cost per ton $200.00

Practical capacity (300,000,000/(.4*120)) 6,250,000 Average capacity (70% of practical) 4,375,000 Fixed manufacturing cost per ton using average (300,000,000/average)

68.57

Total manufacturing cost per ton $268.57 Mark-up @ 35% 94.00

Contract price per ton $362.57

c) The problem is the contract did not specify the basis for computing the fixed manufacturing overhead rate, leaving the company free to charge idle capacity costs to this contract. It seems unfair to expect the government contract to absorb excess capacity costs. The industry average, which reflects general conditions and average industry efficiency in dealing with the conditions faced, appears to be a more reasonable approach.

d) The fixed manufacturing overhead rate itself should be specified in the contract. Not only does this eliminate the idle capacity costs discussed in c), but it also deals with any other fixed manufacturing overhead inefficiencies.

Problem 5

a. Average mark-up percentage =(290,000 - 124,500) / 124,500 = 132.93% b. Plant costs $15,000 Direct labour costs 8,000 Other variable costs 3,000 Total contract related costs 26,000 Mark-up 34,562 Contract price $60,562

Page 424: MODULE 3 Financial Management and Management Accounting 1

Module 3 - Financial Management & Management Accounting 1

Page 424 © CMA Ontario, 2011

Problem 6

a. Target cost = target price – target margin

Target cost = $1000 – 20% * $1000 = $800

b. Target cost = target price – target margin

Target cost = $1000 - .30*(400 + 200 + 50 + 1.2 * 200) = $1000 - $267 = $753

Problem 7

a.

Direct materials $1.45 Direct labour 0.35 Variable overhead 0.20

Total variable cost $2.00 Target margin @ 100% of variable cost $2.00

Target price $3.75 Target margin 2.00

Target variable cost $1.75

b. The re-engineering project would result in a product that would have a contribution margin of $2 per unit. Therefore, the breakeven quantity of lifetime sales of this product would be 500,000 (1,000,000 / 2).

Page 425: MODULE 3 Financial Management and Management Accounting 1

Module 3 - Financial Management & Management Accounting 1

Page 425 © CMA Ontario, 2011

Problem 8

Developing the product that has the functionality provided by the variable cost of $1.70 provides the highest contribution margin.

Variable Cost Target Price CM Demand Total

$1.50 $1.50 $0.00 1,000,000 $0.00 $1.55 $1.88 $0.33 950,000 $313,500.00 $1.60 $2.09 $0.49 900,000 $441,000.00 $1.65 $2.24 $0.59 850,000 $501,500.00 $1.70 $2.34 $0.64 800,000 $512,000.00 $1.75 $2.42 $0.67 750,000 $502,500.00 $1.80 $2.49 $0.69 700,000 $483,000.00 $1.85 $2.54 $0.69 650,000 $448,500.00 $1.90 $2.59 $0.69 600,000 $414,000.00 $1.95 $2.63 $0.68 550,000 $374,000.00 $2.00 $2.66 $0.66 500,000 $330,000.00

Page 426: MODULE 3 Financial Management and Management Accounting 1

Module 3 - Financial Management & Management Accounting 1

Page 426 © CMA Ontario, 2011

Plan

DoAct/Revise

Check

The Budget Domain

11. Budgeting

Learning Objectives

After completing this chapter, you will:

1. Understand the structure of the master budget and the role it plays in planning and control.

2. Be able to develop and interpret a master budget for a simple organization.

3. Be able to develop and interpret inventory, labour and cash budgets.

4. Understand the structure of static and flexible budgets and the role they play in planning and control.

What is a Budget?

In the private sector, there are two types of budgets:

1. Operating budgets summarize the level of activities such as sales, purchasing, production and the acquisition of the various factors of production.

2. Financial budgets identify the expected financial consequences of the activities summarized in the operating budgets. The major financial budgets are the projected balance sheet, the projected income statement and the project statement of cash flows.

In the public sector, budgets reflect the spending authority granted to the unit that will make the expenditures. These notes focus on budgets in the private (for profit) sector.

The Role of Budgets

The organization plan reflects the implementation of the organization’s strategy for a planning period, which is usually one year. The plan results in a set of implemented activities (do). The results of those activities are monitored (check). These results are studied and any required revisions are developed and implemented as a revised plan act/revise).

Budgets are an important part of the organization’s planning and control cycle. The domain of budgeting is shown as the shaded area in this exhibit. Budgets provide quantitative targets (both financial and non-financial) of planned activities. These measures are used to communicate organization objectives, thereby, providing a coordinating and integrating role in the organization.

Page 427: MODULE 3 Financial Management and Management Accounting 1

Module 3 - Financial Management & Management Accounting 1

Page 427 © CMA Ontario, 2011

After the fact, the targets developed in the plan provided the basis against which results are compared. The purpose of the comparison of planned and actual results is two-fold:

1. To identify opportunities to improve organization performance.

2. To assign accountability for performance failures and rewards for performance success.

It should be noted that while most people agree on the important role of using performance results to identify opportunities for improvement, there is strong disagreement about the accountability role. Since much of the information used to set performance targets in the budget comes from the people whose performance will be evaluated by the budget, there is a strong incentive for people to manipulate the information they provide, which is then used to set budget targets. This mitigates getting truthful information for budget setting. Other people argue that holding people accountable for budget numbers focuses attention on debating the reasonableness of performance measures and targets and distracts attention from using the numbers to effect performance improvement. The tension is that accountability is thought to motivate people to strive to achieve budget targets and, without accountability for budget results, a major benefit of budgeting is lost.

These are serious issues that are taken up in the Strategic Leadership Program. In this chapter, we focus on the process of budgeting and not the behavioural issues surrounding budgeting.

The Master Budget

The master budget is the overall budget plan. The following diagram provides a summary view of the components and inter-relationships of the master budget components:

Page 428: MODULE 3 Financial Management and Management Accounting 1

Module 3 - Financial Management & Management Accounting 1

Page 428 © CMA Ontario, 2011

Master Budget Components

Behind the master budget are the organization business strategies and sales forecasts. The results of senior level planning in the organization are the specific organization goals the organization expects to achieve during the budget period.

The organization objectives result in a formal sales plan that identifies the planned level of sales for the organization’s products. The sales plan, combined with the organization’s inventory policy (e.g. produce on demand or hold 30 days of projected sales in inventory) results in the production plan.

The production plan triggers the activities needed to support production. These activities include:

1. The need to acquire additional machinery and equipment.

2. The need to acquire and train or lay off employees

3. The amount and time of the various raw materials needed for production.

Related to the sales and production plans are the discretionary spending plans. Discretionary spending is spending on support activities that is not driven directly by production levels, but are required to support sales and production activities. The most common discretionary spending items are: advertising, research and development, training and travel. Discretionary spending is subject to periodic authorization that reflects the organization’s circumstances and objectives.

Organization Goals

Discretionary Spending Plan

Capacity Plan

Sales Plan

Production PlanLabour Plan

Operating Results

Fro Forma Financial Statements

Inventory Policy Capital Plan

Page 429: MODULE 3 Financial Management and Management Accounting 1

Module 3 - Financial Management & Management Accounting 1

Page 429 © CMA Ontario, 2011

The projected or pro forma financial statements reflect the expected financial consequences of the various operating plans.

The following table summarizes the above discussion of the components of the master budget. It should be noted that, in practice, the operating budgets are often referred to as operating plans.

Financial Budgets Operating Budgets • Projected income statement • Projected balance sheet • Projected statement of cash flows

• Sales budget • Capital spending budget • Production budget • Material purchasing plan • Labour hiring and training plan • Administrative and discretionary spending plan

The Recursive Nature of Master Budget Development

By its nature the development of the master budget and its component budgets is recursive. The following are some situations that would require recycling and redeveloping the budget:

• The initial budget generates forecasted financial results that are deemed unacceptable and a more aggressive budget is demanded.

• The initial budget generates a production plan that requires the acquisition of capital equipment or skilled labour that cannot be accommodated in the time available.

• An unforeseen circumstance (such as a product failure or a competitor’s initiative) arises that requires a major reworking of the planned budget

The Sales and Production Plan

The sales plan provides the foundation for all operational planning, since it is the driver for all production and selling activities. The sales plan will specify the amount and timing of product sales. The following is an example of how the sales plan drives the production plan.

Eva Company is a retail operation selling a single product whose average price is $100 per unit. The inventory policy is to have on hand at the beginning of each month 60% of the month’s budgeted sales. The product purchase price is $40. Purchases are paid in the month following the purchase. All sales are credit sales with 25% collected during the month of sale, 40% in the following month, 30% in the third month and 5% never collected.

The following is the summary for the first six months of the upcoming year, which shows sales, purchases and the net cash flows associated with the sales and purchases activities:

Page 430: MODULE 3 Financial Management and Management Accounting 1

Module 3 - Financial Management & Management Accounting 1

Page 430 © CMA Ontario, 2011

Actual Budgeted

November December January February March April May June July

Sales (units) 1,300 1,000 1,200 1,500 1,400 1,100 1,300 900 1,200

Current month collections - 25%

$32,500 $25,000 $30,000 $37,500 $35,000 $27,500 $32,500 $22,500

Last month - 40% 52,000 40,000 48,000 60,000 56,000 44,000 52,000

Two months previous - 30% 39,000 30,000 36,000 45,000 42,000 33,000

Total collections this month

$109,000 $115,500 $131,000 $128,50

0 $118,50

0 $107,500

Inventory

Opening 780 600 720 900 840 660 780 540

Purchases 1,120 1,120 1,380 1,440 1,220 1,220 1,060 1,080

Available 1,900 1,720 2,100 2,340 2,060 1,880 1,840 1,620

Sales 1,300 1,000 1,200 1,500 1,400 1,100 1,300 900

Ending inventory 600 720 900 840 660 780 540 720

Cash flow

Opening cash $44,494 $108,694 $168,994 $242,394

$322,094

$391,794

Collections 109,000 115,500 131,000 128,500 118,500 107,500

Available $153,494 $224,194 $299,994 $370,894

$440,594

$499,294

Purchases - 100% last month 44,800 55,200 57,600 48,800 48,800 42,400

Ending cash

$108,694 $168,994 $242,394 $322,09

4 $391,79

4 $456,894

• Materials receipt labour hours required = units purchased * .1 = 1220 * .1 = 122 • Materials receipt labour cost = hours required * $20 = 122 * 20 = $2,440 • Total labour budget = $3,960 + $2,440 = $6,400

Other Expenditures

Expenditures, such as advertising and research and development, are driven by periodic budget authorizations. These expenditures reflect ongoing strategic initiatives and operational initiatives and the availability of funds.

Static Budgets

Static budgets are prepared at the start of the budget period to provide an overall financial summary of planned operations.

Consider the operations of Maggie Company, which manufactures plastic rain barrels. Each rain barrel requires:

• 20 units of plastic resin with an expected cost of $2.50 per unit

• Two valves with an expected cost of $.60 each

• .5 hours of finishing labour with an expected cost of $20 per hour

Page 431: MODULE 3 Financial Management and Management Accounting 1

Module 3 - Financial Management & Management Accounting 1

Page 431 © CMA Ontario, 2011

The Maggie Company accountant has estimated variable manufacturing overhead cost as $2.50 per unit and variable selling costs as $4 per unit. Estimated annual fixed manufacturing, selling and administrative costs are $5,000,000. Maggie Company executives believe the company will sell 200,000 rain barrels in the upcoming year at a price of $100 apiece.

The following is the static budget for Maggie Company:

Note that the unit amounts are called standards, for example, the standard price per unit of plastic is $2.50. The standard plastic cost per barrel is $50 (20 units of plastic multiplied by the standard price for plastic of $2.50).

The amounts in the total units and total dollars columns are called budgeted amounts. Budgeted amounts are computed by multiplying the unit standards by the budgeted activity level, which, in this case, is 200,000 rain barrels. For example, since the production plan calls for a production of 200,000 rain barrels and since the standard use of finishing labour is .5 hours per unit, the budgeted labour hours is 100,000 (200,000 *.5) and the budgeted cost of finishing labour is $2,000,000 (100,000 hours * $20 standard price per hour).

We can now flex the static budget by developing different budget amounts of different levels of activity. Assume that during the most recent year, Maggie Company planned on producing 200,000 rain barrels but actually produced 210,000. We can flex the static budget as follows to develop the budget against which the actual results can be compared:

Maggie Company

Page 432: MODULE 3 Financial Management and Management Accounting 1

Module 3 - Financial Management & Management Accounting 1

Page 432 © CMA Ontario, 2011

Standard Static Budget Flexible Budget

Units Price Cost Total Units

Total Dollars

Total Units

Total Dollars

Revenue $100.00 200,000 $20,000,000 210,000 $21,000,000

Variable costs

Plastic 20 $2.50 $50.00 4,000,000 $10,000,000 4,200,000 $10,500,000

Valves 2 0.60 1.20 400,000 240,000 420,000 252,000

Finishing labour 0.5 20.00 10.00 100,000 2,000,000 105,000 2,100,000

Manufacturing overhead

1 2.50 2.50 500,000 525,000

Selling 1 4.00 4.00 800,000 840,000

Total $67.70 $13,540,000 $14,217,000

Contribution margin $6,460,000 $6,783,000

Fixed costs $5,000,000 $5,000,000 $5,000,000

Profit $1,460,000 $1,783,000

Note that the static and flexible budget concepts reflect the discussion of cost volume profit relationships in Chapter 9. The rain barrels have a total variable cost of $67.70 and a contribution margin of $32.30 (100 – 67.70) per unit. Therefore, the profit equation for Maggie Company will be:

Profit = number of rain barrels produced * contribution margin per rain barrel – fixed costs

Profit = number of rain barrels produced * 32.30 - $5,000,000.

Therefore, the flexible budget profit is simply the profit line in the cost volume profit chart.

Page 433: MODULE 3 Financial Management and Management Accounting 1

Module 3 - Financial Management & Management Accounting 1

Page 433 © CMA Ontario, 2011

The Overall Cash Budget

When organizations are under financial stress, senior executives pay huge attention to the cash budget since cash is the organization’s life’s blood. Organizations that hold too much cash create opportunity cost losses, since cash is generally non-productive. Holding too little cash creates the possibility of missed opportunities or having to rely on expensive financing alternatives in the spot market.

Cash flow forecast statements usually separate cash flows relating to ongoing operations from cash flows relating to the acquisition of fixed assets and the effects on cash of financing.

The following example illustrates a cash budget for a simple organization. Sales drives the cash flow items and the major variables in the cash flow statement are:

• Sales are collected as follows: - 20% in the month of sale - 50% in the month following the month of sale - 25% in the second month following the month of sale - 5% never collected

• Other miscellaneous cash inflows are estimated to be $10,000 per month • Materials purchases are 27% of current sales • Direct labour costs are 25% of current sales • Other factory related costs are $170,000 per month plus 15% of sales • Selling costs are 5% of sales • Administrative costs are $100,000 per month plus 4% of sales • Other costs are estimated to be $10,000 per month • The company wants to maintain a minimum cash balance of $100,000 and cash

balances are maintained at this level through the line of credit accounts, which charges or pays interest based on whether the balance is positive or negative

Page 434: MODULE 3 Financial Management and Management Accounting 1

Module 3 - Financial Management & Management Accounting 1

Page 434 © CMA Ontario, 2011

The advantage of modelling cash flows is it allows planners to manage cash more effectively by identifying when significant shortages or surpluses will be generated by planned operations.

January February March April May June

Sales $1,600,000 $2,200,000 $1,750,000 $2,100,000 $1,960,000 $1,700,000

Opening cash $125,000 $161,000 $100,000 $113,000 $100,000 $100,000

Cash inflows Cash sales $320,000 $440,000 $350,000 $420,000 $392,000 $340,000 Collections of credit sales 1,700,000 1,500,000 1,425,000 1,487,500 1,505,000 1,340,000 Other cash inflows 12,000 12,000 12,000 12,000 12,000 12,000 Total cash inflows $2,032,000 $1,952,000 $1,787,000 $1,919,500 $1,909,000 $1,692,000

Cash outflows Materials purchases $432,000 $594,000 $472,500 $567,000 $529,200 $459,000 Direct labour costs 400,000 550,000 437,500 525,000 490,000 425,000 Other factory related costs 410,000 500,000 432,500 485,000 464,000 425,000 Selling costs 80,000 110,000 87,500 105,000 98,000 85,000 Administrative costs 164,000 188,000 170,000 184,000 178,400 168,000 Other costs 10,000 10,000 10,000 10,000 10,000 10,000 Total cash outflows $1,496,000 $1,952,000 $1,610,000 $1,876,000 $1,769,600 $1,572,000

Net operating cash flow $536,000 $0 $177,000 $43,500 $139,400 $120,000

Fixed asset transactions Purchases $0 -$500,000 $0 -$250,000 $0 -$100,000 Sales 0 125,000 0 45,000 0 0 Net fixed asset transactions $0 -$375,000 $0 -$205,000 $0 -$100,000

Cash before financing transactions $661,000 -$214,000 $277,000 -$48,500 $239,400 $120,000

Financing transactions Long-term borrowing $0 $0 $0 $0 $0 $0 Long-term repayments -250,000 0 0 0 0 0 Net stock related transactions 0 150,000 0 0 0 0 Net financing transactions -$250,000 $150,000 $0 $0 $0 $0

Cash before line of credit flows $411,000 -$64,000 $277,000 -$48,500 $239,400 $120,000

Line of credit activities Opening line of credit balance $250,000 $0 $164,000 $0 $148,500 $9,100 Line of credit repayment 250,000 0 164,000 0 139,400 9,100 Line of credit increases 0 164,000 0 148,500 0 0 Ending line of credit balance $0 $164,000 $0 $148,500 $9,100 $0

Cash yield from line of credit activities -$250,000 $164,000 -$164,000 $148,500 -$139,400 -$9,100

Ending cash balance $161,000 $100,000 $113,000 $100,000 $100,000 $110,900

Summary:

Overall, budget plays an important role in planning and providing alignment and direction in an organization. There are important behavioural issues associated with

Page 435: MODULE 3 Financial Management and Management Accounting 1

Module 3 - Financial Management & Management Accounting 1

Page 435 © CMA Ontario, 2011

budgeting and using budgets to provide a basis for accountability. Some of these issues will be discussed in the Strategic Leadership Program.

Page 436: MODULE 3 Financial Management and Management Accounting 1

Module 3 - Financial Management & Management Accounting 1

Page 436 © CMA Ontario, 2011

Problems with Solutions

Multiple Choice Questions Questions 1 – 2 refer to the following: Mitchell Company had the following budgeted sales for the last half of 2009: Cash Sales Credit Sales July $50,000 $150,000 August 55,000 170,000 September 45,000 130,000 October 50,000 145,000 November 60,000 200,000 December 80,000 350,000 The company is in the process of preparing a cash budget and must determine

the expected cash collections by month. To this end, the following information has been assembled: Collections on credit sales: 60% in month of sale, 30% in month following sale, 10% in second month following sale

1. Assume the accounts receivable balance on July 1, 2009, was $75,000. Of this

amount, $60,000 represented uncollected June sales. Given this data, the total cash collected during July would be:

a) $150,000 b) $235,000 c) $215,000 d) $200,000 2. What is the budgeted accounts receivable balance on December 1, 2009? a) $80,000 b) $140,000 c) $94,500 d) $131,300

Page 437: MODULE 3 Financial Management and Management Accounting 1

Module 3 - Financial Management & Management Accounting 1

Page 437 © CMA Ontario, 2011

3. Walman Company is budgeting sales of 42,000 units of product Y for March 2003. To make one unit of Product Y, three kilograms of direct Material A are required. Actual beginning and desired ending inventories of direct Material A and Product Y are:

March 1 March 31 Direct Material A 100,000 kg 110,000 kg Product Y 22,000 units 24,000 units There is no work-in-process inventory for Product Y at the beginning and end of

March. For the month of March, how many kilograms of direct Material A is Walman planning to purchase?

a) 126,000 b) 132,000 c) 136,000 d) 142,000 4. The first step in formulating next year's master budget for a manufacturing

company is to project next year's: a) Capital budget to decide which production machine to buy in order to increase

productivity b) Cash budget to decide if the company needs to take out a bank loan c) Materials and labour budget to decide on next year's direct material costs and

direct labour costs d) Production budget to decide on next year's production schedule e) Sales budget to decide next year's sales volume

Page 438: MODULE 3 Financial Management and Management Accounting 1

Module 3 - Financial Management & Management Accounting 1

Page 438 © CMA Ontario, 2011

5. Huang Company has budgeted sales and production over the next quarter as:

April May June Sales in units 100,000 120,000 ? Production in units 104,000 128,000 156,000 On April 1, the company has 20,000 units of product on hand. A minimum of 20%

of the next month's sales needs (in units) must be on hand at the end of each month. July sales are expected to be 140,000 units. What would be the budgeted sales for June (in units)?

a) 128,000 units b) 160,000 units c) 184,000 units d) 188,000 units 6. BH Wholesalers has a sales budget for December of $800,000. Cost of merchandise

sold is expected to be 30% of sales. 60% of all merchandise is paid in the month of purchase and the remaining 40% is paid in the following month. The merchandise inventory balance on November 30 is $24,000 and the December 31 merchandise inventory balance is budgeted to be $30,000. The merchandise accounts payable balance on November 30 is $102,000. The budgeted accounts payable balance for December 31 is:

a) $138,000 b) $98,400 c) $93,600 d) $147,600 e) $96,000

Page 439: MODULE 3 Financial Management and Management Accounting 1

Module 3 - Financial Management & Management Accounting 1

Page 439 © CMA Ontario, 2011

The following data apply to Questions 7 – 8: Berol Company plans to sell 200,000 units of finished product in July of 2000 and anticipates a growth rate in sales of 5% per month. The desired monthly ending inventory in units of finished product is 80% of the next month's estimated sales. There are 150,000 finished units in inventory on June 30, 2000. Each unit of finished product requires four pounds of direct material at a cost of $1.20 per pound. There are 800,000 pounds of direct material in inventory on June 30, 2000. 7. Berol Company's production requirement in units of finished product for the three

month period ending September 30, 2000 is: a) 712,025 units b) 630,500 units c) 664,000 units d) 665,720 units e) 862,025 units 8. Without prejudice to your answer to Question 7, assume Berol Company plans to

produce 600,000 units of finished product in the three-month period ending September 30, 2000 and have direct materials inventory on hand at the end of the three-month period equal to 25% of the use in that period. The estimated cost of direct material purchases for the three month period ending September 30, 2000 is:

a) $2,200,000 b) $2,400,000 c) $2,640,000 d) $2,880,000 e) $3,600,000

Page 440: MODULE 3 Financial Management and Management Accounting 1

Module 3 - Financial Management & Management Accounting 1

Page 440 © CMA Ontario, 2011

The following data apply to Questions 9 – 10: Esplanade Company has the following historical pattern for its credit sales:

70% collected in month of sale 15% collected in the first month after sale 10% collected in the second month after sale 4% collected in the third month after sale 1% uncollectible

The sales on open account have been budgeted for the past six months of 2000 as: July $60,000 August 70,000 September 80,000 October 90,000 November 100,000 December 85,000 9. The estimated total cash collections during October 2000 from accounts receivable

would be: a) $63,000 b) $84,400 c) $89,100 d) $21,400 e) $83,556 10. The estimated total cash collections during the fourth calendar quarter from sales

made on open account during the fourth calendar quarter would be: a) $172,500 b) $275,000 c) $230,000 d) $251,400 e) $265,400

Page 441: MODULE 3 Financial Management and Management Accounting 1

Module 3 - Financial Management & Management Accounting 1

Page 441 © CMA Ontario, 2011

The following data apply to Questions 11 – 12: Pardise Company budgets on an annual basis for its fiscal year. The following beginning and ending inventory levels (in units) are planned for the fiscal year of July 1, 2000, through June 30, 2001: July1, 2000 June 30, 2001 Direct material* 40,000 50,000 Work-in-process 10,000 20,000 Finished goods 80,000 50,000 *Two (2) units of direct material are needed to produce each unit of finished product. 11. If Pardise Company plans to sell 480,000 units during the 2000 - x1 fiscal year, the

number of units it would have to manufacture during the year would be: a) 440,000 units b) 480,000 units c) 510,000 units d) 450,000 units e) 460,000 units 12. If 500,000 complete units were to be manufactured during the 2000 - x1 fiscal year

by Pardise Company, the units of raw material needed to be purchased would be: a) 1,000,000 units b) 1,020,000 units c) 1,010,000 units d) 990,000 units e) 950,000 units

Page 442: MODULE 3 Financial Management and Management Accounting 1

Module 3 - Financial Management & Management Accounting 1

Page 442 © CMA Ontario, 2011

Problem 1

Down East Planters provides reforestation services to large paper products companies. It must hire one planter for every 10,000 trees it has contracted to plant. The customer provides the trees.

New employees are hired on the first day of the month they are needed. Each employee receives one week of evaluation and training before being profitably employed and, therefore, works only three of the four weeks of the first month of employment. On average, for every five prospective employees who enter training, three are deemed suitable for employment. When cutbacks occur, employees are laid off on the first day of the month. Every employee laid off receives pay equal to one week’s salary, which, on average, is $400. The payment is independent of how long the layoff will last. Laid off employees inevitably drift away and new hires must be trained.

Down East Planters has two trained employees on staff on January 1.

Down East Planters has been offered the following contracts for the upcoming year. Each monthly contract is offered on an accept or reject basis. That is, if a monthly contract is accepted, it must be completed in full. The price paid to Down East Planters is $.20 per tree.

The required tree plantings each month is:

January February March April May June July August September

October November

December

Demand

8,692 5,765 8,134 34,400 558,729 832,251 1,286,700 895,449 733,094 203,525 29,410 9,827

Required:

Develop the cash flow budget for the first six months of the upcoming year.

Page 443: MODULE 3 Financial Management and Management Accounting 1

Module 3 - Financial Management & Management Accounting 1

Page 443 © CMA Ontario, 2011

Problem 2

Madill Company manufactures a single product which is produced by blending two chemicals, A and B. Chemicals A and B are mixed in batches to produce the final product. The batch size is not variable and each batch requires 200 litres of Chemical A, 50 litres of Chemical B, two labour hours, one set of protective sheets and one container, which is used for blending Chemicals A and B. The resulting product is packaged in one litre containers, therefore, each batch yields 250 (250/1) units of final product.

The following table summarizes the amount of each variable item required per batch of product and the total cost of that item per batch of product:

Direct materials Chemical A 200 litres $900.00 Chemical B 50 litres $400.00

Direct labour 5 hours $140.00

Variable overhead Protective sheets 4 units $180.00 Container 1 unit $120.00

Madill has signed a contract to deliver the following amounts over the next six months. Madill is paid $15 per unit for this product. Payment is 50% in the month of delivery and 50% in the month following delivery.

January February March April May June

Sales (units) 2,100 2,400 3,100 2,800 2,300 2,500 Since the product is unstable, Madill produces on demand and holds minimum inventories. All chemical purchases are paid 30% in the month of production, 40% in the second month and 30% in the third month following. Direct labour is paid in the month of production. Variable overhead items are paid in the month following the month of production.

Required:

a. Develop the cash collection budget for this product. b. Develop the production budget for this product. c. Develop the materials purchase budget for this product. d. Develop the direct labour budget for this product. e. Develop the variable overhead budget for this product. f. Prepare a cash flow budget for this product. g. Assume there is a production constraint and the maximum number of batches that

can be produced in any month is 11. Develop a cash flow budget for this product.

Page 444: MODULE 3 Financial Management and Management Accounting 1

Module 3 - Financial Management & Management Accounting 1

Page 444 © CMA Ontario, 2011

Problem 3

Dish Inc., a distributor of fashion items, normally collects its receivables 45% in month of sale, 35% the subsequent month and 17% in the third month. Historically, 3% is not collected (is a bad debt).

Dish Inc. attempts to match its payments of payables closely to its receivables and pays for its purchases 40% in the month of purchase, 40% in the next month and 20% in the third month. Sales during February, March and April were $160,000, $195,000 and $168,000, respectively. Purchases during these three months were $198,000, $182,000 and $220,000 for February, March and April.

The company pays income tax instalments of $4,000 each month. Assuming the beginning balance of cash at February 1 is $39,800 and the company wishes to maintain a cash balance of $30,000, prepare a cash budget for February, March and April. Dish Inc. has a line of credit available at a rate of 10%. Interest is paid when the loan is repaid.

December and January sales were $250,000 and $175,000, respectively. December and January purchases were $195,000 and $200,000, respectively.

Required:

Prepare a cash budget for February, March and April.

Page 445: MODULE 3 Financial Management and Management Accounting 1

Module 3 - Financial Management & Management Accounting 1

Page 445 © CMA Ontario, 2011

Problem 4

The City of Loughborough had the following sales of water for the last half of 2009:

Month Sales Revenue July $120,000 August 85,000 September 115,000 October 75,000 November 150,000

• All sales are on credit. Historically, 40% is collected in the month of the sale, 40% during the first month following the sale and 20% in the second month following the sale.

• Cost of water averages 70% of sales revenue. Water is purchased in month of sale. All purchases are paid during the month following the purchase.

• Operating costs of $25,000 are paid each month.

• The September 1 cash balance is expected to be the minimum balance of $10,000.

• The minimum acceptable cash balance at the end of any month is $10,000. Money can be borrowed from a local bank in increments of $1,000. (Do not include interest charges in your budget.)

Required:

Prepare monthly a cash budget for September, October and November, 2009 and an overall cash budget for the three months together.

Page 446: MODULE 3 Financial Management and Management Accounting 1

Module 3 - Financial Management & Management Accounting 1

Page 446 © CMA Ontario, 2011

Problem 5 The following performance report is for the bottling department Moore Company for the month ending December 31, 2006.

Actual Budget Variance Volume (units) 75,000 90,000 Manufacturing costs: Direct materials $156,000 $180,000 $24,000 (F) Direct labour 835,000 900,000 65,000 (F) Variable overhead 360,000 450,000 90,000 (F) Fixed overhead 60,000 54,000 6,000 (U) Total $1,411,000 $1,584,000 $173,000 (F)

Required: Evaluate the performance report. Using a flexible budgeting approach, prepare a more appropriate performance report (i.e. produce a flexible budget and indicate planning and flexible budget variances).

Page 447: MODULE 3 Financial Management and Management Accounting 1

Module 3 - Financial Management & Management Accounting 1

Page 447 © CMA Ontario, 2011

Problem 6 The Storey Manufacturing Company makes two basic products, Cee and Dee. Data assembled by the managers follows:

Cee Dee Requirements for finished unit: Raw Material 1 10 kg 8 kg Raw Material 2 4 kg Raw Material 3 2 units 1 unit Direct labour 5 hours 8 hours Sales price $100 $150 Sales units 12,000 9,000 Estimated beginning inventory 400 150 Desired ending inventory 300 200

Raw Materials 1 2 3 Cost $2.00 per kg 2.50 per kg $.50 per unit Estimated beginning inventory 3,000 1,500 1,000 Desired ending inventory 4,000 1,000 1,500 The direct labour wage rate is $4 per hour. Overhead is applied on the basis of direct labour hours. The tax rate is 40%. The budgeted sales level is divided into quarters. Storey estimated that 20% of the annual sales will be in the first quarter, 30% in the second and 25% in the third and fourth quarters. The beginning inventory of finished products has the same cost per unit as the ending inventory. The work-in-process inventory is negligible.

Storey Manufacturing Company Sales Forecasts by Products 2001

Cee Dee Total Units Dollars Units Dollars Dollars First quarter 2,400 $ 240,000 1,800 $ 270,000 $ 510,000 Second quarter 3,600 360,000 2,700 405,000 765,000 Third quarter 3,000 300,000 2,250 337,500 637,500 Fourth quarter 3,000 300,000 2,250 337,500 637,500 Total 12,000 1,200,000 9,000 1,350,000 $2,550,000

Page 448: MODULE 3 Financial Management and Management Accounting 1

Module 3 - Financial Management & Management Accounting 1

Page 448 © CMA Ontario, 2011

Factory Overhead Costs

Indirect materials $ 10,000 Miscellaneous supplies and tools 5,000 Indirect labour 40,000 Supervision 20,000 Payroll taxes and fringe benefits 75,000 Maintenance costs – fixed 20,000 Maintenance costs – variable 10,000 Depreciation 70,000 Heat, light and power – fixed 8,710 Heat, light and power – variable 5,090 $263,800

Selling And Administrative Expenses

Advertising $ 60,000 Sales salaries 200,000 Travel and entertainment 60,000 Depreciation – warehouse 5,000 Office salaries 20,000 Executive salaries 250,000 Supplies 4,000 Depreciation – office 6,000 $605,000

Required: Prepare the following: a. Production budget b. Direct materials purchase budget c. Direct labour budget d. Cost of goods sold budget e. Budgeted income statement Note: Because you are given inventory values for the beginning and the end of the year, it is impossible to construct budgets by quarter. The budgets should be presented for the whole year.

Page 449: MODULE 3 Financial Management and Management Accounting 1

Module 3 - Financial Management & Management Accounting 1

Page 449 © CMA Ontario, 2011

Problem 7 The Gowan Corporation manufactures and distributes wooden baseball bats. This is a seasonal business with a large portion of its sales occurring in late winter and early spring. The production schedule for the last quarter of the year is heavy in order to build up inventory to meet expected sales volume. The company experiences a temporary cash strain during this heavy production period. Payroll costs rise during the last quarter because overtime is scheduled to meet the increased production needs. Collections from customers are low because the fall season produces only modest sales. This year the company's concern is intensified because prices are increasing during the current inflationary period. In addition, the sales department forecasts sales of fewer than one million bats for the first time in three years. This decrease in sales appears to be caused by the popularity of aluminum bats. The cash account builds up during the first and second quarters as sales exceed production. The excess cash is invested in Treasury bills and other commercial paper. During the last half of the year, the temporary investments are liquidated to meet the cash needs. In the early years of the company, short-term borrowing was used to supplement the funds released by selling investments, but this has not been necessary in recent years. Because costs are higher this year, the treasurer asks for a forecast for December to judge if the $40,000 in temporary investments will be adequate to carry the company through the month with a minimum balance of $10,000. Should this amount ($40,000) be insufficient, she wants to begin negotiations for a short-term loan. The unit sales volume for the past two months and the estimate for the next four months are:

October (actual) 70,000 November (actual) 50,000 December (estimated) 50,000 January (estimated) 90,000 February (estimated) 90,000 March (estimated) 120,000

The bats are sold for $5 each. All sales are made on account. Half of the accounts are collected in the month of the sale, 40% are collected in the month following the sale and the remaining 10% in the second month following the sale. Customers who pay in the month of the sale receive a 2% cash discount. The production schedule for the six-month period beginning with October reflects the company's policy of maintaining a stable year-round work force by scheduling overtime to meet the following production schedules:

Page 450: MODULE 3 Financial Management and Management Accounting 1

Module 3 - Financial Management & Management Accounting 1

Page 450 © CMA Ontario, 2011

October (actual) 90,000 November (actual) 90,000 December (estimated) 90,000 January (estimated) 90,000 February (estimated) 100,000 March (estimated) 100,000

The bats are made from wooden blocks that cost $6 each. Ten bats can be produced from each block. The blocks are acquired one year in advance so they can be properly aged. Gowan pays the supplier one-twelfth the cost of this material each month until the obligation is retired. The monthly payment is $60,000. The plant is normally scheduled for a 40-hour, five-day work week. During the busy production season, however, the work week may be increased to six 10-hour days. Workers can produce 7.5 bats per hour. Normal monthly output is 75,000 bats. Factory employees are paid $15 per hour for regular time and time and one-half for overtime. Other manufacturing costs include variable overhead of $.30 per unit and annual fixed overhead of $280,000. Depreciation charges totalling $40,000 are included among the fixed overhead. Selling expenses include variable costs of $.20 per unit and annual fixed costs of $60,000. Fixed administrative costs are $120,000 annually. All fixed costs are incurred uniformly throughout the year. The controller has accumulated the following additional information: 1. The balances of selected accounts as of November 30, 2004 are:

Cash $ 12,000 Marketable securities, at market value 40,000 Accounts receivable 96,000 Prepaid expenses 4,800 Accounts payable (arising from raw material purchases) 300,000 Accrued vacation pay 9,500 Equipment note payable 102,000 Accrued income taxes payable 50,000

2. Interest to be received from the company's temporary investments is estimated at

$500 for December. 3. Prepaid expenses of $3,600 will expire during December and the balance of the

prepaid account is estimated at $4,200 for the end of December. 4. Gowan purchased new machinery in 2004 as part of a plant modernization

program. The machinery was financed by a 24-month note of $144,000. The terms call for equal principal payments over the next 24 months with interest paid

Page 451: MODULE 3 Financial Management and Management Accounting 1

Module 3 - Financial Management & Management Accounting 1

Page 451 © CMA Ontario, 2011

at the rate of 1% per month on the unpaid balance at the first of the month. The first payment was made on May 1, 2004.

5. Old equipment, which has a book value of $8,000, is to be sold during December

for $7,500. 6. Each month the company accrues $1,700 for vacation pay by charging vacation

pay expense and crediting accrued vacation pay. The plant closes for two weeks in June when all plant employees take vacation.

7. Quarterly dividends of $.20 per share will be paid on December 15 to

stockholders of record. Gowan Corporation has authorized 10,000 shares. The company has issued 7,500 shares and 500 of these are classified as treasury stock.

8. The quarterly income taxes payment of $50,000 is due on December 15, 2004. Required: Prepare a schedule that forecasts the cash position at December 31, 2004. What action, if any, will be required to maintain a $10,000 cash balance?

Page 452: MODULE 3 Financial Management and Management Accounting 1

Module 3 - Financial Management & Management Accounting 1

Page 452 © CMA Ontario, 2011

Solutions: Multiple Choice Questions 1. d From May sales: $ 15,000 From June Sales: $60,000 ÷ .4 = $150,000 x 30% 45,000 From July sales Cash 50,000 Credit: $150,000 x .6 90,000 $200,000

2. c October sales: $145,000 x 10% $14,500 November sales: $200,000 x 40% 80,000 $94,500 3. d Product Y: 42,000 Sales 24,000 Ending inventory 66,000 Total needs 22,000 Less opening inventory 44,000

Direct Material A: Required for production: 44,000 x 3 kg 132,000 Ending inventory 110,000 Total needs 242,000 Less opening inventory 100,000 142,000 4. e The sales forecast is the usual starting point for budgeting because production

and inventory levels generally depend on the forecasted level of sales. 5. b Needs for June: Sales X + ending inventory: 140,000 x 20% 28,000 - opening inventory 0.2X = June production 156,000 X + 28,000 - .2X = 156,000 0.8X = 128,000 X = 160,000

Page 453: MODULE 3 Financial Management and Management Accounting 1

Module 3 - Financial Management & Management Accounting 1

Page 453 © CMA Ontario, 2011

6. b Budgeted cost of merchandise sold in December = $800,000 x 30% = $240,000 Budgeted merchandise purchases in December = $240,000 + $30,000 ending inventory - $24,000 beginning inventory = $246,000 Budgeted merchandise accounts payable balance = $246,000 x 40% = $98,400

7. d Sales: July: 200,000; Aug: 200,000 x 1.05 = 210,000 September: 210,000 x 1.05 = 220,500; Total = 630,500 September ending inventory requirement = $220,500 x 1.05 x 80% = 185,220 Production: 630,500 sales + 185,220 ending inventory - 150,000 op. inv. = 665,720 units 8. c Ending inventory requirements: 600,000 units x 4 lbs x 25% = 600,000 lbs Unit purchases = (600,000 x 4) prod. + 600,000 ending inv. - 800,000 op. inv. = 2,200,000 x $1.20 = $2,640,000 9. b July – $60,000 x 4% $2,400 August – $70,000 x 10% 7,000 September – $80,000 x 15% 12,000 October – $90,000 x 70% 63,000 $84,400

10. c October – $90,000 x 95% $85,500 November – $100,000 x 85% 85,000 December – $85,000 x 70% 59,500 $230,000

11. d 480,000 + 50,000 - 80,000 = 450,000 12. c (500,000 x 2) + 50,000 - 40,000 = 1,010,000 units

Page 454: MODULE 3 Financial Management and Management Accounting 1

Module 3 - Financial Management & Management Accounting 1

Page 454 © CMA Ontario, 2011

Problem 1

The following is the cash flow budget for the entire year:

End of month 1 1 1 5 73 87 143 90 74 21 3 1

Capacity (d below) 10,000 10,000 10,000 40,000 560,000 835,000 1,290,000 900,000 740,000 210,000 30,000 10,000

Wages (e below) $1,600 $1,600 $1,600 $6,400 $89,600 $133,600 $206,400 $144,000 $118,400 $33,600 $4,800 $1,600

Training costs (e below)

0 0 0 2,800 45,600 9,600 37,600 0 0 0 0 0

Layoff severance 400 0 0 0 0 0 0 21,200 6,400 21,200 7,200 800

Total costs $2,000 $1,600 $1,600 $9,200 $135,200 $143,200 $244,000 $165,200 $124,800 $54,800 $12,000 $2,400

Net cash flow -$262 -$447 $27 -

$2,320 -$23,454 $23,250 $13,340 $13,890 $21,819 -

$14,095 -$6,118 -$435

Beginning capacity 20,000 10,000 10,000 10,000 50,000 730,000 870,000 1,430,000 900,000 740,000 210,000 30,000

Demand shortage (excess)

-11,308

-4,235 -1,866 24,400 508,729 102,251 416,700 -534,551 -166,906 -536,475

-180,590

-20,173

(a) New planters needed

0 0 0 4 68 14 56 0 0 0 0 0

(b) Training = 5/3 × needed

0 0 0 7 114 24 94 0 0 0 0 0

(c) Planters laid off 1 0 0 0 0 0 0 53 16 53 18 2

(d) Trainees add to capacity for only 3 weeks of their first month. For example in April, the capacity is ((1 × 10,000) + (4 × 10,000 × ¾)) = 40,000.

(e) Trainees are hired at the beginning of the month and receive $400 for a week of training and 3 weeks of wages at $400 per week. Trained workers receive $1,600 a month.

Problem 2

a.

January February March April May June

Sales (units) 2,100 2,400 3,100 2,800 2,300 2,500

Sales Collections This Month $15,750 $18,000 $23,250 $21,000 $17,250 $18,750 Last Month 0 15,750 18,000 23,250 21,000 17,250

Total collections $15,750 $33,750 $41,250 $44,250 $38,250 $36,000

Page 455: MODULE 3 Financial Management and Management Accounting 1

Module 3 - Financial Management & Management Accounting 1

Page 455 © CMA Ontario, 2011

b.

January February March April May June

Sales (units) 2,100 2,400 3,100 2,800 2,300 2,500 Opening inventory 0 150 0 150 100 50

Minimum production 2,100 2,250 3,100 2,650 2,200 2,450 Batches required 9 9 13 11 9 10 Production 2,250 2,250 3,250 2,750 2,250 2,500

Ending inventory 150 0 150 100 50 50

c.

January February March April May June

Sales (units) 2,100 2,400 3,100 2,800 2,300 2,500 Opening inventory 0 150 0 150 100 50

Minimum production 2,100 2,250 3,100 2,650 2,200 2,450 Batches required 9 9 13 11 9 10 Production 2,250 2,250 3,250 2,750 2,250 2,500

Ending inventory 150 0 150 100 50 50

Materials purchase Chemical A purchases Amount 1,800 1,800 2,600 2,200 1,800 2,000 Cost $8,100 $8,100 $11,700 $9,900 $8,100 $9,000

Chemical B purchases Amount 450 450 650 550 450 500 Cost $3,600 $3,600 $5,200 $4,400 $3,600 $4,000

Total purchases $11,700 $11,700 $16,900 $14,300 $11,700 $13,000

Payment $3,510 $8,190 $13,260 $14,560 $14,300 $12,870

Page 456: MODULE 3 Financial Management and Management Accounting 1

Module 3 - Financial Management & Management Accounting 1

Page 456 © CMA Ontario, 2011

d.

January February March April May June

Sales (units) 2,100 2,400 3,100 2,800 2,300 2,500 Opening inventory 0 150 0 150 100 50

Minimum production 2,100 2,250 3,100 2,650 2,200 2,450 Batches required 9 9 13 11 9 10 Production 2,250 2,250 3,250 2,750 2,250 2,500

Ending inventory 150 0 150 100 50 50

Labour hours Amount 45 45 65 55 45 50 Cost $1,260 $1,260 $1,820 $1,540 $1,260 $1,400

Payment $1,260 $1,260 $1,820 $1,540 $1,260 $1,400

e.

January February March April May June

Sales (units) 2,100 2,400 3,100 2,800 2,300 2,500 Opening inventory 0 150 0 150 100 50

Minimum production 2,100 2,250 3,100 2,650 2,200 2,450 Batches required 9 9 13 11 9 10 Production 2,250 2,250 3,250 2,750 2,250 2,500

Ending inventory 150 0 150 100 50 50

Variable overhead items Protective sheets 36 36 52 44 36 40 Cost $6,480 $6,480 $9,360 $7,920 $6,480 $7,200

Containers 9 9 13 11 9 10 Cost $1,080 $1,080 $1,560 $1,320 $1,080 $1,200

Total purchases $7,560 $7,560 $10,920 $9,240 $7,560 $8,400

Payment 0 $7,560 $7,560 $10,920 $9,240 $7,560

Page 457: MODULE 3 Financial Management and Management Accounting 1

Module 3 - Financial Management & Management Accounting 1

Page 457 © CMA Ontario, 2011

f. January February March April May June

Sales (units) 2,100 2,400 3,100 2,800 2,300 2,500

Opening inventory 0 150 0 150 100 50 Minimum production 2,100 2,250 3,100 2,650 2,200 2,450

Batches required 9 9 13 11 9 10

Production 2,250 2,250 3,250 2,750 2,250 2,500

Ending inventory 150 0 150 100 50 50

Sales collections

This month $15,750 $18,000 $23,250 $21,000 $17,250 $18,750

Last month 0 15,750 18,000 23,250 21,000 17,250

Total collections $15,750 $33,750 $41,250 $44,250 $38,250 $36,000

Materials purchase

Chemical A purchases

Amount 1,800 1,800 2,600 2,200 1,800 2,000

Cost $8,100 $8,100 $11,700 $9,900 $8,100 $9,000

Chemical B purchases

Amount 450 450 650 550 450 500

Cost $3,600 $3,600 $5,200 $4,400 $3,600 $4,000

Total purchases $11,700 $11,700 $16,900 $14,300 $11,700 $13,000

Payment $3,510 $8,190 $13,260 $14,560 $14,300 $12,870

Labour Hours

Amount 45 45 65 55 45 50

Cost $1,260 $1,260 $1,820 $1,540 $1,260 $1,400

Payment $1,260 $1,260 $1,820 $1,540 $1,260 $1,400

Protective sheets 36 36 52 44 36 40

Cost $6,480 $6,480 $9,360 $7,920 $6,480 $7,200

Containers 9 9 13 11 9 10

Cost $1,080 $1,080 $1,560 $1,320 $1,080 $1,200

Total purchases $7,560 $7,560 $10,920 $9,240 $7,560 $8,400

Payment 0 $7,560 $7,560 $10,920 $9,240 $7,560

Total disbursements $4,770 $17,010 $22,640 $27,020 $24,800 $21,830

Net cash flow $10,980 $16,740 $18,610 $17,230 $13,450 $14,170

Page 458: MODULE 3 Financial Management and Management Accounting 1

Module 3 - Financial Management & Management Accounting 1

Page 458 © CMA Ontario, 2011

g.

January February March April May June Sales (units) 2,100 2,400 3,100 2,800 2,300 2,500

Opening inventory 0 150 500 150 100 50

Minimum production 2,100 2,250 2,600 2,650 2,200 2,450

Batches required 9 11 11 11 9 10

Production 2,250 2,750 2,750 2,750 2,250 2,500

Ending inventory 150 500 150 100 50 50

Sales collections

This month $15,750 $18,000 $23,250 $21,000 $17,250 $18,750

Last month 0 15,750 18,000 23,250 21,000 17,250

Total collections $15,750 $33,750 $41,250 $44,250 $38,250 $36,000

Materials purchase

Chemical A purchases

Amount 1,800 2,200 2,200 2,200 1,800 2,000

Cost $8,100 $9,900 $9,900 $9,900 $8,100 $9,000

Chemical B purchases

Amount 450 550 550 550 450 500

Cost $3,600 $4,400 $4,400 $4,400 $3,600 $4,000

Total purchases $11,700 $14,300 $14,300 $14,300 $11,700 $13,000

Payment $3,510 $8,970 $13,520 $14,300 $13,520 $12,870

Labour hours

Amount 45 55 55 55 45 50

Cost $1,260 $1,540 $1,540 $1,540 $1,260 $1,400

Payment $1,260 $1,540 $1,540 $1,540 $1,260 $1,400

Protective sheets 36 44 44 44 36 40

Cost $6,480 $7,920 $7,920 $7,920 $6,480 $7,200

Containers 9 11 11 11 9 10

Cost $1,080 $1,320 $1,320 $1,320 $1,080 $1,200

Total purchases $7,560 $9,240 $9,240 $9,240 $7,560 $8,400

Payment 0 $7,560 $9,240 $9,240 $9,240 $7,560

Total disbursements $4,770 $18,070 $24,300 $25,080 $24,020 $21,830

Net cash flow $10,980 $15,680 $16,950 $19,170 $14,230 $14,170

Page 459: MODULE 3 Financial Management and Management Accounting 1

Module 3 - Financial Management & Management Accounting 1

Page 459 © CMA Ontario, 2011

Problem 3

Dish Company Cash Budget

for the three months ended April 30, 2000

February March April

Cash balance, beginning $39,800 $30,000 $30,000

Add cash receipts (Schedule 1) 175,750 173,500 171,050

Total cash available 215,550 203,500 201,050

Less: cash disbursements Cash disbursements from operations (Schedule 2) $198,200 $192,000 $200,400 Income taxes 4,000 4,000 4,000

Total disbursements $202,200 $196,000 $204,400

Excess (deficiency) of total cash 13,350 7,500 -3,350 available over total disbursements

Financing: Borrowings (at end of period) (Note 1) 16,650 22,500 33,350

Cash balance, ending $30,000 $30,000 $30,000

Note 1: Dish Company requires a minimum cash balance of $30,000. Therefore, the company must borrow $16,650, $22,500 and $33,350 in February, March and April, respectively.

Schedule 1 - Cash Receipts

Percentage December January February March April

Sales $250,000 $175,000 $160,000 $195,000 $168,000 Collections from: Current month 45% $72,000 $87,750 $75,600 Last month 35% 61,250 56,000 68,250 Second to last month

17% 42,500 29,750 27,200

Total $175,750 $173,500 $171,050

Page 460: MODULE 3 Financial Management and Management Accounting 1

Module 3 - Financial Management & Management Accounting 1

Page 460 © CMA Ontario, 2011

Schedule 2 - Cash Disbursements

Percentage December January February March April

Purchases $195,000 $200,000 $198,000 $182,000 $220,000 Payments for purchases:

Current month 40% $79,200 $72,800 $88,000 Last month 40% 80,000 79,200 72,800 Second to last month

20% 39,000 40,000 39,600

Total $198,200 $192,000 $200,400

Problem 4

July August September October November Total

Sales $120,000 $85,000 $115,000 $75,000 $150,000 $545,000

Cash balance beginning

$10,000 $29,500 $17,000 $10,000

Plus: cash collections:

Month of sale 40% $46,000 $30,000 $60,000 $136,000 Month following 40% 34,000 46,000 30,000 110,000 2nd month 20% 24,000 17,000 23,000 64,000

Total available $114,000 $122,500 $130,000 $320,000

Less cash disbursements

Water 70% $59,500 $80,500 $52,500 $192,500 Operating costs $25,000 25,000 25,000 25,000 75,000

Total disbursements

$84,500 $105,500 $77,500 $267,500

Cash balance ending

$29,500 $17,000 $52,500 $52,500

Page 461: MODULE 3 Financial Management and Management Accounting 1

Module 3 - Financial Management & Management Accounting 1

Page 461 © CMA Ontario, 2011

Problem 5 The performance report provided for the bottling department matches an actual budget for 75,000 units to a budget based on 90,000 units. The favourable variances are misleading because a budget based on 75,000 units would be more appropriate.

Master Budget

Planning Variance

Flexible Budget

Flexible Budget

Variance

Actual

Volume (units) 90,000 15,000 75,000 0 75,000

Manufacturing costs: Direct materials $180,000 ($30,000) $150,000 $6,000 (U) $156,000

Direct labour 900,000 (150,000) 750,000 85,000 (U) 835,000 Variable o/h 450,000 (75,000) 375,000 (15,000) (F) 360,000 Fixed o/h 54,000 0 54,000 6,000 (U) 60,000

Total $1,584,000 ($255,000) $1,329,000 $82,000 (U) $1,411,000

Page 462: MODULE 3 Financial Management and Management Accounting 1

Module 3 - Financial Management & Management Accounting 1

Page 462 © CMA Ontario, 2011

Problem 6 a. Cee Dee Needs: Sales 12,000 9,000 Ending inventory 300 200 Less: beginning inventory (400) (150) Production budget 11,900 9,050 b. DM1 DM2 DM3 Needs: Production (11,900 x 10) + (9,050 x 8) 191,400 9,050 x 4 36,200 (11,900 x 2) + (9,050 x 1) 32,850 Ending inventory 4,000 1,000 1,500 Less: beginning inventory (3,000) (1,500) (1,000) Purchase budget – units 192,400 35,700 33,350 Purchase price $2.00 $2.50 $.50 Purchase budget $384,800 $89,250 $16,675 c. Cee Dee Hours: 11,900 x 5 | 9,050 x 8 59,500 72,400 Hourly rate $4 $4 Direct labour budget $238,000 $289,600 d. Cee Dee Manufacturing cost per unit RM1: 10 x $2 | 8 x $2 $20.00 $16.00 RM2: 4 x $2.50 10.00 RM3: 2 x $.50 | 1 x $.50 1.00 .50 Direct labour: 5 hrs x $4 | 8 hrs x $4 20.00 32.00 Manufacturing overhead Rate: $263,800 / (59,500 + 72,400) = $263,800 / 131,900 hours = $2 per DLH 5 hrs x $2 | 8 hrs x $2 10.00 16.00 $51.00 $74.50

Page 463: MODULE 3 Financial Management and Management Accounting 1

Module 3 - Financial Management & Management Accounting 1

Page 463 © CMA Ontario, 2011

Cost of goods sold Cee Dee Opening inventory:

400 x $51 | 150 x $74.50

$ 20,400

$ 11,175 Cost of goods manufactured 11,900 x $51 | 9,050 x $74.50 606,900 674,225 Ending inventory:

300 x $51 | 200 x $74.50

(15,300)

(14,900) $612,000 $670,500

e. Cee Dee Total Sales $1,200,000 $1,350,000 $2,550,000 Cost of goods sold 612,000 670,500 1,282,500 Gross margin $ 588,000 $ 679,500 1,267,500 Selling and administrative expenses 605,000 Operating income 662,500 Income taxes (40%) 265,000 Net income $397,500

Page 464: MODULE 3 Financial Management and Management Accounting 1

Module 3 - Financial Management & Management Accounting 1

Page 464 © CMA Ontario, 2011

Problem 7 Cash, November 30, 2004 $12,000 Cash collections: Oct. sales: 70,000 x $5 x 10% $ 35,000 Nov. sales: 50,000 x $5 x 40% 100,000 Dec. sales: 50,000 x $5 x 50% x .98 122,500 257,500 Interest from investments 500 Sale of equipment 7,500 Cash disbursements: Direct materials $60,000 Direct labour Regular time: 75,000 bats / 7.5 x $15 150,000 Overtime: 15,000 bats / 7.5 x $15 x 1.5 45,000 Overhead: Variable: 90,000 bats x $.30 27,000 Fixed: ($280,000 – 40,000) / 12 20,000 Selling: Variable: 50,000 x $.20 10,000 Fixed: $60,000 / 12 5,000 Fixed administrative: $120,000 / 12 10,000 Prepaid expenses: $4,200 Ending Balance – ($4,800 Opening Balance – 3,600 Expired in December)

3,000

Income taxes 50,000 Note payment: $144,000 / 24 6,000 Interest on note payable: [$144,000 - (6,000 x 7)] x 1%

1,020

Dividend payment: 7,000 shares x $.20 1,400 (388,420) Cash balance before financing (110,920) Liquidation of temporary investments 40,000 Borrowing 80,920 Cash balance, end $10,000